Download as pdf or txt
Download as pdf or txt
You are on page 1of 421

PRELIM BITS

UPSC/CSE Economy PYQs


1995 - 2023
By Roman
10/22/2023

https://t.me/prelimbits
1

1. Which one of the following is true regarding the Jawahar Rozgar Yojana (JRY)? [1995]
A. It was launched during the Prime Ministership of Indira Gandhi

B. It aims at creating one million jobs annually

C. The target group of JRY are the urban poor living below the poverty line

D. Under the scheme 30% of the employment generated is reserved for women

Ans. D

JAWAHAR ROZGAR YOJANA (JRY)


 Under the Wage Employment Programmes, the National Rural Employment Programme
(NREP) and Rural Landless Employment Guarantee Programme (RLEGP) were started in Sixth
and Seventh Plans.
 The NREP and RLEGP were merged in April 1989 under the Jawahar Rozgar Yojana
(JRY).
 The JRY was meant to generate meaningful employment opportunities for the
unemployed and underemployed in rural areas through the creation of economic
infrastructure and community and social assets.
 The JRY was revamped from 1st April 1999 and came to be known as Jawahar Gram
Samriddhi Yojana (JGSY).
 Later from September 25, 2001, Jawahar Gram Samridhi Yojna was merged with
Sampoorna Grameen Rozgar Yojna. This scheme was merged in 2006 with Mahatma
Gandhi National Rural Employment Guarantee Act (MGNREGA)

2. The Narasimham Committee for financial sector Reforms has suggested


reduction in [1995]
A. SLR and CRR

B. SLR, CRR and Priority Sector Financing

C. SLR and Financing to capital goods sector

D. CRR, Priority Sector Financing and Financing to capital goods sector

Ans. B

NARASIMHAM COMMITTEE I (1991)


 FM Manmohan Singh to examine the functioning of banks. In August 1991, a nine-
member committee was appointed to suggest reforms to the financial system.

 The committee submitted its recommendations and the report in December, 1991 to the
Parliament.

 Recommendations →

o Reduction in SLR and CRR- During 1991, both Statutory Liquidity Ratio (SLR) and
Cash Reserve Ratio (CRR) were extremely high. Due to this, bank resources were
https://t.me/prelimbits
2

not available for government use. The committee recommended reducing


the SLR and CRR from 38.5 percent to 25 percent and from 15 percent to 3 to 5
percent, respectively.

o Reorganization of the Banking sector- Committee recommended reduction in


the number of public sector banks. The committee suggested mergers and
acquisitions to increase the bank‘s efficiency. The Committee recommended
nationwide the national recognition of 8 to 10 banks.

o Establishment of the ARF Tribunal- During the 1991 economic crisis, banks' bad
debts and Non-Performing Assets (NPA) were concerning. The committee
recommended setting up an Asset Reconstruction Fund (ARF) to take over
the proportion of bad and doubtful debts from banks and financial institutions.

o Removal of Dual Control- At that point, the banking sector in India was regulated
by the RBI and the Ministry of Finance. The committee proposed RBI be the
sole primary regulator of banking in India.
o Stop the Directed Credit Program- Directed credit programme should be phased
out gradually. As per the committee, agriculture and small-scale industries (SSIs)
had already grown to a mature stage and they did not require any special
support; two decades of interest subsidy were enough. Therefore, concessional
rates of interest could be dispensed with.

 Directed credit should not be a regular programme - it should be a case of


extraordinary support to certain weak sections—besides, it should
be temporary, not a permanent one.

 Concept of PSL should be redefined to include only the weakest


sections of the rural community such as marginal farmers, rural artisans,
village and cottage industries, tiny sector, etc.

 The ―redefined PSL‖ should have 10 per cent fixed of the


aggregate bank credit.

 The composition of the PSL should be reviewed after every 3 years.

o Interest Rate Determination- The committee highlighted that the interest rates
should be determined based on Market Forces and not by the
Government, which was earlier the case.

o More Freedom to Banks- To improve the workings of banks, the Narasimhan


Committee 1 recommended that every bank be free and autonomous to
carry out its work. Over-regulation and over-administration should be
avoided, and the selection of the Chief Executive and board of directors should be

https://t.me/prelimbits
3

made on merit solely.

o The RBI should allow the establishment of new private-sector banks as


long as they meet the minimum start-up capital and other requirements.

o The government should declare that no more banks will be nationalized.

o Foreign banks are permitted to open branches in India, either wholly-owned or


as

3. Corporation tax: [1995]


A. is levied and appropriated by the States

B. is levied by the Union and collected and appropriated by the States

C. is levied by the Union and shared by the Union and the States

D. is levied by the Union and belongs to it exclusively

Ans. C

CORPORATION TAX

 The Indian government levies corporate taxes on enterprises as a source of income.


Corporation tax is a tax on a company's Net Profits

 Companies that are registered in India under the Companies Act 1956 [pvt & public]

 According to Income Tax Act of 1961, the tax is applied at a certain rate.

 The Minimum Alternate Tax (MAT) does not apply to such companies.

o Minimum Alternate Tax (MAT) →

 MAT is a clause in direct tax laws that restrict tax exemptions granted to
businesses, requiring them to pay a minimum amount of corporate tax to
the government.

 Unlike a conventional corporate tax, which is imposed on taxable


earnings, MAT is levied on Book Profit. [what is BF not Imp]

 MAT was 1st implemented in 1988 to bring zero-tax corporations into the
tax net.

 It was later repealed in 1990. However, the Finance Act of 1996 reinstated it.

https://t.me/prelimbits
4

 The tax applies to all entities functioning in India, regardless of whether


they are owned by Indians or foreigners.

 Life Insurance Firms and Shipping Enterprises liable for tonnage tax
are notable exceptions.

 Companies with no permanent premises in India are also exempt from


paying MAT.

 The government lowered the MAT tax rate from 18.5 % to 15% in
September 2019, while simultaneously lowering the company tax rate
from 30 % to 22 %

 Cooperative Societies is also liable to pay 15% MAT.

 Taxes are levied on a company's taxable income, which comprises →

o Revenue minus cost of goods sold, general and administrative expenses, selling
and marketing, R&D, depreciation, and other operating costs.

 Types of income that a company earns →

o Profits earned by the business: Profits refers to the financial benefits realised by a
company when its total revenue exceeds total expenses.

o Income from renting a property: When a business lets out its property on rent, its
rental income comes under the purview of business income.

o Capital gains: Capital gains refer to the increase in the value of a company‘s
capital assets. A capital gain, in this case, can be short-term or long-term and is
claimed on income taxes.

o Income from other sources: Any other income of an enterprise that is not
specifically taxed under other heads is taxed as earnings from other sources. It
includes income from dividends, interests, etc.

 Companies, both domestic and foreign, are liable to pay an annual corporate tax. It is,
therefore, based on the above income earned in a given financial year.

Corporation Tax Cut (2019)

 Corporate tax rate to be 22 % without exemptions.

o No MAT (Zero TAX companies) applicable on such companies.

o Effective corporate tax rate [ETR] after surcharge and cess to be 25.17 percent.

 Local companies incorporated after Oct. 2019 : 15% till March 23 & ETR 17.01 %

 MAT relief : reduced to 15 % from 18.5 %

 Foreign Company‘s profit from India : 40%+surcharge+cess [No Change]

Corporation Tax: Exemption from audit for small companies

https://t.me/prelimbits
5

 Audit is a systematic and scientific examination of the Balance Sheet/Accounts


Books/Financial Transactions.

 Audit is done by an independent person known as Auditor.

 By default, companies with turnover above ₹1cr, need to present their audited accounts
to the Govt.

 Budget-2021: if the company carries out 95% of their transactions digitally, they'll be
exempted from the audit requirement, if their turnover upto ₹10cr

Corporation Tax on Startups

 Startup is a company not older than 10 years and not having turnover more than 100 cr.
Govt helps them through Startup India.

 Startup can claim 100% deduction on its profits, for 3 years out of the first 10 years of
incorporation.

Domestic Company Foreign Company

Area of Economic transactions take place within Economic transactions take place
operations the geographical boundaries of India. with several countries across the
globe.

Registration Registered under the Companies Act of Not registered under the
India. Companies Act of India.
Also includes companies with a foreign
registration but has control and
management wholly in India.

Currency dealt Single currency Multiple currencies

4. One of the reasons for India‟s occupational structure remaining more or less the
same over the years has been that: [1995]
A. Investment pattern has been directed towards capital intensive industries

B. Productivity in agriculture has been high enough to induce people to stay with agricultural

C. ceiling on land holdings have enabled more people to own land and hence their preference
to stay with agriculture

D. People are largely unaware of the significance of transition from agriculture to industry for
economic development

Ans. A

OCCUPATIONAL STRUCTURE OF INDIA

https://t.me/prelimbits
6

 Occupational structure, usually refer to the percentage of that nation‘s workforce, which
is employed in the diverse economic ventures present in the nation

 Meaning number of the people in the total working population who are employed in the
agricultural, as well as the associated activities along with identifying, how many of these
people are included in the manufacturing and the service sectors. [figure maybe bit old
but you can get the idea]

 During that time, the agricultural sector had appraised for the highest level of share of the
working population, normally prevailing at 70-75%, while the manufacturing sector
along with the services sector of the nation was evaluated for only 10% and 15-20%
respectively.

 According to the three-sector model, the main focus of an economy's activity shifts
from the primary, through the secondary and finally to the tertiary sector.

 But event after the 75 yrs of independence large numbers of working population is still
employed in agriculture sector.

Factors Responsible for Failure of Occupational Structure:

 Indian planners failed to make any serious attempt for the development of rural
economy for utilizing the vast idle labour force and also to raise the productivity of
labourers. Due to poor organisation, the programmes of reducing unemployment and
under-employment problem in the rural areas failed miserably.

 Planners did not make any serious attempt to enlarge the scope of non-agricultural
rural employment.

 Land reforms in India failed miserably to realise its goal and to create small owner
holding. These reforms could not diffuse the ownership of land among a large number of
marginal cultivators.

 Various other facilities provided by the Government such as cheaper credit, marketing,
subsidy on fertilizer price etc. only benefitted rich farmers and poor and marginal

https://t.me/prelimbits
7

farmers could not reap any benefit from these facilities leading to a failure in raising their
agricultural productivity.

 Efforts of the planners to develop industries helped the large scale capital goods
sector and the plans could not create much response to the development of
small scale and cottage industries. This development of large scale highly capital-
intensive industries could not create much employment potential and thus created no
impact on the occupational structure of the country.

5. The main reason for low growth rate in India, inspite of high rate of savings and
capital formation is: [1995]
A. high birth rate

B. low level of foreign and

C. low capital output ratio

D. high capital output ratio

Ans. D

 It is the amount of capital needed to produce one unit of output.

 It depends on factors such as

o Technological progress,

o Prices of capital goods / machinery.

 In India, High Capital Ratio is among the reasons for subdued growth rates.

 Productivity of capital = Output ÷ Capital. Higher is the productivity of capital, it is


good for the economy.

 The inverse of ―productivity of capital‖ is Capital/Output ratio. This is Bad For


Economy.

o If Capital/Output ratio is 3/1, that means Rs. 1 unit of output is produced from
Rs. 3 units of capital.

o And if Capital/Output ratio is 4/1, that means to produce Rs. 1 unit of output, Rs.
4 units of capital is required. So, 3/1 is better than 4/1 for the economy.

 Generally, if an economy has higher savings, higher capital formation happens. But if
Capital/Output ratio in the economy is high, then that means the productivity of the
capital is low, so output production may not increase much even if capital formation is
high.

Incremental Capital Output Ratio (ICOR)

 Another variant of capital output ratio is Incremental Capital Output Ratio (ICOR).

https://t.me/prelimbits
8

 The ICOR indicate additional unit of capital or investment needed to produce an


additional unit of output.

6. Which one of the following Five Year Plans recognised human development as the core
of all development efforts? [1995]
A. The Third Five Year Plan
B. The Fifth Five Year Plan
C. The Sixth Five Year Plan
D. The Eighth Five Year Plan

Ans. D

In the eight five year plan (1992-1997), the top priority was given to the development of the
human resources i.e., employment, education, and public health.

HISTORY OF FIVE YEAR PLANS


 The Idea of Planning as a process of rebuilding the economy gained prominence in the
1940s-50s.
 Various Industrialists came together in 1944 and drafted a joint proposal for setting up a
planned economy in India. It is famously known as the Bombay Plan.
 Planning for development was seen as a crucial choice for the country, following
Independence.
 Joseph Stalin was the first person to implement the Five-Year Plan in the Soviet Union,
in the year 1928.
 India launched a series of Five-Year Plans after independence to build its economy and
attain development.

Concept of FYPs

 The idea of five-year plans is simple- The Government of India prepares a document with all
its income and expenditure for five years.
 The budget of the central government and all the state governments is divided into two
parts: Non-Plan Budget and Plan Budget.
 The non-plan budget is spent on routine items yearly. The planned budget is spent on
a five-year basis as per the priorities fixed by the plan.
 The model of the Indian Economy was premised on the concept of planning based on
five-year plans from 1951-2017.
 The Five Year Plans were formulated, implemented and regulated by a body known as
the Planning Commission.
 The Planning Commission was replaced by a think tank called NITI AAYOG in 2015.
 The Niti Aayog has come out with three documents — 3-year action agenda, 7-year
medium-term strategy paper and 15-year vision document.

Five Year Highlights

https://t.me/prelimbits
9

Plan

First Five-  The First Five Year Plan laid the thrust of economic development in India.
Year Plan  It was presented by the first Indian Prime Minister, Jawaharlal Nehru to
(1951-56) the Parliament of India.

 K.N Raj, a young economist, argued that India should "hasten slowly" for the
first two decades.

 It mainly addressed the agrarian sector, including investment in dams


and irrigation. Ex- Huge allocations were made for Bhakhra Nangal Dam.

 It was based on the Harrod Domar Model and emphasised increasing


savings.

 By the end of 1956, five Indian Institutes of Technology were established.

 The target growth rate was 2.1% and the achieved growth rate was 3.6%.

Second Five  The Second Five year Plan stressed rapid industrialisation and the
Year Plan public sector.
(1956-61)
 It was drafted and planned under the leadership of P.C Mahalanobis.

 It emphasised quick structural transformation.

 The government imposed tariffs on imports to protect domestic industries


under this plan.

 The target growth rate was 4.5% and the actual growth rate was slightly
less than expected, 4.27%.

Third Five  The focus was on agriculture and improvement in the production of
Year Plan wheat.
(1961-66)  States were entrusted with additional development responsibilities. Ex-
States were made responsible for secondary and higher education.

 Panchayat elections were introduced to bring democracy to the grassroots


level.

 The target growth rate was 5.6% and the actual growth rate only
achieved 2.4%
 This indicated a miserable failure of the Third Plan, and the government
had to declare "Plan Holidays" (1966-67, 1967-68, and 1968-69). The Sino-
Indian War and the Indo-Pak War, which caused the Third Five Year
Plan to fail, were the primary causes of the plan holidays.

https://t.me/prelimbits
10

Fourth  It was introduced under the Prime Ministership of Indira Gandhi and
Five-Year attempted to correct the previous failures.
Plan: (1969-
 Based on Gadgil Formula, a great deal of emphasis was laid on
74)
growth with stability and progress towards self-reliance.
 The government nationalised 14 major Indian Banks and the Green
Revolution boosted agriculture.

 The Drought Prone Area Programme was also launched.

 The target growth rate was 5.6%, but the actual growth rate was 3.3%.

Fifth Five-  It laid stress on increasing employment and poverty alleviation (garibi hatao).
Year Plan  In 1975, the Electricity Supply Act was amended, enabling the central
(1974-78) government to enter into power generation and transmission.

 The Indian National Highway System was introduced.

 The Minimum Needs Programme introduced in the first year of this plan,
aimed to provide basic minimum needs. MNP was prepared by D.P. Dhar.

 The target growth rate was 4.4% and the actual growth rate turned out to be
4.8%

 In 1978, the newly elected Morarji Desai government rejected this plan.

Rolling Plan (1978-80)

This was a period of instability. The Janata Party government rejected the fifth five-year Plan and
introduced a new Sixth Five-Year Plan. This, in turn, was rejected by the Indian National Congress
in 1980 upon Indira Gandhi's re-election.

A rolling plan is one in which the effectiveness of the plan is evaluated annually and a new
plan is created the following year based on this evaluation. As a result, throughout this plan, both
the allocation and the targets are updated.

Sixth Five  It underlined the beginning of economic liberation by eliminating price


Year Plan controls.
(1980-85)
 It was seen as the end of Nehruvian Socialism.

 To prevent overpopulation, family planning was introduced.

 On the recommendation of the Shivaraman Committee, the National Bank


for Agriculture and Rural Development was established.

 The target growth rate was 5.2% and the actual growth rate was 5.7%,
implying that it was a success.

https://t.me/prelimbits
11

Seventh  This plan was led by the Prime Ministership of Rajiv Gandhi.
Five Year
 It laid stress on improving Industrial productivity levels through the
Plan (1985-
90)
use of technology.
 Other objectives included increasing economic productivity, increasing the
production of food grains and generating employment by providing
Social Justice.

 The outcome of the Sixth Five-Year Plan provided a robust base for the
success of the seventh five-year plan.

 It emphasised anti-poverty programmes, the use of modern technology,


and the need to make India an independent economy.

 It focused on attaining prerequisites for self-sustained growth by 2000.

 The target growth rate was 5.0%. However, the actual growth rate grew to
reach 6.01%

Annual Plans (1990-92)

The Eight Five Year Plan was not introduced in 1990 and the following years 1990-91 and 1991-
92 were treated as Annual Plans. This was largely because of the economic instability.
India faced a crisis of foreign exchange reserves during this time. Liberalisation,
Privatisation, Globalisation (LPG) was introduced in India to grapple with the problem of
the economy under prime minister P.V Narasimha Rao.

Eighth Five  The Eighth Plan promoted the modernisation of Industries.


Year Plan  India became a member of the World Trade Organisation on 1 January
(1992-97)
1995.

 The goals were to control population growth, reduce poverty,


generate employment, strengthen the development of
infrastructure, manage tourism, focus on human resource
development etc.
 It also laid emphasis on involving the Panchayats and Nagar Palikas
through decentralisation.

 The target growth rate was 5.6% but the actual growth rate was an
incredible 6.8%.

Ninth Five  It marked India's fifty years since Independence and Atal Bihari Vajpayee
Year Plan led the prime ministership.
(1997-  It offered support for social spheres to achieve complete elimination of

https://t.me/prelimbits
12

2002) poverty and witnessed the joint efforts of public and private sectors in
guaranteeing economic development.

 The focus was also to balance the relationship between rapid


growth and the quality of life for the people.
 The objectives, further included, empowering socially disadvantaged
classes, developing self-reliance and primary education for all children
in the country.

 Strategies included enhancing the high rate of export to gain self-reliance,


efficient use of scarce resources for rapid growth etc.

 The target growth rate was estimated at 7.1% but its actual growth rate fell
shorter to 6.8%

Tenth Five  The features of this plan were to promote inclusive growth and
Year Plan equitable development.
(2002-07)
 It intended for an 8% GDP growth per year.

 It aimed at reducing the poverty by half and creating employment for


80million people. Further, it aimed to reduce regional inequalities.

 It also emphasised reducing the gender gaps in the field of education and
wage rates by 2007.

 The target growth rate was 8.1% while the actual growth was 7.6%.

Eleventh  The Eleventh Plan was significant in its aim to increase enrolment in
Five Year higher education and focused on distant education as well as IT
Plan (2007-
institutes. Ex: The Right to Education Act was introduced in
2012)
2009, and came into effect in 2010, making education free and
compulsory for children aged between 6-14 years.

 Its main theme was rapid and more inclusive growth.

 It is aimed at environmental sustainability and reduction in gender


inequality.

 C.Rangarajan prepared the Eleventh Five Year Plan.

 The focus was also laid on providing clean drinking water for all by 2009.

 The target rate was 9% and the actual growth rate was 8%.

https://t.me/prelimbits
13

Twelfth  The last Five Year Plan had "Faster, More Inclusive and
Five Year Sustainable Growth" as its theme.
Plan (2012-
17)  The plan aimed at strengthening infrastructure projects, and
providing electricity supply in all villages.
 It also aimed at removing the gender and social gap in admissions at school
and improved access to higher education.

 Further, it aspired to enhance the green cover by 1 million hectares each


year and to create new opportunities in the non-farming sector.

 The target growth rate was 9% but in 2012, National Development Council
approved a growth rate of 8% for this twelfth plan.

7. Which of the following are among the non-plan expenditures of the Government of
India? [1995, 1997]
1. Defence expenditure
2. Subsidies
3. All expenditures linked with the previous plan periods
4. Interest payment

Select the Correct option

A. 1 and 2
B. 1 and 3
C. 2 and 4
D. 1, 2, 3 and 4

Ans. D

 All expenditures which are done in the name of planning were called plan expenditures
while the rest are placed under non-plan expenditures.
 The government of India has now scrapped the plan and non-plan expenditures in
budget exercise and are replaced by capital and revenue spending classifications.

NON-PLAN EXPENDITURE IS SPREAD ACROSS


 Expenditure incurred on Defence Services
 Interest payments for debt
 Subsidies (food, fertilisers, etc.)
 Salaries and pensions of employees in various government services
 Economic services by the government such as Agriculture, Industry, Power, Science &
Technology,
 Loans and grants
 Social services such as healthcare, education, social security
 Police, etc.
https://t.me/prelimbits
14

8. What is the annual rate aimed in the Eighth Five Year Plan [1995]
A. 5.6%
B. 6%
C. 6.5%
D. 7%

Ans. A

Eighth Five  The Eighth Plan promoted the modernisation of Industries.


Year Plan  India became a member of the World Trade Organisation on 1 January
(1992-97) 1995.

 The goals were to control population growth, reduce poverty,


generate employment, strengthen the development of
infrastructure, manage tourism, focus on human resource
development etc.
 It also laid emphasis on involving the Panchayats and Nagar Palikas
through decentralisation.

 The target growth rate was 5.6% but the actual growth rate was an
incredible 6.8%.

9. The Exim Policy announced in 1992, is for period of : [1995]


A. 3 years
B. 4 years
C. 7 years
D. 5 years

Ans. D

In order to liberalize imports and boost exports, the Government of India for the first time
introduced the Indian Exim Policy on April I, 1992. In order to bring stability and continuity, the
Export Import Policy was made for the duration of 5 years.

EXIM POLICY OF INDIA


 The foreign trade policy (FTP) also known as EXIM (export-import) policy is regulated by
the Foreign Trade Development and Regulation Act, 1992.
 The main governing body in the matters concerning the EXIM policy is DGFT (Directorate
General of Foreign Trade).
 The main objective of this act is to provide the development and regulation of
foreign trade by facilitating imports into and augmenting exports from
India.
 Foreign Trade Act has replaced the earlier law known as the imports and Exports (Control)
https://t.me/prelimbits
15

Act 1947.

Objectives of EXIM Policy of India

 To promote persistent growth in exports to acquire a share of at least 1% of international


merchandise trade
 To encourage stable economic expansion by offering access to necessary capital goods, raw
materials, intermediate products, consumables, installations, and elements essential for
providing services and expanding production.
 To improve the technological potency and productivity of Indian agriculture, companies,
and services, thus enhancing their competitive power while creating fresh employment
possibilities, and to stimulate the accomplishment of globally acknowledged norms of
quality.
 To supply consumers with fine condition goods and services at globally competitive rates
while simultaneously generating a level playing range for domestic production.

EXIM Bank of India

 Export-Import Bank of India (Exim Bank), set up under an Act of Parliament, is the apex
financial institution engaged in financing, facilitating and promoting
India‘s International trade and investment.
 Established in 1982, Export-Import Bank of India is a wholly owned Govt. of India
entity.
 HQ : New Delhi
 Functions of Exim Bank
o The bank offers immediate monetary help to exporters of the plant, equipment,
and corresponding services by means of medium-term credit.
o To guarantee the issue of stocks, bonds, shares, and debentures of any
organization involved in exports.
o The bank put forward rediscount of export bills for a duration no longer than 90 days
against short period usage export invoices depreciated by commercial banks.
o It also provides foreign buyers a credit to overseas importers for import of Indian
industrial products and concerning services.
o To create and fund export-oriented enterprises.
o To accumulate and assemble the market and credit particulars about foreign trade.

NEW FOREIGN TRADE POLICY (FTP)


 The current trade policy was introduced in 2015 for a five-year term.

https://t.me/prelimbits
16

 The current FTP (from 2015-20 but extended thereafter till March 2023) is a policy
document that spells out the objectives and strategies to boost exports.
 The new policy will be a comprehensive and important document that interprets rules,
regulations, and procedures in international trade transactions, which are critical in
facilitating in export-import operations and making the export sector more competitive.
 India‘s strategy is to cash in on a world seeking to become less dependent on China and
to enable exporters (and importers) to plan their investments ahead.
 The new policy will aim to provide a leg-up to exports and address some of industry‘s
key concerns, including a buffer against rising interest rates. It could include measures
to help push up goods and services exports as well as rein in the runaway import bill.
 There is a need for new policy due to global growth slowdown and recession fears after the
Ukraine-Russia war.
 The Indo-Pacific Economic Framework has led to assertions that the Government has ‗no
bandwidth‘ left for new free trade pact negotiations through more countries

10. Agricultural income tax is assigned to the State Government by: [1995]
A. Finance Commission
B. National Development Council
C. Inter-State Council
D. The Constitution of India
Ans. D
LEGAL POSITION OF TAX ON AGRICULTURE
 In the Seventh Schedule, Entry 82 in the Union List mentions taxes other than
agricultural income
 WHILE Entry 46 in the State List mentions taxes on agricultural income. Therefore, it is
in the State List.
 Section 2 (1A) of the Income Tax Act defines agricultural income as rent/revenue from
land, income derived from this land through agriculture and income derived from

https://t.me/prelimbits
17

buildings on that land.


 Section 10 (1) of the Income Tax Act excludes agricultural income from a
computation of total income
 Agricultural income was taxed till 1886.
 Past attempt to bring Agri income under income Tax Net →
o In 1925, a committee was set up to assess the feasibility of taxing agriculture income
o KN Raj committee: The most famous attempt in post-Independence India was the
K.N. Raj committee report of 1972, which also examined feasibility and
implementation issues
o Kelkar task force: The Kelkar task force report of 2002 estimated that 95% of the
farmers were below the tax threshold
11. In which one of the following crops international trade is low in the context of
total produce? [1995]
A. Rice
B. Coffee
C. Rubber
D. Wheat
Ans. A
Rice production is highest in India among cereals but its quality of rice cannot compete with other
rice producing countries. Most of rice produced in India is meant for internal consumption.
India is a major producer and exporter of Basmati rice.

Rice Export (Current Affairs)

 India decided on Dec 2022 to lift the ban on exports of organic non-basmati rice,
including broken rice after the easing of domestic supplies moderated prices.
 The government had, in early September, banned the export of broken rice with an aim to
increase domestic availability.
BASMATI RICE
 Basmati, is a variety of long, slender-grained aromatic rice which is traditionally grown in
India, Pakistan, and Nepal.
 India is known for its Basmati rice, with seven States — Jammu and Kashmir, Himachal
Pradesh, Punjab, Haryana, Delhi, Uttar Pradesh and Uttarakhand — earmarked for
geographical indication.
 The largest area under Basmati rice is in the state of Haryana (60%) followed by Uttar
Pradesh (17.1%) and Punjab (16.1%).
 According to the Indian Government agency Agricultural and Processed Food Products
Export Development Authority (APEDA), a rice variety is eligible to be called basmati if it
has a minimum average precooked milled rice length of 6.61 mm (0.260 in) and
average precooked milled rice breadth of up to 2 mm (0.079 in), among other
parameters.

https://t.me/prelimbits
18

 Exports →
o Basmati, has a market abroad and brings about ₹30,000 crore foreign exchange
every year.
o In 2019, India accounted for 65% of the international trade in basmati rice, while
Pakistan accounted for the remaining 35%.
o While 75% of the export is to West Asian countries, European Union countries
also import Indian Basmati.
 Recently Indian Agriculture Research Institute (IARI) developed five varieties of Basmati
Rice seeds in 2020 and 2021. These paddy seeds can resist diseases and herbicides.
Broken Rice

 It is often used in the manufacture of feed for very young animals and for pets. Further,
it is used for all types of livestock and is particularly suitable due to its rich caloric value
and low fibre content.
 It is also used in the brewing industry, where it is mixed with barley and the production
of arak (aniseed alcoholic drink, distilled, colourless drink).
 It is a raw material for rice flour, used in baby food, breakfast cereals, rice wine, rice liqueur,
sake, and prepackaged and canned foods.

Parboiled rice

 The expression parboiled rice refers to rice that has been partially boiled at the paddy
stage, before milling.
 Parboiling of rice is not a new practice, and has been followed in India since ancient times.
However, there is no specific definition of parboiled rice of the Food Corporation of India or
the Food Ministry.
 All processes generally involve three stages—soaking, steaming and drying. After passing
through these stages, the paddy goes for milling.
 Today, there are several processes for parboiling rice.
 In contrast to the more popular method, which calls for the paddy to be soaked for eight
hours, the Central Food Technological Research Institute (CFTRI), Mysuru, uses a method
that calls for the paddy to be soaked in hot water for just three hours. After draining the
water, the paddy is steam-cooked for 20 minutes.
 Also, the paddy is dried in the shade in the method used by the CFTRI, but is sun-dried in

https://t.me/prelimbits
19

the common method.


 The chromate soaking technique is a procedure used at the Paddy Processing Research
Centre (PPRC), Thanjavur. To get rid of the smell from the wet rice, it employs chromate, a
class of salt in which the anion contains both chromium and oxygen.
 Advantages and disadvantages:
o Parboiling makes rice tougher which reduces the chances of the rice kernel
breaking during milling.
o Parboiling also increases the nutrient value of the rice.
o Parboiled rice has a higher resistance to insects and fungi.
o However, parboiling makes rice darker and may smell unpleasant due to
prolonged soaking. Besides, setting up a parboiling rice milling unit requires a
higher investment than a raw rice milling unit.
Kalanamak Rice

https://www.thehindu.com/sci-tech/agriculture/fragrant-and-nutritious-kalanamak-rice-buddhas-
gift-to-people-gets-new-powers-and-name/article66069818.ece

Recently, Indian Agriculture Research Institute (IARI) successfully tested two new dwarf
varieties of Kalanamak rice i.e., Pusa Narendra Kalanamak 1638 and Pusa Narendra
Kalanamak 1652 in Uttar Pradesh that give double the yield.

 This will address the problem of lodging responsible for low yield seen across the
traditional variety.

 Kalanamak is a traditional variety of paddy with a black husk and a strong fragrance.
 It is considered a ‗gift from Lord Buddha‘ to the people of the Sravasti when he visited the
region after enlightenment.
 It has been awarded under 'One District One Product' (ODOP) scheme as an ODOP
product of Siddharthnagar which is an aspirational district located in Terai belt of Uttar
Pradesh.
 It is grown in 11 districts of the Terai region of northeastern Uttar Pradesh and
in Nepal.
 It is protected under the Geographical Indication (GI) tag system.
Farmer's Benefits from Kalanamak Rice:

 Natural Cultivation: Kala Namak rice is grown mainly without using fertiliser or
pesticide residues, making it perfect for crop production.
 Cost-effective factor: Since pesticides and fertilisers are not used, the expense is lowered
and the grower saves a lot of money.

https://t.me/prelimbits
20

Health Benefits of Kalanamak Rice:

 Kala Namak rice acts as an antioxidant like anthocyanin, which assists in the prevention of
cardiovascular disease and the advancement of skincare.
 Kalanamak rice includes a lot of micronutrients like zinc and iron. As a result, eating this
rice is also said to protect against illness caused by zinc and iron deficits.
 It is claimed that eating Kalanamak rice on a regular basis can help prevent Alzheimer's
disease.
 Kala Namak rice can also help strengthen, regrow, and galvanise the body, as well as
help regulate blood pressure, diabetes, and skin damage.
Issue with the Traditional Variety:

 The problem with the traditional variety of Kalanamak paddy is that it‘s tall and prone to
lodging, which badly impacted grain filling and quality.
 Lodging is a condition in which the top of the plant becomes heavy because of grain
formation, the stem becomes weak, and the plant falls on the ground.
COFFEE

https://www.thehindu.com/todays-paper/tp-opinion/a-crisis-is-brewing-in-the-coffee-
industry/article66042581.ece

 Coffee was introduced to India during the late seventeenth century.


 The story goes that an Indian pilgrim to Mecca smuggled seven beans back to India from
Yemen in 1670 (it was illegal to take coffee seeds out of Arabia at the time) and planted
them in the Chandragiri hills of Karnataka.
 The Dutch (who occupied much of India throughout the 17th century) helped spread the
cultivation of coffee across the country, but it was with the arrival of the British Raj in
the mid-nineteenth century that commercial coffee farming fully flourished.
Coffee in India

 Coffee in India is grown under a canopy of thick natural shade in ecologically sensitive
regions of the Western and Eastern Ghats.
o This is one of the 25 biodiversity hotspots of the world.
 Coffee is predominantly an export oriented commodity and 65% to 70% of coffee
produced in the country is exported while the rest is consumed within the country.
 Coffee contributes significantly to sustain the unique bio- diversity of the region and is
also responsible for the socio-economic development in the remote, hilly areas.
Climatic Conditions Required

 Coffee plants require a hot and humid climate with temperatures ranging between 15°C
and 28 °C and rainfall from 150 to 250 cm.
 Frost, snowfall, high temperature above 30°C and strong sun shine is not good for coffee

https://t.me/prelimbits
21

crops and is generally grown under shady trees.


 Dry weather is necessary at the time of ripening of the berries.
 Stagnant water is harmful and the crop is grown on hill slopes at elevations from 600 to
1,600 metres above sea level.
 Well drained, loams containing a good deal of humus and minerals like iron and calcium
are ideal for coffee cultivation.
Soil Types for Coffee Production:

 Coffee can be grown on lots of soils but the ideal types are fertile volcanic red earth or
deep sandy loam.
 For coffee trees to grow it is important that the soil is well draining which makes heavy clay
or heavy sandy soils inadequate.
Major Areas:

 In India, coffee is traditionally grown in the Western Ghats spread over Karnataka,
Kerala and Tamil Nadu.
 Karnataka is the largest producer accounting for about 70% of the total coffee
production.
o Coffee heartland of Karnataka, comprising the Kodagu, Chikkamagaluru and
Hassan districts,
 Coffee cultivation is also expanding rapidly in the nontraditional areas of Andhra Pradesh
and Odisha as well as in the North East tates.

Main Varieties : Arabica and Robusta are grown in India

 Arabica is mild coffee, but the beans being more aromatic, it has higher market value
compared to Robusta beans.
o On the other hand Robusta has more strength and is, therefore, used in making
various blends.
 Arabica is grown in higher altitudes than Robusta.

https://t.me/prelimbits
22

 Arabica requires more care & nurture and is more suitable for large holdings whereas
Robusta is suitable irrespective of size of the farm.
 Arabica is susceptible to pests & diseases such as White Stem Borer, leaf rust etc., and
requires more shade than Robusta.
 The harvest of Arabica takes place between November to January, while for Robusta it is
December to February.
Indian Coffee in Global Market:

 In the coffee market, India‘s annual coffee output is rising to allow the nation to rank sixth
among the world‘s major coffee producers. India exported up to 80% of its coffee bean
production, accounting for 4% of the world coffee market.
 India is the world‘s sixth-largest producer and fifth-largest exporter of coffee.
 It is the third-largest coffee producer in Asia.
Coffee Board of India

 Coffee Board of India was established under the Coffee Act of 1942 in 1942 and it is
headquartered in Bangalore.
 The organization is managed by the Ministry of Commerce and Industry to promote
coffee production in India.
 Coffee Boards traditional duties include
o Promotion of the sale and consumption of coffee in India and abroad,
o Conducting coffee research,
o Financial assistance to establish small coffee growers,
o Safeguarding working conditions for labourers,
o Managing the surplus pool of unsold coffee.
Top five producers: Brazil, Vietnam, Colombia, Indonesia, and Ethiopia. India is 7th

12. Which one of the following is not an instrument of selective credit control in India?
[1995]
A. Regulation of consumer credit
B. Rationing of credit
C. Margin requirements
D. Variable cash reserve ratios

Ans. D

Variable Reserve Ratio (Cash Reserve Ratio) is aimed to control only volume of credit (quantitative
method) not both volume and purpose of credit for which bank gives loans. (Qualitative method
and selective control method are used for these purposes. It has a number of limitations.

MONETARY POLICY - QUANTITATIVE TOOLS


 Also known as (aka) General or Indirect Tools as they affect the entire economy, and not just
a particular sector.
CRR, SLR (Fight inflation: ↑, deflation: ↓)

https://t.me/prelimbits
23

 CRR and SLR are collectively known as ―Variable Reserve Ratios‖ or ―Statutory Reserve
Ratios‖
CRR SLR
Banks must keep this much deposits (or Banks must keep with themselves this
balance) with RBI. [only in cash] deposits in liquid assets such as cash, gold,
RBI doesn‘t pay interest on this deposit, except in G-Sec, T-Bills, State Development Loan Bonds
extraordinary circumstances like 1999‘s Banking and other securities notified by RBI.
slowdown.
Bank earns no profit / interest, as such. Some profit may be involved.
CRR: first suggested by the British economist J.M. Mandated under Banking Regulation Act,
Keynes & first introduced in US Federal Reserves 1949
(=Central Bank of USA).
Mandated under RBI Act, 1934

RBI can fix any amount of CRR, legally there is Legally, SLR can‘t be more than 40%.
no minimum floor or maximum ceiling. Presently it‘s 18.00% of NDTL.
Presently it‘s 4.50% of Net Demand & Time
Liabilities (NDTL) of a bank
All Scheduled Commercial Banks (SCB) must Similar
keep CRR.
However, RBI may prescribe separate %
norms/slabs for Regional Rural Banks (RRBs) and
Cooperative Banks.

Role of SLR & CRR


 CRR-SLR are counted on fortnightly basis. If not maintained, bank will have to pay penalty
interest rate to RBI. Penalty rate is linked with Bank Rate.
 CRR-SLR ensure monetary stability of India through two primary functions:
o CRR assists in money multiplier effect,
o CRR-SLR provide buffer/protection during a Bank Run
 While in theory CRR/SLR can be used for inflation control but RBI primarily relies on REPO
Rate (=its Policy Rate) to combat inflation, and not CRR/SLR.
Bank rate
 Introduced under RBI act 34
 Bank rate is a rate at which the Reserve Bank of India (RBI) provides the loan to
commercial banks without keeping any security. [For Longer Period]
 There is no agreement on repurchase that will be drawn up or agreed upon with no
collateral as well.
 The RBI allows short-term loans with the presence of collateral. This is known as Repo Rate.
 Bank Rates in India is determined by the RBI. It is usually higher than a Repo Rate on
account of its ability to regulate liquidity.

https://t.me/prelimbits
24

LAF REPO (2000)

RBI‘s Liquidity Adjustment Facility (LAF): has two windows:


Repo Rate [RR] Reverse Repo Rate [RRR]
The Interest rate at which RBI lends short-term It’s the interest rate that clients [Banks] earn when
loans to its clients [Banks or govt.], keeping their parking their surplus funds with the RBI for short
periods, to earn interest. [or when RBI buy money
G-Sec as collaterals.
form Commercial Banks]
Clients enter into an agreement with RBI to Mechanism similar to Repo, RBI gives its G-Sec as
repurchase their G-sec at a future date at a collateral.
(higher) pre-determined price.
Banks can’t pledge their SLR-quota-G-Secs
for this borrowing.
Repo Rate is RBI’s Policy Rate to control inflation. RR is higher than RRR
Inflation - Increase If RBI decreases the Reverse Repo rate, it means RBI
Deflation - Decrease wants expansion of credit.
With this fewer banks will supply their surplus to
central bank because earning is less.
This will affect the financial market as supply of
money in financial market will increase.
Due to increase in the supply of credit in the
market, inflation rate will increase.
Market Repo
 Players other than RBI (such as bank, NBFCs) loaning short money to other
Banks/NBFCs/Corporate Companies, and demanding financial securities (G-Sec/T-
Bill/shares/bonds/commercial paper etc.) as collateral.
What can be the possible results when RBI reduce Reverse Repo Rate
Long Term Repo Operations (LTROs)
 Usually, Repo loans are for short term borrowing from overnight to 14-days.
 But, 2020-Feb: RBI announced to conduct Long Term Repo Operations (LTROs) of 1 year and
3 years tenors.
 RBI will loan total ₹ 1,00,000 crore, in various rounds through E-Kuber platform.
 RBI‘s clients can apply to borrow a minimum ₹1 crore or higher.
 Interest rate: prevailing repo rate. Interest rate will be compounded annually.
 This will increase loanable funds with banks → economic growth can be revived.
As banks get long-term funds at lower rates, their cost of funds falls. In turn, they reduce interest
rates for borrowers. LTRO helped RBI ensure that banks reduce their marginal cost of funds-based
lending rate, without reducing policy rates.
 MSF and (short term) repo lending will also be continued separately as per their own existing
norms. LTRO doesn‘t aim to eliminate / replace them.
MSF (2011)

 Marginal Standing Facility (MSF) is the Interest rate at which RBI lends short-term

https://t.me/prelimbits
25

loans to Scheduled Commercial Banks (SCB) with their SLR-quota G-Sec as collaterals.
 MSF higher than higher than Repo Rate. MSF = Repo% + 0.25%.
 Under MSF, banks can borrow funds up to one percent of their net demand and time
liabilities (NDTL).

 Banks can borrow through MSF on all working days except Saturdays.
 The minimum amount for which RBI receives application is Rs.1 Crore, and afterward in
multiples of Rs.1 Crore.
Policy Corridor
 Policy Corridor: It‘s the width among MSF-Repo-Reverse Repo.
 Window Operations: LAF-MSF ―windows‖ are operated through RBI‘s E-Kuber Core Banking
Solution (CBS) platform.
Market Operations (OMO, MSS): (Inflation → Sell G-Sec, Deflation → Buy)
 OPEN MARKET OPERATIONS: RBI buys and sells Union & State Govts‘ securities to control
money supply.
o RBI BUYING= Money SUPPLY INCREASED/liquidity injected in the market.
o RBI SELLING = Money SUPPLY DECREASED/liquidity absorbed from the market.
 MARKET STABILIZATION SCHEME: RBI sells special type of G-sec, T-Bill & Cash
Management Bills (CMB) to suck excess liquidity. While the money thus collected is not part
of Govt.‘s borrowing, but Govt. pays interest on it. This mechanism was enhanced during
Demonetization to counter excess liquidity and crashing of lending rates. But not really
important topic SO DONOT LOOSE SLEEP
 STERILIZATION / FOREX SWAP: Their primary objective is to control the currency
exchange rate volatility.
Operation Twist
 ‗Operation Twist‘ is RBI‘s simultaneous selling of short-term securities and buying of long
term securities through open market operations (OMO).
 Under this mechanism, the short-term securities are transitioned into long-term securities.
How does RBI manage ‘Operation Twist’?
 This operation involves buying and selling government securities simultaneously in order to
bring down long-term interest rates and bolster short-term rates.
 There is an inverse relationship between the bond prices and their yields. As the central
bank buys long-term securities (bonds), their demand rise which in turn pushes up their
prices. [Bond Yield is the effective rate of return that a bond earns]
o However, the bond yield comes down with an increase in prices. Yield is the return an
investor gets on his (bond) holding/investment.
 The interest rate in an economy is determined by yield. Thus, lower long-term interest
rates mean people can avail long-term loans (such as buying houses, cars or financing
projects) at lower rates.

https://t.me/prelimbits
26

 This also results in a dip in the expected returns from long-term savings which tilts the
balance from saving towards spending. Hence, cheaper retail loans can help encourage
consumption spending which is the largest GDP component in the economy.
How does it affect investors?
 Fixed income investors with higher exposure to long term debt will benefit from easing yield
of long-term bonds.
 Consumers/borrowers will also profit from ‗Operation Twist‘ as the retail loans will now get
cheaper.
 Previously banks were forced to price their retail loans at higher rates owing to high yields on
long-term government borrowings. Cheaper retail loans mean a boost in consumption and
spending in the economy which in turn will revive growth.
Bond & Yield Inversion

 The Bond Yield is the EFFECTIVE RATE OF RETURN that a bond earns. But the rate of
return is not fixed as it changes with the price of the bond.
 Suppose a Rs. 100 bond (this is called issue price or Face Value) is issued @ interest rate
10% (which means that interest in the market is around 10% ).
 Now if the interest rate in the market decreases (the interest rate is decreasing in the
market but the bond which has been issued @10% interest rate → This interest rate is fixed
and it will never change) then the new bonds which will be issued at less interest rate
(say 8%).
 Now if you want to purchase a previously issued bond of Rs. 100 face value→ the
holder of the bond will not give you in Rs. 100, rather he will ask for more Rupees → i.e.
the price of the previously issued bond will increase and suppose you purchase this bond in
Rs. 125 then for you annual RETURN will be = (Rs. 10/Rs. 125)*100 = 8% . This 8% is yield.
 So if bond Price goes up Yield will fall [opposite if price of bond fall down than Yield will
rise]
Yield inversion
 Yield inversion happens when the yield on a longer tenure bond becomes less than the
yield for a shorter tenure bond
 A yield inversion typically portends a recession. An inverted yield curve shows that
investors expect the future growth to fall sharply; in other words, the demand for money
would be much lower than what it is today and hence the yields are also lower.
Soft-landing
 The process of monetary tightening that the US Federal Reserve is currently unveiling
involves not just reducing the money supply but also increasing the cost of
money (that is, the interest rate).
 The US‘ Federal Reserve is doing this to contain soaring inflation.
 When a central bank is successful in slowing down the economy without bringing about
a recession, it is called a soft-landing — that is, no one gets hurt.
 But, when the actions of the central bank bring about a recession, it is called a hard-

https://t.me/prelimbits
27

landing.
Reverse Currency Wars
 A flip side of the US Federal Reserve action of aggressively raising interest rates is that
more and more investors are rushing to invest money in the US.
 This, in turn, has made the dollar become stronger than all the other currencies as the
dollar is more in demand than yen, euro, yuan etc.
 A relative weakness of the local currency of other countries against the dollar makes
their exports more competitive.
 For instance, a Chinese or an Indian exporter gets a massive boost.

 In the past, the US has accused other countries of manipulating their currency (and
keeping its weaker against the dollar) just to enjoy a Trade Surplus against the US.
 This used to be called the currency war.

MONETARY POLICY: QUALITATIVE TOOLS [SELECTIVE CREDIT CONTROL]


 Quantitative tools (SLR, CRR, Repo etc.) control the ‗volume‘ of loans. Whereas, qualitative
tools (PSL,LTV etc.) control the ―distribution‖ of loans to a particular sector of economy
(e.g. agriculture) or particular segment of society (e.g. farmers, women, SC/ST).
 Hence, qualitative tools also known as SELECTIVE or DIRECT Tools.
Moral suasion

 Moral suasion meaning applying ―Persuasion‖ without applying punitive measures. RBI
governor tries this tactic via conferences, informal meetings, letters, seminars etc
 Example, RBI-Governor asking banks to transmit repo-rate cuts, open new branches in rural
areas, spread financial literacy, give loans to farmers beyond PSL quota etc. Similarly, RBI
Governor requesting CM or Finance Minister to control fiscal deficit & subsidy leakage to
enhance the efficacy of RBI‘s monetary policy.
Publicity

 Publicity: RBI governor could give media statement, speech during university convocation.
 By doing so, he can create an effective public opinion which also pressurizes the banks to
stop their thuggery.
Direct Action

 RBI can punish banks (and even non-banks) for not complying with its directives under
RBI Act, Banking Regulation Act, Payment and Settlement Systems Act, Prevention of Money
Laundering Act (PMLA), Foreign Exchange Management Act (FEMA).
 2019: RBI ordered the banks to have a ―Clawback‖ provision in their CEO & Top executives‘
salaries. E.g. If the CEO did any scam/fraud, he‘ll have to return his previously paid salary /
bonus, even if he had retired/left the job afterwards.
Margin Requirements / Loan to Value (LTV)

 RBI can mandate Loan to Value (LTV) for a gold-loan, home loan, auto loan or business loan
etc. so a Bank/NBFC can‘t lend more than x% of the value of the collaterals.
https://t.me/prelimbits
28

 RBI can change this x% to boost / curb demand.


Regulation of Consumer Credit

 Consumer credit supply is regulated by the instalment of sale and hire purchase of
consumer goods.
 Features such as instalment amount, down payment, loan period, and so on are all pre-
determined, which aids in the control of credit and inflation in the country.
 For Instance, for a home loan, the RBI can set a minimum downpayment limit of 15%.
Therefore for a home loan of 1 crore, Rs. 15 lakhs must be paid as a downpayment and avail
85 lakhs as a loan.

Rationing of Credit

 The Reserve Bank of India sets a credit limit for commercial banks. The quantity of credit
accessible to any commercial bank is limited.
 The higher credit limit might be set for certain objectives, and banks must adhere to it.
 This reduces the bank's credit exposure to unfavourable industries. This device also
regulates bill rediscounting.
 For Instance, the banks might be instructed by the RBI not to lend to traders of Onion and
Potato in spite of having eligibility and collateral pledging capacity. This is to ensure there is
no hoarding of essential commodities by using bank loans.

13. Bank Rate implies the rate of interest: [1995]


A. paid by the Reserve Bank of India on the Deposits of Commercial Banks
B. charged by Banks on loans and advances
C. payable on Bonds
D. at which the Reserve Bank of India discounts the Bills of Exchange

Ans. D

BANK RATE
 Introduced under RBI act 34
 Bank rate is a rate at which the Reserve Bank of India (RBI) provides the loan to
commercial banks without keeping any security. [For Longer Period]
 There is no agreement on repurchase that will be drawn up or agreed upon with no
collateral as well.
 The RBI allows short-term loans with the presence of collateral. This is known as Repo Rate.
 Bank Rates in India is determined by the RBI. It is usually higher than a Repo Rate on
account of its ability to regulate liquidity.
14. Consider the following: [1995]
1. Industrial Finance Corporation of India
2. Industrial Credit and Investment Corporation of India
3. Industrial Development Bank of India
4. Unit Trust of India

https://t.me/prelimbits
29

The correct sequence in which the above were established is:

A. 1, 2, 4, 3
B. 1, 3, 2, 4
C. 4, 3, 2, 1
D. 1, 4, 3, 2

Ans. A

IFCI – July 1948; ICICI – 1955; IDBI – July 1964; UTI – 1963

IFCI
 IFCI, previously Industrial Finance Corporation of India, is a development finance
institution under the ownership of Ministry of Finance.
 Established in 1948 as a statutory corporation, IFCI is currently a company listed on BSE
and NSE. IFCI has seven subsidiaries and one associate.
 It provides financial support for the diversified growth of Industries across the
spectrum.
 The financing activities cover various kinds of projects such as airports, roads, telecom,
power, real estate, manufacturing, services sector and such other allied industries.
 During its 70 years of existence, mega-projects like Adani Mundra Ports, GMR Goa
International Airport, Salasar Highways, NRSS Transmission, Raichur Power Corporation,
among others, were set up with the financial assistance of IFCI.

INDUSTRIAL CREDIT AND INVESTMENT CORPORATION OF INDIA (ICICI)


 ICICI was a government institution established on 5 January 1955 and Sir Arcot
Ramasamy Mudaliar was elected as the first Chairman of ICICI Ltd.
 It was structured as a joint-venture of the World Bank, India's public-sector banks and
public-sector insurance companies to provide project financing to Indian
industry.
 ICICI Bank was established by ICICI, as a wholly owned subsidiary in 1994 in Vadodara.
The bank was founded as the Industrial Credit and Investment Corporation of India
Bank, before it changed its name to ICICI Bank. The parent company was later merged
with the bank.

INDUSTRIAL DEVELOPMENT BANK OF INDIA


 IDBI (Industrial Development Bank of India) Bank was established in 1964 by an Act of
Parliament as a wholly owned subsidiary of the Reserve Bank of India (RBI).
It was established as a specialised development financial institution (DFI) – on the lines
of NABARD, NHB and SIDBI – to provide credit and other financial facilities for the
development of Indian industry.
 In 1976, the ownership of IDBI was transferred to the Government of India.
 In 2004 it was converted into a commercial bank when RBI incorporated IDBI as a
'scheduled bank' under the RBI Act, 1934
https://t.me/prelimbits
30

 Operates Credit Guarantee fund, Small Enterprises Development Fund (SEDF).


 Operates udyamimitra.in for loans to small entrepreneurs & SME via schemes like Mudra,
Stand-up-India.

UTI MUTUAL FUND


 UTI Mutual Fund was carved out of the erstwhile Unit Trust of India (UTI) as a Securities
and Exchange Board of India (SEBI) registered mutual fund from 1 February 2003.
 The Unit Trust of India Act 1963 was repealed, paving way for the bifurcation of UTI
into: Specified Undertaking of Unit Trust of India (SUUTI) and UTI Mutual Fund
(UTIMF).
 UTI Mutual Fund is the oldest and one of the largest mutual funds in India with over 10
million investor accounts under its 230 domestic schemes/plans as of September 2017.
 UTI Mutual Fund has a nationwide distribution network, which is spread across the length
and breadth of the country

15. As part of the liberalisation programme and with a view to attract foreign exchange,
the government and the RBI have, devised two scheme known as FCNR „A‟ and FCNR
„B‟. Which of the following is/are true regarding these two schemes? [1995]
1. Under scheme ‗A‘ RBI bears exchange rate fluctuations.
2. Under scheme ‗B‘ other banks are to meet out the difference in exchange rate fluctuations.
3. Both the schemes stand withdrawn now.
4. Only scheme ‗A‘ has been withdrawn

Option

A. 3 only
B. 1 and 2
C. 1, 2 and 3
D. 1, 2 and 4

Ans. D

Under the FCNR (A) Scheme, the RBI bore any exchange rate risk, while in the case of FCNR (B)
Scheme banks have to bear the exchange rate risk. The FCNR (A) Scheme was replaced by the
FCNR (B) Scheme in 1994.

FCNR (B)
 FCNR (B) accounts. Any Non-Resident Indian – NRI and PIO cardholder – can open an
FCNR (B) account.
 It is a term deposit with a stipulated maturity date and can be held in a foreign currency.
 The main benefit of an FCNR (B) account is that the principal amount and interest
amount are tax-free and fully repatriable.
 Since the account is denominated in a foreign currency, you are not exposed to the risk of
exchange rate fluctuations. Banks offer competitive interest rates on these deposits to NRIs.

https://t.me/prelimbits
31

16. Which one of the following is correct regarding stabilization and structural adjustment
as two components of the new economic policy adopted in India? [1996]
1. Stabilization is a gradual, multi-step process while structural adjustment is a quick
adaptation process
2. Structural adjustment is a gradual multi-step process, while stabilization is a quick
adaptation process
3. Stabilization and structural adjustment are very similar and complimentary policies. It is
difficult to separate one from the other
4. Stabilization mainly deals with a set of policies which are to be implemented by the Central
government while structural adjacent is to be set it motion by the State governments

Ans. A

New Economic Policy was adopted in 1991 based on Rao-Manmohan model. Stabilization
component of any economy is essentially a short-term programme while the structural
readjustment component is a long term process.

NEW ECONOMIC POLICY (NEP) 1991


 The New Economic Policy (NEP) of India was launched in the year 1991 under the
leadership of P. V. Narasimha Rao & Finance Minister was Manmohan Singh
 New economic policy was undertaken in view of the 1991 financial crisis that arose due to
reasons like the gulf war that pushed up oil prices and lower remittances from the gulf,
foreign reserves at an all-time low, hyperinflation occurring at the same time
 This was in line with the International Monetary Funds (IMF) regulations to lend to
India. The credibility of the country's economy was decreasing, with no country willing
to lend loans. This period also saw a decrease in the foreign exchange reserves of the
country. This is also known as the LPG Model of growth.
 It was a set of policy measures that emphasized LIBERALIZATION, PRIVATIZATION, and
its outcome was GLOBALIZATION.
 It included various policy measures such as stabilization measures (to control inflation
and the correct balance of payments) and various structural reform measures (to
improve the efficiency of the economy and increase international competitiveness by
removing rigidity in various economic segments).

Branches of New Economic Policy

 Liberalization
o All commercial Banks become free to determine the rate of interest in the banking
system without the influence of RBI.
o Increasing investment limit for small-scale industries to upgrade their machinery
and improve their efficiency.
o Indian industries would have the freedom to import capital goods such as to buy
machines and raw materials from foreign nations for their development.

https://t.me/prelimbits
32

o Industries have freedom towards diverse production capacities and reduction in


the cost of production.
o Doing away with restrictive trade practices, for instance replacing the Monopolies
and Restrictive Trade Practices Act 1969 with Competition Act, 2002.
 Privatization
o Selling shares of Public Sector Undertakings (PSUs) to public and financial
institutions.
o Disinvestment in PSUs incurring losses by selling out these industries to the private
sector.
o Minimizing the role of the public sector with a view to help in industrialization and
economic growth. Industries reserved for the public sector were reduced from 17 to
 Globalization
o Reduction in custom duties and tariffs imposed on imports and exports to make
the Indian economy attractive to international investors.
o Enforce long-term foreign trade policy involving open competition, removing
restrictions on foreign trade, etc.
o Partial convertibility of the Indian rupee to increase the inflow of foreign
investment through Foreign Institutional Investment (FII) and Foreign Direct
Investment (FDI).
o Increasing the equity limit for foreign investment from 40% to 100% percent, for
which the foreign exchange management act was enforced.

17. One of the important goals of the economic liberalisation policy is to achieve full
convertibility of the Indian rupee. This is being advocated because: [1996]
A. convertibility of the rupee will stabilize its exchange value against major currencies of the
world
B. it will attract more foreign capital inflow in India
C. it will help to promote exports
D. it will help India secure loans from the world financial markets at attractive terms

Ans. B

 The full convertibility of the Indian currency means that the rupee is freely exchangeable
into other international currencies and vice versa. Also, this would mean that
international investors can buy and sell Indian assets at will.
 After 1994, the rupee has been partially convertible.

INTERNATIONALIZATION OF RUPEE
https://indianexpress.com/article/business/banking-and-finance/internationalisation-of-rupee-has-
risks-but-they-are-unavoidable-rbi-deputy-governor-8224133/

 Internationalisation of the rupee is a process that involves increasing use of the local
currency in cross-border transactions.

https://t.me/prelimbits
33

 It involves promoting the rupee for import and export trade and then other current
account transactions followed by its use in capital account transactions.

 As far as the rupee is concerned, it is →

o Fully convertible in the current account.

 The full convertibility of the Indian currency means that the rupee is freely
exchangeable into other international currencies and vice versa.

o But Partially in capital account.

 Current and Capital account are the two components of Balance of


payments.

 While the current account deals mainly with the import and export of goods
and services, the capital account is made up of the cross-border movement of
capital by way of investments and loans.

Need for Internationalisation of Rupee

 The dollar accounts for 88.3% of global foreign exchange market turnover, followed by
the euro, Japanese Yen and Pound Sterling; the rupee accounts for a mere 1.7%,
underlining the need for pushing the currency much farther to get an international tag.

 In the case of the dollar, which is an international currency, the ‗exorbitant‘ privileges
include immunity from Balance of Payments crises as the USA can pay for its external
deficits with its own currency.

Advantages of Internationalisation of the Rupee

 Use of Rupee in cross-border transactions mitigates currency risk for Indian business.

o Protection from currency volatility

o Reduces cost of doing business

o Enables better growth of business & foreign investments

o Improve the chances for Indian business to grow globally

 It reduces the need for holding foreign exchange reserves.

o While reserves help manage exchange rate volatility and project external stability,
they impose a cost on the economy.

 Reducing dependence on foreign currency makes India less vulnerable to external shocks.

o For example, during phases of monetary tightening in US and strengthening


dollar, excessive foreign currency liabilities of domestic business results in a de
facto domestic tightening. Reduced exposure to currency risk would substantially
mitigate the pain of reversal of capital flows.

 As the use of Rupee becomes significant, the bargaining power of Indian business would

https://t.me/prelimbits
34

improve adding weight to the Indian economy, enhancing India‘s global stature and
respect.

Challenges in Internationalisation of Rupee

 India is a capital deficient country, and hence needs foreign capital to fund its growth.

o If a substantial portion of its trade is in Rupee, non-residents would hold Rupee


balances in India which would be used to acquire Indian assets.

o Large holdings of such financial assets could heighten vulnerability to external


shocks, managing which would necessitate more effective policy tools.

 A reduced role for convertible currencies in external transactions could lead to reduced
reserve accretion. At the same time, however, the need for reserves would also reduce to
the extent the trade deficit is funded in Rupees.

 Non-resident holdings of Rupees could exacerbate pass-through of external stimulus to


domestic financial markets, increasing volatility. For instance, a global risk-off phase could
lead non-residents to convert their Rupee holdings and move out of India.

Steps taken for the Internationalisation of the Rupee

 In July, 2022 the RBI has introduced a mechanism to facilitate international trade in
rupees.

 Enabling external commercial borrowings in Rupees (especially Masala Bonds).

 The Asian Clearing Union is also exploring a scheme of using domestic currencies for
settlement. An arrangement, bilateral or among trading blocs, which offers importers of
each country the choice to pay in domestic currency is likely to be favoured by all countries,
and therefore, is worth exploring.

18. Given below are two statements, one labelled as Assertion (A) and the other
labelled as Reason (R): [1996]
Assertion (A): Though India‟s national income has gone up several fold since 1947,
there has been no marked improvement in the per capita income level.
Reason (R): Sizeable proportion of the population of India is still living below the
poverty line
In the context of the above two statements which one of the following is correct?

A. Both A and R are true and R is the correct explanation of A

B. Both A and R are true but R is not a correct explanation of A

C. A is true but R is false

D. A is false but R is true

Ans. B

https://t.me/prelimbits
35

 The per capita income doesn‘t accurately reflect the standard of living for individuals
residing in the specific area.

 We can‘t say Per capita income is a reliable indicator to signify of the standard of living as
Per Capita Income is obtained by dividing the Total Income of the population by its
population.

 As result, economic inequality may not be taken into account by per capita
income.

PER CAPITA INCOME


 Per capita Income or PCI is the average income earned by every individual in a certain
time period of a given area (say, a country, state, city, or others).

 It can be calculated by dividing the total income of a specific area by the total
population of that area.

o Per capita income or PCI = Total income earned by Population / Population of a


specific area

 In order to assess a population‘s standard of living, per capita income, is used to


calculate the average per-person income.

Limitations of per capita income

 Inflation

o While determining the per capita income we do not calculate the inflation rate (the
rate of price rise) into the account.

o Consumers‘ purchasing power is reduced by inflation, which also restricts income


growth. Because of this, per capita income can exceed a population‘s income.

 Children

o The calculations of per capita Income include those non-earning Individuals such
as children and even newborns. The newborns or children are counted although
they do not generate any revenue, As result, they don‘t increase the average
income of the nation.

o Therefore, while calculating the Average income of


a country‘s economy having a large child
population in the per capita income or PCI
parameter, the outcome is biased and
inadequate.

 Economic Welfare

o A country‘s standard of living can be determined


using the per capita income.

https://t.me/prelimbits
36

o But factors like the standard of the workplace, the standard of working
conditions, the number of hours worked literacy rates, and overall health benefits
are not always considered in the calculation of Per capita income.

o If the rate of population increase outpaces the rate of national income growth,
economic welfare, and the availability of goods and services then the per capita
income will decline.

 Poverty

o As per capita income is determined by dividing total income by the population of


that area.

o So it is the Overall idea of a specific geographical area, not actual numbers.


Despite the increase in national revenue, its distribution makes the wealthiest
further richer and the poor even poorer.

 Savings and Wealth

o A person‘s savings or wealth are not included in the per capita income calculation.
If a wealthy individual may not work and have a minimal annual income, they still
use their money to maintain a satisfactory level of living.

o The wealthy person would appear to be a low earner using the per capita income
statistic.

 Livings Standards

o The per capita income doesn‘t accurately reflect the standard of living for individuals
residing in the specific area. We can‘t say Per capita income is a reliable indicator to
signify of the standard of living as Per Capita Income is obtained by dividing the
Total Income of the population by its population. As result, economic inequality may
not be taken into account by per capita income.

19. A redistribution of income in a country can be best brought about through: [1996]
A. progressive taxation combined with progressive expenditure

B. progressive taxation combined with regressive expenditure

C. regressive taxation combined with regressive, expenditure

D. regressive taxation combined with progressive expenditure

Ans. B

 A progressive tax : is a tax that takes a larger percentage from high-income earners
than it does from low-income individuals.

 Regressive expenditure : If the government tends to spend more on poor in


comparison to rich people then it is called regressive expenditure and if government tends
to spend more on rich than on poor then it is called progressive expenditure.

https://t.me/prelimbits
37

o In other words : If the government spending decreases with the increase in


income of people then it is called as REGRESSIVE EXPENDITURE. On the other
hand if the government spending increases with the increase in income then it is
called as progressive expenditure.

TYPES OF TAXES: PROGRESSIVE VS REGRESSIVE VS PROPORTIONAL


Types Example
Proportional If Govt. had a single 10% Flat Rate direct tax on income irrespective of
whether you‘re a poor, middle class, upper middle class or a rich person.
Then each taxpayers‘ same proportion of income (10%) will go into taxes.
Progressive 5%-20%-30% income tax slabs depending on income.
Thus, richer the person, bigger proportion of his income will go into taxes.
Thus, DIRECT TAXES ARE PROGRESSIVE in nature.
So → Tax rate increases with increase in the taxpayer‘s income
Regressive 18% GST on Biscuits worth ₹100 = ₹18 paid as (indirect) tax
When rich one buys one packet, and a poor man buys one packet, greater
proportion of poorman‘s income is gone in taxes. Thus, INDIRECT TAXES
ARE REGRESSIVE in nature.
So → Tax Rate decreases with increase in income.

20. [1996]
Assertion (A): An important policy instrument of economic liberalization is reduction in
import duties on capital goods.
Reason (R): Reduction in import duties would help the local entrepreneurs to improve
technology to face the global markets.
In the context of the above two statements, which one of the following is correct?

A. Both A and R are true and R is the correct explanation


B. Both A and R are true R is not a correct explanation
C. A is true but R is false
D. A is false but R is true

Ans. A

LIBERALISATION
 Liberalisation is the process or means of removing the state's control over economic
activities. It gives business enterprises more autonomy in decision-making and eliminates
government interference.
 It is used in the context of a government relaxing its previously imposed restrictions on
economic or social policies. In India, liberalization was done through deregulating the
industrial sector, financial sector reforms, tax reforms and foreign exchange reforms.
 In India, it began with the introduction of a new economic policy to tide over conditions
of the balance of payment crisis.
 Liberalization was undertaken to attain objectives like industrialization, expansion in the

https://t.me/prelimbits
38

role of private and foreign investment, and the introduction of a free market system.

Need for Liberalization

 Following independence, India erected barriers to foreign trade and investment in order to
protect domestic producers from foreign competition. It was believed that imports would
have stifled the growth of these industries.
 There is a widespread belief that the mixed economy framework that has prevailed since
independence has resulted in the establishment of a slew of rules and laws, culminating in
permit licence raj.
 During 1990-1991, the government was unable to make repayments on its foreign
borrowings.
 Low Forex reserve coverage to cover imports: In 1990-91, India's foreign exchange
reserves were so low that they were insufficient to cover a 10-day import bill. Forex
reserves fell sharply from Rs. 8,151 crore in 1986-87 to Rs. 6,252 crore in 1989-90.
 Poor Fiscal Management: During the 1980s, government spending outpaced revenue, and
continued spending on development programmes did not generate additional revenue.
 The government was unable to generate enough revenue from internal sources such as
taxation. A greater proportion of spending has gone to areas such as the social sector and
defence.
 Loss making PSUs: There were only 5 public-sector enterprises in India in 1951, but by
March 1991, the number had grown to 246. Several thousand crores of rupees have been
invested in their growth. Their performance was encouraging for the first 15 years, but then
the majority of them began to lose money. Public-sector enterprises have devolved into
liabilities as a result of poor performance.
 Money borrowed from foreign governments/multinational institutions was used to meet
the government's consumption needs.
 Borrowing for Interest coverage: At one point, there was insufficient foreign exchange
to pay the interest owed to international lenders.
 India approached international financial institutions such as the IBRD and the IMF for
loans, and while granting loans, the international agencies expected India to liberalise
and open up the economy by removing restrictions on the private sector while also
reducing the government's role in many areas.
 This eventually resulted in the New Economic Reforms of 1991.

Objectives of Liberalization

 To encourage the participation of multinational firms and private companies in India


 To enable globalization of the Indian economy.
 To increase exports, promote foreign trade in the country.
 To overcome India‘s balance of payment crisis.
 To boost the participation of the private sector in the development of India‘s economy.
 To increase the amount of foreign direct investment in Indian industries
 To induce a competitive environment amongst domestic businesses.
https://t.me/prelimbits
39

 Characteristics

Characteristics of Liberalization

 Deregulation of the Industrial Sector


o Pre-1991 crisis
 Industrial licencing required every businessman to obtain permission from
government officials to open or close a business.
 The number of goods that could be produced was also subject to government
approval.
 In many industries, the private sector was not permitted.
 Some goods could only be manufactured on a small scale.
 The government used to control prices and distribute only certain industrial
products, which resulted in corruption.
o Post-1991 crisis
 Since 1991, many of the restrictions mentioned above have been lifted.
 Except for the following five industries, industrial licencing was abolished for
almost all products: (a) liquor, (b) cigarettes, (c) defence equipment, (d)
industrial explosives, and (e) dangerous chemicals.
 The number of industries reserved for the public sector was reduced from
17 to 8 under the new industrial policy. In 2010-11, the number of these
industries was reduced to just two, namely (i) Nuclear energy; and (ii) railways.
 Many production areas that were previously reserved for SSI (small-scale
industries) have been de-reserved. Market forces were allowed to
determine resource allocation (rather than the directive policy of the
government).
 The market has been allowed to determine prices in many industries.
 Financial Sector Reforms
o Liberalization entailed a significant shift in the RBI's role in the financial sector
from "regulator" to "facilitator."
o Private sector banks, both domestic and international, such as ICICI, Kotak, and
HDFC, were established.
o FDI and FPI limits were gradually increased in various sectors.
o Banks were permitted to generate funds from both India and abroad.
o Several reforms were implemented in the insurance, money, and capital markets,
among other areas.
o In response to the new realities of the Indian financial sector, new institutional
regulators and structures such as SEBI, BSE, NSE, PFRDA, and IRDA were
established.
 Tax Reforms
o Since 1991, there has been a steady reduction in individual income taxes.
o It was believed that high income tax rates were a major source of tax evasion, so

https://t.me/prelimbits
40

moderate income tax and corporate tax rates were implemented.


o Many procedures have been streamlined.
o Indirect tax reforms have been implemented, the most recent of which is the Goods
and Services Tax (GST).
 Foreign Exchange Reforms
o The rupee has been devalued in relation to foreign currencies. It was
done primarily to boost exports and, ultimately, to increase foreign exchange
reserves.
o The Indian rupee's depreciation against foreign currencies increased the supply of
foreign exchange in the Indian economy.
o As a result, demand and supply of foreign currency determined exchange rates,
and government intervention was minimal in this regard. The RBI rarely intervenes,
which is known as 'managed float.'
 Trade and Investment Policy
o Reforms to trade and investment policies:
o Import quantitative restrictions were gradually eased.
o Except for hazardous and environmentally sensitive industries, import licencing was
abolished.
o Import quantitative restrictions on manufactured consumer goods and agricultural
products were also lifted in April 2001.
o Export duties have been eliminated in order to increase the competitiveness of
Indian goods in international markets.
o FDI/FPI flowed in gradually.

Benefits of Liberalization

 Liberalisation ushered in the free inflow of capital in the country by enabling businesses to
access the same from investors.
 It enabled diversification of investor portfolios which helped in increasing profits
amongst businesses.
 Decreasing the economic regulations lead to an increase in the stock market‘s value, thus
resulting in better trading among investors.
 It improved the agricultural sector by resulting in greater investments, diversification of
cropping patterns, etc.
 Limitations

Limitations of Liberalization

 It caused economic destabilization due to radical reforms introduced in political and


economic domains.
 Increased competition from large multinationals threatened the existence of several
smaller firms.
 Removal of restrictions from sectors like banking and insurance led to a downfall in the
government‘s stakes in both these sectors.

https://t.me/prelimbits
41

 Increased potential for mergers and acquisitions posed a threat to the employees of
smaller firms

21. The Eighth Five Year Plan is different from the earliest ones. The critical difference lies
in the fact that: [1996]
A. it has a considerably larger outlay compared to the earlier plans
B. it has a major thrust on agricultural and rural development
C. considerable emphasis is placed on infrastructure growth
D. industrial licensing has been abolished

Ans. A

Eighth Five Year Plan (1992-97) had a bigger outlay with energy being given 26.6% of total outlay
to a cheque a targeted growth rate of 6.78% per annum.

Eighth Five  The Eighth Plan promoted the modernisation of Industries.


Year Plan  India became a member of the World Trade Organisation on 1 January
(1992-97) 1995.

 The goals were to control population growth, reduce poverty,


generate employment, strengthen the development of
infrastructure, manage tourism, focus on human resource
development etc.
 It also laid emphasis on involving the Panchayats and Nagar Palikas
through decentralisation.

 The target growth rate was 5.6% but the actual growth rate was an
incredible 6.8%.

22. Most international agencies which find Development Programme in India on


intergovernmental bilateral agreements, mainly provide: [1996]
1. Technical assistance
2. Soft loans which are required to be paid back with interest
3. Grants, not required to be paid back
4. Food assistance to be paid back

Consider the following Options:

A. 2 and 4 are correct


B. 1, 2 and 3 are correct
C. 1, 2 and 4 are correct
D. 3 and 4 are correct

Ans. B

Food assistance is not provided by international agencies which fund developmental programmes
https://t.me/prelimbits
42

in India under intergovernmental and bilateral agreements.

 A soft loan is a loan with a below market rate of interest. It also includes concessions to
borrowers such as long repayment periods or interest holidays.
 Technical assistance is aid involving highly educated or trained personnel, such as
doctors, who are moved into a developing country to assist with a program of
development.
 Grant is usually given to governments through individual countries, international aid
agencies and through multilateral institutions such as the world Bank and by through
development charities.

23. In India, rural incomes are generally lower than the urban incomes. Which of the
following reasons account for this? [1996]
1. A large number of farmers are illiterate and know little about scientific agriculture
2. Prices of primary products are lower than of manufactured products
3. Investment in agriculture has been low when compared to investment in industry

Select the correct answer by using the codes given below:

A. 1, 2 and 3
B. 1 and 3
C. 1 and 3
D. 2 and 3

Ans. D

 Prevailing illiteracy in rural areas does not lower productivity. The farmer has adequate
knowledge of farming techniques. Low prices of primary products compared to the
manufactured products and investment in the agriculture sector compared to the industry
are major factors which accounts for low incomes in rural economy than in Urban economy.

24. Local supply of coal is not available to: [1996]


A. TISCO, Jamshedpur
B. VSL, Bhadravati
C. HSL, Durgapur
D. HSL, Bhilai

Ans. B

 VSL Bhadravati located in Karnataka is the only iron and state plant which does not have
capative coal mines and is situated outside the coal producing area. The plant switched over
to electric furnaces after the completion of Mahatma Gandhi Hydel Power Station.

25. Hawala transactions relate to payments: [1996]


A. received in rupees against overseas currencies and vice versa without going through the
official channels

https://t.me/prelimbits
43

B. received for sale/transfer of shares without going through the established stock exchanges
C. received as commission for services rendered to overseas investors/buyers/sellers in
assisting them to get over the red tape and/or in getting preferential treatment
D. made to political parties or to individuals for meeting election expenses

Ans. A

HAWALA

 Hawala is an illegal method of remittance across countries. == > Black Money


 There are money brokers who are the middle men who undertake hawala transfer. This
method of remittance does not involve physical movement of cash. It is also known as
Hundi.
 The word Hawala means trust. The Hawala system works as it is based on mutual trust
between the hawala agents. It works outside the banking system and legal financial
systems.
 This works through codes, contacts and trust with no paperwork at all.
 The remittance happen based on communication between the hawala agents. It is an
alternate to traditional remittance system.

26.Human Development Index comprises literacy rates, life expectancy at birth and
[1997]
A. Gross National Product per head in the US dollars

B. Gross Domestic Product per head at real purchasing power

C. Gross National Product in US dollars

D. National Income per head in US dollars


https://t.me/prelimbits
44

Ans. D

HUMAN DEVELOPMENT INDEX (HDI)


 The Human Development Index (HDI) is a statistic composite index of →

o Life expectancy,

o Education (mean years of schooling completed and expected years of schooling


upon entering the education system), and

o Per capita income indicators

 A country scores a higher level of HDI when the lifespan is higher, the education level
is higher, and the gross national income GNI (PPP) per capita is higher.

 It was developed by Pakistani economist Mahbub ul Haq and was further used to
measure a country's development by the United Nations Development Programme
(UNDP)'s Human Development Report Office.

 The 2010 Human Development Report introduced an Inequality-adjusted Human


Development Index (IHDI).

 While the simple HDI remains useful, it stated that "the IHDI is the actual level of human
development (accounting for inequality), while the HDI can be viewed as an index of
'potential' human development (or the maximum level of HDI) that could be achieved if
there were no inequality.‖

 The index does not take into account several factors, such as the net wealth per capita
or the relative quality of goods in a country. This is why even some of the most advanced
countries, including the G7 members (Canada, France, Germany, Italy, Japan, the UK, the
US, and the European Union) and others, do not do too well on HDI. That's why
countries like Switzerland rank high on HDI.

New method (2010 HDI onwards)

Published on 4 November 2010 (and updated on 10 June 2011), the 2010 Human Development
Report calculated the HDI combining three dimensions:

 A long and healthy life: Life expectancy at birth

 Education: Mean years of schooling and expected years of schooling

 A decent standard of living: GNI per capita (PPP international dollars)

According to the Human Development Report 2021-22, India‘s rank on the Human
Development Index (HDI) has slipped from 130 in 2020 to 132 in 2022, in line with a global fall in
HDI scores in the wake of the Covid-19 pandemic.

27. In India, inflation is measured by the: [1997]


A. Wholesale Price Index number
B. Consumers Price Index for urban non-manual workers

https://t.me/prelimbits
45

C. Consumers Price Index for agricultural workers


D. National Income Deflation

Ans. *

 Wholesale Price Index (WPI) is an index used by the Reserve Bank of India till 2014 to
measure inflation. WPI is the price of a representative basket of wholesale goods.
 The RBI, ex-governor Raghuram Rajan, Shifted to consumer Price Index (CPI) is
BECAUSE WPI neglects services and the bottlenecks between a wholesaler and a retailer.

WHOLESALE PRICE INDEX


 It was launched in 1942
 It measures the changes in the prices of goods [NOT SERVICES] sold and traded in
bulk by wholesale businesses to other businesses.
 Published by the Office of Economic Adviser, Ministry of Commerce and Industry.
 It is the most widely used inflation indicator in India.
 It is also used as GDP deflators
 Major criticism for this index is that the general public does not buy products at wholesale
price.
 The base year of All-India WPI has been revised from 2004-05 to 2011-12 in 2017
 Calculated every month in india.
 Presently it covered 697 commodities

CONSUMER PRICE INDEX


 It measures price changes from the perspective of a retail buyer. It is released by
the National Statistical Office (NSO).
 The CPI calculates the difference in the price of COMMODITIES & SERVICES such as
food, medical care, education, electronics etc, which Indian consumers buy for use.
 The CPI has several sub-groups including food and beverages, fuel and light, housing and
clothing, bedding and footwear.
 Types of CPI are as follows:
o CPI for Industrial Workers (IW). [used to revise dearness allowance (DA)]
o BY – 2016
o CPI for Agricultural Labourer (AL). [used to decide to MGNREGA]
o CPI for Rural Labourer (RL).
o CPI (Rural/Urban/Combined).
o CPI UNME

Of these, the first three are compiled by the Labour Bureau in the Ministry of Labour
and Employment. Fourth & 5th is compiled by the NSO in the Ministry of Statistics
and Programme Implementation.

 Base Year for CPI is 2012.


 The Monetary Policy Committee (MPC) uses CPI data to control inflation. In April 2014,

https://t.me/prelimbits
46

the Reserve Bank of India (RBI) had adopted the CPI as its key measure of inflation.

28. National Income is: [1997]


A. Net National Product at market price
B. Net National Product at factor cost
C. Net Domestic Product at market price
D. Net Domestic Product at factor cost

Ans. B

NATIONAL INCOME

 National Income is the money value of all final goods and services produced in
an economy during a financial year. At the level of an economy, value of final
goods and services is equal to the total income of all factors of production viz labour,
capital, land and entrepreneurship.
 It does not include taxes, depreciation and non-factor inputs (raw materials)

Domestic Income

 Domestic Income – Total value of final goods and services produced within a domestic
territory during an accounting year, after adjusting depreciation.
o It is NDP at FC
o Both NNP and NDP can be measured at constant prices (real income) or market
prices (nominal income)
o Domestic Income + NFIA = National Income

MEASUREMENT OF NATIONAL INCOME


Income Method

 Estimated by adding all the factors of production (rent, wages, interest, profit) and the
mixed-income of self-employed.

Production Method

 Estimated by adding the value added by all the firms.


 Value-added = Value of Output – Value of (non-factor) inputs
o This gives GDP at Market Price (MP) – because it includes depreciation (therefore
‗gross‘) and taxes (therefore ‗market price‘)
o To reach National Income (that is, NNP at FC)
 Add Net Factor Income from Abroad: GNP at MP = GDP at MP + NFIA
 Subtract Depreciation: NNP at MP = GNP at MP – Dep
 Subtract Net Indirect Taxes: NNP at FC = NNP at MP – NIT

Expenditure Method

 The expenditure method to measure national income can be understood by the equation
given below:

https://t.me/prelimbits
47

Y = C + I + G + (X-M),
 where Y = GDP at MP, C = Private Sector‘s Expenditure on final consumer goods, G = Govt‘s
expenditure on final consumer goods, I = Investment or Capital Formation, X = Exports, I =
Imports, X-M = Net Exports

29. The Minimum Alternative Tax (MAT) was introduced in the Budget of the Government of
India for the year: [1997]
A. 1991-92
B. 1992-93
C. 1995-96
D. 1996-97

Ans. D

MINIMUM ALTERNATE TAX (MAT)


 MAT is a clause in direct tax laws that restrict tax exemptions granted to businesses,
requiring them to pay a minimum amount of corporate tax to the government.
 Unlike a conventional corporate tax, which is imposed on taxable earnings, MAT is levied
on Book Profit. [what is BF not Imp]
 MAT was 1st implemented in 1988 to bring zero-tax corporations into the tax net.
 It was later repealed in 1990. However, the Finance Act of 1996 reinstated it.
 The tax applies to all entities functioning in India, regardless of whether they are
owned by Indians or foreigners.
 Life Insurance Firms and Shipping Enterprises liable for tonnage tax are notable
exceptions.
 Companies with no permanent premises in India are also exempt from paying MAT.
 The government lowered the MAT tax rate from 18.5 % to 15% in September 2019,
while simultaneously lowering the company tax rate from 30 % to 22 %
 Cooperative Societies is also liable to pay 15% MAT.

30. Match List-I with List-II and select the correct answer. [1997]

Code

A. A- 1, B- 4, C-2, D -3
B. A- 4, B- 2, C-1, D -3

https://t.me/prelimbits
48

C. A- 4, B- 1, C-2, D -3
D. A- 1, B- 3, C-4, D -2

Ans. B

 Chelliah Committee on Tax Reforms – 1991


 Omkar Goswami Committee on Industrial Sickness – 1994
 Rangarajan Committee on Disinvestment of Shares in PSUs – April 1993
 RN Malhotra Committee on Reforms in Insurance Sector – 1993.

TAX REFORM COMMITTEE [CHELLIAH COMMITTEE]


 The Government appointed a Tax Reforms Committee in 1991 under Prof Raja
Chelliah to lay out the agenda for reforming India‘s tax system.
 It was primarily established to reform direct and indirect taxes prevailing in India.
 This TRC came up with three reports in 1991, 1992, and 1993 with several measures,
which can be summarized in the following points:
o Reforming the personal taxation system by reducing the marginal tax rates.
o Improving the quality of tax information.
o Simplifying the excise duties and its integration with a Value-Added Tax (VAT)
system.
o Bringing the service sector in the tax net within a VAT system.
o Reduction of corporate tax rates.
o Reducing the cost of imported inputs.
o By lowing the customs duties.
o Reduction in the number of Customs tariff rates and its rationalization.
o Broadening the tax base.
o Building tax information and computerization.
 Dr Chelliah was awarded Padma Vibushan in 2007 and is often referred to as ‗The Father of
Tax Reforms‘.

Other tax reform Com.

REKHI COMMITTEE ON INDIRECT TAX REFORM


 The Rekhi Committee was constituted in 1992 under the chairmanship of K.L. Rekhi. The
recommendations are
o A Tribunal should be set up to deal with problems between taxpayers and tax
collectors.
o A High Level all India classification committee with trade and industry representative
should be set up.
o Important consignments should be cleared within 3 days.
o The monopoly of one nominated bank in each state should be supplemented by
another bank.
o Coercive measures must not be used for recovery of disputed duty amount when the
assessee files a stay application.
https://t.me/prelimbits
49

KELKAR COMMITTEE REPORT ON TAX REFORMS


 A task force on Direct and Indirect Taxes was constituted under the chairmanship of DR.
Vijay Kelkar in 2002. The recommendation of the Vijay Kelkar committee are
o The income tax exemption limit must be increased to Rs. 1 Lakh from the present
50,000 tax exemption limit for senior citizens and widows should be Rs. 1.5 lakhs.
o There should be a two-tier income tax structure with a 20% tax for earning od Rs. 1
lakh to 4 lakh and 30% tax for over Rs. 4 lakhs. The standard deduction must be
abolished but suggested an exemption for conveyance allowance.
o There should be the abolition of long-term capital gains tax, dividend tax, and wealth
tax. There should not be any surcharge on income tax.
o There should be an interest subsidy of 2% for housing loans up to Rs. 5 lakh.
o There must be a 30% corporate tax for domestic companies and 35% for foreign
companies and there should be no MInimum Alternate Tax (MAT).
o There Should be a 14% Central Value Added Tax (CENNAT) rate.
o There should be nationwide VAT and Comprehensive Service Tax.
o Exemptions for life-saving drugs, security items, and farm products.
o Exemption of tax for small scale units with a turnover up to Rs. 50 lakh.
 Different tax reforms measures were being undertaken by the government based on
the recommendations made by the committees on tax reforms in India and the reforms
are still continuing to this day. The biggest and the recent tax reforms in India that were
undertaken is the introduction of the Goods and Services Tax (GST). The Good and
Service Tax replaced almost all the existing indirect taxes throughout the country except
customs duty and it came into force effective from 1st July. 2017.

OMKAR GOSWAMI COMMITTEE


 During the phase of liberalization initiated in 1991, it was felt essential to take some
preventive steps to check growing industrial sickness in the country.
 In April 1993, a committee on industrial sickness and corporate restructuring was
constituted and Dr. Omkar Goswami of Indian Statistical Institute was appointed
chairman of this committee. The committee submitted its report to the Ministry of Finance
on July 13,1993.
 The committee suggested a new definition of industrial sickness and also gave
recommendations to modify SICA [ Sick Industrial Companies Act (SICA, repealed in
2016] and the role of BIFR. [Board for Industrial and Financial Reconstruction, Dissolved on
30 November 2016]
 The committee made it clear that in previous years BIFR adopted the policy of rehabilitation
of sick units instead of closing them. The committee objected this approach with the
view that any financial assistance granted under rehabilitation policy is indirectly a
reward to inefficient and irresponsible entrepreneurs. The Committee also made it
clear that this rehabilitation certainly provides a relief to the laborers in short run but
this relief is not long lasting.

https://t.me/prelimbits
50

RANGARAJAN COMMITTEE OF 1993


 The Rangarajan Committee of 1993 was constituted by the government for making
recommendations in context with the Disinvestment.
 The committee said that →
o The units to be disinvested should be identified and disinvestment could be made up
to any level, except in defense and atomic energy where the government should
retain the majority holding in equity.
o Disinvestment should be a transparent process duly protecting the right of the
workers.
o An autonomous body for the smooth functioning and monitoring of the
disinvestment program should be established.
 This recommendation led to the Disinvestment Commission in 1996 as an advisory body
having a full-time chairman and four part-time members.
 It suggested four modes of disinvestment viz. Trade sale, Strategic Sale, Offer of
shares, and Closure or Sale of Assets.

31. The Sixth and the Eighth Five Year Plans covered the period 1980-1985 and 1992-1997
respectively. The Seventh Five Year Plan covered the period: [1997]
A. 1987-1992
B. 1986 -1991
C. 1985-1990
D. 1988 -1994

Ans. C

Period between 1990-92 was second plan holiday. The first plan holiday was between 1966-69.

Sixth Five  It underlined the beginning of economic liberation by eliminating price


Year Plan controls.
(1980-85)
 It was seen as the end of Nehruvian Socialism.

 To prevent overpopulation, family planning was introduced.

 On the recommendation of the Shivaraman Committee, the National Bank


for Agriculture and Rural Development was established.

 The target growth rate was 5.2% and the actual growth rate was 5.7%,
implying that it was a success.

Seventh  This plan was led by the Prime Ministership of Rajiv Gandhi.
Five Year
 It laid stress on improving Industrial productivity levels through the
Plan (1985-
use of technology.
90)
 Other objectives included increasing economic productivity, increasing the

https://t.me/prelimbits
51

production of food grains and generating employment by providing


Social Justice.

 The outcome of the Sixth Five-Year Plan provided a robust base for the
success of the seventh five-year plan.

 It emphasised anti-poverty programmes, the use of modern technology,


and the need to make India an independent economy.

 It focused on attaining prerequisites for self-sustained growth by 2000.

 The target growth rate was 5.0%. However, the actual growth rate grew to
reach 6.01%

Annual Plans (1990-92)

The Eight Five Year Plan was not introduced in 1990 and the following years 1990-91 and 1991-
92 were treated as Annual Plans. This was largely because of the economic instability.
India faced a crisis of foreign exchange reserves during this time. Liberalisation,
Privatisation, Globalisation (LPG) was introduced in India to grapple with the problem of
the economy under prime minister P.V Narasimha Rao.

Eighth Five  The Eighth Plan promoted the modernisation of Industries.


Year Plan  India became a member of the World Trade Organisation on 1 January
(1992-97) 1995.

 The goals were to control population growth, reduce poverty,


generate employment, strengthen the development of
infrastructure, manage tourism, focus on human resource
development etc.
 It also laid emphasis on involving the Panchayats and Nagar Palikas
through decentralisation.

 The target growth rate was 5.6% but the actual growth rate was an
incredible 6.8%.

32. Match List-I with List-II and select the correct answer:

https://t.me/prelimbits
52

Code

A. A -5; B- 1; C -2; D- 3
B. A -3; B- 1; C -4; D -2
C. A -1; B-5; C - 4; D- 3
D. A -3; B - 4; C-1; D -2

Ans. B

 Iron ore is exported to Japan.


 Leather goods are exported to Russia.
 Tea is exported to U.K.
 Cotton fabric is exported to U.S.A.

33. The number of industries for which industrial licensing is required has now been
reduced to: [1997]
A. 15
B. 6
C. 35
D. 20

Ans. B

Industrial Licensing was abolished for all except short list of 18 industries in New Industrial
Policy 1991.

Currently, data, only five industries are under compulsory licensing mainly on account of
environmental, safety and strategic considerations. They are:

1. Distillation and brewing of alcoholic drinks


2. Cigars and Cigarettes of tobacco and manufactured tobacco substitutes.
3. Electronic Aerospace and defense equipment: all types
4. Industrial explosives including detonating fuses, safety fuses, gun powder, nitrocellulose and
matches.
5. Specified Hazardous chemicals i.e. (i) Hydrocyanic acid and its derivatives (ii) Phosgene and
its derivatives and (iii) Isocyanates & diisocyanates of hydrocarbon, not elsewhere specified
(example Methyl isocyanate).

34. Which of the following places are known for paper manufacturing industry ? [1997]
1. Yamuna Nagar
2. Guwahati
3. Shahabad
4. Ballarpur

Choose the correct answer using the codes given below:

A. 1, 2 and 3
B. 1, 2 and 4
https://t.me/prelimbits
53

C. I, 3 and 4
D. 2, 3 and 4

Ans. B

Yamuna Nagar, Guwahati and Ballarpur are famous for paper manufacturing industry. Yamuna
Nagar is in Haryana, Guwahati is in Assam and Ballarpur is in Maharashtra.

DISTRIBUTION OF PAPER INDUSTRY


 Maharashtra and Andhra Pradesh are India's major paper-producing states. Maharashtra
accounts for 16.52 percent of installed capacity and 18 percent of India's paper
production.
 Nearly 65 percent of overall manufacturing capacity is held by Maharashtra, Andhra
Pradesh, Madhya Pradesh, and Karnataka.
 Tamil Nadu, Kerala, Assam, Haryana, Bihar, and Gujarat are the producers.
 According to existing estimations, Assam has the most potential for developing the paper
sector due to the existence of extensive forests as a raw material.

35. Match List-I with List-II and select the correct answer: [1997]

Code

A. A-3; B-1; C-2; D-4


B. A-1; B-3; C-4; D-2
C. A-3; B-1; C-4; D-2
D. A-1; B-3; C-2; D-4

Ans. A

36. The sum of which of the following constitutes Broad Money in India? [1997]
1. Currency with the public

https://t.me/prelimbits
54

2. Demand deposits with banks


3. Time deposits with banks
4. Other deposits with RBI

Choose the correct answer using the codes given below:

A. 1 and 2
B. 1, 2 and 3
C. 1, 2, 3 and 4
D. 1, 2 and 4

Ans. B

MONETARY AGGREGATES
 Monetary aggregates are the measures of the money supply in a country.
 Very often, the money supply in the economy is represented using a monetary aggregate
called ‗Broad Money‘, also denoted as M3.
 In India, Reserve Bank of India (RBI), measures the money supply and publishes it on a
weekly or fortnight basis.
 There are also different other monetary aggregates.
M1 = C + DD (Narrow Money)

 C - Currency held by the public


 DD- Demand Deposits with Banks [CASA]
Why Narrow

 Bcoz it includes only 100% liquid deposits which is a very narrow definition of the money
supply.
M2

 M1 + Savings account deposits with Post Offices


Note- Post offices have no facility for the opening of current accounts. The types of accounts that
can be opened are – Savings account, Fixed deposit, and Recurring deposit.
M3 = M1 + TD (Broad Money)

 TD – Time Deposits with Banks Includes fixed deposits, Recurring deposits, and time
liability of Savings accounts
 The Most Common measure used for money supply is M3
 Central Bank tracks the growth of broad money to help forecast inflation
M4 = M3 + Total Deposits with Post Office

 As the total deposits with the post office are negligible there is not much difference
between M3 and M4.
We are only counting the ―NET Demand / NET Time deposits‖ i.e. only public deposits in bank.
We are not counting interbank deposits i.e. one commercial bank‘s deposit in other commercial
banks.
37. The Capital Account Convertibility of the Indian Rupee implies: [1998]
A. that the Indian Rupee can be exchanged by the authorised dealers for travel

https://t.me/prelimbits
55

B. that the Indian Rupee can be exchanged for any major currency for the purpose of trade in
goods and services
C. that the Indian Rupee can be exchanged for any major currency for the purpose of trading
financial assets
D. None of the above

Ans. C

CAPITAL ACCOUNT CONVERTIBILITY


 CAC means the freedom to convert rupee into any foreign currency (Euro, Dollar, Yen,
Renminbi etc.) and foreign currency back into rupee for capital account transactions. In
very simple terms it means, Indian‘s having the freedom to convert their local financial
assets into foreign ones at market determined exchange rate.
 CAC will lead to a free exchange of currency at a lower rate and an unrestricted
movement of capital
 Capital Account Convertibility is widely regarded as the hallmark of developed countries.
It is also seen as the major comfort factor for foreign investors since it allows them to
reconvert local currency back into their own currency and move out from India.
 After the recommendations of the S.S. Tarapore Committee (1997) on Capital Account
Convertibility, India has been moving in the direction of allowing full convertibility in
this account, but with required precautions. India is still a country of partial
convertibility (40:60) in the capital account, but inside this overall policy, enough reforms
have been made, and to certain levels of foreign exchange requirements, it is an economy
allowing full capital account convertibility. Following steps have been taken in the direction
of capital account convertibility.

CURRENT ACCOUNT CONVERTIBILITY


 Current Account Convertibility allows free inflows and outflows of foreign currency for all
purpose including resident Indians buying foreign goods and services (imports), Indians
selling foreign goods and services (exports), Indians receiving and sending remittances,
accessing foreign currency for travel, study abroad, medical tourism purpose etc.
 Current account is today fully convertible (operationalized on August 19, 1994).
 It means that the full amount of the foreign exchange required by someone for current
purposes will be made available to him at the official exchange rate and there could
be an unprohibited outflow of foreign exchange (earlier it was partially convertible).
 India was obliged to do so as per Article VIII of the IMF which prohibits any exchange
restrictions on current international transactions (keep in mind that India was under pre-
conditions of the IMF since 1991).

38. A consumer is said to be in equilibrium, if: [1998]


A. he is able to fulfil his need with a given level of income
B. he is able to live in full comforts with a given level of income

https://t.me/prelimbits
56

C. he can fulfil his needs without consumption of certain items


D. he is able to locate new sources of income

Ans. A

EQUILIBRIUM IN THE MARGINAL UTILITY


 A consumer is said to be in equilibrium if he is able to fulfill his needs with a given level
of income.
 Equilibrium in the marginal utility, from an economic point of view, is derived from money
paid and well consumed.
 If the satisfaction derived from each commodity would be equal to each other, then
the consumers would be said to be in equilibrium.
 If a consumer cannot change his condition either by spending or earning more, then he is
said to be in equilibrium.

39. The supply-side economics lays greater emphasis on the point of view of: [1998]
A. producer
B. global economy
C. consumer
D. middle-man

Ans. A

SUPPLY-SIDE ECONOMICS
https://www.indeed.com/career-advice/career-development/supply-side-vs-demand-side

 Supply-side economics describes when wealthy individuals or large corporations receive


tax cuts.
 The hope is that these individuals use tax cuts to their advantage to make investments,
hire additional employees and complete other business initiatives that
help stimulate the economy.
 In supply-side economics, there's a prioritization of the production volume of
goods for consumers to purchase. Supply side economics consists of three policies →
o Monetary policy: In supply-side economics, monetary policies include increasing
the amount of money in circulation to allow for lower interest rates or lower
inflation. This helps businesses feel more comfortable about making investments.
o Tax policy: Supply-side economics encourages the government to reduce taxes on
businesses and wealthy individuals and redistribute tax percentages across the
population. Their goal is to inspire businesses to lower market prices, expand into
new markets and open up new jobs for consumers.
o Regulatory policy: The government enacts regulatory policies that offer businesses
more freedom regarding international trading, unemployment benefits and other
areas that would cost more with heightened regulations.

https://t.me/prelimbits
57

DEMAND-SIDE ECONOMICS
 Demand-side economics represents the idea that providing tax cuts to wealthy individuals
doesn't help the economy.
 Demand-side economics focuses on government works projects and other
government initiatives that create jobs.
 By increasing job opportunities through government projects, more consumers may
feel comfortable spending more, increasing economic growth. Demand-side economics
consists of these types of policies:
o Monetary policy: In demand-side economics, the government creates monetary
policies to reduce interest rates. This makes it easier for consumers to pay off debts
and encourages them to make major purchases on things like cars or homes.
o Fiscal policy: The government creates fiscal policies to rebalance tax rates so that
consumers receive a lower tax rate than more wealthy individuals or businesses.

40. Human Poverty Index was introduced in the Human Development Report of the year:
[1998]
A. 1994
B. 1995
C. 1996
D. 1997

Ans. D

HUMAN POVERTY INDEX (HPI)


 The Human Poverty Index (HPI) was an indication of the Standard of living in a country,
developed by the United Nations (UN) to complement the Human Development Index
(HDI) and was first reported as a part of the Human Development Report in 1997.
 In 2010, it was supplanted by the UN‘s Multidimensional Poverty Index.

GLOBAL MULTIDIMENSIONAL POVERTY INDEX (MPI) 2022

Global Multidimensional Poverty Index (MPI) 2022 was released by the United Nations
Development Programme (UNDP) and the Oxford Poverty and Human Development
Initiative (OPHI).

Global Data

 1.2 billion people are multidimensionally poor.

 Nearly half of them live in severe poverty.

 Half of poor people (593 million) are children under age 18


https://t.me/prelimbits
58

 The number of poor people is highest in Sub Saharan Africa (579 million), followed by South
Asia (385 million). The two regions together are home to 83% of poor people.

Key Findings about India

 India has by far the largest number of poor people worldwide at 22.8 crore, followed
by Nigeria at 9.6 crore.

 Two-thirds of these people live in a household in which at least one person is deprived
of nutrition.

Reduction in Poverty

 The incidence of poverty fell from 55.1% in 2005/06 to 16.4% in 2019/21 in the country.

 The deprivations in all 10 MPI indicators saw significant reductions as a result of which
the MPI value and incidence of poverty more than halved.

 As many as 41.5 crore people moved out of poverty in India during the 15-year period
between 2005-06 and 2019-21.

o Improvement in MPI for India has significantly contributed to the decline in


poverty in South Asia.

o South Asia now has not the lowest number of poor people than Sub-Saharan
Africa.

Relative Reduction in Poverty:

 The relative reduction from 2015/2016 to 2019/21 was faster: 11.9% a year compared
with 8.1% from 2005/2006 to 2015/2016.

Performance of States:

 Bihar, the poorest state in 2015-16, saw the fastest reduction in MPI value in absolute
terms.

 The percentage of poor in Bihar fell from 77.4 % in 2005-06 to 52.4 % in 2015-16 and
further to 34.7 % in 2019-21.

https://t.me/prelimbits
59

 However, in relative terms, the poorest states have not quite caught up.

o Of the 10 poorest states in 2015/2016, only one (West Bengal) have emerged out of
the list in 2019-21.

o The rest (Bihar, Jharkhand, Meghalaya, Madhya Pradesh, Uttar Pradesh, Assam,
Odisha, Chhattisgarh and Rajasthan) remain among the 10 poorest.

 Across states and union territories in India, the fastest reduction in relative terms was in
Goa, followed by Jammu and Kashmir, Andhra Pradesh, Chhattisgarh and Rajasthan.

Poverty among Children:

 Poverty among children fell faster in absolute terms, although India still has the highest
number of poor children in the world.

 More than one in five children in India are poor compared with around one in seven
adults.

Reduction of Poverty Region Wise:

 The incidence of poverty fell from 36.6% in 2015-2016 to 21.2% in 2019-2021 in rural
areas and from 9.0% to 5.5% in urban areas.

Global Multidimensional Poverty Index

 The index is a key international resource that measures acute multidimensional poverty
across more than 100 developing countries.

 It was first launched in 2010 by the OPHI and the Human Development Report Office of
the UNDP.

 The MPI monitors deprivations in 10 indicators spanning health, education and standard
of living and includes both incidence as well as intensity of poverty.

https://t.me/prelimbits
60

 A person is multidimensionally poor if she/he is deprived in one third or more (means


33% or more) of the weighted indicators (out of the ten indicators). Those who are
deprived in one half or more of the weighted indicators are considered living in extreme
multidimensional poverty.

41. The current Price Index (base 1960) is nearly 330. This means that the price of: [1998]
A. all items cost 3.3 times more than what they did in 1960
B. the price of certain selected items have gone upto 3.3 times
C. weighted mean of price of certain items has increased 3.3 times
D. gold price has gone up 3.3 times

Ans. C

 A price index is a normalized average (typically a weighted average) of price relatives for a
given class of goods or services in a given region, during a given interval of time. The
current Price Index (base 1960) is nearly 330. This means that the price of the weighted
mean of price of certain items has increased 3.3 times.

42. The price of any currency in international market is decided by the:


1. World Bank
2. Demand for goods/services provided by the country concerned
3. Stability of the government of the concerned country
4. Economic potential of the country in question of these

Options

A. 1, 2, 3 and 4 are correct


B. 2 and 3 are correct
C. 3 and 4 are correct
D. 1 and 4 are correct

Ans. B

 World Bank is important to the source of financial and technical assistance to developing
countries around the world.
 It has no relation to the price of the currency in the International Market. Hence statement
1 is not correct.
 The price of any currency is determined by forces of demand and supply. Demand for a
currency depends on two factors- its exports to other countries and investments that
people want to make in that currency. Hence statement 3 is correct.
 The stability of the government is a very important factor too as an unstable govt may not
be able to take effective economic decisions which will, in turn, affect export and
import. Hence statement 2 is correct.
 The economic potential of the country is not related to the price of any currency in the
international market. Hence statement 4 is not correct.

https://t.me/prelimbits
61

43. Some time back, the Government of India, decided to de-license „white goods‟ industry.
„White goods‟ include: [1998]
A. stainless steel and aluminium utensils
B. milk and milk products
C. items purchased for conspicuous consumption
D. soaps, detergents and other mass consumption goods

Ans. C

WHITE GOODS
 White goods are large home appliances such as, LED Bulbs, refrigerators, freezers,
washing machines, tumble driers, dishwashers, and air conditioners. They are large
electrical goods for the house which were traditionally available only in white

44.The banks are required to maintain a certain ratio between their cash in hand and total
assets. This is called: [1998]
A. SBR (Statutory Bank Ratio)
B. SLR (Statutory Liquid Ratio)
C. CBR (Central Liquid Reserve)
D. CLR (Central Liquid Reserve)

Ans. B

CRR, SLR (FIGHT INFLATION: ↑, DEFLATION: ↓)


 CRR and SLR are collectively known as ―Variable Reserve Ratios‖ or ―Statutory Reserve
Ratios‖

CRR SLR
Banks must keep this much deposits (or Banks must keep with themselves this
balance) with RBI. [only in cash] deposits in liquid assets such as cash, gold,
RBI doesn‘t pay interest on this deposit, except in G-Sec, T-Bills, State Development Loan Bonds
extraordinary circumstances like 1999‘s Banking and other securities notified by RBI.
slowdown.
Bank earns no profit / interest, as such. Some profit may be involved.
CRR: first suggested by the British economist J.M. Mandated under Banking Regulation Act,
Keynes & first introduced in US Federal Reserves 1949
(=Central Bank of USA).
Mandated under RBI Act, 1934

RBI can fix any amount of CRR, legally there is Legally, SLR can‘t be more than 40%.
no minimum floor or maximum ceiling. Presently it‘s 18.00% of NDTL.
Presently it‘s 4.50% of Net Demand & Time
Liabilities (NDTL) of a bank
All Scheduled Commercial Banks (SCB) must Similar
keep CRR.
However, RBI may prescribe separate %

https://t.me/prelimbits
62

norms/slabs for Regional Rural Banks (RRBs) and


Cooperative Banks.

Role of SLR & CRR


 CRR-SLR are counted on fortnightly basis. If not maintained, bank will have to pay penalty
interest rate to RBI. Penalty rate is linked with Bank Rate.
 CRR-SLR ensure monetary stability of India through two primary functions:
o CRR assists in money multiplier effect,
o CRR-SLR provide buffer/protection during a Bank Run
 While in theory CRR/SLR can be used for inflation control but RBI primarily relies on REPO
Rate (=its Policy Rate) to combat inflation, and not CRR/SLR.
45. The accounting year of the Reserve Bank of India is: [1998]
A. April-March
B. July-June
C. October-September
D. January-December

Ans. A

RBI ACCOUNTING YEAR


Before After
RBI‘s financial year For 2020: a smaller balancesheet prepared from
 1934 onwards: January to December 1/July/2020 to 31/March/2021.
 1940 onwards: July to June Then From 1st April, 2021, RBI has switched to
annual April-March format. [Bimal Jalan
Committee]
But Govt‘s financial year is April-March so, After this reform, no need for interim dividend
RBI had to give interim dividend to govt
until RBI‘s final balancesheet was prepared,
then another (full) dividend was given.

46.
Assertion (A): Devaluation of a currency may promote export. [1999]
Reason (R): Price of the country‟s products in the international market may fall due to
devaluation.
A. Both A and R are true and R is the correct explanation of A
B. Both A and R are true but R is not a correct explanation of A
C. A is true but R is false
D. A is false but R is true

Ans. A

https://t.me/prelimbits
63

CURRENCY DEVALUATION AND REVALUATION

 Under a fixed exchange rate system, devaluation and revaluation are official
changes in the value of a country‘s currency relative to other currencies.
 Devaluation →
o is when the price of the currency is officially decreased in a fixed exchange rate
system.
 Revaluation
o is when the price of the currency is increased within a fixed exchange rate system.
 For example, suppose a government has set 10 units of its currency equal to one rupee. To
devalue, it might announce that from now on 20 of its currency units will be equal to one
rupee. This would make its currency half as expensive to Indian, and the Indian Rupee twice
as expensive in the devaluing country.
 To revalue, the government might change the rate from 10 units to one rupee, to five units
to one rupee. This would make the currency twice as expensive to Indians, and the rupee
half as costly at home.
Reasons of currency devaluation

 Untenable fixed exchange rate system: To sustain a fixed exchange rate, a country must
have sufficient foreign exchange reserves, often dollars, and be willing to spend them, to
purchase all offers of its currency at the established exchange rate. When a country is
unable or unwilling to do so, then it must devalue its currency to a level that it is able
and willing to support with its foreign exchange reserves.
 Boost aggregate demand: Rather than implementing unpopular fiscal spending policies, a
government might try to use devaluation to boost aggregate demand in the economy in
an effort to fight unemployment.
Effects of devaluation

 Cheaper exports: Devaluation makes the country‘s exports relatively less expensive for
foreigners.
 Expensive imports: Devaluation makes foreign products relatively more expensive for
domestic consumers, thus discouraging imports.
 Rise in inflation: For an import dependent country like India, Currency depreciation would
mean increase in prices of imports, thus leading to inflation.
 Lower investor confidence: Devaluation may dampen investor confidence in the country‘s
economy and hurt the country‘s ability to secure foreign investment

(SIMILAR CONCEPT) → CURRENCY APPRECIATION AND DEPRECIATION


 Currency appreciation is an increase in the value of country‘s currency with respect to
one or more foreign reference currencies, in a floating exchange rate system.
o There are number of reasons that contribute currency appreciation, including

https://t.me/prelimbits
64

government policy, interest rates, trade balances and business cycles.


 Currency depreciation is the loss of value of a country‘s currency with respect to one or
more foreign reference currencies, in a floating exchange rate system.
o Depreciation of currency can occur due to any number of reasons – economic
fundamentals, interest rate differentials, political instability, risk aversion among
investors, etc.
Effects on exports and imports

 Since the exchange rate has an effect on the trade surplus or deficit, a weaker domestic
currency stimulates exports and makes imports more expensive. Conversely, a strong
domestic currency hampers exports and makes imports cheaper.
 For example: A good priced at $10 in the U.S. will be exported to India. With an assumption
that the exchange rate is 50 rupees to USD and ignoring transportation and other
transaction costs, the $10 item would cost the Indian importer 500 rupees.
 In a situation where, the dollar strengthens against the Indian rupee to a level of Rs. 55, and
the price of good remains constant at $10, the increased price would be 550 rupees ($10 x
Rs.55). This may force the importer to look for cheaper components from other locations.
 The 10% appreciation in the dollar versus the rupee has diminished the exporter‘s
competitiveness in the Indian market.
 To conclude, when a country has stronger value of currency or appreciation, they can
import more goods and services from another country (assuming that the currency of
exporting country remains the same). Similarly, currency depreciation leads to buying
lesser in the same amount of money.

(SIMILAR CONCEPT) → CURRENCY MONITORING LIST || CURRENCY MANIPULATION


The US Department of Treasury recently removed India along with Italy, Mexico, Thailand and
Vietnam from its Currency Monitoring List of major trading partners that merit close attention to
their currency practices and macroeconomic policies.

 Currency manipulation refers to a process defined by the USDT (US Treasury Department)
for countries that engage in unfair currency practices to gain a trade advantage.

 It is an attempt made by a country‘s central bank to decrease the value of their currency
with respect to foreign currency exchange rates, the dollar, in this case.

 To weaken its currency, a country sells its currency and buys foreign currency—usually
USD.

 This results in weak demand for the local currency and increased demand for US
dollars.

 The USTD uses Three Benchmarks to judge whether a country has manipulated its
currency:

o a Bilateral Trade Surplus with the US of more than $20 billion

https://t.me/prelimbits
65

o a Current Account Surplus of at least 3 % of GDP

o Net Purchases Of Foreign Currency of 2 % of GDP over 12 months

Remaining Countries

 China, Japan, Korea, Germany, Malaysia, Singapore, and Taiwan are the seven economies
that are a part of the current monitoring list.

47.
Assertion (A): Fiscal deficit is greater than budgetary deficit.
Reason (R): Fiscal deficit is the borrowing from the Reserve Bank of India plus other
liabilities of the Government to meet its expenditure. [1999]
A. Both A and R are true and R is the correct explanation of A
B. Both A and R are true but R is not a correct explanation of A
C. A is true but R is false
D. A is false but R is true

Ans. A

 Budget deficit = Total Receipts – Total Expenditure


 Fiscal deficit is the sum of Budget deficit plus Borrowings and other liabilities. Hence,
fiscal deficit is greater than budgetary deficit.

TYPES OF DEFICITS
 If government‘s income > its expenditure it will have a Surplus Budget
 If government‘s expenditure = its income, it will be a Balanced Budget
 If government‘s expenditure > its income, it‘ll be a Deficit Budget

Deficit Formula
Revenue Deficit Revenue expenditure minus Revenue receipts
Effective Revenue Deficit minus Grants for creation of capital assets
Revenue Deficit
Budget Deficit Total Receipts minus Total Expenditure
Effects of a budget deficit →
 Rise in national debt
 Higher debt interest payments
 Fall in value of currency
 Increase in currency circulation
 Increase in Aggregate Demand (AD)
 Possible increase in public sector investment
 May cause crowding out and higher bond yields
Fiscal Deficit Budget Deficit plus Borrowing

Primary Deficit Fiscal Deficit minus interest to be paid on previous loans

https://t.me/prelimbits
66

48. Persons below the poverty line in India are classified as such based whether: [1999]
A. they are entitled to a minimum prescribed food basket
B. they get work for a prescribed minimum number of days in a year
C. they belong to agricultural labourer household and the scheduled caste/tribe social group
D. their daily wages fall below the prescribed minimum wages

Ans. A

POVERTY LINE
 Poverty lines are defined as a level of income or spending below which it is reasonable
to conclude that someone is poorer than the rest of society.
 It is a measure of income or consumption spending that distinguishes the poor from the
rest of the population.
 The Tendulkar Committee proposed a poverty level of Rs 29 per person per day in urban
areas and Rs 22 per person per day in rural areas.
 There are two reasons for choosing a poverty line.
o To create policies that are tailored to the needs of the poor.
o To determine if government programs have been successful or unsuccessful over
time.

Poverty Line – Background

 Dadabhai Naoroji was the first to estimate a poverty line in the nineteenth century,
though he did not use the term "poverty line" himself.
 Dadabhai Naroji →
o Poverty assessment in India dates back to the 19th century when Dadabhai
Naoroji's efforts and careful research led him to establish a subsistence-based
poverty level at 1867-68 prices, though he never used the term "poverty line."
o It was calculated using the cost of a subsistence diet of "rice or flour, dhal, mutton,
vegetables, ghee, vegetable oil, and salt."
o According to him, subsistence is what a human being requires to meet his basic
needs and maintain his normal health and decency.
o His research included diet scales, and he arrived at a poverty level based on
subsistence prices that ranged from Rs.16 to Rs.35 per capita per year in diverse
parts of India. England's per capita income was Rs. 450 at the time.
o However, because India's essentials were only approximately a third of what they
were in England at the time, the real differential in terms of buying power parity was
only five times, not fifteen.
 National Planning Committee
o Subhash Chandra Bose, the president of the Congress, established the National
Planning Committee (NPC) in 1938, with Jawaharlal Nehru as chairman and
Professor K. T. Shah as secretary, to create an economic plan with the primary goal
of ensuring a sufficient quality of living for the masses.
https://t.me/prelimbits
67

o At pre-war prices, the Committee considered the irreducible minimum income to


be between Rs. 15 and Rs. 25 per capita per month.
o However, this was not labelled as a country's poverty line.
 First Planning Commission working group
o The concept of the poverty line was first introduced by a Planning
Commission working group in 1962 and then expanded by a task force in
1979.
o At 1960-61 prices, the national minimum for each household of five people should
be Rs 100 per month in rural areas and Rs 125 in urban areas, according to the
1962 working group.
o The expenditures on health and education, which were both supposed to be
provided by the state, were not included in these estimations.
 Y K Alagh Committee
o Until 1979, the standard method of calculating poverty was based on a lack of
money.
o Later, it was decided to assess poverty in terms of starving, i.e. how much people
eat.
o The YK Alagh Committee's suggestion in 1979, which said that persons
who consume less than 2100 calories in urban areas or less than 2400
calories in rural regions are poor, was the first to use this approach.
o The discrimination between rural and urban areas was based on the fact that rural
people do more physical labour.
o Moreover, it was assumed that the states would look for the people's health and
education.
o As a result, YK Alagh established India's first poverty line.
 Lakdawala Formula
o Official poverty lines were based exclusively on the recommendations of the
Lakdawala Committee from 1993 until 2011.
o This poverty limit was chosen such that anyone earning more than it could afford
2400 and 2100 calories worth of food, clothing, and shelter in rural and urban
regions, respectively.
o These calorie intakes were obtained solely from the YK Alagh committee.
o A poor person, according to the Lakdawala Committee, is one who is unable to meet
these average energy needs.
o However, in comparison to earlier models, the Lakdawala formula differed in the
following ways.
o Both health and education were previously removed from the estimates because
they were assumed to be provided by the states.
o The poverty line was established by this committee based on household per
capita consumption expenditure.

https://t.me/prelimbits
68

o The poverty line was calculated using the CPI-IL (Consumer Price Index for
Industrial Laborers) and CPI-AL (Consumer Price Index for Agricultural Laborers)
indexes.
o The approach for assessing poverty includes first estimating the per capita household
spending at which the average energy norm is satisfied, and then using that
expenditure as the poverty line, defining the poor as all people living in homes with
per capita expenditures less than the estimated value.
o As a result of the Lakdawala formula, the number of individuals living in poverty
nearly doubled.
o In 1993-94, the number of individuals living in poverty was 16.7% of the
population. According to the Lakdawala formula, it was 36.3%.
 Suresh Tendulkar Committee
o The Planning Commission established the Suresh Tendulkar committee in 2005.
o The current poverty estimates are based on the committee's recommendations.
o This committee suggested discarding the calorie-based methodology in favour of
a more broad-based poverty level that included monthly spending on education,
health, electricity, and transportation.
o It was highly advised to focus on nutritional outcomes rather than calories,
i.e., intake nutrition support should be counted instead of calories.
o The Tendulkar panel set a daily per capita expenditure benchmark of Rs. 27 in
rural regions and Rs. 33 in urban areas, respectively, and arrived at a poverty
cut-off of around 22% of the population.
o For the fiscal year 2004-05, the Lakdawala and Tendulkar Committees calculated the
percentage of the population living below the poverty line.
o However, because the amount was so small, it drew swift criticism from all corners
of the media and society.
o The government formed a new committee to study the poverty measurement
technique, led by Prime Minister's Economic Advisory Council Chairman C.
Rangarajan, because the statistics were unrealistic and too low.
o Despite the Tendulkar Committee's objections, the Rangarajan
Committee increased these limitations to Rs. 32 and Rs. 47,
respectively, and calculated a poverty line of over 30%.
o Poverty was estimated to be over 30% in 2011-12, according to the Rangarajan
committee. In 2011-12, India's poor were estimated to number 36.3 crore.
 Current Status: Arvind Panagariya Task Force
o The discussions over the Lakdawala Formula, the Suresh Tendulkar Committee, and
the Rangarajan Committee indicate that setting a poverty line in India has been a
controversial topic in India since the 1970s.
o Rangarajan Formula was the most recent poverty line to be established. This
report, however, did not satisfy the critics. This report was also rejected by the new

https://t.me/prelimbits
69

NDA government.
o To establish the poverty level, the NDA government formed a 14-member task
force led by Arvind Panagariya, vice-chairman of the NITI Aayog, to make
proposals for a realistic poverty limit.
o This task force likewise failed to reach a consensus on a poverty level after one and a
half years of effort.
o It proposed to the government in September 2016 that another panel of experts is
called to conduct this job if the poverty level was to be defined.
o This committee formally supported the poverty limit proposed by the Tendulkar
Committee.
o The NITI Aayog released a National Multidimensional Poverty Index in 2021 by
ranking states and indices on 12 parameters divided among Health, Education and
Standard of Living. The NITI Aayog pegs that nearly 25% of the Indian
population is Multidimensionally poor.
WORLD BANK POVERTY LINE
 The World Bank Poverty Line is otherwise called as the International Poverty Line.
 In 2008, it was set at consumption levels of $1.25 per day. In 2015, it was updated to
$1.90 per day which means that anyone living below that threshold level is considered to
be in extreme poverty.
 The World Bank defines poverty as having incomes between $3.20 and $5.50 per day, as
well as a multidimensional spectrum that includes access to education and basic
infrastructure.
 The global extreme poverty rate is expected to rise by 1.3 percent to 9.2 percent in 2020. If
the pandemic had not occurred, the poverty rate was expected to fall to 7.9 percent in 2020.
 The World Bank releases a biennial Poverty and shared prosperity report highlighting
the poverty scenario across the world.
 According to the 2020 report, 9.2% of the world's population was in extreme poverty
in 2017.

WHY IS DEFINING THE POVERTY LINE A CONTROVERSIAL ISSUE?


 Because this topic is not only politically sensitive but also has deeper fiscal repercussions,
most governments have put the recommendations of committees and panels on hold.
 If the poverty line is set too high, many people will be left out; on the other hand, if it
is set too low, the government's fiscal health will suffer. Finally, there is a lack of
agreement among states.
 Some states, such as Odisha and West Bengal, backed the Tendulkar Poverty Line, whilst
others, such as Delhi, Jharkhand, and Mizoram, backed the Rangarajan Poverty Line.
 As a result, no one, even NITI Aayog, wants to be the first to tally the number of poor
people in the country.

https://t.me/prelimbits
70

49. Which one of the following statements regarding the levying, collecting and
distribution of Income Tax is correct? [1999]
A. The Union levies, collects and distributes the proceeds of income tax between itself and the
states
B. The Union levies, collects and keeps all the proceeds of income tax to itself
C. The Union levies and collects the tax but all the proceeds are distributed among the states
D. Only the surcharge levied on income tax is shared between the Union and the states

Ans. A

Income tax is levied and collected by Union government or the central government and distributed
between itself and states.

INCOME TAX
 It is levied on a Hindu Undivided Family or an individual's net taxable income during a
fiscal year that begins on April 1 and ends on March 31 of the following year.
 It is calculated using an individual's or a business's net taxable income.
 The government of India has set certain slab rates, i.e., higher and lower slab rates, for
the convenience of its citizens.
 The government announced a new regime of income tax which is optional in Union
Budget 2020-2021.

Background : James Wilson (financial member of the Council of India, founder of the Economist
magazine and Standard Chartered Bank) introduced income tax in India on 24 July 1860 to
compensate the British losses during 1857‘s Sepoy mutiny. So, 24th July is celebrated as
Income Tax Day

50. The first Indian State to have its Human Development Report prepared and released by
Amartya Kumar Sen in Delhi is: [1999]
A. West Bengal
B. Kerala
C. Madhya Pradesh
D. Andhra Pradesh

Ans. C

Madhya Pradesh was the first state to have calculated Human Development Report in 1995, under
the guidance of Prof. Amartya Sen.

HUMAN DEVELOPMENT REPORT 2021-22

https://indianexpress.com/article/india/india-slips-two-places-on-hdi-as-covid-19-reverses-global-

https://t.me/prelimbits
71

gains-8139750/

According to the Human Development Report 2021-22, India‘s rank on the Human
Development Index (HDI) has slipped from 130 in 2020 to 132 in 2022, in line with a global fall
in HDI scores in the wake of the Covid-19 pandemic

 Human Development Index: India‘s HDI value stood at 0.633 in 2021, which was lower
than the world average of 0.732. In 2020, too, India recorded a decline in its HDI value
(0.642) in comparison to the pre-Covid level of 2019 (0.645).
Human Development Report

 Human Development Reports (HDRs) have been released since 1990 and have explored
different themes through the human development approach.
 It's published by the Human Development Report Office for the United Nations
Development Programme (UNDP).
 Goal: The goal is to contribute toward the expansion of opportunities, choice and freedom.
 Theme: The theme for Human Development Report 2021-22 is Uncertain Times,
Unsettled Lives: Shaping our Future in a World in Transformation.
Human Development Index

 HDI is a composite index that measures average achievement in human development


taking into account four indicators:
o Life expectancy at birth (Sustainable Development Goal 3),
o Expected years of schooling (Sustainable Development Goal 4.3),
o Mean years of schooling (Sustainable Development Goal 4.4),
o Gross national income (GNI) (Sustainable Development Goal 8.5).

51. Question 14: Which one of the following is the objective of National Renewal Fund?
[1999]
A. To safeguard the interests of workers who may be affected by technological upgradation of
industry or closure of sick units
B. To develop the core sector of the economy
C. For the development of infrastructure such as energy, transport communications and
irrigation
D. For human resource development such as full literacy, employment population control,
housing and drinking water

Ans. A

 The concept of the National Renewal Fund was announced by the Government as a part of
the New Industrial Policy, 1991. The Government established the National Renewal Fund
(NRF) by a Government of India resolution on 3rd February , 1992.
 The main objective of the National Renewal Fund was to provide a social safety net to the
workers who are likely to be affected by technological up - gradation and
modernisation in the Indian industry.

https://t.me/prelimbits
72

 It was abolished later in 2001 as it did not serve adequately the purpose of retraining and
rehabilitation

52. The Employment Assurance Scheme envisages financial assistance to rural areas for
guaranteeing employment to at least: [1999]
A. 50 percent of the men and women seeking jobs in rural areas
B. 50 percent of the men seeking jobs in rural areas
C. one man and one woman in a rural family living below the poverty line
D. one person in a rural landless household living below the poverty line

Ans. C

EMPLOYMENT ASSURANCE SCHEME


 The EAS was introduced where the Revamped Public Distribution System was in operation
the rural districts and blocks in 1993 which were drought prone areas, desert, tribal and
hilly regions.
 In 1994, the scheme was extended from 1778 blocks to further 409 blocks which were
mostly tribal regions. Later in 1995, the scheme was extended to 256 blocks in the states of
Uttar Pradesh, Bihar, Assam and Jammu and Kashmir.
 In 1996, Jawahar Rozgar Yojana was merged with Employment Assurance Scheme and
presently the scheme is available throughout the country without any special conditions
through a restructured program started in the year 2000.
 In 2006, the scheme was totally merged with Mahatma Gandhi National
Rural Employment Guarantee Act.
Objective and Work

 Primary objective of the scheme was to provide employment to the poor during
the lean agricultural months where they cannot find work and earn from manual
labor and the secondary objective of this scheme is to provide better infrastructure and
community assets for creating sustained employment for the livelihood.

Eligibility Norms

 Men and women between 18- 60 years of age living in villages were covered under the
scheme on minimum 100 days employment for two adults from a family.
 The eligible persons had to get registered with Gram Panchayats and have to obtain an
identity/family card.

53. Regional disparities in India are high and have been rising in recent years because:
1. There is persistent investment over time only in select locates.
2. Some areas are agro-climatically less conducive to development.
3. Some areas continue to face little or no agrarian transformation and the consequent lack of
social and economic opportunities.

https://t.me/prelimbits
73

4. Some areas have faced continuous political instability.

Which of the above statements are correct?

A. 1, 2 and 3
B. 1, 2 and 4
C. 1, 3 and 4
D. 2, 3 and 4

Ans. A

 The most appropriate answer is (a). Investment, weather conditions and agricultural
transformation are the best indicators of agricultural development. Political stability plays
a very small role in agricultural development.

54. Indian farmers are unhappy over the introduction of “Terminator Seed Technology”
because the seeds produced by this technology are expected to: [1999]
A. show poor germination
B. from low-yielding plants despite the high quality
C. give rise to sexually sterile plants
D. give rise to plants incapable of forming viable seeds

Ans. C

TERMINATOR GENE TECHNOLOGY


 Terminator gene technology refers to plants that have been genetically modified to render
sterile seeds at harvest, it is also called Genetic Use Restriction Technology or GURT.
 Terminator technology was developed by the multinational seed / agrochemical
industry and the united-states govt. to prevent farmers from saving and re-planting
harvested seed.
 The terminator gene technology, or genetic use restriction technology (GURT), is the
genetic modification of plants to make them produce sterile seeds in
second generation which is also famous as a ―suicide seeds‖.
 Terminator alters the expression of certain genes in plants so that plants terminate their
reproductive switch, about the embryo and make themselves sterile. Such plants then
produce seed that cannot germinate.
 This technology was patented by U.S. Department of Agriculture and the seed company,
Delta and Pine Land Company – a subsidiary of the seeds and agrochemical multinational
Monsanto/American Home products. During 2002 Monsanto acquired Delta and Pine
land (DPL).

55. Small-scale industries are, in most cases, not as efficient and competitive as the
large-scale ones. Yet the Government provides preferential treatment and
reservations in a range of products to the small firms because small-scale industries:
1. provide higher employment on a per unit capital deployment basis

https://t.me/prelimbits
74

2. promote a regional dispersion of industries and economical activities


3. have performed better in export of manufactured products than the large scale ones
4. provide jobs to low-skill workers, who otherwise may not find employment avenues
elsewhere

Which of the above statements are correct?

A. 1 and 4
B. 1 and 2
C. 2 and 3
D. 3 and 4

Ans. B

 Earlier industries that manufactured goods, produced goods and provided services on a
small scale or micro-scale were granted Small Scale Industries (SSI) registration.
Subsequently, the government passed the MSME (Micro, Small and Medium Enterprises)
Act in 2006. The small and micro industries came under the MSME Act. Thus, both SSI
and MSME are the same. The government broadened the scope of the SSI and termed it
as MSME.
 On 9 May 2007, subsequent to an amendment of the Government of India (Allocation of
Business) Rules, 1961, the Ministry of Small Scale Industries and the Ministry of Agro
and Rural Industries were merged to form the Ministry of Micro, Small and Medium
Enterprises. Thus, the SSIs are included under the Ministry of MSME now.

MICRO, SMALL AND MEDIUM ENTERPRISE (SMALL-SCALE INDUSTRIES)


 Essentially the small scale industries are generally comprised of those industries which
manufacture, produce and render services with the help of small machines and less
manpower. These enterprises must fall under the guidelines, set by the Government of
India.
 The SSI‘s are the lifeline of the economy, especially in developing countries like India.
These industries are generally labour-intensive, and hence they play an important role in the
creation of employment. SSI‘s are a crucial sector of the economy both from a financial
and social point of view, as they help with the per capita income and resource utilisation
in the economy.

Characteristics of SSI

 Ownership : SSI‘s generally are under single ownership. So it can either be a sole
proprietorship or sometimes a partnership.
 Management : Generally, both the management and the control is with the owner/owners.
Hence the owner is actively involved in the day-to-day activities of the business.
 Labor Intensive : SSI‘s dependence on technology is pretty limited. Hence they tend to use
labour and manpower for their production activities.
 Flexibility : SSI‘s are more adaptable to their changing business environment. So in case of

https://t.me/prelimbits
75

amendments or unexpected developments, they are flexible enough to adapt and carry on,
unlike large industries.
 Limited Reach : Small scale industries have a restricted zone of operations. Hence, they
can meet their local and regional demand.
 Resources Utilisation : They use local and readily available resources which helps the
economy fully utilise natural resources with minimum wastage.

Role in the Indian Economy

 Employment : SSI‘s are a major source of employment for developing countries like India.
Because of the limited technology and resource availability, they tend to use labour and
manpower for their production activities.
 Total Production : These enterprises account for almost 40% of the total production of
goods and services in India. They are one of the main reasons for the growth and
strengthening of the economy.
 Export Contribution : India‘s export industry majorly relies on these small industries
for their growth and development. Nearly half of the goods that are exported from
India are manufactured or produced by these industries.
 Public Welfare : These industries have an opportunity to earn wealth and create
employment. SSIs are also important for the social growth and development of our country.
 Seedbed for Large Scale Industries : SSI acts as the seedbed for Large Scale Industries
(LSI) as it provides conducive conditions for the development and growth of entrepreneurs.
Small enterprises require low investment and simple technology and use local resources to
meet local demands through personal contacts. Thus, it creates scope for the growth and
development of LSI.

Objectives of SSI

 The objectives of the small scale industries are:


 To create more employment opportunities.
 To help develop the rural and less developed regions of the economy.
 To reduce regional imbalances.
 To ensure optimum utilisation of unexploited resources of the country.
 To improve the standard of living of people.
 To ensure equal distribution of income and wealth.
 To solve the unemployment problem.
 To attain self-reliance.
 To adopt the latest technology aimed at producing better quality products at lower costs.

Examples and Ideas of SSIs in India

Bakeries Agarbatti making Chocolate making


School stationeries Chalk making Water bottles manufacturing
Water bottles Biodiesel production Toothprick making
Leather belt Rice mill Xerox and printing
Small toys Toys making Pickle manufacturing industry
https://t.me/prelimbits
76

Paper Bags Honey processing Incense stick manufacturing


Photography Slippers making industry
Beauty parlours Detergent powder making Paper plate manufacturing
Spinning and weaving industry Spices making industry
Coconut oil making Candle manufacturing
Cashew nut processing
Clay products

56. From the balance sheet of a company, it is possible to: [1999]


A. judge the extent of profitability of this company
B. assess the profitability and size of the company
C. determine the size and composition of the assets and liabilities of the company
D. determine the market share, debts and assets of the company

Ans. C

BALANCE SHEET
In financial accounting, a balance sheet or statement of financial position is a summary of the
financial status of an organisation which can be a sole proprietorship, a business partnership or
a company. Assets, liabilities and ownership of equity are listed as on a specific date, which is
normally the end of the financial year. A balance sheet is the ―snapshot of a company‘s financial
condition‖.

TWIN BALANCE SHEET PROBLEM & NPA


 Any asset which stops giving returns to its investors for a specified period [mostly 90d]
of time is known as Non-Performing Asset (NPA).
 This led the problem of twin Balance Sheet.

 TBS deals with Two Balance Sheet Problems. One with Indian companies and the other
with Indian Banks.

o Overleveraged companies – Debt accumulation on companies is very high and


thus they are unable to pay interest payments on loans. Note: 40% of corporate
debt is owed by companies who are not earning enough to pay back their interest
payments. In technical terms, this means that they have an interest coverage ratio
less than 1.

o Bad-loan-encumbered-banks – Non Performing Assets (NPA) of the banks is


around 5% for the total banking system of India. Largest NPA is in Public Sector
Banks
 As companies fail to pay back principal or interest, banks are also in trouble.

Origin of TBS or NPA


 In the period from 2004 to 2009, there was a huge growth in the economy, which led to
firms taking bank loans very aggressively.

https://t.me/prelimbits
77

 Most of the investment was in infrastructure sectors like roads, power, aviation, steel

 Laxity in lending norms by the banks, without analysing the financial health of the
companies and their credit ratings

 Global Financial Crisis (2007-08), banning of mining projects, delay in environment


permit, led to a rise in prices of raw materials and a big gap in demand and supply
thereby affecting the power, steel, and iron industries. This affected the capacity of the
companies to repay the loans to banks which resulted in Non-Performing Assets
(NPA).

57. Match List-I with List-II and select the correct answer using the codes given the lists:
[1999]

Code
A. A-2; B-1; C-4; D-3
B. A-2; B-1; C-3; D-4
C. A-1; B-2; C-4; D-3
D. A-1; B-2; C-3; D-4
Ans. A

58. Consider the following statements: [1999]


Industrial development in India, to an extent, is constrained by:
1. lack of adequate entrepreneurship and leadership in business
2. lack of savings to invest
3. lack of technology, skills and infrastructure
4. limited purchasing power among the larger masses
Which of the above statements are correct?
A. 1, 2 and 3
B. 1, 3 and 4

https://t.me/prelimbits
78

C. 2, 3 and 4
D. 1, 2 and 4
Ans. C
Industrial development in India is constrained by luck of savings to invest, lack of technology, skills
and infrastructure and limited purchasing capacity among the larger masses.

59. Tourism industry in India is quite small compared to many other countries in terms of
India‟s potential size. Which one of the following statements is correct in this regard?
[1999]
A. Distances in India are too far apart and luxury hotels are too expensive for western tourists.
B. For most of months India is too hot for western tourists to feel comfortable.
C. Most of the picturesque resorts in India such as in the North East and Kashmir are, for all
practical purposes, out of bounds.
D. In India, the infrastructure required for attracting tourists is inadequate.
Ans. D

CHALLENGES FACED BY THE TOURISM SECTOR IN INDIA


 Awareness:
o Despite promotional campaigns by the Government, the awareness regarding India
as a tourist destination remains low.
o Even among domestic tourists, the choice is limited to few popular destinations
which remain overcrowded, while many other potential places receive low
footfalls of tourists.
o The information portals and centres are poorly managed. There is lack of
promotional campaigns in foreign countries.
o The absence of online branding campaigns fail to provide information to attract
tourists.
 Infrastructure and Safety
o Many popular destinations lack air connectivity, especially in the hilly regions.
Moreover, there is lack of proper hygienic facilities in may places. Lack of
cleanliness is off-putting to many tourists.
o In addition there are safety concerns especially among foreign visitors because of
few cases of harassment.
o Poor experience of some tourists leads to bad word-of-mouth information impacting
perception of potential tourists.
 Communication
o Many tourists face communication problem while in India. This makes them
dependent on tourist guides or travel operators to curate their travel in India.
 Lack of Skilled Manpower
o There is dearth of skilled manpower especially multi-lingual tour guides or hotel staff.
o The sector is dominated by small unorganized players who can‘t spend on skilling
their employees or sensitising them to cultural values of the foreign tourists. This
impacts tourist experience.

https://t.me/prelimbits
79

 Visa Process
o The Government had started the e-visa process (online) which has led to increase in
foreign tourists. However, the visa-on-arrival facility is limited to very few
countries, limiting foreign tourists.
 Currency Fluctuations
o Another issue is the fluctuations in the currency exchange rates. The inability to
know the value of a currency means that long-range tourism prices are especially
hard to predict and the fallout from this monetary instability is already impacting
multiple tourism support systems.
What steps have been taken for the development of Tourism Sector in India?
 Infrastructure
o The Government has been increasing investments in strengthening of the country‘s
road and rail networks and promoting port development is a significant driver for the
growth of the Tourism sector.
o The Adarsh Station Scheme is helping modernize railway stations, while the
Regional Connectivity Scheme – UDAN (Ude Desh ka Aam Nagrik), is helping make
air travel more economical and widespread to hitherto unserved routes.
o The Swadesh Darshan and PRASHAD schemes aim to stimulate growth in niche
tourism segments such as religious, heritage, wellness, medical, adventure, MICE,
wildlife etc.
o Under the Swadesh Darshan Scheme, the Government has launched several
theme based circuits like Buddhist circuit which covers destinations associated with
the life of Lord Buddha.
 Promotional Campaign
o Promotional activities such as the Incredible India 2.0 campaign focuses on niche
tourism products including yoga, wellness, luxury, cuisine wildlife among others.
o ―Find the Incredible You‖ Campaign focuses on the promotion of niche tourism
products of the Country on digital and social media.
 Information Helpline
o The government has introduced the concept of e-tourist and e-medical visas which
has helped increase inbound tourists to the country. Additional initiatives such
as Atithi Devo Bhava, a 24×7 multi-lingual Tourist Helpline, among others have
helped improve the safety and security of tourists. On a pilot basis, an ‗Incredible
India Helpline‘ has been set up to guide the tourists.
 Safety
o The Ministry of Tourism has adopted a code of conduct for safe tourism, which
contains a set of guidelines to encourage tourism activities to be undertaken with
respect to basic rights like dignity, and safety of both tourists and local residents, in
particular women and children.
 Investment
o The government allows 100% Foreign Direct Investment in the Travel and Tourism
sector through the automatic route to increase investments across the sector.
o More recently, the GST rate cut on hotel room tariffs across the board has been a

https://t.me/prelimbits
80

positive move for the industry and is expected to boost the sector‘s competitiveness
globally.
 Cleanliness and Hygiene
o Major cleanliness campaign has been launched under the Swachh Bharat movement
for protecting and preserving the sanctity of monuments of national heritage.
o The Ministry of Tourism has also launched awareness campaign to ensure cleanliness
of surroundings and help create a Swachh Bharat, Swachh Smarak.
 Assistance to States
o Financial assistance to states, including places of religious importance, for various
tourism projects in consultation with them subject to availability of funds, inter-se
priority, liquidation of pending utilisation certificates and adherence to the scheme
guidelines.
 Digital Database
o In September 2021, the Government launched NIDHI 2.0 (National Integrated
Database of Hospitality Industry), a scheme which will maintain a hospitality
database comprising accommodation units, travel agents, tour operators and others.
NIDHI 2.0 will facilitate digitalisation of the tourism sector by encouraging hotels
to register themselves on the platform.
 Skilling
o The Ministry of Tourism has introduced the Incredible India Tourist Facilitator (IITF)
and Incredible India Tourist Guide (IITG) Certification Programme to create an
online learning platform of well-trained tourist facilitators and guides across the
country.
o The Ministry of Tourism had launched an initiative called SAATHI (System for
Assessment, Awareness & Training for Hospitality Industry) by partnering with the
Quality Council of India (QCI) in October 2020. The initiative was focused on effective
implementation of guidelines/SOPs issued with reference to COVID-19 for safe
operations of hotels, restaurants, and other units.

60. Match List-I with List-II and select the correct answer using the codes given below the
lists: [1999]

Codes
A. A – 1; B – 4; C – 2; D – 3
B. A – 1; B – 4; C – 3; D – 2
C. A – 4; B – 1; C – 2; D – 3
D. A – 4; B – 1; C – 3; D – 2
https://t.me/prelimbits
81

Ans. D
 Atlas cycle company is situated at Sonepat in Haryana.
 Bharat Earth Movers Limited is in Banglore, Karnataka.
 Indian Farmers Fertilizers Co-operative Ltd is present in Kalol in Gujarat and
 National Aluminium Company Limited is located at Bhubaneswar.

61. [1999]
Assertion (A): Information technology is fast becoming a very important field of activity in
India.
Reason (R): Software is one of the major exports of the country and India has a very
strong base in hardware.
A. Both A and R are true and R is the correct explanation of A
B. Both A and R are true but R is not a correct explanation of A
C. A is true but R is false
D. A is false but R is true
Ans. C

ASSERTION
 Information technology (IT) is an example of a general-purpose technology that has the
potential to play an important role in economic growth, as well as other dimensions of
economic and social development.
 Indian IT industry set its primary focus on back-end support services for delivering
cost benefits to customers. It has become very important field of activity in India.

REASON
 The Indian software industry consists of a large and growing number of firms and one of
the world‘s successful information technology industries. India has its mark in developing
software products like system software, enterprise applications. Software is one of the major
exports of India.
 But much of India‘s hardware industry consists of assembly tasks, almost entirely for
the domestic market, rather than for export.
 Despite India‘s emphasis on import-substituting industrialization, it has not developed
robust, world-class manufacturing industry, and this includes IT hardware.

62. The farmers are provided credit from a number of sources for their short and long
term needs. The main sources of credit to the farmers include: [1999]
A. the Primary Agricultural Cooperative Societies, commercial banks, RRBs and private money
lenders
B. the NABARD, RBI, commercial banks and private money lenders
C. the District Central Cooperative Banks (DCCB), the lead banks, IRDP and JRY
D. the Large Scale Multi-purpose Adivasis Programme, DCCB, IFFCO and commercial banks
Ans. A

https://t.me/prelimbits
82

PRIMARY AGRICULTURAL CREDIT SOCIETIES (PACS)

 PACS are the ground-level cooperative credit institutions that provide short-term,
and medium-term agricultural loans to the farmers for the various agricultural and
farming activities.
 It works at the grassroots gram Panchayat and village level.
 The first Primary Agricultural Credit Society (PACS) was formed in the year 1904.
 The PACS functioning at the base of the co-operative banking system constitute the
major retail outlets of short term and medium term credit to the rural sector.
 Objectives →
o To raise capital for the purpose of making loans and supporting members'
essential activities.
o To collect deposits from members with the goal of improving their savings habit.
o To supply agricultural inputs and services to members at reasonable prices,
o To arrange for the supply and development of improved breeds of livestock for
members.
o To make all necessary arrangements for improving irrigation on land owned by
members.
o To encourage various income-generating activities through supply of necessary
inputs and services.
 Significance of PACS →
o They are multifunctional organizations that provide a variety of services such as
banking, on-site supplies, marketing produce, and consumer goods trading.
o They function as mini-banks to provide finance, as well as counters to provide
agricultural inputs and consumer goods.
o These societies also provide warehousing services to farmers in order to preserve
and store their food grains.
o PACS account for 41% (3.01 crore farmers) of the Kisan Credit Card (KCC) loans
given by all entities in the country, and 95% of these KCC loans (2.95 crore farmers)
through PACS are to small and marginal farmers.
 Aim of Digitizing PACS →
o Recently, the Union Budget 2023 has announced Rs 2,516 crore for digitization of
63,000 Primary Agricultural Credit Societies (PACS) over the next five years.
o It aims at bringing greater transparency and accountability in their
operations and enabling them to diversify their business and undertake more
activities.
o It aims to help PACS become a nodal centre for providing various services such
as Direct Benefit Transfer (DBT), Interest Subvention Scheme (ISS), Crop
Insurance Scheme (PMFBY), and inputs like fertilizers and seeds.

REGIONAL RURAL BANKS (RRB)


 Based on M.Narasimham‘s Committee on Financial Inclusion in 1970s
 Setup under the provisions of RRB act 1976 & its amendment in 2015.
 Stakeholders: Central government + State Government + Sponsor Bank = 50:15:35

https://t.me/prelimbits
83

 Subjected to CRR, SLR norms but RBI could prescribe separate norms.
 PSL: 75%.
 Main objectives of RRBs are →
o To Provide Credit and other facilities to the small and marginal farmers,
agricultural labourers, artisans and small entrepreneurs in rural areas. Their loan
interest rates can‘t be more than prevailing lending rates of Cooperative Banks in the
area.
o To check the outflow of rural deposits to urban areas and reduce regional imbalances
and increase rural employment generation
 After the reforms in the 1990s, the government in 2005-06 initiated a consolidation
program that resulted in the number of RRBs declining from 196 in 2005 to 43 in FY21,
and 30 of the 43 RRBs reported net profits.
 Ultimate Regulator: RBI but immediate regulator NABARD.
NATIONAL BANK FOR AGRICULTURE AND RURAL DEVELOPMENT (NABARD)
 NABARD is the country's premier financial institution for agricultural finance and rural
development.
 It was formed in July 1982 by combining the Reserve Bank of India's Agriculture Credit
Department, Rural Planning and Credit Cell, and the entire Agriculture Refinance and
Development Corporation.
 It was established to meet the credit needs of agriculture and rural development.
 NABARD was envisioned as the national apex institution for the entire rural credit system,
providing supplemental funding to all rural credit institutions and
coordinating their operations. [Not directly to Farmers]

Historical Background

 The importance of institutional credit in boosting the rural economy was obvious to the
Government of India from the beginning of its planning process.
 As a result, at the request of the Government of India, the Reserve Bank of India (RBI)
formed a Committee to Review the Arrangements for Institutional Credit for
Agriculture and Rural Development (CRAFICARD) to investigate these critical issues.
 The Committee was established on March 30, 1979, under the chairmanship of B.
Sivaraman, a former member of the Government of India's Planning Commission.
 The interim report of the Committee, submitted on November 28, 1979, outlined the need
for a new organizational device to provide undivided attention, forceful direction, and
pointed focus to credit-related issues related to rural development.
 Its recommendation was to establish a one-of-a-kind development financial institution to
address these aspirations, and the formation of National Bank for Agriculture and Rural
Development (NABARD) was approved by Parliament throughAct 61 of 1981.
 On July 12, 1982, NABARD was established by transferring the agricultural credit functions
of the RBI and the refinance functions of the then Agricultural Refinance and

https://t.me/prelimbits
84

Development Corporation (ARDC).


 On November 5, 1982, late Prime Minister Smt. Indira Gandhi dedicated it to the service of
the nation.
 It was founded with an initial capital of Rs.100 crore and had a paid-up capital of Rs.14,080
crore as of March 31, 2020.
 NABARD today is fully owned by the Government of Indiaas a result of a change in the
composition of share capital between the Government of India and the Reserve Bank of
India.

NABARD – Functions

 Financial Functions
o NABARD provides financial assistance to the farm sector through refinancing for a
variety of agriculture and allied activities such as minor irrigation, plantation and
horticulture, land development, farm mechanization, and animal husbandry.
o Commercial banks, state cooperative banks, regional rural banks, and state land
development banks are eligible for refinancing.
o NABARD is also empowered to extend loans and advances against the security of
stocks and promissory notes.
o Aside from refinancing, NABARD also makes short-term loans to State Co-
operative Banks and Regional Rural Banks to fund seasonal agricultural
operations, crop marketing, and agricultural input purchases.
 Developmental Functions
o NABARD engages in a variety of developmental activities, including the
development of credit plans, the establishment of institutions, and the promotion of
research and technology.
o It also coordinates rural credit agencies and develops expertise to address
agricultural and rural issues.
o NABARD has a Research and Development Fund (RDF) to assist and promote
agricultural and rural development research, including the provision of research and
training facilities.
o NABARD's Central Board is also empowered to establish a "Reserve Fund" or any
other fund it deems appropriate.
 Supervisory Functions
o The Banking Regulation Act of 1949 gave NABARD the authority to inspect the
operations of Regional Rural Banks and Co-operative Banks.
o Before the RBI will grant permission to open a new branch, it must receive its
recommendation from NABARD.
o Governance

NABARD - Governance

 A Board of Directors governs NABARD's operations. In accordance with the NABARD Act,
the Board of Directors is appointed by the Government of India.

https://t.me/prelimbits
85

 The Chairperson and other directors (except those elected by shareholders and Central
Government officials) will be appointed by the Central Government in consultation with the
RBI.
 The Board of Directors may appoint an Executive Committee comprised of the prescribed
number of directors (referred to as Executive Directors).
 The Executive Committee shall perform such functions as the Board may prescribe or
delegate to it.
 The NABARD (Amendment Bill) 2017 which was passed in 2018 allowed the Union
Government to increase NABARD's authorized capital from Rs. 5,000 crore to Rs. 30,000
crore.

63. Since 1980, the share of the tertiary sector in the total GDP of India has: [1999]
A. shown an increasing trend
B. shown a decreasing trend
C. remained constant
D. been fluctuating
Ans. A

SECTORS IN THE ECONOMY


 Sectors are groups of economic activity classified according to certain characteristics.
 On the basis of ownership, labour conditions, and the nature of the operations, the Indian
economy can be divided into numerous sectors.
 But the three main sectors of the Indian Economy are: the Primary sector, the Secondary
sector, and the Tertiary sector are discussed below.
 During the early stages of civilization, the primary sector accounted for all economic
activity.
 People's demand for other items grew as a result of the surplus food production, resulting
in the growth of the secondary sector.
 During the nineteenth century's Industrial Revolution, the secondary sector expanded its
significance.
 To facilitate industrial activities, a support system was required. Certain industries, such
as transportation and finance, were critical in sustaining industrial activity.
 On the Basis of Activity Nature, the three main sectors of the Indian Economy are the
primary sector, the secondary sector, and the tertiary sector.
Primary Sector

 The activities of the primary sector of the economy are carried out by utilising natural
resources directly.
 Agriculture, mining, fishing, forestry, dairy, and other industries fall into this category.
 It is thus named because it serves as the foundation for all other sectors.
 About 54.6 % of the total workforce in the country is still engaged in agricultural and
allied sector activities.
 Agriculture, together with fisheries and forestry collectively make up one-third of India's

https://t.me/prelimbits
86

GDP. The primary sector generates 18.20 % of the GDP.


 It is also known as the Agriculture and Allied Sector since agriculture, dairy, forestry, and
fishing provide the majority of the natural items we consume.
 Due to the nature of their profession, people who engage in primary activities are referred
to as red-collar employees.
 Underemployment and Covert Employment are the two main problems this industry is
dealing with.
Secondary Sector

 Secondary Sector covers industries that manufacture finished goods from natural
materials harvested in the primary sector.
 This sector includes operations such as industrial production, cotton fabric manufacture,
sugar cane production, and so on.
 As a result, rather than producing raw materials, it is the sector of a country's economy that
manufactures goods.
 This sector is often known as the industrial sector because it is involved with various types
of industries.
 Blue-collar employees are those who engage in secondary activities.
 The contribution of the industrial sector has been constantly declining since 2011-12.
Tertiary Sector/Service Sector

 The Tertiary Sector covers the services provided in the economy and not the goods or
tangible items.
 Services sector‘s significance in the Indian economy has been steady, with the sector now
accounting for over 54 % of the economy and almost four-fifths of total FDI inflows.
 The activities of this sector contribute to the growth of the primary and secondary
sectors.
 Economic activities in the tertiary sector do not produce things on their own, but they do
help or assist production.
 The sector includes goods transported by trucks or trains, as well as banking, insurance,
and finance.
 It adds value to a product in the same way that the secondary sector does.
 These sector jobs are called white-collar jobs.

Quaternary Sector

 These are specialised tertiary operations in the 'Knowledge Sector,' therefore thus
require their own classification.
 The intellectual side of the economy is the quaternary sector. It is the procedure that
allows entrepreneurs to innovate and increase the economy's service quality.
 This category includes employees who work in office buildings, elementary schools, and
university classrooms, hospitals and doctors' offices, theatres, accounting, and
brokerage firms.
 Quaternary activities, like other tertiary functions, can be outsourced.

https://t.me/prelimbits
87

Quinary Sector
 The quinary sector is the segment of the economy that makes the highest-level decisions.
 This includes the government, which is in charge of enacting legislation. It also includes the
most powerful decision-makers in industry, trade, and education.
 These are services that focus on the development, reorganisation, and
interpretation of new and existing ideas, as well as data interpretation
and the use and evaluation of new technology.
 Senior business executives, government officials, research scientists, financial and
legal consultants, and other professionals in this category are often referred to as 'gold
collar' professionals.
 They represent another subdivision of the Tertiary Sector, representing special and highly
paid skills peoples.

WHY DID INDIA SHIFT FROM THE PRIMARY SECTOR TO THE SERVICES SECTOR AND NOT THE
SECONDARY SECTOR?

 A country's normal economic path is from Agrarian to Industrial to a Service Economy,


but India has jumped ahead of the curve from agrarian to service economy.
 Diversification towards services has been a notable element of India's recent prosperity,
with the services sector accounting for the majority of GDP.
 India has become a prominent services exporter thanks to its success in software and
IT-enabled services (ITeS), with its share of global services exports rising from 0.6 % in
1990 to 7.4% of India's GDP in FY22.
 Other factors for the country's quick expansion in the service industry include well-educated
and vast human resources, fluency in English, and the availability of cheap labour.
 On the other hand, low growth in the Secondary sector can be attributed to:
o The license Raj
o Restrictions on foreign investment
o Lack of measures to promote private industry
o Power Deficit
o Stringent Labour laws
o Lack of skilled labour
o Delays in Land Acquisition and environmental clearances
o Import of cheap manufactured goods etc.

64. In an open economy, the national income (Y) of the economy is: (C, I, G, X, M stand for
Consumption, Investment, Govt. Expenditure, total exports and total imports
respectively.) [2000]
A. Y = C + I + G + X
B. Y = I + G –X + M
C. Y = C + I + G + (X – M)
D. Y = C – G + I + (X – M)
Ans. C

https://t.me/prelimbits
88

Consumption (C), Investment (I), Government purchases (G), X stands for exports and M for
imports. Y = C + I + G + X – M

GDP (NATIONAL INCOME) CALCULATION METHOD → VIA EXPENDITURE


If anything is produced in India then someone must have paid money for that. So, accordingly we
can derive GDP = C + I + G + X – M

GDP Expenditure Method Description and Examples


components
(C) Consumption of final  Purchasing new car, mobiles, computer etc. Both India
goods and services made & (Imported) foreign made are counted.
 If existing house, its ‗notional rent‘ is counted (i.e. even if you
did not rent the property.)
 IGNORE purchase of second hand goods, because we are
only measuring ‗new‘ things ―MADE in India‖ in present year.
 IGNORE of new house not counted here, it‘s counted in (I)
(I) Investments  Purchase of tangible capital assets like New House, Land,
Building, Factory, Truck, Machinery.
 Purchase of intangible capital assets like IPR / Patents,
Computer Software etc.
 Purchase of raw material & intermediate goods, wages to
workers for production.
 UNSOLD inventory.
 IGNORE savings in bank, shares and bonds etc. (because it‘d
have been given to entrepreneur as ‗Capital‘ to buy above
things).
(G) Government Purchases  Salaries to employees, Procurement of computer,
stationery, fans, tube lights, vehicles etc.
 IGNORE Government‘s scholarship, subsidy etc. ‗Transfer
Payments‘. They‘re counted in ―C‖ (Private) consumption
by the respective beneficiaries.
(X-M) Export MINUS  Export is added because it means a foreigner must have
Imports bought goods/services ―MADE in India‖ so it‘s part of India‘s
GDP.
 Whereas, Import is subtracted because some Indians must
have Consumed (C) foreign products that were not ―MADE
in India‖, So if you do not subtract the ‗Import (M)‘, it will
give wrong estimation of India‘s GDP.
Re-exports
Means, something imported and then again exported. Re-
exports are export of foreign goods in the same state as previously
imported.
For example, if a machinery has been imported in to a country for
testing purpose and after necessary testing, the said machinery is
sent back. Here, the process of sending back such machinery is
called re-exports.
So, whatever (margin) the country will charge for the testing

https://t.me/prelimbits
89

purpose will be counted in its own GDP. And this margin charged is
the value addition of the country which is importing and then again
re-exporting.
Total = GDP The GDP thus arrived is called GDP at Current Market Price
When we adjust it with inflation against base year 2011 → GDP
at Constant Market Price

65. Match List I with List II and select the correct answer using the codes given below the
lists: [2000]

A. A-1; B-2; C-3; D-4


B. A-1; B-2; C- 4; D-3
C. A-2; B-1; C- 4; D-3
D. A-2; B-1; C-3; D-4
Ans. A

BUSINESS CYCLES
 Income growth in any economy occurs by increasing the level of production in the
economy, i.e., real gross national product (GNP).
 It means that development necessitates greater growth, i.e., higher levels of economic
activity.
 The government of an economy strives to maintain a higher level of economic activity by
enacting appropriate economic policies.
 However, the economy frequently fails to achieve this goal. As a result, economies fluctuate
between the best and worst levels of economic activity, referred to in economics as a Boom
and a Depression, respectively.
 They can be categorized as different stages of an economy's economic activities.
 Between boom and bust, there may be many other economic activity situations, such
as stagnation, slowdown, recession, and recovery.
 Economists refer to fluctuations in the level of economic activity between depressions
and booms as the Business Cycle or Trade Cycle, with Recession and Recovery serving as
the main intermediate stages.
 Stagnation and slowdown are also intermediate stages of the business cycle.

https://t.me/prelimbits
90

Depression

 Despite the fact that depression only visited the world economy once, in 1929, economists
have identified enough characteristics to identify it.
 The following are some of the major characteristics of depression →
o an extremely low aggregate demand in the economy causes activities to
decelerate;
o the inflation being comparatively lower;
o the employment avenues start shrinking forcing the unemployment rate to grow
fast;
o to keep the business going, production houses go for forced labor cuts or
retrenchment (to cut down production cost and be competitive in the market,), etc.
 During the depression, economic situations become so chaotic that governments have
almost no control over the economy.
 The 1929 Great Depression gave rise to ideas of strong government intervention in
the economy, such as deficit financing and monetary management, and so on.
Great Depression (1929–1939)
 The Great Depression (1929–1939) was an economic shock that impacted most countries
across the world. It was a period of economic depression that became evident after a
major fall in stock prices in the United States
 The economic contagion began around September and led to the Wall Street stock
market crash of October 24 (Black Thursday). It was the longest, deepest, and most
widespread depression of the 20th century.
 Between 1929 and 1932, worldwide gross domestic product (GDP) fell by an estimated
15%. By comparison, worldwide GDP fell by less than 1% from 2008 to 2009.

https://t.me/prelimbits
91

The unemployment rate in the U.S. during 1910–60, with the years of the Great Depression (1929–
39) highlighted
The Great Depression on the Indian Economy
 The Great Depression had a severe impact on Indian Economy as well, it affected the
economies, societies and lives of people.
 The Great depression had an immediate impact on Indian trade.
 In the 19th century, colonial India was an importer of manufactured goods and
exported agricultural goods.
 Prices in India also crashed, as the international prices plunged.
 Between 1928 and 1934, wheat prices in India fell by 50%.
 Between 1928 and 1934, the exports and imports of India, reduced by half.
 More than the urban dwellers it was the farmers and peasants who suffered the most.
 The colonial British Government refused to reduce their revenue demands, despite massive
reduction in agricultural prices.
 The worst hit in India due to the Great Depression was the peasants who were producing
goods for the world market.
Bengal – Jute Producers
 Peasants who had borrowed more money with the hope of fetching higher income and
producing more were the worst hit.
 They kept falling deeper into debts as the prices kept falling.
 The price of raw jute had collapsed by 60% due to the collapse of gunny bags exports.
 The raw jute that was grown in Bengal, was later processed in factories to manufacture
gunny bags.
Effects of Great Depression on Peasants Across India
 Peasants‘ indebtedness increased across India.
 Peasants had to sell whatever valuable they possessed.
 Peasants were forced to sell their precious metals, jewellery, mortgaged lands and savings.
Effects of Great Depression – Urban India
 Under pressure from Indian Nationalists, industries received tariff protection from the British
Government. Hence Industrial investments grew.
 People in urban India did not face the same kind of crisis faced by people in rural India.
 middle-class salaried employees, town-dwelling landowners were people who received fixed
incomes, they were in a better position as the prices kept falling and everything cost less.

https://t.me/prelimbits
92

Global Economic Recovery Due to India – John Maynard Keynes


 India became an exporter of precious metals, especially Gold, during the years of the Great
Depression.
 As per John Maynard Keynes, a world famous economist, global economic recovery
after the Great Depression was promoted by Indian Gold exports.
 These Gold exports helped the British, but it hardly had any positive effect
on the Peasants suffering in India.
 In 1931, Mahatma Gandhi launched the civil disobedience movement, it was the time when
rural India was seething with unrest.
Recovery

 To survive, an economy attempts to break out of its Low-Production Phase.


 When an economy is in a low-production phase, it may be a depression, a recession, or a
slowdown, with the latter being the worst and most common.
 Governments take many new Fiscal and Monetary measures to boost demand and
production, and eventually an economy recovers.
 The business cycle of recovery may exhibit the following major economic characteristics

o an increase in aggregate (total) demand, which must be accompanied by an
increase in the level of production;
o the production process expands and new investments become appealing;
o as demand rises, inflation rises as well, making borrowing cheaper for investors;
o with an increase in production, new employment avenues are created and
the unemployment rate begins to fall; and so on.
 With the aforementioned symptoms, people's income rises, creating new demand, and a
cycle of demand and production (supply) begins to play hand-in-hand to help the economy
recover.
 To stimulate an economy, governments typically implement tax breaks,
interest rate reductions, wage increases for their employees, and other measures.
 The incorporation of innovations by entrepreneurs and the search for new frontiers of an
enterprise play a critical role in the recovery process, provided that these activities are
initially subsidized by governments.
 With the assistance of the aforementioned measures, the Euro-American
economies recovered from the Great Depression.
 Such recoveries have occurred numerous times around the world as economies recovered
from slowing or recessionary phases.
Boom

 A strong upward fluctuation in economic activity is referred to as a boom.


 As economies try to recover from the slowdown, recession, and depression, the
measures taken by governments and the private sector may place economic activities in
such a way that the economic systems fail to digest. This is the peak period of the boom.

https://t.me/prelimbits
93

 The following are the major economic characteristics of a boom →


o an accelerated and prolonged increase in demand;
o demand peaks to levels that exceed sustainable output/production levels;
o the economy heats up and a demand-supply lag is visible;
o market forces mismatch (i.e., demand and supply disequilibrium) and tend to
create a situation in which inflation begins to rise.
o the economy may face structural issues such as a lack of investible capital, lower
savings, a falling standard of living, and the emergence of a sellers' market.
 The recovery phase is beneficial to the economy, and it progresses to the boom stage,
which is preferable. However, there are some drawbacks to the boom.
 Typically, a boom is followed by a price increase. Because a boom is characterized by a
strong upward fluctuation in an economy, the supply-side pattern of the economy begins
to lag behind the accelerated aggregate demand.
 The dilemma of recovery, on the other hand, puts every economy on the path to boom—
this was the experience in the developed world during the 1990s, particularly in the US
economy.
 The same scenario played out in India after the economy recovered from the recessionary
period of 1996–97 by 2002–03 when inflation reached nearly 8% for a few months.
 The majority of experts believed that the Indian economy was in a boom phase at the time,
and we have seen how the government has struggled to keep inflation around the 5% mark.
 By mid-2007, even the government admitted that the economy was Overheating. The
symptoms of overheating are as follows →
o there is a decline in aggregate demand as demand falls overall;
o as demand falls, the level of production (output) in the economy falls as well.
o as producers reduce their production levels, new employment opportunities are not
created—thus the employment growth rate falls;
o as demand continues to fall, producers usually begin cutting down their labor
force to adjust their overhead expenditure and the cost of production (labor-cut is
not 'forced' here, but 'voluntary')—resulting in an increase in the unemployment
rate; and
o if the government fails to rescue the economy from the phase of recession.
Recession

 This is similar to the 'depression' phase — and it is fatal for economies because it can
lead to depression if not handled with care and in a timely manner.
 The financial crises that followed the US 'sub-prime crisis' in almost the entire Euro-
American economies have essentially brought in 'severe recessionary' trends.
 Major characteristics of recession, which are similar to those of 'depression' can be
summarised as follows:
o there is a general decline in demand as economic activity slows;
o inflation remains low or shows further signs of falling;
o employment falls/unemployment rises; and

https://t.me/prelimbits
94

o industries resort to 'price cuts' to maintain their business.


 In the fiscal year 1996–97, the Indian economy was engulfed in a recessionary cycle,
owing primarily to a general decline in domestic as well as foreign demand, which was
precipitated by the South East Asian Currency Crisis of the mid-1990s.
 The entire economic reform plan in India was derailed, and the economy was only able to
recover by the end of 2001–02.
 The following are the standard treatments that a government can do to bring the economy
out of a slump:
o Direct and indirect taxes should be reduced so that consumers have higher
disposable incomes (income after paying direct tax, i.e., income tax) on the one hand
and goods become cheaper on the other, resulting in an increase in demand.
o The burden of direct taxes, particularly income tax, dividend tax, and interest tax, is
reduced in order to increase disposable income.
o Salaries and wages should be revised by the government to encourage general
consumer spending (as the Government of India did without much deliberation in
1996–97 when it implemented the recommendations of the fifth pay commission).
o Indirect taxes, such as customs duty, excise duty, and sales tax, should be reduced
so that manufactured goods reach the market at lower prices.
o The government usually follows a cheap money supply policy by lowering interest
rates across the board and liberalizing the lending procedure.
o Tax breaks for new investments in the economy are announced.
 Technical recession is defined by the World Bank and IMF →
o A technical recession occurs when an economy's GDP falls for two consecutive
quarters.
 Though agencies use data on employment, oil demand, and other factors to declare a
global recession, it is also defined by the global economic growth rate—a global growth
rate of less than 2.5 % is considered technical recession (2.5 % is the threshold growth
rate for technical global recession).
International Recession Events

 Gulf War Recession (July 1990 – March 1991): Iraq‘s invasion of Kuwait resulted in a spike
in the price of oil in 1990, which caused manufacturing trade sales to decline.
 The 9/11 Recession (March 2001 – November 2001): Various factors contributed to this,
such as the collapse of the dotcom bubble, the 9/11 attacks that lead to mild contraction of
the U.S. economy.
 The Subprime mortgage crisis/ Great Recession (2007): This period was characterized by
a general recession observed in national economies globally. It was concluded as the most
severe economic and financial meltdown since the Great Depression by the IMF.
66. Indian Human Development Report does not give for each sample village: [2000]
A. Infrastructure and Amenities Index
B. Education Related Index

https://t.me/prelimbits
95

C. Health Related Index


D. Unemployment Related Index
Ans. D

 The National Human Development Report 2001, is an attempt to map the state of human
development in India. While considering the state of human development, and the related
economic, educational and health attainments, the report analyses governance for human
development.
 To facilitate monitoring, many process indicators have been created. The Human
Development Index, Gender Disparity Index and Human Poverty Index have been calculated
for India after modifying the UNDP methodology.
67. Consider the following statements about the megacities of India: [2000]
1. Population of each megacity is more than 5 million
2. All the megacities are important sea ports
3. Megacities are either national or state capitals
Which of these statements are correct ?

A. 1, 2 and 3
B. 1 and 2
C. 2 and 3
D. 1 and 3
Ans. D

Megacities of India have more than 5 million population. But not all the major megacities are
located on the sea coast or are sea ports. Delhi, which is the national capital is not a sea port.

 A megacity is usually defined as a metropolitan area with a total population in excess of


ten million people.
 A megacity can be a single metropolitan area or two or more metropolitan areas that
converge.
 The terms conurbation, metropolis and metroplex are also applied to the latter. Megacity is
formed by the spatial merging of two or more cities along major transpiration corridors. It is
a generic term used to refer to urban coalescence.
 Initially the United Nations used the term to describe cities of 8 million or more inhabitants,
but now uses the threshold of 10 million.
 India at present, is home to five megacities
o Delhi
o Mumbai
o Kolkata
o Chennai
o Bengaluru
 Tokyo is the world‘s largest metropolitan area, while Shanghai has the world‘s largest city
proper population.

https://t.me/prelimbits
96

68. Which one of the following ports of India handles the highest tonnage of import cargo?
[2000]
A. Calcutta
B. Kandla
C. Mumbai
D. Vishakhapatnam
Ans. *

VOLUME OF TOTAL CARGO HANDLED ACROSS INDIA IN FINANCIAL YEAR 2021, BY MAJOR PORT

MAJOR SEA PORTS IN INDIA


 India has 13 major seaports (12 Government-owned and one private) and 205 notified
minor and intermediate ports that handle a huge volume of traffic. Ports play a very
central role in the development of a nation. Cost, capacity, and easiness of transportation
via ports are most effective compared to other modes available.
 About 95 percent by volume and 70 percent by value of India‘s total international trade
are carried on through maritime transportation.
 All ports in India are situated in the 9 coastal states of India namely Kerala, Karnataka,
Maharashtra, Goa, Gujarat, West Bengal, Odisha, Andhra Pradesh, and Tamil Nadu.
India‘s extended coastline forms one of the major portions of land that juts out into a water
body. Thirteen major ports in the country handle a lot of volume of container and cargo
traffic.
 On the west coast, there are the ports of Mumbai, Kandla, Mangalore, JNPT, Vadhavan,
Mormugao, and Cochin.
 The ones on the east coast are the ports at Chennai, Tuticorin, Visakhapatnam, Paradip,
Kolkata, and Ennore. The last one, Ennore is a registered public company with the

https://t.me/prelimbits
97

government owning a 68% stake.


 Mumbai is the largest natural port in India.
 As we know, Indian government has a federal structure, and according to its
constitution, maritime transport is to be administered by both the Central and the
State governments.
o While the Central Government‘s shipping ministry administers the major ports,
o Minor and intermediate ports are administered by the relevant departments or
ministries in the nine coastal states – Andhra Pradesh, Goa, Gujarat, Karnataka,
Kerala, Maharashtra, Odisha, Tamil Nadu and West Bengal.
 Major Ports are administered under the Indian Ports Act 1908 and the Major Port Trust
Act, 1963.
o Recently, the government has prepared the Draft Indian Ports Bill, 2022. The draft
Indian Ports Bill,2022 seeks to repeal and replace the existing Indian Ports Act
1908, which is more than 110 years old, becoming imperative that the Act is
revamped to reflect the present-day frameworks.
The list of important major ports in India is given below:

Zone State Port


Eastern Coast Tamil Nadu Chennai
Western Coast Kerala Kochi (Cochin)
Eastern Coast Tamil Nadu Ennore
Eastern Coast West Bengal Kolkata (Haldia)
Western Coast Gujarat Kandla
Western Coast Karnataka Mangalore
Western Coast Goa Mormugao
Western Coast Maharashtra Mumbai Port Trust
Western Coast Maharashtra Jawaharlal Nehru Port Trust (JNPT) also known
as Nhava Sheva, Navi Mumbai
Western Coast Maharashtra Vadhavan Port
Eastern Coast Odisha Paradip
Eastern Coast Tamil Nadu Tuticorin
Eastern Coast Andhra Pradesh Visakhapatnam
 Chennai Port
o Chennai Port, formerly known as Madras Port, is the second-largest container port
of India, behind Mumbai‘s Nhava Sheva (JNPT). The port is the largest one in the
Bay of Bengal.
o It is an artificial and all-weather port with wet docks.
o It is due to the existence of the port that the city of Chennai eventually became
known as the Gateway of South India.
 Kochi (Cochin) Port
o Cochin Port or Kochi Port is a major port on the Arabian Sea – Laccadive Sea –
Indian Ocean sea-route in the city of Kochi and is one of the largest ports in India. It
is also the first transshipment terminal in India.

https://t.me/prelimbits
98

o The port lies on two islands in the Lake of Kochi: Willingdon


Island and Vallarpadam, towards the Fort Kochi river mouth opening onto the
Laccadive Sea.
o Kochi Port in Kerela is the example of Tidal port .
o The port is generally called as the natural gateway for the industrial and
agricultural produce markets of South-West India.
o Exports of spices, tea, and coffee.
o It is one of the centers for shipbuilding.
 Ennore Port (Kamarajar Port Ltd.)
o It is the 12th major port of India and the first port in India which is a public
company.
o It is located on the Coromandel Coast about 24 km north of the Chennai Port.
o Trades: Iron Ore, Coal, petroleum products and chemicals.

 Kolkata Port
o Port of Kolkata or Kolkata Port, officially known as Syama Prasad Mookerjee Port
Trust, is the only riverine major port of India, located in the city of Kolkata, West
Bengal, around 203 kilometres from the sea.
o It is the oldest operating port in India and was constructed by the British East India
Company.
o Kolkata is a freshwater port with no variation in salinity.
o Known as Diamond Harbour
o Known for twin dock systems viz. Kolkata Dock on the eastern bank and Haldia
Dock on the western bank of river Hooghly
o Trade: Jute, tea, Coal, Steel
 Kandla Port
o Deendayal Port Trust, located in town of Kandla, is a seaport and town in Kutch
district of Gujarat state in Western India, near the city of Gandhidham.
o Located on the Gulf of Kutch, it is one of major ports on the west coast.
o Known as Tidal Port
o It was constructed after partition when Karachi Port was transferred to Pakistan.

https://t.me/prelimbits
99

o It also relieves the congestion of Mumbai Port


o Largest port by volume of cargo handled.
o It has been acknowledged as Trade Free Zone.
 Mangalore Port
o It is deep water, all weathered port.
o Deals with the iron ore exports
o It is the only major port of the coastal state of Karnataka.
 Mormugao Port
o Situated on the estuaries of the river Juari
o It is a natural harbour
o It was awarded the status of a major port in the year 1963.
o It is a leading iron ore exporting port in India.
 Mumbai Port Trust
o Largest Natural Port and harbor in India
o Earlier, this port location was used by the navies of Shivaji.
o This port has 3 enclosed wet docks:
 Prince‘s Dock
 Victoria Dock
 Indira Dock
o The busiest Port in India
o Jawahar Dweep is an island in the harbor, for Crude and petroleum products
handling.
 Jawaharlal Nehru Port Trust (JNPT)
o Jawaharlal Nehru Port Trust (JNPT) also known as Nhava Sheva (Navi Mumbai) is
the first major port of the country to become a 100% Landlord port of India
having all berths being operated on the PPP model.
 The name Nhava Sheva is given because of the names of two villages that
existed in that area.
o Largest Artificial Port and leading container port in India.
o It was commissioned in 1989 and in three decades of its operations, JNP has
transformed from a bulk- cargo terminal to become the premier container port in
the country.
o It is located on the eastern shore of Mumbai harbor off Elephanta Island and can
be accessed via Thane Creek.
o This port is the terminal point of the Western Dedicated Freight Corridor of Indian
Railways.
o With its state-of-the-art facilities JNP meets all the international standards,
user-friendly atmosphere, and excellent connectivity by rail and road to the
hinterland.
o It is presently handling 9000 twenty-foot equivalent units (TEUs) capacity
vessels and with the up-gradation, it can handle 12200 TEUs capacity vessels.

https://t.me/prelimbits
100

o Trade: Textiles, sporting goods, carpets, pharmaceuticals, chemicals, etc.


Landlord Port
 In this model, the publicly governed port authority acts as a regulatory body and as a
landlord, while private companies carry out port operations—mainly cargo-handling
activities.
 Here, the port authority maintains ownership of the port while the infrastructure is
leased to private firms that provide and maintain their own superstructure and install their
own equipment to handle cargo.
 In return, the landlord port gets a share of the revenue from the private entity.
Service Port Model
 In service ports, the port authority does the administration and operation of port
activities.
 The port operation includes providing navigational services, warehouse facilities, cranes, and
skilled employees/laborers. the construction of infrastructure, superstructure, and providing
employees, becomes the responsibility of the port authority.
 Even if the port authority act in the public interest full ownership of the port remains
with the state or the government.
 Vadhavan Port
o This will be 13th major port in India.
o With the development of this port, India will become one of the countries in the
top-10 container ports in the world.
o A special purpose vehicle (SPV) will be formed with Jawaharlal Nehru Port Trust
(JNPT) as the lead partner, with equity participation equal to or more than 50% to
implement the project.
o The port will be developed on the landlord model.
o Vadhavan port has been planned by the JNPT as an ‗All Weather, All Cargo‘
satellite port to enhance capabilities in handling deep draft ships and larger vessels.
 Paradip Port
o First Major Port commissioned after Independence.
o Located at the confluence of the Mahanadi river and the Bay of Bengal.
o Deals with the export of iron and aluminum and Iron ore is exported to Japan in
huge quantities.
 Tuticorin Port
o This port has been renamed as V.O.Chidambaranar Port.
o It is an artificial port located in the Gulf of Mannar.
o It is famous for pearl fishery in the Bay of Bengal and is thus also known as the
pearl city.
o Trade: coal, salt, petroleum products, and fertilizers.
 Visakhapatnam Port
o This port is a natural harbor and also is the 2nd largest port by volume of cargo
handled.
o Port is located midway between the Chennai Port and Kolkata Port.
o The deepest port of India deals with the export of iron ore to Japan. Amenities for

https://t.me/prelimbits
101

building and fixing of ships are available.


o Trade: Iron Ore, Coal, Alumina, and oil.
69. A rise in „SENSEX‟ means: [2000]
A. a rise in prices of shares of all companies registered with Bombay Stock Exchange
B. a rise in prices of shares of all companies registered with National Stock Exchange
C. an overall rise in prices of shares of group up companies registered with Bombay Stock
Exchange
D. a rise in prices of shares of all companies belonging to a group of companies registered
with Bombay Stock Exchange
Ans. C

Increase in SENSEX reflects the overall mood of the economy. A rise in it means that investors
and FIIs are positive about the growth of Indian economy and expect that it will be sustained in
future.

STOCK EXCHANGES / SECONDARY MARKET


Shares are issued through IPO @Primary market. Then, they can be resold at secondary market,
commonly known as Share market or Stock Exchange or Bourses.
 World‘s Oldest: Amsterdam Stock exchange, Netherlands (1602)
 Asia‘s Oldest: Bombay Stock Exchange (BSE: 1875)
 India‘s stock exchanges chronology: BSE → A‘bad → Kolkata → NSE (early 90s)
 Just like the Banks have Core Banking Solutions for e-banking, Stock exchanges also have
their electronic platforms for trading. E.g. BOLT (BSE's On-line Trading System) and NEAT
(National Exchange for Automated Trading). They run using internet facility from VSAT
(Very Small Aperture Terminal) Satellite.
Bombay Stock Exchange

 Indian stock exchange located on Dalal Street in Mumbai.


 Established in 1875 by cotton merchant Premchand Roychand, a Rajasthani Jain
businessman, it is the oldest stock exchange in Asia, and also the tenth oldest in the
world.
 The BSE is the 8th largest stock exchange with an overall market capitalisation of more
than ₹276.713 lakh crore, as of January 2022.
 Only in 1995 did BSE switch to electronic trading after following a paper trading pattern
since 1875.
 BSE‘s Stock Index – SENSEX gives top 30 stock index
 Recognized stock exchange in 1957
 BSE promotes trading in debt instruments, mutual funds and currencies
National Stock Exchange of India

 NSE was established in 1992 & recognized as a stock exchange in 1993

https://t.me/prelimbits
102

 It was the first exchange in India to provide fully computerized electronic trading. NSE is
one of the pioneers in technology and innovation which ensured the high-end performance
of its systems.

 HQ : Mumbai

 It is the world‘s largest derivatives exchange in 2021 by number of contracts traded.

 Its automated system makes it more reliable and efficient in comparison to the Bombay
Stock Exchange (BSE).
 Total market capitalization of more than US$3.4 trillion, making it the world's 10th-largest
stock exchange as of August 2021.
 NSE's flagship index, the NIFTY 50, a 50 stock index is used extensively by investors in India
and around the world as a barometer of the Indian capital market.
 NSE promotes trading in equity, debts and currency derivatives
Secondary Market: Significance

 It reduce Cost of Transactions . You don't have to search around for buyers/sellers manually.
 Providing Liquidity to Financial Assets.
 Facilitating Price Discovery of shares / bonds
 Investor sells securities in secondary market→ ₹₹ could be re-invested to a new company‘s
IPO in primary market → Contributes to Economic Growth
NOTABLE INDICES
 SENSEX : Sensitive Index. It‘s the weighted average of Free Float Market Capitalization
(FFMC) of 30 companies, selected by BSE‘s ―Index Cell‖.

 NIFTY : NSE index of 50 companies.

 Nikkei : Tokyo Stock Exchange index of 225 companies

SENSEX – when does it go up Goes down when


RBI‘s Soft /Easy Monetary Policy → cheap loan Tight monetary policy
& credit cards → consumers to spend more →
more profit to company → more dividend :
investor thinks to buy more shares to get
more dividend
Peace, Economic boom / prosperity, Political War, recession, political instability → Bearish
Stability market.
When govt. hikes foreign investment limits Inverse.
Merger-Acquisition, New product launched, CEO/MD arrest/FIR, Courts slapping fine,
Environmental clearance given to factory media exposing scandal…

https://t.me/prelimbits
103

70. [2000]
Assertion (A): The rate of growth of India‟s exports has shown an appreciable increase
after 1991.
Reason (R): The Govt. of India has resorted to devaluation.
A. Both A and R are true and R is the correct explanation of A
B. Both A and R are true but R is not a correct explanation of A
C. A is true but R is false
D. A is false but R is true
Ans. A [let‘s keep ans A. since it is asking appreciable increase not continuously increasing]

 Devaluation of currency is conscious decision taken by Central bank of country to lower


the external value of domestic currency. As a result of this Indian goods become cheaper
for Foreigners. Import decrease and exports increases.
RECENT EXPORT TREND

 Exports from India decreased 4.6 percent from a year earlier to USD 32.91 billion in January
2023, amid weaker global demand. Sales were down for cotton, engineering goods, iron
ore, and gems and jewelry, while exports of petroleum products, electronic goods and
chemicals continued to be higher than a year
 In recent years, India exported mostly: pearls, precious and semi-precious stones and
jewelry (16 percent of total shipments); mineral fuels, oils and waxes and bituminous
substances (12 percent); vehicles, parts and accessories (5 percent); nuclear reactors,
boilers, machinery and mechanical appliances (5 percent); pharmaceutical products (5
percent); and organic chemicals (4 percent).
 India‘s main export partners are: United States (15 percent of the total exports), United
Arab Emirates (11 percent), Hong Kong (5 percent), China (4 percent), Singapore (4 percent)
and United Kingdom (3 percent).
71. “...instil into the vast millions of workers, men and women, who actually do the job, a
sense of partnership and of cooperative performance...” The above passage relates to:
[2000]
A. Planned Development

https://t.me/prelimbits
104

B. Community Development
C. Panchayati Raj System
D. Integrated Development Programme
Ans. B

COMMUNITY DEVELOPMENT PROGRAMME (CDP) IN INDIA


 The Community Development Programme (CDP) was the first major rural development
programme launched after independence in 2nd October 1952.
 It constitutes the first organised effort at rural reconstruction. The CDP was conceived as
an instrument to transform the social and economic life of the village community as a
whole cutting across caste, religious and economic differences.
 Initially, it covered 55 projects with a wide range of programmes for developing
agriculture, animal husbandry, rural industries, education, housing, rural communication etc.
It was, however, soon realised that covering the entire country under CDP was not possible
due to shortage of funds and personnel.
 The National Extension Service (NES) was launched in October 1953 with a reduced
number of personnel and more modest financial provision, so that development work could
proceed on the basis of self-help efforts. The NES was initially launched for a period of
three years, with the provision of 4.5 lakhs, per block with stress on agriculture, animal
husbandry, rural communication and social education schemes.
 After three years, the blocks covered under NES were to be converted into CDP blocks
with the provision of 15 Iakhs for three years for more intensive development work in
agriculture and other activities such as co-operation, health and sanitation, rural industries,
housing etc.
 It was presumed that the blocks covered in the first two phases would have attained
the desired self-sufficiency and therefore, the quantum of special government grants
could be reduced. The major contribution in financial and technical assistance came
from the United States of America under a scheme of technical co-operation.
 Funds were also drawn from the central and state governments to meet the local
expenditure on the implementation of the CDP.
 The only difference between the CDP and NES was that the CDP was a more intensive
programme than the NES. Both these schemes virtually worked hand in hand. CDP was a
programme which sought to change the economic, social and cultural life
of the rural population and to transform the stagnant villages into progressive rural
communities.
Aims and Objectives of CDP →

 CDP intended to promote rural welfare through the improvement of education, public
health and sanitation, medical facilities, housing, drinking water, hospitals, community
centres and sports and cultural activities.
 To secure total development of the material and human resources in rural areas.
 To develop local leadership and self-governing institutions.

https://t.me/prelimbits
105

 To raise the living standards of the rural poor by means of rapid increase in food and
agricultural produce.
 To ensure a change in the mind set of people instilling in them a mission for higher
standards.
 It intended to create an interest among the rural people for better economic, social and
cultural life and make them satisfy their interest by self-help.
 It also aimed at encouraging community thinking and collective action.
 It intended to increase people‘s participation in developmental programmes.
72. Economic liberalisation in India started with: [2000]
A. substantial changes in industrial licensing policy
B. the convertibility of Indian rupee
C. doing away with procedural formalities for foreign direct investment
D. significant reduction in tax rates
Ans. A

For More refer : Q20

LIBERALISATION
 Liberalisation is the process or means of removing the state's control over economic
activities. It gives business enterprises more autonomy in decision-making and eliminates
government interference.
 It is used in the context of a government relaxing its previously imposed restrictions on
economic or social policies. In India, liberalization was done through deregulating the
industrial sector, financial sector reforms, tax reforms and foreign exchange reforms.
 In India, it began with the introduction of a new economic policy to tide over conditions
of the balance of payment crisis.
 Liberalization was undertaken to attain objectives like industrialization, expansion in the
role of private and foreign investment, and the introduction of a free market system.
73. Consider the following statements: [2000]
The Indian rupee is fully convertible:

1. in respect of Current Account of Balance of payment


2. in respect of Capital Account of Balance of payment
3. into gold
Which of these statements is/are correct ?

A. 1 alone
B. 3 alone
C. 1 and 2
D. 1, 2 and 3
Ans. A

https://t.me/prelimbits
106

CAPITAL ACCOUNT CONVERTIBILITY


 CAC means the freedom to convert rupee into any foreign currency (Euro, Dollar, Yen,
Renminbi etc.) and foreign currency back into rupee for capital account transactions. In
very simple terms it means, Indian‘s having the freedom to convert their local financial
assets into foreign ones at market determined exchange rate.
 CAC will lead to a free exchange of currency at a lower rate and an unrestricted
movement of capital
 Capital Account Convertibility is widely regarded as the hallmark of developed countries.
It is also seen as the major comfort factor for foreign investors since it allows them to
reconvert local currency back into their own currency and move out from India.
 After the recommendations of the S.S. Tarapore Committee (1997) on Capital Account
Convertibility, India has been moving in the direction of allowing full convertibility in
this account, but with required precautions. India is still a country of partial
convertibility (40:60) in the capital account, but inside this overall policy, enough
reforms have been made, and to certain levels of foreign exchange requirements, it is an
economy allowing full capital account convertibility. Following steps have been taken in the
direction of capital account convertibility.
CURRENT ACCOUNT CONVERTIBILITY
 Current Account Convertibility allows free inflows and outflows of foreign currency for all
purpose including resident Indians buying foreign goods and services (imports), Indians
selling foreign goods and services (exports), Indians receiving and sending remittances,
accessing foreign currency for travel, study abroad, medical tourism purpose etc.
 Current account is today fully convertible (operationalized on August 19,
1994).
 It means that the full amount of the foreign exchange required by someone for current
purposes will be made available to him at the official exchange rate and there could
be an unprohibited outflow of foreign exchange (earlier it was partially convertible).
 India was obliged to do so as per Article VIII of the IMF which prohibits any exchange
restrictions on current international transactions (keep in mind that India was under pre-
conditions of the IMF since 1991).
74. The growth rate of per capita income at current prices is higher than that of per capita
income at constant prices, because the latter takes into account the rate of : [2000]
A. growth of population
B. increase in price level
C. growth of money supply
D. increase in the wage rate
Ans. B

The growth rate of per capita income at current prices is higher than that of per capita income at
constant prices, because current prices are influenced by the effect of price

https://t.me/prelimbits
107

inflation
MARKET PRICE [OR CURRENT PRICE]
 Market Price is the current price at which an asset or service can be bought or sold. After
goods and services are manufactured, they are sold at market prices.
 In other words, the market price is the price at which customers will pay for a product
when they buy it from a retailer.
 To arrive at the market price, government taxes will be added to the factor cost, while
subsidies will be subtracted from the factor cost.
 Removing Subsidies: Subsidies are removed because subsidies recompense the factor
cost or lower the market price and the factor cost.
 Adding Taxes: Taxes are added because they are costs that raise the price, and subsidies
are reduced because they compensate for the factor cost or reduce the market price and
the factor cost.
Market price = Basic price + Product taxes – Product subsidy

Or

Market Price = Factor cost + Net indirect taxes

( Net indirect taxes = Indirect taxes – Subsidy)

 Excise tax, sales tax, service tax, and import and export charges are all examples of product
taxes.
 Food, petroleum, and fertilizer subsidies are examples of product subsidies.
 A shift in the market price for an item or service can be caused by shocks to the supply or
demand for that good or service.
o A Supply Shock occurs when an unforeseen occurrence occurs that causes the
supply of an item or service to abruptly shift.
 Interest rate reduction, tax cuts, government stimulus, terrorist attacks, natural
catastrophes, and stock market collapses are all instances of a supply shock.
o A Demand Shock occurs when the demand for an item or service suddenly surges or
declines.
 A sharp increase in the price of oil or other commodities, political unrest,
natural calamities, and technological advancements are all instances of a
demand shock.
Influencing Factors

Apart from supply and demand, other factors can have an impact on the market price of goods
and services. The influencing factors of the market price are →

 Natural Disasters: Any natural disaster can result in a significant rise or fall in the cost of
goods, assets, and commodities. For instance, if a flood hits a place that produces wheat in
the greatest number, it would undoubtedly boost the price of wheat in all other regions
throughout the world due to the concern that there will soon be no more wheat available.

https://t.me/prelimbits
108

 International Events: International events such as war, military activity, international


relations, etc. have an impact on the market price. For example, the conflict between Russia
and Ukraine has significantly increased fuel prices, which has caused concern among
consumers worldwide about long-term price increases.
 Employment & Wages: The employment rate and wages/salaries also have an impact on
market prices. People's purchasing power will suffer if they are unemployed and have no
income. They won't be able to purchase anything as a result. As a result, under pressure,
brands either raise or lower their pricing in response to the circumstances, which could have
a negative impact on economies.
Why is Market Price Important?

 One of the key reasons the market price is significant is that it gives a precise way of
establishing what an item is worth that eliminates ambiguity or confusion.
 Customers and sellers in the marketplace frequently hold opposing views about a product's
worth.
 The fundamental purpose of assessing market price is to arrive at an accurate appraisal of
the asset's worth or value.
 It is, simply said, the price at which the item would typically be sold. Buyers can choose to
pay, while sellers can take more or less than market value.
BASE YEAR [CONSTANT PRICING YEAR] FOR GDP CALCULATIONS
 The base year of the national accounts is chosen to enable inter-year comparisons. It gives
an idea about changes in purchasing power and allows calculation of inflation-
adjusted growth estimates.
 The last series has changed the base to 2011-12 from 2004-05.
 The Central Statistical Organisation (CSO)(now NSO) of India issued the first estimates of
national income in 1956, using 1948-49 as the base year.
 The technique was altered as the availability of data improved over time.
 Previously, CSO relied on population estimates from the National Census to determine the
economy's workforce.
 As a result, the base year was always the same as the census year, such as 1970-71, 1980-81,
and so on.
 As a result, the CSO determined that the National Sample Survey (NSS) data on workforce
size were more accurate and that the base year would be changed every five years when the
NSS performed such a survey.
 From 1999, when the base year was changed from 1980-81 to 1993-94, this approach was
implemented.
 The government switched to a new base year of 2011-12 for national accounts in
January 2015, replacing the previous base year of 2004-05.
 With the switch to the new base year, the economy's growth rate was predicted to be 6.9%
in 2013-14, up from 4.7 percent in 2004-05. Similarly, the 2012-13 growth rate was
increased to 5.1 percent from 4.5 percent.

https://t.me/prelimbits
109

Importance

 The base year prices are also known as constant prices since they reduce all of the
data to a similar baseline, the base year price.
 The base year is a representative year that is free of unusual events such as droughts,
floods, earthquakes, and so on.
 It is a year that's very close to the one for which the national accounts statistics are
being compiled.
 The base year must be updated regularly to reflect structural changes in an economy,
such as a rising percentage of services in GDP. The data will be more accurate if the base
year can be updated more regularly.
 The Ministry of Statistics and Programme Implementation (MOSPI) is considering
changing of base year for GDP calculation from 2011-12 to 2017-18.
Need for Change Base Year

 Accuracy: Change of base year to calculate GDP is done in line with the global exercise
to capture economic information accurately.
 Globally Aligned: GDP based on 2011-12 did not reflect the current economic situation
correctly. The new series will be in compliance with the United Nations guidelines in
System of National Accounts-2008.
 Ideally, the base year should be changed after every five years to capture the changing
economy.
75. The correct sequence in decreasing order of the four sugarcane producing States in
India is: [2000]
A. Maharashtra, U .P., Tamil Nadu, Andhra Pradesh
B. U. P., Maharashtra, Tamil Nadu, Andhra Pradesh
C. Maharashtra, U. P., Andhra Pradesh, Tamil Nadu
D. U. P., Maharashtra, Andhra Pradesh, Tamil Nadu
Ans. *

LARGEST PRODUCER OF SUGARCANE IN INDIA


 India is the second-highest producer of sugarcane in the world after Brazil. The largest
producer of sugarcane in India is Uttar Pradesh, which produced over 177 million tonnes of
sugarcane in 2021. Sugarcane is a multipurpose crop, used in making sugar, jaggery,
khansari, molasses, and even paper.
 In India, approximately 60% of the population is involved in agriculture and among the
many crops cultivated in the nation, sugarcane is one of the most important Kharif crops.
The climate of the country supports the plantation of sugarcane throughout the year.
 The largest sugarcane producing state in India is Uttar Pradesh. The state has
an area of 2.27 million hectares that produces sugarcane. The average annual production of
sugarcane in Uttar Pradesh is 135.64 million tonnes.
o Uttar Pradesh, Karnataka, and Maharashtra together contribute to 80% of the
https://t.me/prelimbits
110

total sugarcane production in India.

S.No States / UT 2018-19 2019-20 2020-21* Average % to Rank


total
(2015-16 to
2019-20)

1 Uttar Pradesh 179.71 179.54 177.67 164.37 45.40 1st

2 Maharashtra 89.77 69.31 101.59 73.60 20.33 2nd

3 Karnataka 42.41 38.18 42.09 35.39 9.77 3rd

4 Tamil Nadu 17.14 14.12 12.80 18.58 5.13 4th

5 Bihar 20.12 13.58 10.71 14.64 4.04 5th

6 Gujarat 11.33 11.57 15.85 11.61 3.21 6th

7 Haryana 8.51 7.73 8.53 8.16 2.25 7th

8 Andhra 8.09 6.72 4.12 7.96 2.20 8th


Pradesh

9 Punjab 7.77 7.30 7.49 7.37 2.04 9th

10 Uttarakhand 6.33 6.94 6.96 6.38 1.76 10th

Uttar Pradesh [Largest Sugarcane Producer]

 Uttar Pradesh is the largest cultivator of sugarcane in India. With more than 177 million
tonnes produced in 2021, UP tops the list of sugarcane producers. The main reason behind
this is the perennial water supply provided by the river Ganga.
 Meerut, Bareilly, Saharanpur, and Bulandshahr are the major districts producing
sugarcane in UP.
Maharashtra

 Maharashtra ranks second in the list of largest sugarcane producing state in India. For
2021-22, the area reserved for sugarcane plantation was increased to 12.32 lakh hectares.
Consequentially, the per hectare production of sugarcane is expected to be 97 tonnes.
Pune, Satara, Solapur, Ahmednagar, and Aurangabad are the major producers.
MAHARASHTRA → LARGEST SUGAR PRODUCING STATE
 Maharashtra has once again the top sugar producer state in India after five years. It has
overtaken Uttar Pradesh in sugar production.
 The overall production of sugar by Maharashtra accounts for 138 lakh tonnes for the
year 2021-22.
o The total sugar produced by the Uttar Pradesh in the year 2021-22 accounts

https://t.me/prelimbits
111

for 105 lakh tonnes.


Reasons for the Huge Production of Sugar in Maharashtra

 Abundant Supply of Water:


o Sugarcane is a water intensive crop which needs a huge water supply which
farmers from Maharashtra were getting properly through rainfall, water
reservoirs, network of canals and from groundwater.
o Maharashtra has been receiving more than sufficient rainwater since 2019 during
the south-west monsoon season.
o Groundwater aquifers and other reservoirs were filled by water due to sufficient
rainfall. These sources of water play a key role in agricultural production.
 Underreporting of Cane Production:
o Data regarding actual production of the sugarcane in the state of Maharashtra was
not quite accurate.
o Keeping this in mind the concerned administration tried to make corrections in the
recorded data of the sugarcane production.
o This ultimately resulted in increased acreage under the sugarcane production
from 11.42 lakh hectare to 12.4 lakh hectares.
o Thus, Maharashtra harnessed the benefits from increased acreage under sugarcane
in 2021-22.
Why did Sugar Production in Uttar Pradesh Decline?

 Uttar Pradesh has become the largest ethanol producer because a large amount of
sugarcane production in Uttar Pradesh diverted toward the production of ethanol.
 It has been estimated that 12.60 lakh tonnes equivalent of sugar from cane have been
diverted for making ethanol in the year 2021-22 compared to 7.19 lakh tonnes in 2020-
21 and 4.81 lakh tonnes in 2019-20 and 0.31 lakh tonnes in 2018-19.
o Uttar Pradesh also has achieved the highest blending of ethanol in petrol
ratio among all states.
 Excess rain with water logging problems are associated with the state of Uttar Pradesh
which incurred heavy loss of sugarcane crops.
 Majority of the land in sugarcane area in Uttar Pradesh(87%) is planted under a single
variety of sugarcane (Co-0238). This variety is not a high yield variety of the sugarcane.
 Adverse impact of red rot fungal disease on sugarcane crop is a severe cause for the
decline of sugarcane production in Uttar Pradesh.
o Co-0238 variety of the sugarcane is highly susceptible to the red rot fungal
diseases.
o It should be replaced by the new varieties, such as Co-0118 and Co-15023 because
both of them are resistant to red rot fungal disease.
Sugarcane
 Temperature: Between 21-27°C with hot and humid climate.

https://t.me/prelimbits
112

 Rainfall: Around 100-150 cm.


 Soil Type: Deep rich loamy soil.
 Top Sugarcane Producing States: Uttar Pradesh > Maharashtra > Karnataka > Tamil Nadu
> Bihar.
 India is the second largest producer of sugarcane after Brazil.
 It can be grown on all varieties of soils ranging from sandy loam to clay loam given these
soils should be well drained.
 It needs manual labour from sowing to harvesting.
 It is the main source of sugar, gur (jaggery), khandsari and molasses.
 Scheme for Extending Financial Assistance to Sugar Undertakings
(SEFASU) and National Policy on Biofuels are two of the government initiatives to
support sugarcane production and sugar industry.
Ethanol Blending:
 Ethanol: It is one of the principal biofuels, which is naturally produced by the fermentation
of sugars by yeasts or via petrochemical processes such as ethylene hydration.
 Ethanol Blending Programme (EBP): It is aimed at reducing the country‘s dependence on
crude oil imports, cutting carbon emissions and boosting farmers‘ incomes.
 Blending Target: The Government of India has advanced the target for 20% ethanol
blending in petrol (also called E20) to 2025 from 2030.

76. Consider the following statements: [2000]


1. Maharashtra has the highest acreage under Jawar in India
2. Gujarat is the largest producer of groundnut in India.
3. Rajasthan has the largest area of cultivable wastelands in India
4. Andhra Pradesh has the highest per hectare yield of maize in India
Which of these statements are correct?

A. 1 and 4
B. 2 and 3
C. 1 and 3
D. 2 and 4
Ans. *

JOWAR [SORGHUM]

 Jowar being a tropical crop grows well within temperature 25 C and 32 C but the
temperature goes below 16 C creates a problem for this crop.
 This crop requires rainfall about 40 cm annually.
 For this crop, the pH value of soil ranges between 6 to 7.5 and this range is ideal for its

https://t.me/prelimbits
113

cultivation and better growth.


 Jowar crop has the adaptability for a wide range of soil but grows well in sandy, loamy
soil having good drainage.
 Maharashtra is the largest producer of Jowar
GROUNDNUT
 Peanut or groundnut (Arachis hypogaea), is a species in the legume or "bean" family. The
peanut was first domesticated and cultivated in the valleys of Paraguay.
 India is the second largest producer of groundnuts in the world.
 China is the largest producer as well as consumer of groundnut in the world.
 Indian groundnuts are available in different varieties: Bold or Runner, Java or Spanish and
Red Natal.
 The main Groundnut varieties produced in India are Kadiri-2, Kadiri-3, BG-1, BG-2, Kuber
, GAUG-1, GAUG-10, PG-1 , T-28, T-64, Chandra, Chitra, Kaushal, Parkash, Amber etc.
 Ground Nuts are important protein crops in India grown mostly under rain-fed conditions.
 Major Growing States: Gujarat, Andhra Pradesh, Tamil Nadu, Karnataka, Maharashtra
,Rajasthan, Madhya Pradesh, Orissa, and Uttar Pradesh
 Major Export Destinations (2020-21): Indonesia, Vietnam Soc Rep, China P Rp,
Philippines, Malaysia.
LARGEST AREA UNDER WASTELAND IN INDIA (IN RAJASTHAN)
 Rajasthan has the largest wasteland area in India, followed by Madhya
Pradesh and Maharashtra in terms of the maximum area of wastelands.
 Wastelands are formed for both natural and man-made reasons such as snow-covered
areas, coastal saline area forest blank, barren hill-ridges, etc.
 Deforestation, overgrazing, over-cultivation, and unskilled irrigation also majorly
contribute to the formation of wastelands.
 The wastelands have the problems of nitrogen and phosphorus deficiency that can be
improved by the plantation of certain seedlings and heavy afforestation programs.
 Of the national land area of 3,166 lakh hectares, 15% is categorized as wasteland.
 Rajasthan 25% of its geographical area is a wasteland, accounting for 18% of the
wasteland nationally.
MAIZE
 As per the Economic Survey 2020-21, around 38% of India‘s total maize production is
from 3 states in India. The top 3 maize-producing states of India, as of 2020-21, are listed
below:
o Karnataka – Karnataka contributed 16.45% of the total maize production in India.
o Madhya Pradesh – Madhya Pradesh contributed 11.37% of the total maize
produced in India.
o Maharashtra – Maharashtra accounted for 10.91% of the total maize produced in
India.
 Temperature: Between 21-27°C

https://t.me/prelimbits
114

 Rainfall: High rainfall.


 Soil Type: Old alluvial soil.
 Maize cultivation happens in two seasons – Kharif and Rabi. In Bihar, maize is cultivated
during the Rabi season.
 83% of the maize area under cultivation is grown during the Kharif season.
 Maize is used both as food and fodder.
 The United States of America (USA) is the largest producer of maize contributes nearly
36% of the total production in the world.
 India is the seventh largest producer of Maize representing around 4% of world maize
area and 2% of total production.
77. The most appropriate measure of economic growth is its: [2001]
A. Gross Domestic Product of a country‘s
B. Net Domestic Product
C. Net National Product
D. Per Capita Real Income
Ans. D

GDP → GNP TO NNP TO PER CAPITA INCOME


 Primary income (or factor income) = wages, interest, profit, rent
 Secondary income (or transfer payments) = gifts, donations, charities, fines
Now, first, we‘ve to derive Gross National ―Product‖ i.e.

 India‘s GDP
 PLUS Primary income earned by residents from overseas.
 MINUS Primary income earned by non-residents from India.
IGNORE secondary income.

IGNORE the incomes from sale of second hand (=used) goods.

GNP (Market Prices) = GDP + ―NET‖ factor income from abroad.


 Whenever something is produced, capital assets get consumed due to wear and tear. This
wear and tear is called Depreciation. Since, depreciation does not become part of
anybody‘s income, so it has to be subtracted.
Net National Product (NNP@Market Price) = GNP MINUS Depreciation.
 However, here we are getting the NNP at ‗Market Prices‘. We‘ve to convert it to Factor cost.
NNP (Factor Cost) = NNP (Market Price) (-) Indirect Taxes (+) Subsidies.
NNP (Factor Cost) is the National Income of India, says NCERT Class12.
 Per Capita Income = NNP ÷ population of India. It means national income is sum total of
all factor incomes adjusted for increase in prices.
PER CAPITA INCOME
 Per capita income is the measure of the amount of money earned per person in a country
or a geographic region. This per capita income is often used to evaluate how much income

https://t.me/prelimbits
115

there is per person living in a particular area, which in turn tells you about the
standard of living and quality of life in that particular area.
 When calculating the per capita income of a nation, the National Income of the country
is divided by its Population.
 Per capita ratio is often used to compare economic indicators of countries with different
population sizes.
 Every person in the population including men, women, children and even newborn
babies are counted by the per capita income as every one of them is a part of the region‘s
or nation‘s population.
78. Match List I with List II and select the correct answer using the codes given below the
lists: [2001]

Code

A. A-3; B-1; C-2; D-4


B. A-4; B-3; C-2; D-1
C. A-1; B-3; C-2; D-4
D. A-3; B-1; C-4; D-2
Ans. A

TYPES OF DEFICITS
 If government‘s income > its expenditure it will have a Surplus Budget
 If government‘s expenditure = its income, it will be a Balanced Budget
 If government‘s expenditure > its income, it‘ll be a Deficit Budget
Deficit Formula
Revenue Deficit Revenue expenditure minus Revenue receipts
Effective Revenue Deficit minus Grants for creation of capital assets
Revenue Deficit
Budget Deficit Total Receipts minus Total Expenditure
Effects of a budget deficit →
 Rise in national debt
 Higher debt interest payments

https://t.me/prelimbits
116

 Fall in value of currency


 Increase in currency circulation
 Increase in Aggregate Demand (AD)
 Possible increase in public sector investment
 May cause crowding out and higher bond yields
Fiscal Deficit Budget Deficit plus Borrowing

Primary Deficit Fiscal Deficit minus interest to be paid on previous loans

79. Consider the following taxes: [2001]


1. Corporation tax
2. Customs duty
3. Wealth tax
4. Excise duty
Which of these is/are indirect taxes?

A. 1 only
B. 2 and 4
C. 1 and 3
D. 2 and 3
Ans. B

INDIRECT TAX
 An indirect tax is a tax that is collected through a middleman from the person who
suffers the tax's ultimate economic burden.
 It is possible for the taxpayer to transfer it to someone else.
 The intermediary prepares a tax return and sends the tax proceeds to the government
along with it.
 In this sense, an indirect tax differs from a direct tax, which is collected directly by the
government from the individuals (legal or natural) who are subjected to it.
 Indirect taxes are based on an individual's expenses rather than their income.
 Indirect taxes are levied on suppliers of goods and services, but the tax is passed on to
the consumers, who are indirectly paying the tax.
 In other words →
o Tax incidence = @Person from whom govt collects the tax. (e.g. shopkeeper)
o Tax impact = @Person who finally bears the tax & can‘t pass its burdenon further.
(e.g. Consumer)
 In the indirect taxes, tax incidence and tax impact does not fall on the
same person.
 E.g. Customs Duty on import and export, Excise duty on manufacturing
of goods, Service tax on services, Sales Tax, Value Added Tax (VAT), and
Goods and Services tax (GST).

https://t.me/prelimbits
117

 Indirect taxes fall under the Ambit of FinMin → Department of Revenue → Central Board
of Indirect Taxes and Customs (CBIC)
Examples of Indirect Tax

 Customs Duty
o When commodities are transferred across international boundaries, customs
duty is applied as a tariff or tax.
o Its goal is to safeguard the country's economy.
o Various sorts of duties are imposed under customs rules, including Basic Duty,
Countervailing Duty, Protective Duty, Anti-Dumping Duty, and Export Duty.
o Import duties are used not just to generate revenue for the government, but also
to regulate commerce.
o In India, import duties are calculated on an ad valorem basis.
 Sales Tax
o In India, a sales tax is a type of tax levied by the government on the sale or
purchase of a certain commodity within the country.
o Sales tax is levied by both the central and state governments.
o It has now been replaced by IGST.
 Excise Duty
o Excise duty is a commodities tax in the proper sense because it is collected on the
manufacturing of products in India rather than the sale of the product.
o Except for alcoholic drinks and narcotics, the central government imposes an explicit
excise levy.
o It has now been replaced by CGST.
 Service Tax
o In India, a service tax is levied on all services rendered.
o In 1994-95, a service tax was imposed on three services: telephone services, general
insurance, and stockbroking.
o Every year since then, the service net has widened by adding more and more
services.
o In India, the current rate of service tax was 15% before it was replaced by the Goods
and Services Tax.
 Value Added Tax
o The VAT is constructed in such a way that it eliminates distortions.
o As a result, all states and union territories in India have implemented VAT (except
UTs of Andaman Nicobar and Lakshadweep).
o The tax is imposed on a variety of commodities sold in the state, and the
amount is set by the state.
o State VAT, which was in effect until July 1, 2017, had replaced the previous Sales Tax
of States.
o It has now been replaced by SGST.
 Dividend Distribution Tax

https://t.me/prelimbits
118

o A dividend is a payment made by a corporation to its stockholders from the


company's profits in a given year. Dividends are income in the hands of
shareholders, and they should ideally be subject to income tax.
o Dividend distribution tax is a tax levied by the Indian government on Indian
corporations based on the amount of dividends paid to shareholders.
o DDT was first introduced in 1997, and it was regulated under Section 115 O of the
Income Tax Act.
o The Finance Minister eliminated the Dividend Distribution Tax in Budget 2020.
o The burden of dividend taxation has now been shifted from corporations to
individuals.
 GST
o GST (Goods and Services Tax) is a national indirect tax applied on the manufacture,
sale, and consumption of goods and services. [Destination based Taxation]
o It has supplanted all indirect taxes levied by the central and state governments on
goods and services.
80. The term National Income represents: [2001]
A. gross national product at market prices minus depreciation
B. gross national product at market prices minus depreciation plus net factor income from
abroad
C. gross national product at market prices minus depreciation and indirect taxes plus subsidies
D. gross national product at market prices minus net factor income from abroad
Ans. C

Gross national product at market price minus depreciation minus indirect tax plus subsidies.

81. [2001]
Assertion (A): Ceiling on foreign exchange for a host of current account transaction heads
was lowered in the year 2000.
Reason (R): There was a fall in foreign currency assets also.
A. Both A and R are individually true and R is the correct explanation of A
B. Both A and R are individually true but R is not the correct explanation of A
C. A is true but R is false
D. A is false but R is true
Ans. C

 The foreign exchange reserves of the country consist of foreign currency assets held by the
RBI, gold holdings of the RBI and SDRs. Foreign currency assets at the end of March 2000
amounted to US $35.06 billion, showing an increase of US $5.54 billion during 1999-2000.
 The legal framework for administration of exchange control in India is provided by the
Foreign Exchange Management Act, 1999. Under the Act, freedom has been granted for
buying and selling of foreign exchange for undertaking current account transactions.
However, the Central Government has been vested with powers in consultation with Reserve
https://t.me/prelimbits
119

Bank to impose reasonable restrictions on current account transactions. Accordingly, the


Government has issued Notifications GSR.381(E) dated May 3, 2000, and S.O. 301(E) dated
March 30, 2001, imposing certain restrictions on current account transactions in public
interest.
FOREIGN EXCHANGE MANAGEMENT ACT, 1999
 The legal framework for the administration of foreign exchange transactions in India is
provided by the Foreign Exchange Management Act, 1999.
 Under the FEMA, which came into force with effect from 1st June 2000, all transactions
involving foreign exchange have been classified either as capital or current account
transactions.
o Current Account Transactions:
 All transactions undertaken by a resident that do not alter his / her assets or
liabilities, including contingent liabilities, outside India are current account
transactions.
 Example: payment in connection with foreign trade, expenses in connection
with foreign travel, education etc.
o Capital Account Transactions:
 It includes those transactions which are undertaken by a resident of India
such that his/her assets or liabilities outside India are altered (either increased
or decreased).
 Example: investment in foreign securities, acquisition of immovable
property outside India etc.
 Resident Indians
o A 'person resident in India' is defined in Section 2(v) of FEMA, 1999 as:
o Barring few exceptions, a person residing in India for more than 182 days during
the course of the preceding financial year.
o Any person or body corporate registered or incorporated in India.
o An office, branch or agency in India owned or controlled by a person resident
outside India.
o An office, branch or agency outside India owned or controlled by a person
resident in India.
 FEMA replaced an act called Foreign Exchange Regulation Act (FERA).
 FERA (Foreign Exchange Regulation Act) legislation was passed in 1973. It came into effect
on January 1, 1974. FERA was introduced when the Forex reserves of the country were very
low.
 FERA did not comply with the post-liberalization policies of the Government.
 Main change brought in FEMA compared to FERA : It made all the criminal offences as
civil offences.
Main Features of Foreign Exchange Management Act, 1999

 It gives powers to the Central Government to regulate the flow of payments to and from

https://t.me/prelimbits
120

a person situated outside the country.


 All financial transactions concerning foreign securities or exchange cannot be carried
out without the approval of FEMA. All transactions must be carried out through
―Authorised Persons.‖
 In the general interest of the public, the Government of India can restrict an authorized
individual from carrying out foreign exchange deals within the current account.
 Empowers RBI to place restrictions on transactions from capital Account even if it is
carried out via an authorized individual.
 As per this act, Indians residing in India, have the permission to conduct a foreign exchange,
foreign security transactions or the right to hold or own immovable property in a foreign
country in case security, property, or currency was acquired, or owned when the individual
was based outside of the country, or when they inherit the property from individual staying
outside the country.
Structure of FEMA

 The Head Office of FEMA, also known as Enforcement Directorate, headed by the
Director is located in New Delhi.
 There are 5 zonal offices in Delhi, Mumbai, Kolkata, Chennai, and Jalandhar, each office is
headed by Deputy Director.
 Every 5 zones are further divided into 7 sub-zonal offices headed by Assistant Directors and
5 field units headed by Chief Enforcement Officers.

FOREIGN EXCHANGE RESERVES:


 Foreign exchange reserves are assets held on reserve by a central bank in foreign
currencies, which can include bonds, treasury bills and other government securities.
 It needs to be noted that most foreign exchange reserves are held in US dollars.
 India‘s Forex Reserve include →
o Foreign Currency Assets
o Gold reserves
o Special Drawing Rights
o Reserve position with the International Monetary Fund (IMF).
Objectives of Holding Forex Reserves:

 Supporting and maintaining confidence in the policies for monetary and exchange
rate management.
 Provides the capacity to intervene in support of the national or union currency.
 Limits external vulnerability by maintaining foreign currency liquidity to absorb shocks
during times of crisis or when access to borrowing is curtailed.
Significance of Rising Forex Reserves:

 Comfortable Position for the Government: The rising forex reserves give comfort to the
government and the RBI in managing India‘s external and internal financial issues.

https://t.me/prelimbits
121

 Managing Crisis: It serves as a cushion in the event of a Balance of Payment (BoP) crisis
on the economic front.
 Rupee Appreciation: The rising reserves have also helped the rupee to strengthen against
the dollar.
 Confidence in Market: Reserves will provide a level of confidence to markets and investors
that a country can meet its external obligations.
82. The prices at which the government purchases food grains for maintaining the public
distribution system and for building up buffer-stock is known as: [2001]
A. minimum support prices
B. procurement prices
C. issue prices
D. ceiling prices
Ans. B

PROCUREMENT PRICE
 It is the price at which govt purchases the crop after harvesting, the main difference
between Procurement Price and MSP is that MSP is declared before sowing while PP is
declared after harvesting.
 Since 1968-69 there is no practice of declaring separate Procurement Price and MSP is
usually considered as the Procurement Price.
 The procurement agencies step in to procure the crop and support the prices when the
market price falls below the MSP and the procured farm products are kept in government
warehouses and distributed through the PDS and various food security programmes; FCI
(Food Corporation of India), the nodal central agency of Government of India, along with
other State Agencies undertakes procurement of wheat and paddy.
 Hence, the price at which the Government purchases food grains for maintaining
public distribution is known as Procurement Price.
 Minimum price is given in 1st option can be answered but the government does not
declare MSP of each crop, but the government can procure any crop if required. For
example, if the prices of onions are spiking then the government can directly purchase it
from farmers and increase the supply in the market. Such a crop will have procurement
price but does not have an MSP. Hence, the best option is Procurement Price
MINIMUM SUPPORT PRICE (MSP)
 Agro Ministry‘s Commission for Agricultural Costs and Prices (CACP) recommends MSP
(& FRP for sugar) → Cabinet Committee on Economic Affairs chaired by PM approves &
announces MSP

 Under the State APMC Acts, the first sale of agriculture commodity can occur at
Agricultural Produce Market Committee (APMC) Mandis only. However, a farmer may
not get remunerative prices at the Mandi due to following reasons:

o Bumper production /supply which brings down the market prices AND / OR

https://t.me/prelimbits
122

o Cartelization / price-fixing by the mandi-merchants.

 So, to protect the farmers, Government of India announces MSP BEFORE each crop
sowing season. [fixed twice a year]

 Note: currently MSP does not have any legal backing = EXCEPT

o The only crop where MSP payment has some statutory element is sugarcane. This is
due to its pricing being governed by the Sugarcane (Control) Order, 1966 issued
under the Essential Commodities Act. The responsibility to make FRP payment to
farmers within 14 days of cane purchase lies solely with the sugar mills.

For 22 crops (MSP) + 1 sugarcane (FRP) = 23 viz.

14 kharif crops  Kharif: Grown In Summer / Monsoon Season.


 Paddy, Jowar, Bajra, Maize, Ragi, Arhar, Moong, Urad, Groundnut-
In-Shell, Soyabean, Sunflower, Sesamum, Nigerseed And Cotton;
6 rabi crops  Rabi: grown in winter season.
 Wheat, Barley, Gram, Masur(Lentil), Rapeseed/Mustard And
Safflower
3commercial /cash  Jute, Copra and Sugarcane.
crops  For Sugarcane, mechanism is different: It requires the sugar mill
companies to pay the minimum Fair and Remunerative Price (FRP)
 fixed by Govt. (Whereas for other 22, Govt itself procures @MSP)
How the cost of agricultural produce is calculated? →

The CACP considered various factors while recommending the MSP

 Cost of cultivation
 the supply and demand situation for the commodity
 market price trends (domestic and global) and parity vis-à-vis other crops
 implications for consumers (inflation), environment (soil and water use)
 terms of trade between agriculture and non-agriculture sectors
The CACP calculates cost of production at three levels:

1. A2, which includes cost of inputs such as seeds, fertilizer, labour;

2. FL, which includes the implied cost of family labour (FL); and

3. C2, which includes the implied rent on land and interest on capital assets

Govt. formula →

The CACP‘s ‗Price Policy for Kharif Crops: The Marketing Season 2018-19‘ report stated that its
MSP recommendation was based on 1.5 times the A2+FL costs.

 A2

 A2+FL includes A2 plus an imputed value of unpaid family labour. ‗

https://t.me/prelimbits
123

It does not take into account C2 costs. ‗C2‘ is a more comprehensive cost that factors in
rentals and interest forgone on owned land and fixed capital assets, on top of A2+FL.

Factors that are considered before recommending MSP are →

 Cost of production,
 Overall demand-supply situations,
 Domestic and international prices,
 Inter-crop price parity,
 terms of trade between agriculture and non-agriculture sector.
CEILING PRICE
 Price ceiling is a situation when the price charged is more than or less than the
equilibrium price determined by market forces of demand and supply. It has been found
that higher price ceilings are ineffective. The price ceiling has been found to be of great
importance in the house rental market.
ISSUE PRICE
 In stock exchange the price at which a new issue of shares is offered to the public.
83. Consider the following: [2001]
1. Market borrowing
2. Treasury bills
3. Special securities issued to RBI
Which of these is/are components(s) of internal debt?

A. 1 only
B. 1 and 2
C. 2 only
D. 1, 2 and 3
Ans. D

Internal debt is that part of the total debt that is owed to lenders within the country. Market
borrowings, Treasury bills and special securities issued to RBI are all sources of internal borrowings

GOVERNMENT BORROWINGS
 Government Borrowing is a loan taken by the government through issue of government
https://t.me/prelimbits
124

securities called G-secs and Treasury Bills.


 It is the total amount of money that the government borrows to fund its spending on public
services and benefits.
 It falls under capital receipts in the Budget document.
 The government raises money from the market to fund its fiscal deficit.
 If the government borrows more than what it has projected in the Budget document, its
interest costs go up risking higher fiscal deficit.
 Article 292 deals with the borrowing by the Central Government.
 Article 293 deals with the borrowing by State Governments. The State Governments,
which are indebted, must seek the consent of the Central Government before raising further
borrowings.
84.Consider the following factors regarding an industry [2001]
1. Capital investment
2. Business turnover
3. Labour force
4. Power consumption
Which of these determine the nature and size of the industry?

A. 1, 3 and 4
B. 1, 2 and 4
C. 2, 3 and 4
D. 2 and 3
Ans. A

The factors which are responsible for an industry are capital investment, labour force and power
consumption. However, nature and size of business is not determined by the turnover of business.

FACTORS RESPONSIBLE FOR DETERMINING THE SIZE OF THE FIRM


Entrepreneurial Skill:

 The most important factor of comes is the skill, initiative and resourcefulness of the
entrepreneur. Everything depends on his judgment and ability. An entrepreneur of
outstanding ability will be able to procure as much finance as he may need, hire the
requisite labor force and build up a huge business. But an entrepreneur of moderate ability
will run business on a moderate scale and a man of limited entrepreneurial skill will be
content with a small business
Managerial Ability

 For running the routine part of the business, managers are appointed. If a firm is lucky
enough to have a manager of great ability, the size of the firm will grow to considerable
dimensions. On the other hand, a mediocre manager will have a small-sized firm to manage.
Availability of Finance

 It is finance which oils the wheels of business machine. If ample funds are available, it will

https://t.me/prelimbits
125

help the entrepreneur to make his business grow to a big size. This requires a proper
development, of the banking system so that savings of the community can be effectively
mobilized and utilized in the development of trade and industry.
Availability of Labour

 Another factor on which the size of the firm depends is the availability of labour of requisite
skill. After all, what can the entrepreneur even with large capital do, if the labour to man the
business is not available? What is required is efficient and skilled labour.
Nature of Business:

 Much also depends on the nature of business. If the business obeys the law of increasing
Returns, it will grow to a big size, otherwise, in the case of diminishing returns it will remain
stunted, and in the case of constant returns it will remain stagnant.
Extent of the Market

 The size of the firm also depends on the extent of the market. If the commodity in which the
firm deals or which it-manufactures has a wide market, naturally the business will assume a
large scale. But if the demand for the commodity is fitful or limited, the size of the firm will
continue to be small. These are some of the factors on which the size of an average firm in a
country depends.
Higher power consumption

 Higher power consumption will reduce its capital available to other productive works such
as investment in machinery.
85. [2001]
Assertion (A): India‟s software exports increased at an average growth rate of 50% since
1995-96.
Reason (R): Indian software companies were cost-effective and maintained international
quality.
A. Both A and R are individually true and R is the correct explanation of A
B. Both A and R are individually true but R is not the correct explanation of A
C. A is true but R is false
D. A is false but R is true
Ans. A

SURVEY ON COMPUTER SOFTWARE AND INFORMATION TECHNOLOGY ENABLED SERVICES EXPORTS:


2021-22
 India‘s exports of software services (excluding exports through commercial presence) are
estimated to have increased by 17.2 per cent to US$ 156.7 billion during 2021-22
 Computer services continued to account for over two-thirds of total software services
exports.
 BPO services accounted for nearly 84 per cent of exports of information technology (IT)
enabled services

https://t.me/prelimbits
126

 Private limited companies accounted for 60 per cent of the total exports of software
services whereas exports by public limited companies declined during the year
 The United States and Canada were the top destination for software exports with 55.5
per cent share, followed by Europe, of which, nearly half was attributed to the United
Kingdom
 The US dollar remained the principal invoicing currency for software exports, followed
by the Euro and Pound sterling
86. Consider the following states: [2001]
1. Gujarat
2. Karnataka
3. Maharashtra
4. Tamil Nadu
The descending order of these states with reference to their level of Per Capita Net State Domestic
Product is:

A. 1, 3, 4, 2
B. 3, 1, 2, 4
C. 1, 3, 2, 4
D. 3, 1, 4, 2
Ans. *

2018-19
Gujarat 153495
Karnataka 153276
Maharashtra 147450
Tamil Nadu 142941
PER CAPITA NET STATE DOMESTIC PRODUCT
2018 -19
Andhra Pradesh 107241
Arunachal Pradesh 93191
Assam 60695
Bihar 28668
Chandigarh 234998
Chhattisgarh 69500
Delhi 269505
Goa 368685
Gujarat 153495
Haryana 169409
Himachal Pradesh 139469
Jammu and Kashmir 65178
Jharkhand 54982
Karnataka 153276
Kerala 148078

https://t.me/prelimbits
127

Madhya Pradesh 56498


Maharashtra 147450
Manipur 51180
Meghalaya 62458
Mizoram 107853
Nagaland 73276
Odisha 76417
Puducherry 142583
Punjab 115882
Rajasthan 78570
Sikkim 242002
Tamil Nadu 142941
Telangana 143618
Tripura 82632
Uttar Pradesh 44421
Uttarakhand 155151
West Bengal 67300

87. Consider the following: [2002]


1. Currency with the public
2. Demand deposits with banks
3. Time deposits with banks
Which of these are included in Broad Money (M3) in India ?

A. 1 and 2
B. 1 and 3
C. 2 and 3
D. 1, 2 and 3
Ans. D

MONETARY AGGREGATES
 Monetary aggregates are the measures of the money supply in a country.
 Very often, the money supply in the economy is represented using a monetary aggregate
called ‗Broad Money‘, also denoted as M3.
 In India, Reserve Bank of India (RBI), measures the money supply and publishes it on a
weekly or fortnight basis.
 There are also different other monetary aggregates.
M1 = C + DD (Narrow Money)

 C - Currency held by the public


 DD- Demand Deposits with Banks [CASA]
Why Narrow

 Bcoz it includes only 100% liquid deposits which is a very narrow definition of the money
supply.
M2

https://t.me/prelimbits
128

 M1 + Savings account deposits with Post Offices


Note- Post offices have no facility for the opening of current accounts. The types of accounts that
can be opened are – Savings account, Fixed deposit, and Recurring deposit.
M3 = M1 + TD (Broad Money)

 TD – Time Deposits with Banks Includes fixed deposits, Recurring deposits, and time
liability of Savings accounts
 The Most Common measure used for money supply is M3
 Central Bank tracks the growth of broad money to help forecast inflation
M4 = M3 + Total Deposits with Post Office

 As the total deposits with the post office are negligible there is not much difference
between M3 and M4.
We are only counting the ―NET Demand / NET Time deposits‖ i.e. only public deposits in bank.
We are not counting interbank deposits i.e. one commercial bank‘s deposit in other commercial
banks.
88. With reference to the Indian Public Finance, consider the following statements: [2002]
1. External liabilities reported in the Union Budget are based on historical exchange rates
2. The continued high borrowing has kept the real interest rates high in the economy
3. The upward trend in the ratio of Fiscal Deficit of GDP a recent years has an adverse effect on
private investment
4. Interest payments is the single largest component of the non-plan revenue expenditure of
the Union Government
Which of these statements are correct?

A. 1, 2 and 3
B. 1 and 4
C. 2, 3 and 4
D. 1, 2, 3 and 4
Ans. D

Interest to be paid on previous loans is Revenue Expenditure during 2022-22 was 9.40 Lcr.
[highest]

https://t.me/prelimbits
129

89. Consider the following statements: [2002]


Full convertibility of the rupee may mean:

1. Its free float with the international currencies


2. Its direct exchange with any other international currency at any prescribed place inside and
outside the country
3. It acts just like any other international currency.
Which of these statements are correct?

A. 1 and 2
B. 1 and 3
C. 2 and 3
D. 1, 2 and 3
Ans. A

Also refer Q. 37

 The exchange rate was earlier controlled by RBI for the conversion of Indian currency
into foreign exchange.
 All the foreign exchange earned was to be sold to an authorized dealer.
 One had to seek the permission of the central bank to purchase foreign exchange.
 The main purpose behind this was to utilize the foreign exchange earned by the
residents as per the priorities fixed by the government.
 India is a fast developing country now and one of the most preferred countries for
investment by foreigners.
 India could not restrict its foreign trade as It needs to grow further. So the government
has allowed the convertibility of rupee in a phased manner on current account
transactions.
 But full convertibility of currency for capital account transactions is still a distant dream.
CONVERTIBILITY OF CURRENCY
 Convertibility of currency means when the currency of a country can be freely
converted into the foreign exchange at the market-determined rate of exchange.
 The exchange rate is determined by the demand for and supply of a currency.
 In general, the convertibility of rupee means that those who have a foreign
exchange (e.g. US dollars, Pound Sterlings, etc.) can get them converted into rupees
and vice-versa at the market-determined rate of exchange.
 The rupee is both convertibles on capital account and current account.
90. Global capital flows to developing countries increased significantly during the nineties.
In view of the East Asian financial crisis and Latin American experience, which type of
inflow is good for the host country? [2002]
A. Commercial loans
B. Foreign Direct Investment

https://t.me/prelimbits
130

C. Foreign Portfolio Investment


D. External Commercial borrowings
Ans. B

FOREIGN DIRECT INVESTMENT


 A Foreign Direct Investment (FDI) is an investment made by a firm or individual in one
country into business interests located in another country.
o FDI lets an investor purchase a direct business interest in a foreign country.
 Investors can make FDI in a number of ways.
o Some common ones include establishing a subsidiary in another
country, acquiring or merging with an existing foreign company, or starting a joint
venture partnership with a foreign company.
 Apart from being a critical driver of economic growth, FDI has been a major non-debt
financial resource for the economic development of India.
 It is different from Foreign Portfolio Investment where the foreign entity merely buys
stocks and bonds of a company. FPI does not provide the investor with control
over the business.
Components →

 Equity capital:
o It is the foreign direct investor‘s purchase of shares of an enterprise in a
country other than its own.
 Reinvested earnings:
o It comprises the direct investors‘ share of earnings not distributed as dividends by
affiliates, or earnings not remitted to the direct investor.
 Such retained profits by affiliates are reinvested.
 Intra-company loans:
o These refer to short- or long-term borrowing and lending of funds between direct
investors (or enterprises) and affiliate enterprises.
Routes of FDI

 Automatic Route:
o In this, the foreign entity does not require the prior approval of the government
or the RBI (Reserve Bank of India).
o In India FDI up to 100% is allowed in non-critical sectors through the automatic
route, not requiring security clearance from the Ministry of Home Affairs (MHA).
o Prior government approval or security clearance from MHA is required
for investments in sensitive sectors such as defence, media, telecommunication,
satellites, private security agencies, civil aviation and mining, besides any
investment from Pakistan and Bangladesh.
 Government Route:
o In this, the foreign entity has to take the approval of the government.

https://t.me/prelimbits
131

o The Foreign Investment Facilitation Portal (FIFP) facilitates the single window
clearance of applications which are through approval route. It is administered by
the Department for Promotion of Industry and Internal Trade (DPIIT), Ministry
of Commerce and Industry.
FOREIGN PORTFOLIO INVESTMENT
 Foreign portfolio investment (FPI) consists of securities and other financial
assets passively held by foreign investors.

 It does not provide the investor with direct ownership of financial assets and is relatively
liquid depending on the volatility of the market.

 The investor does not actively manage the investments through FPIs, he does not have
control over the securities or the business.

 The investor‘s goal is to create a quick return on his money.

o FPI is often referred to as ―hot money‖ because of its tendency to flee at the first
signs of trouble in an economy.

 FPI is more liquid and less risky than Foreign Direct Investment (FDI).

EXTERNAL COMMERCIAL BORROWINGS (ECBS)


 External commercial borrowings (ECBs) are loans in India made by non-resident lenders
in foreign currency to Indian borrowers. They are used widely in India to facilitate
access to foreign money by Indian corporations and PSUs (public sector undertakings).
 ECBs include commercial bank loans, buyers' credit, suppliers' credit, securitized
instruments such as floating rate notes and fixed rate bonds etc., credit from official export
credit agencies and commercial borrowings from the private sector window of multilateral
financial Institutions such as International Finance Corporation (Washington), ADB, AFIC,
CDC, etc.
 ECBs cannot be used for investment in stock market or speculation in real estate. The
DEA (Department of Economic Affairs), Ministry of Finance, Government of India along with
Reserve Bank of India, monitors and regulates ECB guidelines and policies.
91. Five Year Plan in India is finally approved by: [2002]
A. Union Cabinet
B. President on the advice of Prime Minister
C. Planning Commission
D. National Development Council
Ans. D

Refer Q. 6

In India plans are formulated by Planning Commission and are finally approved by National
Development Council. All state chief ministers are members of NDC. The NDC is headed by prime
minister of India.

https://t.me/prelimbits
132

NATIONAL DEVELOPMENT COUNCIL


 The National Development Council (NDC) or Rashtriya Vikas Parishad is the apex body for
decision creating and deliberations on development matters in India, presided over by the
Prime Minister.
 It was set up on 6 August 1952 to strengthen and mobilize the effort and
resources of the nation in support of the Five Year Plans made by
Planning Commission, to promote common economic policies in all vital
spheres, and to ensure the balanced and rapid development of all parts of the country.
 The Council comprises the Prime Minister, the Union Cabinet Ministers, Chief Ministers
of all States or their substitutes, representatives of the Union Territories and the
members of the NITI Aayog (erstwhile Planning Commission).
 NDC (National Development Council) has been proposed to be abolished. But till date no
resolution has been passed to abolish it.
 Since the inception of NITI Aayog's Governing Council (which has almost the same
composition and roles as NDC), the NDC has had no work assigned to it nor did it have
any meetings.
 During the tenure of former Prime Ministers Atal Bihari Vajpayee and Manmohan Singh it
was felt that Planning Commission has outlived its life and needs some reform.
 In 2014, Prime Minister Narendra Modi announced Planning Commission's abolition and
created NITI Aayog through an executive resolution. It is neither a constitutional body
nor a statutory body.
92. A country is said to be a debt trap if: [2002]
A. it has to borrow to make interest payments on outstanding loans
B. It has to abide by the conditionalities imposed by the International Monetary fund.
C. it has been refused loans or aid by creditors abroad
D. the World Bank charges a very high rate of interest on outstanding as well as new loans
Ans. A

 Debt-trap diplomacy is an international financial relationship where a creditor country


or institution extends debt to a borrowing nation partially, or solely, to
increase the lender's political leverage.
 The creditor country is said to extend excessive credit to a debtor country with the
intention of extracting economic or political concessions when the debtor country
becomes unable to meet its repayment obligations.
 The conditions of the loans are often not publicized. The borrowed money
commonly pays for contractors and materials sourced from the creditor
country.
 The term was coined by Indian academic Brahma Chellaney to describe how the Chinese
government leverages the debt burden of smaller countries for geopolitical ends.

https://t.me/prelimbits
133

Other analysts have described the idea of a Chinese debt trap as a "myth" or "distraction"
93. With reference to the Public Sector Undertakings in India, consider the following
statements: [2002]
1. Minerals and Metals Trading Corporation of India Limited is the largest non-oil importer of
the country
2. Project and Equipment Corporation of India Limited is under the Ministry of Industry
3. One of the objectives of Export Credit Guarantee Corporation of India Limited is to enforce
quality control and compulsory pre-shipment inspection of various exportable commodities
Which of these statements is/are correct?

A. 1 only
B. 1 and 2
C. 2 and 3
D. 3 only
Ans. A

MMTC LTD
 MMTC Ltd., Metals and Minerals Trading Corporation of India, is one of the two highest
earners of foreign exchange for India and India's largest public sector trading body.
 Not only handling the export and import of primary products such as coal, iron ore, agro
and industrial products, MMTC also exports and imports important commodities such
as ferrous and nonferrous metals for industry, and agricultural fertilizers.
 MMTC's diverse trade activities cover third country trade, joint ventures and link deals and
all modern forms of international trading.
 The company has a vast international trade network, spanning almost in all countries
in Asia, Europe, Africa, Oceania, and in the United States and also includes a wholly
owned international subsidiary in Singapore, MTPL.
 It is one of the Miniratnas companies.
 MMTC is one of the two highest foreign exchange earner for India (after petroleum
refining companies).
 It is the largest international trading company of India and the first public sector
enterprise to be accorded the status of Five Star Export Houses by Government of
India for long standing contribution to exports
 Being the largest player in bullion trade, including retailing, MMTC's share was 146 tonnes
of gold out of the total import of 600 tonnes of the precious metal in 2008–09.
 MMTC is the largest non-oil importer in India. Project and Equipment Corporation of India
Limited is under the Ministry of Commerce & Industry.
 The company was incorporated on 26 September 1963 at New Delhi.
 The Main Objectives of the company was export of mineral ores and import of essential
metals. According to a latest news, MMTC is Asia's biggest gold and silver importer

https://t.me/prelimbits
134

PEC LIMITED
 PEC Ltd (formerly – The Project and Equipment Corporation of India Ltd.) was carved out of
the STC in 1971-72 to take over the canalized business of STC‘s (State Trading Corporation
of India Ltd.) railway equipment division, to diversify into turn-key projects especially
outside India and to aid and assist in promotion of exports of Indian engineering
equipment.
 Thereafter, from 27th March, 1991, PEC Ltd. became an independent company directly
owned by the Government of India. PEC Limited also has a subsidiary Tea Trading
Corporation Limited which is under liquidation.
 PEC Limited is incurring losses since Financial Year 2014-15 and has stopped all business
activities since September, 2019.
EXPORT CREDIT GUARANTEE CORPORATION OF INDIA
 ECGC Ltd is wholly owned by the Ministry of Commerce and Industry.

 The Government of India had initially set up Export Risks Insurance Corporation in
1957.

 After the introduction of insurance covers to banks during the period 1962-64, the name
was changed to Export Credit & Guarantee Corporation Ltd in 1964.

o It was changed to ECGC Ltd in August 2014.

 Its objective was to promote exports from the country by providing credit
risk insurance and related services for exports.
94. HINDALCO, an aluminium factory located at Renukut owes its site basically to: [2002]
A. proximity of raw materials
B. abundant supply of power
C. efficient transport network
D. proximity to the market
Ans. B

HINDALCO, an aluminium factory located at Renukut due to abundant supply of power from
National Thermal Power Station, Rihand and a capative power plant at Renu Sagar. It has a
cogeneration unit at the plant itself.

95. Consider the following financial institutions of India: [2002]


1. Industrial Finance Corporation of India (IFCI)
2. Industrial Credit and Investment Corporation of India (ICICI)
3. Industrial Development Bank of India (IDBI)
4. National Bank for Agriculture and Rural Development (NABARD)
The correct chronological sequence of the establishment of these institution is :

A. 1, 2, 3, 4

https://t.me/prelimbits
135

B. 2, 3, 4, 1
C. 3, 4, 1, 2
D. 4, 1, 2, 3
Ans. A

IFCI – 1948; ICICI – 1955; IDBI – 1964; NABARD – 1982

INDUSTRIAL FINANCE CORPORATION OF INDIA (IFCI)


 IFCI, previously Industrial Finance Corporation of India, is a development finance
institution under the ownership of Ministry of Finance.
 Established in 1948 as a statutory corporation, IFCI is currently a company listed on BSE
and NSE. IFCI has seven subsidiaries and one associate.
 It provides financial support for the diversified growth of Industries across the
spectrum.
 The financing activities cover various kinds of projects such as airports, roads, telecom,
power, real estate, manufacturing, services sector and such other allied industries.
 During its 70 years of existence, mega-projects like Adani Mundra Ports, GMR Goa
International Airport, Salasar Highways, NRSS Transmission, Raichur Power Corporation,
among others, were set up with the financial assistance of IFCI.
INDUSTRIAL CREDIT AND INVESTMENT CORPORATION OF INDIA (ICICI)
 ICICI was a government institution established on 5 January 1955 and Sir Arcot
Ramasamy Mudaliar was elected as the first Chairman of ICICI Ltd.
 It was structured as a joint-venture of the World Bank, India's public-sector banks and
public-sector insurance companies to provide project financing to Indian
industry.
 ICICI Bank was established by ICICI, as a wholly owned subsidiary in 1994 in Vadodara.
The bank was founded as the Industrial Credit and Investment Corporation of India
Bank, before it changed its name to ICICI Bank. The parent company was later merged
with the bank.
INDUSTRIAL DEVELOPMENT BANK OF INDIA
 IDBI (Industrial Development Bank of India) Bank was established in 1964 by an Act of
Parliament as a wholly owned subsidiary of the Reserve Bank of India (RBI).
It was established as a specialised development financial institution (DFI) – on the lines
of NABARD, NHB and SIDBI – to provide credit and other financial facilities for the
development of Indian industry.
 In 1976, the ownership of IDBI was transferred to the Government of India.
 In 2004 it was converted into a commercial bank when RBI incorporated IDBI as a
'scheduled bank' under the RBI Act, 1934
 Operates Credit Guarantee fund, Small Enterprises Development Fund (SEDF).
 Operates udyamimitra.in for loans to small entrepreneurs & SME via schemes like Mudra,
Stand-up-India.

https://t.me/prelimbits
136

NATIONAL BANK FOR AGRICULTURE AND RURAL DEVELOPMENT (NABARD)


 NABARD is the country's premier financial institution for agricultural finance and rural
development.
 It was formed in July 1982 by combining the Reserve Bank of India's Agriculture Credit
Department, Rural Planning and Credit Cell, and the entire Agriculture Refinance and
Development Corporation.
 It was established to meet the credit needs of agriculture and rural development.
 NABARD was envisioned as the national apex institution for the entire rural credit system,
providing supplemental funding to all rural credit institutions and
coordinating their operations. [Not directly to Farmers]

96. Consider the following statements: [2003]


In India, stamp duties on financial transactions are:

1. levied and collected by the State Government


2. appropriated by the Union Government
Which of these statements is/are correct?

A. Only 1
B. Only 2
C. Both 1 and 2
D. Neither 1 nor 2
Ans. D

Stamp duty is a form of tax that is levied on documents. It is levied by the union or the central
government and collected and appropriated by the state governments. (Entry91 in the union
list; entry 63 in the state list of Seventh Schedule of Constitution of India)

STAMP DUTY

 Stamp duty is a legal tax payable in full and acts as evidence for any sale or purchase
of a property. It is payable under Section 3 of the Indian Stamp Act, 1899.
 The levy of stamp duty is a state subject and thus the rates of stamp duty vary from
state to state.
 The Centre levies stamp duty on specified instruments and also fixes the rates for these
instruments.
 It is usually paid by the buyer with regardless of agreement and in case of property
exchange, both seller and the buyer has to share the stamp duty equally.
 A stamp duty paid instrument/document is considered a proper and legal
instrument/document and has evidentiary value and is admitted as evidence in courts.
 Up till now, we knew that stamp duties are levied on property transactions,
registrations etc. With the Finance Act 2019, the stamp duties are also levied on
Mutual Funds.

https://t.me/prelimbits
137

97. Which one among the following States has the highest female literacy rate as per the
Census 2001? [2003]
A. Chhattisgarh
B. Madhya Pradesh
C. Orissa
D. Rajasthan
Ans. A

As per the census of 2001, Chhattisgarh has the highest female literacy of 51.85, Orissa has 50.51%,
Madhya Pradesh - 50.29 and Rajasthan has 43.85%.

LITERACY IN INDIA (CENSUS 2011)


 The 15th official census in India was calculated in the year 2011. In a country like India,
literacy is the main foundation for social and economic growth. When the British rule ended
in India in the year 1947 the literacy rate was just 12%. Over the years, India has changed
socially, economically, and globally. After the 2011 census, literacy rate India 2011 was
found to be 74.04%.
 Kerala is the only state in India to have 100% literacy rate. It is followed by Goa,
Tripura, Mizoram, Himachal Pradesh, and Maharashtra, Sikkim. The lowest literacy rate in
India is seen in the state of Bihar.
https://www.census2011.co.in/literacy.php

# State Literacy Male Female % Change


- India 74.04 82.14 65.46 8.66
1 Kerala 94.00 96.11 92.07 3.14
2 Lakshadweep 91.85 95.56 87.95 5.19
3 Mizoram 91.33 93.35 89.27 2.53
4 Goa 88.70 92.65 84.66 6.69
5 Tripura 87.22 91.53 82.73 14.03
6 Daman and Diu 87.10 91.54 79.55 8.92
7 Andaman and Nicobar 86.63 90.27 82.43 5.33
Islands
8 Delhi 86.21 90.94 80.76 4.54
9 Chandigarh 86.05 89.99 81.19 4.11
10 Puducherry 85.85 91.26 80.67 4.61
11 Himachal Pradesh 82.80 89.53 75.93 6.32
12 Maharashtra 82.34 88.38 75.87 5.46
13 Sikkim 81.42 86.55 75.61 12.61
14 Tamil Nadu 80.09 86.77 73.44 6.64
15 Nagaland 79.55 82.75 76.11 12.96
16 Uttarakhand 78.82 87.40 70.01 7.2
17 Gujarat 78.03 85.75 69.68 8.89
18 Manipur 76.94 83.58 70.26 10.33
19 West Bengal 76.26 81.69 70.54 7.62

https://t.me/prelimbits
138

20 Dadra and Nagar Haveli 76.24 85.17 64.32 18.61


21 Punjab 75.84 80.44 70.73 6.19
22 Haryana 75.55 84.06 65.94 7.64
23 Karnataka 75.36 82.47 68.08 8.72
24 Meghalaya 74.43 75.95 72.89 11.87
25 Orissa 72.87 81.59 64.01 9.79
26 Assam 72.19 77.85 66.27 8.94
27 Chhattisgarh 70.28 80.27 60.24 5.62
28 Madhya Pradesh 69.32 78.73 59.24 5.58
29 Uttar Pradesh 67.68 77.28 57.18 11.41
30 Jammu and Kashmir 67.16 76.75 56.43 11.64
31 Andhra Pradesh 67.02 74.88 59.15 6.55
32 Jharkhand 66.41 76.84 55.42 12.85
33 Rajasthan 66.11 79.19 52.12 5.7
34 Arunachal Pradesh 65.38 72.55 57.70 11.04
35 Bihar 61.80 71.20 51.50 14.8

98. Which one of the following committees recommended the abolition of reservation of
items for the small scale sector in industry? [2003]
A. Abid Hussian Committee
B. Narasimham Committee
C. Nayak Committee
D. Rakesh Mohan Committee
Ans. A

ABID HUSSAIN COMMITTEE


 Abid Hussain committee report on Small Enterprises was submitted in 1997.
 There was a total of 9 members including the chairman.
 The recommendations of committee are:-
o Protection should be replaced by promotion as the cornerstone of future policy.
o The corporatization of government extension agencies.
o Training and Marketing Assistance.
o Infrastructure development in clusters.
 Shri Abid Hussain was the former member of the Planning Commission.
NARASIMHAM COMMITTEE
 From the 1991 India economic crisis to its status of third largest economy in the world by
2011, India has grown significantly in terms of economic development, so has its banking
sector. During this period, recognizing the evolving needs of the sector, the Finance
Ministry of the Government of India set up various committees with the task of analyzing
India's banking sector and recommending legislation and regulations to make it more
effective, competitive and efficient.
 Two such expert Committees were set up under the chairmanship of Maidavolu
Narasimham. They submitted their recommendations in the 1990s in reports widely known

https://t.me/prelimbits
139

as the Narasimham Committee-I (1991) report and the Narasimham Committee-II


(1998) Report. These recommendations not only helped unleash the potential of banking in
India, they are also recognized as a factor towards minimizing the impact of global financial
crisis starting in 2007.
 The first Narasimhan Committee (Committee on the Financial System – CFS) was
appointed by Manmohan Singh as India's Finance Minister on 14 August 1991, and the
second one (Committee on Banking Sector Reforms) was appointed by P.Chidambaram
as Finance Minister in December 1997
 The purpose of the Narasimham-I Committee was to study all aspects relating to
the structure, organisation, functions and procedures of the financial
systems and to recommend improvements in their efficiency and
productivity. The Committee submitted its report to the Finance Minister in November
1991 which was tabled in Parliament on 17 December 1991.[6]
 The Narasimham-II Committee was tasked with the progress review of the
implementation of the banking reforms since 1992 with the aim of further
strengthening the financial institutions of India. It focussed on issues like size of banks
and capital adequacy ratio among other things. M. Narasimham, chairman, submitted
the report of the Committee on Banking Sector Reforms (Committee-II) to the Finance
Minister Yashwant Sinha in April 1998.
Recommendations of Narasimham Committee I (1991)

 Reduction in SLR and CRR- During 1991, both Statutory Liquidity Ratio (SLR) and Cash
Reserve Ratio (CRR) were extremely high. Due to this, bank resources were not
available for government use. The committee recommended reducing the SLR
and CRR from 38.5 percent to 25 percent and from 15 percent to 3 to 5 percent,
respectively.

 Reorganization of the Banking sector- Committee recommended reduction in the


number of public sector banks. The committee suggested mergers and
acquisitions to increase the bank‘s efficiency. The Committee recommended nationwide
the national recognition of 8 to 10 banks.

 Establishment of the ARF Tribunal- During the 1991 economic crisis, banks' bad debts
and Non-Performing Assets (NPA) were concerning. The committee recommended
setting up an Asset Reconstruction Fund (ARF) to take over the proportion of bad
and doubtful debts from banks and financial institutions.

 Removal of Dual Control- At that point, the banking sector in India was regulated by the
RBI and the Ministry of Finance. The committee proposed RBI be the sole
primary regulator of banking in India.
 Stop the Directed Credit Program- Directed credit programme should be phased out

https://t.me/prelimbits
140

gradually. As per the committee, agriculture and small-scale industries (SSIs) had
already grown to a mature stage and they did not require any special support; two
decades of interest subsidy were enough. Therefore, concessional rates of interest could
be dispensed with.

o Directed credit should not be a regular programme - it should be a case of


extraordinary support to certain weak sections—besides, it should be
temporary, not a permanent one.

o Concept of PSL should be redefined to include only the weakest sections


of the rural community such as marginal farmers, rural artisans, village and cottage
industries, tiny sector, etc.

o The ―redefined PSL‖ should have 10 per cent fixed of the aggregate
bank credit.

o The composition of the PSL should be reviewed after every 3 years.

 Interest Rate Determination- The committee highlighted that the interest rates
should be determined based on Market Forces and not by the Government,
which was earlier the case.

 More Freedom to Banks- To improve the workings of banks, the Narasimhan Committee 1
recommended that every bank be free and autonomous to carry out its work.
Over-regulation and over-administration should be avoided, and the selection of the
Chief Executive and board of directors should be made on merit solely.

 The RBI should allow the establishment of new private-sector banks as long as
they meet the minimum start-up capital and other requirements.

 The government should declare that no more banks will be nationalized.

 Foreign banks are permitted to open branches in India, either wholly-owned or as


Recommendations of Narasimham Committee II

 Need for Stronger Banking System


o The Narasimham committee has proposed a stronger banking system in the country,
particularly in the context of capital account convertibility (CAC), which would
involve a large amount of capital inflow and outflow, complicating exchange rate
management and domestic liquidity.
o To deal with this, India would require a strong and resilient banking and financial
system.
 Concept of Narrow Banking
o The Narasimham committee is deeply concerned about the rehabilitation of weak
public sector banks that have amassed a high percentage of Non-Performing
Assets (NPA), in some cases as much as 20% of their total assets.

https://t.me/prelimbits
141

o To rehabilitate such weak banks, they proposed the concept of narrow banking.
 Small Local Banks
o According to the Narasimham committee, ―while two or three banks with an
international orientation and eight to ten larger banks should take care of their
needs of the large and medium corporate sector as well as larger of the small
enterprises, a large number of local banks will still be required.‖
o The committee has proposed the establishment of small local banks that would be
limited to states or clusters of districts in order to serve local trade, small industry,
and so on.
 Capital Adequacy Ratio
o The Narasimham committee also suggested that the government consider raising
the prescribed capital adequacy ratio to improve banks' inherent strength and risk-
taking ability.
 Public Ownership and Real Autonomy
o According to the Narasimham committee, government ownership and management
of banks does not increase autonomy and flexibility in the operation of public sector
banks.
o As a result, the committee has recommended that the functions of bank boards be
reviewed in order to make them accountable for increasing shareholder value
through the formulation of corporate strategy.
 Review and Update Banking Laws
o The Narasimham committee has recommended that the provisions of the RBI Act,
Banking Regulation Act, State Bank of Act, and other laws be reviewed and
amended as soon as possible in order to bring them in line with current banking
needs.
 Aside from these major recommendations, the committee has also advocated for faster
computerization, technological advancement, staff training, depoliticization of banks,
professionalism in banking, and a review of bank recruitment, among other things.
NAYAK COMMITTEE
 The Reserve Bank of India constituted on 9 December 1991, a Committee under the
Chairmanship of Shri P.R. Nayank, Deputy Governor to examine the difficulties
confronting the small-scale industries (SSI) in the country in the matter of
securing finance.
P J Nayak Committee
 The P J Nayak Committee or officially the Committee to Review Governance of Boards of
Banks in India was set up by the Reserve Bank of India (RBI) to review the governance of
the board of banks in India. The Committee was set up in January 2014. The Committee
was chaired by P J Nayak, the former CEO and Chairman of Axis Bank.
Bimal Jalan Committee
 A committee on Economic Capital Framework, under the chairmanship of Bimal Jalan
(Former RBI Governor), is known as Bimal Jalan Committee.

https://t.me/prelimbits
142

RAKESH MOHAN COMMITEE ON INDIAN RAILWAYS 2002


 Rakesh Mohan Committee observed that Indian Railways over the past decade (1991-2002)
has fallen into a vicious cycle of under investment, mis-allocation of scarce resources,
increasing indebtedness, poor customer service and rapidly deteriorating economics.
The root cause of the decade of decline is an unstable political system increasingly driven
by short-term political compulsions.
 This committee came up with radical suggestion of corporatization of Indian Railways. It
suggested, that Indian Railways must eventually be corporatized into the Indian Railways
Corporation (IRC).
Other committee related to railway reforms
 Prakash Tandon Committee (1994), Khanna Committee (1998), Sam Pitroda Committee
(2012) and Bibek Debroy committee (2015).

99. With reference to India, which one of the following statements is NOT correct? [2003]
A. IPCL is India's largest petrochemical company
B. RIL is the largest private sector company in India
C. MTNL is listed on NYSE
D. BSNL is the first telecom service organization in India to launch a nationwide cellular service
at one time
Ans. *

OIL AND NATURAL GAS CORPORATION (ONGC)


 The Oil and Natural Gas Corporation (ONGC) is an Indian central public sector undertaking
under the ownership of Ministry of Petroleum and Natural Gas, Government of India. It is
headquartered in Dehradun.
 ONGC was founded on 14 August 1956 by the Government of India. It is the largest
government-owned-oil and gas explorer and producer in the country, and produces
around 70% of India's crude oil (equivalent to around 57% of the country's total demand)
and around 84% of its natural gas. In November 2010, the Government of India conferred
the Maharatna status to ONGC.
RELIANCE INDUSTRIES LIMITED
 Reliance Industries Limited is an Indian multinational conglomerate company,
headquartered in Mumbai. It has diverse businesses including energy, petrochemicals,
natural gas, retail, telecommunications, mass media, and textiles. Reliance is one of the most
profitable companies in India, the largest publicly traded company in India by market
capitalisation, and the largest company in India as measured by revenue. It is also one of
the largest employers in India, with over 300,000 employees in the world.
 The company is ranked 100th on the Fortune Global 500 list of the world's biggest
corporations as of 2022. Reliance continues to be India's largest exporter, accounting for 7%
of India's total merchandise exports and it has access to markets in over 100 countries.
Reliance is responsible for almost 5% of the Government of India's total revenue from

https://t.me/prelimbits
143

customs and excise duty. It is also the highest income tax payer in the private sector in
India.
 The company has relatively little free cash flow and high corporate debt.
MAHANAGAR TELEPHONE NIGAM LIMITED (MTNL)
 Mahanagar Telephone Nigam Limited (MTNL) (d/b/a MTNL) is a wholly owned subsidiary
of Bharat Sanchar Nigam Limited which is in turn under the ownership of Department of
Telecommunications, Ministry of Communications, Government of India.
 Headquartered in New Delhi, India. MTNL Provides services in the metro cities of
Mumbai and New Delhi in India and in the island nation of Mauritius in Africa.
 It had a monopoly in Mumbai and New Delhi until 1992, when the telecom sector was
opened to other service providers. "Transparency makes us different" is its motto.
 The Bharat Sanchar Nigam Limited currently holds 100% of its stock.
 As of June 2021, it has 3.28 million subscribers. It is also a wholly owned subsidiary of a
central public sector unit.
 In 2012 MTNL moves from NYSE to OTCQX.
https://www.thehindubusinessline.com/markets/stock-markets/MTNL-moves-from-NYSE-
to-OTCQX/article20562795.ece
BHARAT SANCHAR NIGAM LIMITED (D/B/A BSNL)
 Bharat Sanchar Nigam Limited (d/b/a BSNL) is a central public sector undertaking
headquartered in New Delhi, India. It is under the ownership of Department of
Telecommunications, Ministry of Communications, Government of India.
 It was incorporated on 1 October 2000 by the Government of India. Its top official is
designated as Chairman and Managing Director who is a central government civil servant of
the Indian Communication Finance Service cadre or a central government engineer of the
Indian Telecommunications Service cadre.
 It provides mobile voice and internet services through its nationwide telecommunications
network across India. It is the largest government-owned-wireless telecommunications
service provider in India.
 The government-owned BSNL launched a nationwide mobile phone service on September
2, 2002

100. Which one of the following statements is NOT correct? [2003]


A. India is the second largest producer of nitrogenous fertilizers in the world.
B. India is the ninth largest steel producing country in the world.
C. India is the second largest producer of silk in the world.
D. India ranks third in the world in coal production.
Ans. *

https://t.me/prelimbits
144

PRODUCTION VOLUME OF NITROGEN FERTILIZER WORLDWIDE IN 2018

COUNTRIES BY STEEL PRODUCTION

WORLD SILK PRODUCTION BY COUNTRY


 Worldwide 609,332 tonnes of silk is produced per year.
 China is the largest silk producer in the world with 403,021 tonnes production per year.
 China produces alone more than 60 % of world's silk.
 India comes second with 161,127 tonnes yearly production.
 With 17,912 tonnes of production per year, Uzbekistan is the third largest producer of silk.

https://t.me/prelimbits
145

LIST OF COUNTRIES BY COAL PRODUCTION

101. During the year 2000-01 which one of the following Industries recorded the highest
growth rate in India? [2003]
A. Cement
B. Coal
C. Electricity
D. Steel
Ans. D

The steel industry recorded highest growth rate of 7%, electricity, generation growth rate was 4%,
coal industry growth was 3.3%.

102. Which one of the following statements is correct? [2003]


A. Alliance Air is a wholly-owned subsidiary of Indian Airlines
B. The Airports Authority of India manages seven of the country's international airports
C. The Airports Authority of India is the regulatory organisation for enforcing civil air
regulations in India
D. It is the function of Directorate General of Civil Aviation to plan and construct runways and
terminal buildings and to provide air safety services
Ans. * A

ALLIANCE AIR
 Alliance Air (formerly Air India Regional) is a wholly-owned subsidiary of AIAHL (AI Assets
Holding Ltd.) which is a Special Purpose Vehicle (SPV ) formed by Government of India after
the disinvestment of Air India Limited.
 It was founded in April 1996 as a wholly-owned subsidiary of Indian Airlines (later
merged with Air India in 2011 and remained a subsidiary till 2022) and mainly operates
domestic routes as part of the government's Regional Connectivity Scheme.
AIRPORTS AUTHORITY OF INDIA
 Airports Authority of India (AAI) was constituted by an Act of Parliament and came into

https://t.me/prelimbits
146

being on 1st April 1995 by merging erstwhile National Airports Authority and
International Airports Authority of India.
 AAI, is a public sector enterprise under the ownership of the Ministry of Civil Aviation,
Government of India.
 It is responsible for creating, upgrading, maintaining, and managing civil
aviation infrastructure in India.
 It provides Communication Navigation Surveillance/Air Traffic Management (CNS/ATM)
services over the Indian airspace and adjoining oceanic areas. AAI currently manages a total
of 137 airports, including 34 international airports, 10 Customs Airports, 81 domestic
airports, and 23 Civil enclaves at Defense airfields.
 Functions →
o AAI's main functions include, the building, alteration, and administration of
passenger terminals, the development and management of cargo terminals,
and the creation and maintenance of apron infrastructure such as runways,
parallel taxiways, and apron.
o International and domestic airports, as well as civil enclaves, are designed, built,
operated, and maintained.
o Control and management of Indian airspace beyond the country's geographical
bounds, as recognised by International Civil Aviation Organization (ICAO).
o Passenger terminal construction, modification, and management.
o It also offers CNS (Communication Navigation Surveillance).
o It is in charge of developing, managing, maintaining, and upgrading the
country's aviation infrastructure. The AAI is in charge of international airports,
domestic airports, customs airports, and civil enclaves on defence airfields.
DIRECTORATE GENERAL OF CIVIL AVIATION

103. Debenture holders of a company are its: [2003]


A. shareholders
B. creditors

https://t.me/prelimbits
147

C. debtors
D. directors
Ans. B

DEBENTURE
 Debenture is a long-term bond issued by a company in return for a loan which have a
fixed rate of interest.
Shareholders Debenture holders
Shareholders are the owners of the company. Debenture holders are merely lenders to the
company and are considered to be creditors.
Shareholders actively participate in the Debenture holders cannot participate in the
decision making process of the company. decision making process.
Shareholders are entitled to receive dividends, Since they have lent money to the company,
which is basically a share in the profits of the debenture holders are entitled to receive
company. interest.
Despite generating profits, a company may Whether it generates profits or not, a company
choose not to pay dividends to its is obligated to pay interest to its debenture
shareholders. holders.
DEBTOR
 A debtor is a person or an entity that owes money to another, which could be any
individual or institution (including the government). In most cases, the debtor has to pay
interest on debt along with the principal debt.

104. Which of the following is not a recommendation of the task force on direct taxes
under the chairmanship of Dr. Vijay L. Kelkar in the year 2002? [2004]
A. Abolition of Wealth Tax
B. Increase in the exemption limit of personal income to Rs. 1.20 lakh for widows
C. Elimination of standard deduction
D. Exemption from tax on dividends and capital gains from the listed equity
Ans. B

VIJAY KELKAR COMMITTEE (2002) RECOMMENDATIONS ON DIRECT TAXES


 Impetus to direct tax reforms in India, came with the recommendations of the Task Force
on Direct & Indirect Taxes under the chairmanship of Vijay Kelkar in 2002.
 The main recommendations of this task force related to the direct taxes related to
increasing the income tax exemption limit, rationalization of exemptions, abolition of
long term capital gains tax, abolition of wealth tax etc.
 Its key recommendations were as follows →
 Administration of Direct Tax
o The taxpayer services should be extended both in quality and quantity and taxpayers
should get easy access through internet and email.
o PAN (Permanent Account Number) should be expanded and it should cover all

https://t.me/prelimbits
148

citizens.
o Block assessment of search and seizure cases should be abolished.
o To clear the backlog, the department should outsource the data entry work.
o All returns and issue of refunds should be completed in a four month period.
Dispatch of refunds should be outsourced.
o Government should establish a Tax Information Network to modernize, simplify and
rationalize tax collection, particular TDS and TCS.
o Abolish the requirement of Tax Clearance Certificate on leaving the country.
o Empower CBDT with appropriate administrative and financial powers.
 Personal income tax
o Increase in exemption limit to Rs.1 lakh for the general categories of taxpayers and
further exemption for senior citizens and widows [1.50 lakh].
o Rationalize income tax slabs, eliminate surcharge on personal income tax.
o Incentivise home loans by providing interest subsidy on home loans @2%.
o Increase deduction under Section 80C for contribution to pension funds.
 Corporation Tax
o Reduce the Corporate tax to 30% for domestic companies and 35% for foreign
companies.
o The listed companies should be exempted from tax on dividends and capital gains.
o Increase rate of depreciation for plant and machinery.
o Abolish Minimum Alternate Tax.
 Wealth Tax
o Abolition of wealth tax

105. As per 1991 Census, which one of the following groups of Union Territories had the
highest literacy rate? [2004]
A. Chandigarh and Dadra & Nagar Haveli
B. Delhi and Andaman & Nicobar Islands
C. Andaman & Nicobar Islands and Pondichery
D. Pondicherry and Delhi
Ans. D

As per 1991 census, Pondicherry and Delhi had the literacy rates of 75.3 and 74.7 respectively.

106. Consider the following statements: [2004]


As per 2001 Census

1. the two States with the lowest sex ratio are Haryana and Punjab
2. the two States with the lowest population per sq. km. of area are Meghalaya and Mizoram
3. Kerala has both the highest literacy rate and sex ratio
Which of the statements given above is/are correct?

A. 3 only

https://t.me/prelimbits
149

B. 2 and 3
C. 1 and 2
D. 1 and 3
Ans. [options does not match]

Census 2001, Sex ratio

 Haryana = 861
 Sikkim = 875
 Punjab = 876
Population per sq. km [2001]

 Arunachal = 13
 Mizoram = 42
 Meghalaya =103
Highest Literacy Rate And Sex Ratio [2001]

 Kerala has both the highest literacy rate and sex ratio.
SEX RATIO IN INDIA
Sex ratio is used to describe the number of females per 1000 of males. Sex ratio is a valuable
source for finding the population of women in India and what is the ratio of women to that of men
in India. In the Population Census of 2011 it was revealed that the population ratio in India 2011 is
940 females per 1000 of males.

The Sex Ratio 2011 shows an upward trend from the census 2001 data. Census 2001 revealed
that there were 933 females to that of 1000 males. Since decades India has seen a decrease in the
sex ratio 2011, but since the last two of the decades there has been in slight increase in the sex
ratio. Since the last five decades the sex ratio has been moving around 930 of females to that of
1000 of males.

Latest Data 2001 Census


Rank State Sex Ratio Child Sex Ratio Child Sexratio
Sexratio
- India 943 919 933 927
1 Kerala 1084 964 1058 960
2 Puducherry 1037 967 1001 967
3 Tamil Nadu 996 943 987 942
4 Andhra Pradesh 993 939 978 961
5 Chhattisgarh 991 969 989 975
6 Meghalaya 989 970 972 973
7 Manipur 985 930 974 957
8 Orissa 979 941 972 953
9 Mizoram 976 970 935 964
10 Goa 973 942 961 938
11 Karnataka 973 948 965 946
12 Himachal Pradesh 972 909 968 896

https://t.me/prelimbits
150

13 Uttarakhand 963 890 962 908


14 Tripura 960 957 948 966
15 Assam 958 962 935 965
16 West Bengal 950 956 934 960
17 Jharkhand 948 948 941 965
18 Lakshadweep 946 911 948 959
19 Arunachal Pradesh 938 972 893 964
20 Madhya Pradesh 931 918 919 932
21 Nagaland 931 943 900 964
22 Maharashtra 929 894 922 913
23 Rajasthan 928 888 921 909
24 Gujarat 919 890 920 883
25 Bihar 918 935 919 942
26 Uttar Pradesh 912 902 898 916
27 Punjab 895 846 876 798
28 Sikkim 890 957 875 963
29 Jammu and Kashmir 889 862 892 941
30 Haryana 879 834 861 819
31 Andaman and Nicobar 876 968 846 957
Islands
32 Delhi 868 871 821 868
33 Chandigarh 818 880 777 845
34 Dadra and Nagar Haveli 774 926 812 979
35 Daman and Diu 618 904 710 926
DENSITY OF INDIA
As a result of the gradual increase in the population of India with the passing of each day, the
population density of India per square km is also quickly on the rise. A survey of the Indian
population density 2011 shows quite a considerable rise in the figures of population density in
India. The records of population density 2011 of India state that the density 2011 has increased
from a figure of 324 to that of 382 per square kilometre, which is considerably higher than the
average population density of the world 2011, which are 46 per square kilometre.

While the National Capital Region area of Delhi possesses the highest of the population density
2011 among the states of India having a statistics of 11,297 per square kilometre, the state of
Arunachal Pradesh has the lowest record of population density having just 17 per square
kilometre.

# State Area Sq. Km Density 2011 Density


2001
- India (Average) 3,287,240 382 324
1 Uttar Pradesh 240,928 829 690
2 Maharashtra 307,713 365 315
3 Bihar 94,163 1,106 881
4 West Bengal 88,752 1,028 903
5 Andhra Pradesh 275,045 308 277

https://t.me/prelimbits
151

6 Madhya Pradesh 308,252 236 196


7 Tamil Nadu 130,060 555 480
8 Rajasthan 342,239 200 165
9 Karnataka 191,791 319 276
10 Gujarat 196,244 308 258
11 Orissa 155,707 270 236
12 Kerala 38,852 860 819
13 Jharkhand 79,716 414 338
14 Assam 78,438 398 340
15 Punjab 50,362 551 484
16 Chhattisgarh 135,192 189 154
17 Haryana 44,212 573 478
18 Delhi 1,483 11,320 9,340
19 Jammu and Kashmir 222,236 56 46
20 Uttarakhand 53,483 189 159
21 Himachal Pradesh 55,673 123 109
22 Tripura 10,486 350 305
23 Meghalaya 22,429 132 103
24 Manipur 22,327 128 103
25 Nagaland 16,579 119 120
26 Goa 3,702 394 364
27 Arunachal Pradesh 83,743 17 13
28 Puducherry 490 2,547 2,034
29 Mizoram 21,081 52 42
30 Chandigarh 114 9,258 7,900
31 Sikkim 7,096 86 76
32 Andaman and Nicobar Islands 8,249 46 43
33 Dadra and Nagar Haveli 491 700 449
34 Daman and Diu 111 2,191 1,413
35 Lakshadweep 30 2,149 1,895

107. Consider the following statements: [2004]


1. Regarding the procurement of food-grains, Government of India follows a procurement
target rather than an open–ended procurement policy.
2. Government of India announces minimum support prices only for cereals.
3. For distribution under Targeted Public Distribution System (TPDS), wheat and rice are issued
by the Government of India at uniform central issue prices to the States/Union Territories.
Which of the statements given above is/are correct?

A. 1 and 2
B. 2 only
C. 1 and 3
D. 3 only
Ans. C

 Regarding the procurement of food-grains, Government of India follows a procurement

https://t.me/prelimbits
152

target rather than an open-ended procurement policy. In case, there is no procurement


target. The government allows the procurement agencies like the FCI to buy whatever
is offered by the farmers for sale at MSP.
 Minimum Support Price is announced for 22 crops. [+ Sugarcane]
 Wheat and Rice are issued by the central Government at Uniform Central Issue Prices
(CIPs) to states and Union Territories for distribution under TPDS. The difference between
the economic cost and issue price of food grains is reimbursed to the FCI by the central
Government in the form of subsidy.
CENTRAL ISSUE PRICE UNDER NFSA
 Foodgrains under NFSA were to be made available at subsidized prices of Rs.3/2/1 per kg
for rice, wheat and coarse grains respectively for an initial period of three years from the
date of commencement of the Act (July 13, 2013).
 Thereafter, prices were to be fixed by the Central Government from time to time, but not
exceeding MSP. Government has decided from time to time to continue the above
mentioned subsidized prices under NFSA.
 It has been decided with the approval of Prime Minister‘s Office to continue to existing
Central Issue Prices of foodgrains i.e. Rs. Rs.3/2/1 per kg for rice, wheat and coarse
grains respectively under NFSA until further orders. [The states fix retail price to be
charged at fair price shops.]
PROCUREMENT POLICY
 State Government agencies and Food Corporation of India (FCI) purchase wheat & paddy
within the stipulated period with prescribed Fair Average Quality (FAQ) specification at
Minimum Support Price (MSP) for Central Pool.
 The estimates for procurement of wheat and paddy are finalized by Government of India in
consultation with State Governments and Food Corporation of India, before the
commencement of each marketing season based upon estimated production, marketable
surplus and agricultural crop pattern.
 Further, the different types of coarsegrains are procured by State Governments itself in
consultation with FCI to the extent that the concerned State Govt. may utilise the same for
distribution under National Food Security Act (NFSA) as well as Other Welfare Schemes
(OWS).
Centralized Procurement System

 Under Centralized Procurement System, the procurement of foodgrains in Central Pool are
undertaken either by FCI directly or State Government agencies procures the foodgrains
and handover the stocks to FCI for storage and subsequent issue against GOI allocations in
the same State or movement of surplus stocks to other States.
 The cost of the foodgrains procured by State agencies is reimbursed by FCI as soon as the
stocks are delivered to FCI as per cost-sheets issued by GOI.
Decentralized Procurement System (DCP)

 The scheme of Decentralized Procurement of foodgrains was introduced by the


https://t.me/prelimbits
153

Government in 1997-98 with a view to enhancing the efficiency of procurement and PDS
and encouraging local procurement to the maximum extent thereby extending the benefits
of MSP to local farmers as well as to save on transit costs. This also enables procurement of
foodgrains more suited to the local taste.
 Under this scheme, the State Government itself undertakes direct purchase of
paddy/rice and wheat and also stores and distributes these foodgrains under NFSA
and other welfare schemes.
 The Central Government undertakes to meet the entire expenditure incurred by the State
Governments on the procurement operations as per the approved costing. The Central
Government also monitors the quality of foodgrains procured under the scheme and
reviews the arrangements made to ensure that the procurement operations are carried
smoothly.

108. India continues to be dependent on imports to meet the requirement of oilseeds in


the country because: [2004]
1. farmers prefer to grow food grains with highly remunerative support prices.
2. most of the cultivation of oilseed crops continues to be dependent on rainfall.
3. oils from the seeds of free origin and rice bran have remained unexploited.
4. it far cheaper to import oilseeds than to cultivate the oilseed crops.
Which of the statements given above are correct?

A. 1 and 2
B. 1, 2 and 3
C. 2 and 3
D. 1, 2, 3 and 4
Ans. B

WHY INDIA IS NOT SELF-SUFFICIENT IN OILSEED PRODUCTION

 Micro-irrigation, quality seeds, marketing infrastructure and government


policies are the four main concerns for oil seed and oil producers in India. Policy goof-
ups, as in the above paragraph add to the woes of farmers.
 When it comes to crop-wise coverage of irrigated area (2016-17) almost 70 per cent of
cereals and 52 per cent of foodgrains are covered under irrigation. But when it comes oil
seeds, the coverage of the irrigated area is 27.8 per cent.
 From 5.16 MT in 1950-51 to 33. 42 MT in 2019-20, oil seed production in India has
witnessed growth at a slow pace. The average per hectare yield of major oilseeds is over 50
per cent lower than average world yields in several crops.
 At present, India meets nearly 55% to 60% of its edible oil demand through
imports. Therefore, India needs to be independent in oil production to meet the domestic
consumption demand.
 Palm oil (Crude + Refined) constitutes roughly around 62% of the total edible oils
imported and are imported mainly from Indonesia and Malaysia, while Soyabean oil
https://t.me/prelimbits
154

(22%) is imported from Argentina and Brazil and Sunflower oil (15%) is imported mainly
from Ukraine and Russia.
 The major edible oils consumed in the country are mustard, soyabean, groundnut,
sunflower sesame oil, niger seed, safflower seed, castor, and linseed (primary source)
and coconut, palm oil, cottonseed, rice bran, solvent extracted oil, tree and forest
origin oil.
 Solvent Extractors Association of India (SEA) members have been demanding higher
Minimum Support Price (MSP) for oilseeds, so that more farmers are drawn towards oilseed
cultivation. The demand is also to link import duty on edible oils to MSP. Industry players
say that India‘s domestic oilseed production needs to go up about 54 MT by 2025 to reduce
edible oil imports by 10 MT annually.
Related Government Initiatives

 The government of India launched National Mission on Edible Oils-Oil Palm as a


centrally-sponsored scheme, being implemented jointly by the central and state
governments with a special focus in the northeast region and the Andaman and
Nicobar Islands.
 It is proposed to have an additional 6.5 lakh hectares for palm oil by 2025-26.
109. [2004]
Assertion (A): India does not export natural rubber.
Reason (R): About 97% of India's demand for natural rubber is met from domestic
production.
A. Both A and R are true and R is the correct explanation of A
B. Both A and R are true but R is NOT a correct explanation of A
C. A is true but R is false
D. A is false but R is true
Ans. *

India does Export natural Rubber.

https://www.researchgate.net/publication/338449846_Export_-
_Import_Performance_of_Natural_Rubber_in_India

NATURAL RUBBER
 Rubber consists of polymers of the organic compound isoprene, with minor impurities of
other organic compounds.
 The main chemical constituents of rubber are elastomers, or ―elastic polymers,‖ large
chainlike molecules that can be stretched to great lengths and yet recover their original
shape.
 Thailand and Indonesia are two of the leading rubber producers.
 Hevea brasiliensis,or rubber tree or rubber plant, is a flowering plant belonging to the
spurge family Euphorbiaceae. The milky latex extracted from the tree is the primary source

https://t.me/prelimbits
155

of natural rubber.
 Brasiliensis is a tall deciduous tree.
 The tree requires a tropical or subtropical climate with a minimum of about 1,200 mm
per year of rainfall, and no frost. If frost does occur, the results can be disastrous for
production.
 In the wild, the tree can reach a height of up to 140 feet.
 In plantations, the trees are generally smaller for two reasons:
o Trees grow more slowly when they are tapped for latex, and
o Trees are generally cut down after only 30 years, because latex production declines
as trees age, and they are no longer economically productive.
 The natural rubber tree takes between seven and ten years to deliver the first harvest.
 As latex production declines with age, rubber trees are generally felled when they reach the
age of 25 to 30 years.
 Originally, the South American rubber tree grew only in the Amazon rainforest.
 Early attempts were made in 1873 to grow brasilensis outside Brazil.
 Gradually, rubber was extensively propagated in the British colonies including India.
Where does India stand in terms of the production and consumption of natural rubber?

 The British established the first rubber plantation in India in 1902 on the banks of the
river Periyar in Kerala.
 India is currently the fifth largest producer of Natural Rubber in the world with one of
the highest productivity.
 The production of the material improved by 8.4%, to 7,75,000 tonnes, during 2021-22
compared to 2020-21.
 It also remains the second biggest consumer of the material globally.
 About 40% of India‘s total natural rubber consumption is currently met
through imports.
 Top Rubber Producing States: Kerala > Tamil Nadu > Karnataka.
110. In the last one decade, which one among the following sectors has attracted the
highest Foreign Direct Investment inflows into India? [2004]
A. Chemicals other than fertilizers
B. Services sector
C. Food processing
D. Telecommunication
Ans. B

INVESTMENT CLIMATE IN INDIA HAS IMPROVED CONSIDERABLY SINCE THE OPENING UP OF THE
ECONOMY IN 1991.

 This is primarily attributed to ease in FDI rules in India. India, today is a part of the top 100
clubs on Ease of Doing Business (EoDB).FDI inflows in India stood at $45.15 bn in 2014-15
and have consistently increased since then. Moreover, total FDI inflow grew by 65.3%, i.e.

https://t.me/prelimbits
156

from $266.21 bn in 2007-14 to $440.01bn in 2014-21 and FDI equity inflow also increased
by 68.6% from $185.03 bn during 2007-14 to $312.05 bn (2014-21).
 India has attracted a total FDI inflow of $27.37 bn during the first four months of F.Y. 2021-
22 which is 62% higher as compared to the corresponding period of F.Y. 2020-21 ($ 16.92
bn).
 India received the highest annual FDI inflows of $84,835 mn in FY 21-22 overtaking last
year‘s FDI by $2.87 bn. Also, FDI equity inflow in FY 2021-22 were $ 59,825 mn.
 FDI Equity inflow in Manufacturing Sectors have increased by 76% in FY 2021-22 ($ 21.34
bn) compared to previous FY 2020-21 ($ 12.09 bn).
 Total FDI inflows in the country in the last 22 years (April 2000 - March 2022) are $ 847 bn
while the total FDI inflows received in the last 8 years (April 2014- March 2022) was $ 523
bn which amounts to nearly 40% of total FDI inflow in last 22 years.
 In FY 2014-15, FDI inflow in India stood at mere $ 45.15 bn, which increased to $ 60.22 bn in
2016-17 and further to the highest ever annual FDI inflow of $ 83.57 bn reported during the
FY 2021-22.
 Total FDI inflows in the country in the second quarter of FY 2022 (July - September) is $ 16.6
Bn and total FDI equity inflows stands at $ 10.3 Bn.
 Singapore (27.01%), USA (17.94%), Mauritius (15.98%), Netherland (7.86%) and
Switzerland (7.31%) emerge as top 5 countries for FDI equity inflows into India FY 2021-22.
 Top 5 sectors receiving highest FDI Equity Inflow during FY 2021-22 are
o Computer Software & Hardware (24.60%),
o Services Sector (Fin., Banking, Insurance, Non Fin/Business, Outsourcing, R&D,
Courier, Tech. Testing and Analysis, Other) (12.13%),
o Automobile Industry (11.89%),
o Trading 7.72% and
o Construction (Infrastructure) Activities (5.52%).
 Top 5 States receiving highest FDI Equity Inflow during FY 2021-22 are Karnataka
(37.55%), Maharashtra (26.26%), Delhi (13.93%), Tamil Nadu (5.10%) and Haryana (4.76%)
111. Consider the following statements: [2004]
1. The National Housing Bank the apex institution of housing finance in India, was set up as a
wholly-owned subsidiary of the Reserve Bank of India
2. The Small Industries Development Bank of India was established as a wholly owned
subsidiary of the Industrial Development Bank of India
Which of the statements given above is/are correct?

A. 1 only
B. 2 only
C. Both 1 and 2
D. Neither 1 nor 2
Ans. C

https://t.me/prelimbits
157

NATIONAL HOUSING BANK (NHB)


 The National Housing Bank (NHB) was established in July 1988 as the apex bank in the
field of housing finance by the National Housing Bank Act, 1987. It is the primary agency
responsible for promoting housing finance institutions at the regional and local levels, as
well as providing financial and other assistance to such institutions.
 National Housing Bank is a government-owned corporation. The government took
over the NHB from the RBI in 2019 after purchasing the entire stake for
Rs. 1,450 crores.
o The move is in response to the recommendation of the Narasimham-II
committee report from October 2001.
 NHB raises funds via debt instruments such as bonds, debentures, and borrowings.
 Its bonds, which are guaranteed by the Government of India, are eligible securities for
commercial banks to use in order to meet the statutory liquidity requirements under
the Banking Regulation Act.
 Furthermore, it receives external assistance from international organisations such as USAID
and OECF Japan.
 It accepts deposits through commercial banks' Home Loan Accounts.
 The National Housing Bank refinances Housing Finance Companies, which are located
throughout the country and account for the majority of the market, followed by commercial
banks, co-operative banks, and Land Development Banks.
 National Housing Bank offers refinancing to housing finance companies at varying rates
based on loan size.
Historical perspective

 The Sub-Group on Housing Finance for the Seventh Five Year Plan (1985-1990) identified
the lack of long-term finance to individual households on a significant scale as a major
impediment to housing sector progress and recommended the establishment of a national
level institution.
 The Committee of Secretaries considered the recommendation and formed the High-Level
Group, chaired by Dr C. Rangarajan, the then-Deputy Governor of the Reserve Bank of
India, to examine the proposal and recommend the establishment of the National Housing
Bank as an autonomous housing finance institution.
 The High-Level Group's recommendations were accepted by the Government of India.
 On February 28, 1987, the Prime Minister of India announced the decision to establish
the National Housing Bank (NHB) as an apex level institution for housing finance while
presenting the Union Budget for 1987-88.
 Following that, the National Housing Bank Bill, which provided the legislative framework
for the establishment of NHB, was passed by Parliament in the winter session of 1987 and
became an Act of Parliament with the assent of the President of India on December 23,
1987.
 The National Housing Policy of 1988 called for the establishment of the NHB as the apex

https://t.me/prelimbits
158

housing institution.
 NHB was established on July 9, 1988, under the National Housing Bank Act of 1987.
 The entire paid-up capital was contributed by the Reserve Bank of India.
 The general supervision, direction, and management of NHB's affairs and business are
vested in a Board of Directors under the Act.
 NHB's headquarters are in New Delhi.
Objectives

 To promote a sound, healthy, viable, and cost-effective housing finance system capable of
serving all segments of the population, as well as to integrate the housing finance system
with the overall financial system.
 To encourage the development of a network of dedicated housing finance institutions to
adequately serve various regions and income groups.
 To increase resources for the sector and direct them toward housing.
 To make mortgages more affordable.
 To monitor the activities of housing finance companies as per the supervisory powers it
derives from the Act.
 To encourage the expansion of the supply of buildable land as well as building materials for
housing, as well as to upgrade the country's housing stock.
 Encourage government agencies to become facilitators and suppliers of serviced land for
housing.
Eligibility Criteria for obtaining refinance from NHB

 The housing finance company must have a minimum share capital of Rs. 3 crore and
a promoter contribution of at least 25% of the total capital.
 It must be registered as a publiclimited company. Long-term financing for house
construction/purchase for residential purposes must account for at least 75% of loans.
 It must not be a subsidiary of any construction firm.
 The top management of the Housing Finance Company should not hold a similar position in
the promoters' Construction Company.
National Housing Bank – Role & Importance

 Ensuring adequate financing for housing infrastructure development, as well as a


continuous flow of liquidity to various housing finance institutes for timely financing to all
income segments.
 Ensure proper regulation and oversight of all housing finance companies operating
throughout the country.
 The NHB is also in charge of auditing such companies, ensuring their compliance with the
relevant guidelines, and ensuring that the organisations make credit available at affordable
rates in order to provide housing facilities for all.
 The NHB was also created with the goal of increasing the number of housing units in the
country.
o As a result, the National Housing Bank plays an important role in making land

https://t.me/prelimbits
159

available for housing development by acting as a facilitator to enable companies


in the housing sector to raise funds and smooth the entire function, resulting in
increased efficiency and productivity.
National Housing Bank – Functions

 It is primarily in charge of registering and supervising all Housing Finance Companies


(HFCs), as well as maintaining surveillance through on-site and off-site mechanisms and
coordinating with other regulators.
 Supervision and control of housing companies operating in India under the authority
granted by the National Housing Bank Act.
 Raising funds on a large scale and refinancing for housing finance companies, cooperative
banks, and other housing agencies for onward lending to individuals and housing
infrastructure companies.
 Regulating and ensuring that housing finance companies meet all regulatory capital
requirements outlined in the BASEL guidelines.
 Assuring that they have a proper risk management system in place, as well as good
governance practises, and so on.
National Housing Bank – RESIDEX

 The NHB RESIDEX is India's first official housing price index, created by the National
Housing Bank (NHB) in response to Ministry of Finance, Government of India directives. A
few key points regarding NHB RESIDEX are:
o The NHB RESIDEX was developed with the assistance of a Technical Advisory
Committee (TAC) comprised of housing stakeholders.
o Since its inception in July 2007, the NHB RESIDEX has been updated on a regular
basis until March 2015, with the year 2007 serving as the baseline. During this time,
the NHB RESIDEX gradually expanded to cover 26 cities across the country.
o The NHB RESIDEX was initially calculated using market data. However, beginning in
2010, it was based on valuation data obtained from housing finance companies
(HFCs) and banks.
o The NHB conducted a review of the processes used to compute the index, as well as
the base year, in order to make the RESIDEX more current and in line with the current
macroeconomic scenario.
o Following that, a redesigned NHB RESIDEX with broader geographical coverage and
a broader scope was released. It was calculated using the fiscal year 2012-13 as the
base year and is updated tillMarch 2018.
o With effect from April 2018, a new series of the NHB RESIDEX has been published,
with the fiscal year 2017-18 as the new base year, and is updated up to the current
quarter.
SMALL INDUSTRIES DEVELOPMENT BANK OF INDIA (SIDBI)
 Small Industries Development Bank of India (SIDBI) is the apex regulatory body for overall
licensing and regulation of micro, small and medium enterprise finance companies in

https://t.me/prelimbits
160

India. It is under the jurisdiction of Ministry of Finance , Government of India


headquartered at Lucknow and having its offices all over the country.
 The SIDBI was established on April 02, 1990 by Government of India, as a wholly
owned subsidiary of IDBI Bank .
 It was delinked from IDBI w.e.f. March 27, 2000.
 Its purpose is to provide refinance facilities to banks and financial institutions and engage
in term lending and working capital finance to industries, and serves as the principal
financial institution in the Micro, Small and Medium Enterprises (MSME) sector.
 SIDBI is one of the four All India Financial Institutions regulated and supervised by the
Reserve Bank of India; other three are India Exim Bank, NABARD and NHB.
 But recently NHB came under government control by taking more than 51% stake. They
play a statutory role in the financial markets through credit extension and refinancing
operation activities and cater to the long-term financing needs of the industrial sector.[4]
 SIDBI is active in the development of Micro Finance Institutions through SIDBI
Foundation for Micro Credit, and assists in extending microfinance through the Micro
Finance Institution (MFI) route. Its promotion & development program focuses on rural
enterprises promotion and entrepreneurship development.[
 In order to increase and support money supply to the MSE sector, it operates a refinance
program known as Institutional Finance program. Under this program, SIDBI extends
Term Loan assistance to Banks, Small Finance Banks and Non-Banking Financial Companies.
Besides the refinance operations, SIDBI also lends directly to MSMEs.
NHB (1988) SIDBI (1990)
National Housing Bank Small Industries Development Bank of India
Original boss: RBI (100%). But, 2019-Apr: RBI BOSS: SBI, LIC, IDBI other public sector banks,
sold 100% to Govt. insurance companies etc.
Regulate by RBI Regulated by RBI
Finance to banks and NBFCs for housing Operates Credit Guarantee fund, Small
projects. Enterprises Development Fund (SEDF).

RESIDEX index to monitor residential real Operates udyamimitra.in for loans to small
estate prices. entrepreneurs & SME via schemes like Mudra,
Stand-up-India.

112. Consider the following statements: [2004]


1. Reserve Bank of India was nationalized on 26 January, 1950
2. The borrowing programme of the Government of India is handled by the Department of
Expenditure, Ministry of Finance
Which of the statements given above is/are correct?

A. 1 only
B. 2 only
C. Both 1 and 2

https://t.me/prelimbits
161

D. Neither 1 nor 2
Ans. D

RBI was established in 1935 and was nationalized on 1 January, 1949. RBI handles the borrowing
programme of the central and State Governments.

At present, the government debt, including market borrowings, is managed by the Reserve Bank of
India.

EVOLUTION OF RBI
1913 Commercial banks were required to register under the Companies Act, but monitoring
was lax. No CRR, SLR, BASEL Norms.
1926 Royal Commission on Indian Currency (Hilton Young Commission) recommends setting
up a central bank named 'Reserve Bank of India'.
1929 Great Depression in USA leads to collapse of 450+ banks in India. So British Indian
Government becomes serious about setting up RBI.
1934 Reserve Bank of India Act was enacted.
1935, April RBI becomes operational from 1st April, with 1st Governor Sir RBI Gov:
1935, July Commercial Banks fulfilling certain conditions were listed in the 2nd Schedule of RBI Act,
& such “Scheduled Banks” were required to keep CRR with RBI.
1943-49 C.D. Deshmukh 2nd FM of India becomes the first INDIAN Governor of RBI. He had also
participated in the Bretton Woods Conference, USA (1944).
1948-49 All private investors’ shares transferred to Govt of India under the RBI transfer of
ownership act 1948. Therefore, RBI governor answerable to Parliament, has to pay
dividend to Government from its profits.
1949 Banking Regulation Act 1949, empowered the RBI to
 Give license to companies to open banks, give permission banks to open new
branches.
 Prescribe auditing and liquidity norms for Banks such as SLR.
 Protect interest of depositors. Force elimination / merger of weak banks.

RBI Governor & Dy Governor

https://t.me/prelimbits
162

 25th Governor: Shaktikanta Das (Retd. IAS, Former finance secretary, Member of 15th
Finance Commission). He replaced Urjit Patel (2018-Dec).
 RBI Act (Section 8) provides for ―NOT MORE than 4‖ Dy. Governors
 Governor and Dy.Governors‘ tenure usually (not always) 3 years.
 Re-appointment is possible.
 They‘re selected by Financial sector regulatory appointment search committee
(FSRASC) headed by the Cabinet Secretary (IAS) → successful candidates‘ names sent to
Appointments Committee of the Cabinet headed by the Prime Minister for final approval.
RBI Offices & Departments

 RBI 4 regions: Northern: Delhi, Eastern: Kolkata, Southern: Chennai, Western: Mumbai
 RBI has various departments looking after Banks, NBFCs, Payment Systems, Foreign
Exchange Management etc.
 Previously individual departments directly acted against violators. WEF 1/4/2017: A New
Enforcement Dept was setup in RBI for centralized action against violators.
RBI: Functions

 Controller of Money Supply: Issues M0 under RBI Act, Makes Monetary Policy.
 Controller of Foreign Exchange: through FEMA Act.
 Banker to Governments & Public Debt Manager
 Banker‘s Bank: Lender of Last resort, Advises in monetary matters.
 Regulator of all ―BANKS‖: through BR Act‘49, Payment Systems‘07
 Regulator of AIFI, NBFC-D & others.
 Promotional Roles: Customer protection through Ombudsman, Financial Inclusion through
PSL norms, 25% rural branch requirements.
 Data Publication & awareness e.g. annual Financial Stability Report
 International Cooperation e.g. BASEL, IMF, G20‘s Financial Stability Board etc.
Balance Sheet of Reserve Bank

ASSETS LIABLITIES
 Gold  Currency
 Foreign exchange [4th in world]  Currency held by public
 Government securities  Vault cash held by commercial banks
 Loan to commercial bank  Deposits of commercial banks with RBI
 Treasury deposits of GOI
Source of RBI‘s Powers

 Reserve Bank of India Act, 1934


 Public Debt Act, 1944
 Government Securities Regulations, 2007
 Banking Regulation Act, 1949
 Foreign Exchange Management Act, 1999
 Securitisation and Reconstruction of Financial Assets and Enforcement of Security Interest
Act, 2002
 Credit Information Companies (Regulation) Act, 2005
 Payment and Settlement Systems Act, 2007

https://t.me/prelimbits
163

113. Consider the following statements: [2005]


1. Poverty Reduction and Growth Facility (PRGF) has been established by the International
Development Association (IDA) to provide further assistance to low income countries facing
high level of indebtedness.
2. Singapore regional Training Institute (STI) is one of the institutes that provides training in
macroeconomic analysis and policy and related subject as a part of programme of the IMF
institute.
Which of the statements given above is/are correct?

A. 1 only
B. 2 only
C. Both 1 and 2
D. Neither 1 nor 2
Ans. B

POVERTY REDUCTION AND GROWTH FACILITY (PRGF)


 The Poverty Reduction and Growth Facility (PRGF) is an arm of the International
Monetary Fund which lends to the world's poorest countries.
 It was created on September 16, 1999, replacing the Enhanced Structural Adjustment
Facility.
 In 2009 the organization sold 212 metric tons of gold in separate off-market transactions
and then gradually sold 191.3 tons of gold to fund the SDR 1¼ billion yearly lending
program to the targeted nations
IMF – SINGAPORE REGIONAL TRAINING INSTITUTE (STI),
 The IMF – Singapore Regional Training Institute (STI), is the International Monetary Fund‘s
(IMF) regional training center for the Asia and Pacific region. It provides government
officials from 37 countries with training and technical assistance in
macroeconomic and financial management and related legal and
statistical issues. Most training takes place in two-week seminars, with shorter events
organized for senior officials.
 The IMF, the Government of Singapore, and the Government of Japan jointly fund the
STI and together guide the training program. STI helps further the work of the Singapore
Cooperation Program, which coordinates the resources available in Singapore for technical
assistance to other countries. It is a key part of Japan‘s financing to the IMF for capacity
development.
 Each year, more than 800 officials participate in STI training in Singapore, and another 100–
200 attend STI courses elsewhere in the region
114. Consider the following statements: [2005]
1. Sensex is based on 50 of the most important stocks available on the Bombay stock
Exchange (BSE).

https://t.me/prelimbits
164

2. For calculating the Sensex, all the stock are assigned proportional weightage.
3. New York Stock Exchange is the oldest stock exchange in the world.
Which of the statements given above is/are correct?

A. 2 only
B. 1 and 3
C. 2 and 3
D. None
Ans. A

 The ‗BSE SENSEX‘ is a value-weighted index composed of 30 stocks and was started in 1
January, 1986.
 The origin of the NYSE can be traced to 17 May, 1792.
 Amsterdam stock exchange (1602) is considered oldest in the world and was established
by the Dutch East India company.
STOCK EXCHANGES / SECONDARY MARKET
Shares are issued through IPO @Primary market. Then, they can be resold at secondary market,
commonly known as Share market or Stock Exchange or Bourses.
 World‘s Oldest: Amsterdam Stock exchange, Netherlands (1602)
 Asia‘s Oldest: Bombay Stock Exchange (BSE: 1875)
 India‘s stock exchanges chronology: BSE → A‘bad → Kolkata → NSE (early 90s)
 Just like the Banks have Core Banking Solutions for e-banking, Stock exchanges also have
their electronic platforms for trading. E.g. BOLT (BSE's On-line Trading System) and NEAT
(National Exchange for Automated Trading). They run using internet facility from VSAT
(Very Small Aperture Terminal) Satellite.
BOMBAY STOCK EXCHANGE
 Indian stock exchange located on Dalal Street in Mumbai.
 Established in 1875 by cotton merchant Premchand Roychand, a Rajasthani Jain
businessman, it is the oldest stock exchange in Asia, and also the tenth oldest in the
world.
 The BSE is the 8th largest stock exchange with an overall market capitalisation of more
than ₹276.713 lakh crore, as of January 2022.
 Only in 1995 did BSE switch to electronic trading after following a paper trading pattern
since 1875.
 BSE‘s Stock Index – SENSEX gives top 30 stock index
 Recognized stock exchange in 1957
 BSE promotes trading in debt instruments, mutual funds and currencies
115. Which one of the following statements is not correct? [2005]
A. Rourkela Steel Plant, the first integrated steel plant in the Public Sector of India was set up
with the Soviet Union collaboration.
B. Salem Steel Plant is a premier producer of stainless steel in India.
C. Maharashtra Elektrosmelt Ltd. is a subsidiary of the Steel Authority of India Ltd.

https://t.me/prelimbits
165

D. Visakhapatnam Steel Plant is a unit of the Rashtriya Ispat Nigam Ltd.


Ans. A

ROURKELA STEEL PLANT (RSP)


 Rourkela Steel Plant (RSP) is the first intergrated steel plant in the public sector in India. It
was set up with German collaboration with an installed capacity of 1 million tonnes in
1959.
 RSP was the first plant in India to incorporate LD technology of steel making. It is also
the first steel plant in SAIL and the only one presently where 100% of slabs are produced
through the cost-effective and quality-centric continuous casting route.

 It is SAIL‘s only plant that produce silicon steels for the power sector and high quality
pipes for the oil & gas sector.
 Its wide and sophisticated product range includes various flat, tubular and coated products.
SALEM STEEL PLANT (SSP) [OWNER SAIL]
 Salem Steel Plant (SSP), a unit of Steel Authority of India Limited (SAIL), is a steel plant
involved in the production of stainless steel.
 It is located along the Salem — Bangalore National Highway 44 in the foothills of
Kanjamalai in Salem district, Tamil Nadu, India.
 The plant has an installed capacity of 70,000 tonnes per annum in its cold rolling mill and
3,64,000 tonnes per annum in the hot rolling mill.
 It also has the country's first stainless steel blanking facility.
MAHARASHTRA ELECTROSMELT LIMITED
 Maharashtra Electrosmelt Limited is a Public incorporated on 17 April 1974.
 It is inolved in Manufacture of electronic valves and tubes and other electronic components
 Maharashtra Elektrosmelt Ltd (MEL), the 99.12% subsidiary of Maharatna Steel Authority of
India Limited (SAIL), has been merged with SAIL. The process of merger of MEL with SAIL
was started in April 2006 and culminated with the receipt of the final order from the
Ministry of Corporate Affairs on June 14, 2011.

https://t.me/prelimbits
166

VISAKHAPATNAM STEEL PLANT


 Visakhapatnam Steel Plant is the integrated steel plants of Rashtriya Ispat Nigam Limited in
Visakhapatnam.
 Founded in 1971, the plant focuses on producing value-added steel, producing 5.773
million tonnes of hot metal, 5.272 million tonnes of crude steel and 5.138 million tonnes of
saleable steel in the 2021-2022 financial year
116. Which one of the following statements is correct? [2006]
Fiscal Responsibility and Budget Management Act (FRBMA) concerns:

A. Fiscal Deficit only


B. Revenue deficit only
C. Both fiscal deficit and revenue deficit
D. Neither fiscal deficit nor revenue deficit
Ans. C

FRBM ACT
 The Fiscal Responsibility and Budget Management (FRBM) Act, 2003 is an act to provide for
the responsibility of the Central Government to ensure intergenerational equity in
fiscal management and long-term macro-economic stability by removing fiscal
impediments in the effective conduct of monetary policy.
 The Act sets a target for the government to establish financial discipline in the economy,
improve public funds management and reduce fiscal deficit.
Why FRBM Act was needed?

 During 1990s and the early 2000s, the borrowing level were very high. It had led to high
fiscal deficit, high revenue deficit, and high debt-to-GDP ratio.
 The high government borrowing and the resultant debt had severely impacted the
health of the Indian economy.
 Moreover, the borrowings were more to pay interest than for any capital formation, which
means we were on the verge of falling into a debt trap.
 All these reasons led to passing of Fiscal Responsibility and Budget Management Act in
2003
FRBM Act objectives

 The FRBM Act aimed to introduce transparency in India‘s fiscal management systems.
 It also aimed to bring fiscal discipline, efficient debt management, and macroeconomic
stability.
 The Act also aimed at maintaining better coordination between fiscal and monetary
policy.
 The long-term vision was to introduce more equitable distribution of India‘s debt over the
years.
Fiscal policy statements to be laid before Parliament

https://t.me/prelimbits
167

 Along with the annual financial statement (Budget) and demands for grants, the Central
Government shall lay in each financial year before both Houses of Parliament the
following statements of fiscal policy namely:
o the Medium-term Fiscal Policy Statement;
o the Fiscal Policy Strategy Statement;
o the Macro-economic Framework Statement;
o the Medium-term Expenditure Framework Statement..
Initial FRBM Targets

These targets were scheduled to be met by 2008-09.

 Revenue Deficit (RD): – RD should be completely eliminated by 2009. The minimum


annual reduction target was 0.5% of GDP.
 Fiscal Deficit (FD): FD should be reduced to 3% of GDP by 2009. The minimum annual
reduction target was 0.3% of GDP.
 Contingent Liabilities – The Central Government shall not give incremental guarantees
aggregating an amount exceeding 0.5% of GDP in any financial year beginning 2004-05.
 Additional Liabilities – Additional liabilities should be reduced to 9% of the GDP by 2004-
05. The minimum annual reduction target in each subsequent year to be 1% of GDP.
 RBI purchase of government bonds – to cease from 1 April 2006. This indicates the
government not to borrow directly from the RBI.
 The targets were not achieved. So, the Act was amended first in 2012 and then in 2015
to relax the realisation of fiscal targets.
N.K.Singh committee recommendations

 Central government believed that the targets are too rigid. So, it set up a committee
under N.K.Singh to review the FRBM Act.
 Targets: The committee suggested using debt as the primary target for fiscal policy and
that the target must be achieved by 2023.
 Fiscal Council: The committee proposed to create an autonomous Fiscal Council with a
chairperson and two members appointed by the Centre (not employees of the government
at the time of appointment)
 Deviations: The committee suggested that the grounds for the government to deviate from
the FRBM Act targets should be clearly specified
 Borrowings: According to the suggestions of the committee, the government must not
borrow from the RBI, except when…
o the Centre has to meet a temporary shortfall in receipts
o RBI subscribes to government securities to finance any deviations
o RBI purchases government securities from the secondary market.
Latest FRBM targets

 Fiscal Deficit: The Central Government shall take appropriate measures to limit the fiscal
deficit upto 3% of GDP by the 2021.
 The Central Government shall also ensure that the general Government debt does not

https://t.me/prelimbits
168

exceed 60%.
 The Central Government debt does not exceed 40%. of GDP by the end of financial year
2024-2025.
 The Central Government does not give additional guarantees with respect to any loan
on security of the Consolidated Fund of India in excess of 0.5% of GDP, in any financial
year.
FRBM escape clause

 The FRBM Act also allows invoking of an escape clause in situations of calamity and
national security. In such situations, the government can deviate from its annual fiscal
deficit target.
Govt may not amend FRBM Act again, but aims for 5.5-6% deficit in FY24

 The central government is unlikely bring any amendments to the Fiscal Responsibility and
Budget Management Act for the third year in a row as it expects that the impact of the
global slowdown on the Indian economy in 2023 may impact expenditure commitments in
financial year 2023-24 (FY24).
 Ministry is likely to stick to its internal fiscal consolidation roadmap and the 2023 Union
Budget may target a fiscal deficit of between 5.5-6 percent of nominal GDP in FY24. The
roadmap aims for a fiscal deficit target of 4.5 percent of GDP by 2025-26.
https://www.business-standard.com/budget/article/govt-may-not-amend-frbm-act-again-but-
aims-for-5-5-6-deficit-in-fy24-
123011800970_1.html#:~:text=The%20last%20amendment%20to%20the,as%20permitted%20und
er%20the%20Act.

117. Which one of the following is the correct statement? Service tax is a/an: [2006]
A. direct tax levied by the Central Government.
B. indirect tax levied by the Central Government.
C. direct tax levied by the State Government.
D. indirect tax levied by the State Government.
Ans. B*

Service Tax has been replaced by the Goods and Services Tax (GST) starting 1 July 2017.

SERVICE TAX
 In India, a service tax is levied on all services rendered.
 In 1994-95, a service tax was imposed on three services: telephone services, general
insurance, and stockbroking.
 Every year since then, the service net has widened by adding more and more services.
 In India, the current rate of service tax was 15% before it was replaced by the Goods and
Services Tax.

https://t.me/prelimbits
169

118. [2006]
Assertion (A): Balance of Payments represents a better Picture of a country economic
transactions with the rest of the world than the Balance of Trade
Reason (R): Balance of Payments takes into account the exchange of both visible and
invisible items whereas balance of Trade does not.
A. Both ‗A‘ and ‗R‘, are individually true and ‗R‘ is the correct explanation of ‗A‘.
B. Both ‗A‘ and ‗R‘ are individually true but ‗R‘ is not the, correct explanation of ‗A‘ .
C. ‗A‘ is true but ‗R‘ is false.
D. ‗A‘ is false but ‗R‘ is true.
Ans. A

Balance of payments of a nation consists of visible account, invisible account and capital account.
Whereas balance of trade = Export of goods – import of goods. If a country imports are more than
exports the deficit can be made up by invisibles like remmitance by NRI and FDI investment.

BALANCE OF PAYMENTS
 It summarizes all transactions that a country‘s individuals, companies, and government
bodies complete with individuals, companies, and government bodies outside the
country.
 These transactions consist of Imports and Exports of goods, services, and capital, as
well as transfer payments, such as foreign aid and remittances.
 The sum of all transactions recorded in the balance of payments must be zero, as
long as the capital account is defined broadly.
 Balance of Payments is made up of 3 components →
o Current Account – Deals with inflow and outflow of goods and services between
countries.
o Capital Account – Deals with foreign exchange reserves, investments, loans &
borrowings.
o Financial Account – Deals with investments in real estates, business ventures,
Foreign Direct Investments (FDI).
RBI‘s (Actual) Method Of Classifying BOP

Current Account Capital & Financial Account


Goods and services  Direct Investment (FDI)
 Primary Income: wages, dividend,  Portfolio Investment (FPI)
interest  Loans / ECB
 Secondary income: remittance, gift,  Non-resident‘s investment in Bank,
donation Insurance, Pension schemes.
 RBI‘s foreign exchange reserve

https://t.me/prelimbits
170

Balance of Payment Surplus

 The overall balance is determined by the sum of the current and capital accounts,
which may be in surplus or deficit.
 The net credit in the Current and Capital Accounts is the Balance of Payments.
 When a country's overall exports exceed its imports, it has a balance of
payments surplus.
 A BOP surplus is accompanied by the central bank's building of foreign exchange
reserves.
 Balance of payment surplus occurs when: (Current account + capital account receipts)
> (current account + capital account payments)
Significance of Balance of Payment Surplus

 When a country has a balance of payments surplus, it exports more than it imports.
 It offers sufficient capital to cover all domestic production costs.
 It's possible that the country will lend outside its borders.
 In the short run, a surplus stimulates economic growth.
 Increased exports enhance production in the company's plants, allowing it to hire more
workers.
 In the long run, the country's growth becomes overly reliant on exports. It needs to
encourage residents to spend more money.
 The country will be protected from exchange rate swings by having a larger local
market.
 It also enables its businesses to develop goods and services by utilizing its own
workforce as a test market.
 BoP Surplus: Correction
Balance of Payment Surplus: Correction
https://t.me/prelimbits
171

 To remove the surplus government will:


o Invest the excess foreign currency in the Foreign Exchange Reserves.
o This is a government-facilitated transaction aimed solely at restoring balance to the
Balance of Payments.
Balance of Payment Deficit Balance of Payment Surplus
Import > Export Export > Import
The country has to borrow in order to fund the The country provides enough to pay for
imports. domestic production.
Short Run: fuels economic growth. Short Run: boosts economic growth.
Long Run: economy goes into debt to pay for Long Run: economy becomes too dependent
consumption. on exports.
BALANCE OF TRADE
 It‘s the difference between the value of import and export (of goods and services)
 If -ve = Trade Deficit (i.e. Import > Export)
 If +ve = Trade Surplus (i.e. Export > Import);
119. Which one of the following Indian banks is not a nationalized bank? [2006]
A. Corporation Bank
B. Dena Bank
C. Federal Bank
D. Vijaya Bank
Ans. C

Federal Bank is a major Indian commercial bank in the private sector, headquartered at Kochi,
Kerala.

NATIONALIZATION OF BANKS
 The nationalization of banks began on July 19, 1969, during the tenure of Indira Gandhi.
 The whole process took place in two phases.
 In 1969, 14 Private Sector Banks were nationalized that accounted for around 80% of
the money deposited in banks.
 This major financial step was taken up to reform the lending sector and provide credit to
some priority sectors like agriculture and small scale businesses.
 Then in 1980, another 6 banks were nationalized.
 Aside from the aforementioned 20 banks, seven SBI subsidiaries were nationalised in 1959.
 The government merged Punjab National Bank and New Bank of India in 1993. It was the
only merger between nationalised banks, which reduced the number of nationalised banks
from 20 to 19.
 With the nationalisation of the Imperial Bank of RBI in 1955, the central government
entered the banking business, taking a 60% stake and forming a new bank, SBI. The
nationalisation of banks broadened the scope of public sector banking, which had
previously been limited to the State Bank of India.

https://t.me/prelimbits
172

 The primary goal of this move was to reduce the concentration of power
and wealth in the hands of a few families who owned and controlled
these financial institutions.
Reasons for Nationalisation of Banks

 To boost private sectors - Banks were collapsing at an alarming rate – 361 banks failed
between 1947 and 1955, equating to approximately 40 banks per year! Customers' deposits
were forfeited, and there was no way to recover them.
 To assist agricultural sector - Banks favored big industries and businesses while ignoring
the rural sector. Nationalization was accompanied by a promise to support the agricultural
sector.
 To grow India‘s banking network - Nationalisation facilitated the establishment of new
branches, ensuring that banks were well-represented throughout the country.
 To mobilize individual savings - Nationalizing the banks would give people more access
to banks and encourage them to save, bringing in more revenue to a cash-strapped
economy.
 Economic & Political Factors - The two wars in 1962 and 1965 had wreaked havoc on the
economy. The nationalisation of Indian banks would boost the economy by increasing
deposits.
Nationalisation of Banks – Significance

 The opening of new branches resulted in a significant increase in savings. Gross domestic
savings nearly doubled as national income increased in the 1970s.
 Banks' efficiency increased as a result of increased accountability. It also increased public
trust.
 Small scale industries (SSIs) were given a boost, resulting in a proportionate improvement
in the economy.
 Overall statistics for the banking sector and the Indian economy improved noticeably
between 1969 to 1991.
o It reflected on parameters such as the share of bank deposits to GDP, the gross
savings rate, the share of advances to DGP, and the gross investment rate.
 Banks were no longer limited to metropolitan areas. Branches were planted in the farthest
reaches of the country.
 Banks' expanded reach aided the growth of small businesses, agriculture, and the export
sector. This expansion was accompanied by an increase in public deposits in proportion.
 The Green Revolution, one of the government's top priorities, received a boost thanks to
the assistance provided to the agricultural sector by newly nationalised banks.
120. Consider the following statements: [2006]
1. Life Insurance Corporation of India is the oldest insurance company in India.
2. National Insurance Company Limited was nationalized in the year 1972 and made a
subsidiary of General Insurance Corporation of India.

https://t.me/prelimbits
173

3. Headquarters of United Indian Insurance Company Limited are located at Chennai.


Which of the statements given above are correct?

A. 1, 2 and 3
B. 1 and 2 only
C. 2 and 3 only
D. 1 and 3 only
Ans. C

SOME OF THE IMPORTANT MILESTONES IN THE LIFE INSURANCE BUSINESS IN INDIA ARE:

 1818: Oriental Life Insurance Company, the first life insurance company on
Indian soil started functioning.

 1870: Bombay Mutual Life Assurance Society, the first Indian life insurance company started
its business.

 1912: The Indian Life Assurance Companies Act enacted as the first statute to regulate the
life insurance business.

 1928: The Indian Insurance Companies Act enacted to enable the government to collect
statistical information about both life and non-life insurance businesses.

 1938: Earlier legislation consolidated and amended to by the Insurance Act with the
objective of protecting the interests of the insuring public.

 1956: 245 Indian and foreign insurers and provident societies are taken over by the central
government and nationalised. LIC formed by an Act of Parliament, viz. LIC Act, 1956,
with a capital contribution of Rs. 5 crore from the Government of India.

 The General insurance business in India, on the other hand, can trace its roots to the
Triton Insurance Company Ltd., the first general insurance company established
in the year 1850 in Calcutta by the British.

SOME OF THE IMPORTANT MILESTONES IN THE GENERAL INSURANCE BUSINESS IN INDIA ARE:
 1907: The Indian Mercantile Insurance Ltd. set up, the first company to transact all classes
of general insurance business.

 1957: General Insurance Council, a wing of the Insurance Association of India, frames a code
of conduct for ensuring fair conduct and sound business practices.

 1968: The Insurance Act amended to regulate investments and set minimum solvency
margins and the Tariff Advisory Committee set up.

 1972: The General Insurance Business (Nationalisation) Act, 1972 nationalised the general
insurance business in India with effect from 1st January 1973. 107 insurers amalgamated
and grouped into four companies viz. the

o National Insurance Company Ltd.,

https://t.me/prelimbits
174

o the New India Assurance Company Ltd.,

o the Oriental Insurance Company Ltd. and

o the United India Insurance Company Ltd.

 GIC incorporated as a company.

121. Which one of the following is the correct sequence in the decreasing order of
contribution of different sectors to the Gross Domestic Product of India? [2007]
A. Services - Industry - Agriculture
B. Services - Agriculture - Industry
C. Industry - Services - Agriculture
D. Industry - Agriculture – Services

Ans. A

Refer Q. 63

Exact % is not imp for CSE. Just Remember the overall Share [like Service sector Contribute more
than 50%]

https://statisticstimes.com/economy/country/india-gdp-
sectorwise.php#:~:text=Sector%2Dwise%20GDP%20of%20India&text=The%20services%20sector%
20accounts%20for,and%20allied%20sector%20share%2020.19%25.

122. Participatory Notes (PNs) are associated with which one of the following? [2007]
A. Consolidated Fund of India

https://t.me/prelimbits
175

B. Foreign Institutional Investors


C. United Nations Development Programme
D. Kyoto Protocol

Ans. B

PARTICIPATORY NOTES (P-NOTES)


 Participatory Notes or P-Notes (PNs) are financial instruments issued by a registered
foreign institutional investor (FII) to an overseas investor who wishes to invest in
Indian stock markets without registering themselves with the market
regulator, the Securities and Exchange Board of India (SEBI).
 SEBI permitted FIIs to register and participate in the Indian stock market in 1992.
 P-Notes are Offshore Derivative Investments (ODIs) with equity shares or debt securities
as underlying assets.
 They provide liquidity to the investors as they can transfer the ownership by endorsement
and delivery.
 While the FIIs have to report all such investments each quarter to SEBI, they need not
disclose the identity of the actual investors.
What are govt & regulator‘s concerns?

 The primary reason why P-Notes are worrying is because of the anonymous nature of the
instrument as these investors could be beyond the reach of Indian regulators.
 Further, there is a view that it is being used in money laundering with wealthy Indians, like
the promoters of companies, using it to bring back unaccounted funds and to manipulate
their stock prices.
123. Consider the following statements: [2007]
1. The repo rate is the rate at which other banks borrow from the Reserve Bank of India.
2. A value of 1 for Gini Coefficient in a country implies that there is perfectly equal income for
everyone in its population.

Which of the statements given above is/are correct?

A. 1 only
B. 2 only
C. Both 1 and 2
D. Neither 1 nor 2

Ans. A

Repo Rate is the rate at which commercial banks borrow funds from RBI. A reduction in the repo
rate will help banks to get money from the central bank at a cheaper rate. When the repo rate
increases borrowing from RBI becomes more expensive.

A value of (0) for Gini Coefficient in a country implies that there is perfect equality in the
system. If the value is 1 then there is complete inequality in the country.

https://t.me/prelimbits
176

REPO RATE
 Repo Rate stands means Repurchasing Option.
 Commercial Banks borrow money from the Reserve Bank of India at some rate which is
called Repo Rate. [by keeping their G-Sec as collaterals.]
 Clients enter into an agreement with RBI to repurchase their G-sec at a future date at a
(higher) pre-determined price.
 Banks can‘t pledge their SLR-quota-G-Secs for this borrowing.
 Repo rate is applicable to short term borrowings, for example for 7 or 14 or 28 days.
o Another related term is Reverse Repo Rate, which is the rate provided by RBI when
commercial banks keep their surplus with it.
 Repo Rate as a tool used in Monetary policy. RBI keeps on regulating Repo rate as a part
of its monetary policy.
 Easy/ Cheap money policy (lower repo rate):
o Banks borrow money from RBI at lower rates and hence the banks can also lend
money to their borrowers at cheaper rates.
o Money is available to spend at lower rates.
o Increases the liquidity in the market.
o It controls deflation and also boosts the purchasing power of consumers.
 Tight/ Dear money policy (higher repo rate):
o Banks borrow money from RBI at higher rates and hence the people have to borrow
the money from banks at higher rates.
o People refrain from borrowing money from banks.
o Decreases the liquidity in market controls inflation.

GINI COEFFICIENT

https://t.me/prelimbits
177

124. The National Housing Bank was set up in India as a wholly owned subsidiary of
which one of the following? [2007]
A. State Bank of India
B. Reserve Bank of India
C. ICICI Bank
D. Life Insurance Corporation of India

Ans. B

Refer Q. 111

 National Housing Bank has been set up under the National Housing Bank Act of 1987,
which was passed on 9th July, 1988. It is wholly owned by the Reserve bank of India and was
created to encourage housing, finance institutions and provide them with financial support.

125. Which of the following pairs about India‟s economic indicator and agricultural
production (all in rounded figures) are correctly matched? [2008]
1. GDP per capita (current prices): ₹ 37,000
2. Rice : 180 million tons
3. Wheat : 75 million tons

Select the correct answer using the code given below:

A. 1, 2 and 3
B. 1 and 2 only
C. 2 and 3 only
D. 1 and 3 only

Ans. D

Rice production was 96.69 Million tonnes in 2007-08.

126. Consider the following statements with reference to Indira Gandhi National Old Age
Pension Scheme (IGNOAPS): [2008]
1. All persons of 60 years or above belonging to the households below poverty line in rural
areas are eligible.
2. The Central Assistance under this Scheme is at the rate of ₹ 300 per month per beneficiary.
Under the Scheme, States have been urged to give matching amounts.

Which of the statements given above is/are correct?

A. 1 only
B. 2 only
C. Both 1 and 2
D. Neither 1 nor 2

Ans. D

https://t.me/prelimbits
178

NATIONAL SOCIAL ASSISTANCE PROGRAMME (NSAP)


 For Below Poverty Line (BPL) people, Rural Development Ministry‘s National Social
Assistance Programme (NSoAP) 1995, where direct money is given without asking for
any premium from the beneficiary .
 It‘s a CORE OF THE CORE scheme with 100% cost is paid by Union.
 It‘s optional for state govt. to contribute money.
NSoAP Components Union State (Optional to give extra)
Old age pension @60 ₹200-500* (depending on May give extra & rename it e.g.
[2007] how old) Samajwadi Pension
Widow pension ₹200 May give extra and rename it e.g.
Aged 40-59 UP Vidhva Pension @1k
Disability pension: Aged Same as widow e.g. Guj: Sant Surdas Scheme
18-59
National Family Benefit Death of bread winner: ₹10k e.g. Guj: Sankat Mochan (Total 20k)
INSURANCE
Annapurna (2001) If senior citizen uncovered in May give additional.
any other scheme 10kg grain
/ pm for FREE.

127. Consider the following: [2009]


1. Fringe Benefit Tax
2. Interest Tax
3. Securities Transaction Tax

Which of the above is/are Direct Tax/Taxes?

A. 1 only
B. 1 and 3 only
C. 2 and 3 only
D. 1, 2 and 3

Ans. D

FRINGE BENEFITS TAX (FBT)


 Was the tax applied to most, although not all, fringe benefits. A new tax was imposed on
employers by India‘s Finance Act 2005 and was introduced for the financial year
commencing April 1, 2005.
 The Fringe Benefit Tax was abolished in the Finance Bill of 2009.

SECURITIES TRANSACTION TAX (STT)


 Securities Transaction Tax (STT) is the tax payable on the value of taxable securities
transaction.
 STT was introduced in India by the 2004 budget and is applicable with effect from 1st
October 2004.

https://t.me/prelimbits
179

INTEREST TAX
 Interest tax is imposed as a special tax on interest accrued in specified cases. The Interest
Tax Act of 1974 governs the tax process associated with the imposition and collection of
interest tax. T
 he Act applies to all scheduled Banks that are required to pay tax on the chargeable interest
amounts they have on their deposits.
 The act is no longer applicable with regard to chargeable interest accruing after 31 March
2000.

128. During which Five Year Plan was the Emergency clamped, new elections took place
and the Janata Party was elected? [2009]
A. Third
B. Fourth
C. Fifth
D. Sixth

Ans. C

This all happened from 1975-78 during fifth five year plan the period of which is (1974-78)

Fifth Five-  It laid stress on increasing employment and poverty alleviation (garibi hatao).
 In 1975, the Electricity Supply Act was amended, enabling the central
Year Plan
government to enter into power generation and transmission.
(1974-78)  The Indian National Highway System was introduced.
 The Minimum Needs Programme introduced in the first year of this plan,
aimed to provide basic minimum needs. MNP was prepared by D.P. Dhar.
 The target growth rate was 4.4% and the actual growth rate turned out to be
4.8%
 In 1978, the newly elected Morarji Desai government rejected this plan.
Rolling Plan (1978-80)

This was a period of instability. The Janata Party government rejected the fifth five-year Plan and
introduced a new Sixth Five-Year Plan. This, in turn, was rejected by the Indian National Congress
in 1980 upon Indira Gandhi's re-election.

A rolling plan is one in which the effectiveness of the plan is evaluated annually and a new
plan is created the following year based on this evaluation. As a result, throughout this plan, both
the allocation and the targets are updated.

129. Consider the following statements regarding Indian Planning: [2009]


1. The Second Five-Year Plan emphasized on the establishment of heavy industries.
2. The Third Five-Year Plan introduced the concept of import substitution as a strategy for
industrialization.

Which of the statements given above is/are correct?

https://t.me/prelimbits
180

A. 1 only
B. 2 only
C. Both 1 and 2
D. Neither 1 nor 2

Ans. C

 The second five-year plan emphasized on the development of the public sector and ―rapid
industrialization.
 The third five –year plan, stressed on agriculture and improvement in the production of
food grains with long term development == > Import substitution

Second  The Second Five year Plan stressed rapid industrialisation and the
Five Year public sector.
Plan (1956-
61)  It was drafted and planned under the leadership of P.C Mahalanobis.

 It emphasised quick structural transformation.

 The government imposed tariffs on imports to protect domestic industries


under this plan.

 The target growth rate was 4.5% and the actual growth rate was slightly
less than expected, 4.27%.

Third Five  The focus was on agriculture and improvement in the production of wheat.
Year Plan  States were entrusted with additional development responsibilities. Ex- States
(1961-66) were made responsible for secondary and higher education.

 Panchayat elections were introduced to bring democracy to the grassroots


level.

 The target growth rate was 5.6% and the actual growth rate only
achieved 2.4%
 This indicated a miserable failure of the Third Plan, and the government
had to declare "Plan Holidays" (1966-67, 1967-68, and 1968-69). The Sino-
Indian War and the Indo-Pak War, which caused the Third Five Year
Plan to fail, were the primary causes of the plan holidays.

130. Which one of the following brings out the publication called “Energy Statistics” from
time to time? [2009]
A. Central Power Research Institute
B. Planning Commission
C. Power Finance Corporation Ltd.
D. Central Statistical Organization

https://t.me/prelimbits
181

Ans. D

It is published by Ministry of Statistics and Programme Implementation, Central Statistical


Organization.

MINISTRY OF STATISTICS AND PROGRAMME IMPLEMENTATION (MOSPI)


 The Ministry of Statistics and Programme Implementation (MoSPI) is a ministry of
Government of India concerned with coverage and quality aspects of statistics released. The
surveys conducted by the Ministry are based on scientific sampling methods.
 The Ministry of Statistics and Programme Implementation (MOSPI) came into existence as
an Independent Ministry on 15 October 1999 after the merger of the Department of
Statistics and the Department of Programme Implementation.
 The Ministry has two wings, one relating to Statistics and the other Programme
Implementation. The Statistics Wing called the National Statistical Office (NSO) consists of
the Central Statistical Office (CSO), the Computer center and the National Sample Survey
Office (NSSO).

RESTRUCTURING OF INDIAN STATISTICAL SYSTE → NATIONAL STATISTICAL OFFICE (NSO)


 2019: The government has formed an overarching body NSO by merging the National
Sample Survey Office (NSSO), Computer Centre and Central Statistical Office (CSO).
 NSO was first envisaged by Rangarajan Commission to implement and maintain
statistical standards and coordinate statistical activities of Central and State agencies as
laid down by the National Statistical Commission (NSC).
 NSO is headed by Secretary (Statistics and Programme Implementation).
 CSO coordinates the statistical activities in the country and also evolves statistical standards.
 NSSO is responsible for the conduct of large scale sample surveys in diverse fields on an all
India basis.
 Release: it release Consumer price index

131. In the context of independent India‟s economy, which one of the following was the
earliest event to take place? [2009]
A. Nationalization of Insurance companies
B. Nationalization of State Bank of India
C. Enactment of Banking Regulation Act
D. Introduction of First Five-Year Plan

Ans. C

 Nationalization of State Bank of India – 1955


 Introduction of First Five-Year Plan – 1951
 Enactment of Banking Regulation Act – 1949
 Nationalization of Insurance Companies – 1955-56

https://t.me/prelimbits
182

HISTORY OF INSURANCE IN INDIA


 (1818): Europeans started insurance companies in India, but they charged higher premium
on Indian clients with racist bias that Indians belong to an inferior race = higher probability
to die.

 (1870): Bombay Mutual Life Insurance was the first Swadeshi life insurance company
and they did not charge extra premium on Indian clients.

 (1912): Life Insurance Companies Act to regulate them, but the norms were lax, so just
like the banking industry, the insurance industry too faced problems in the aftermath of
Great Depression in USA. So, 1938: Insurance Act for tougher regulation.

 Just like the banking industry, the insurance industry had to be nationalized after
independence due to scams, financial inclusion and Five-Year Plans.

Chronology Bank Insurance


1948-49 RBI nationalized
1955 SBI nationalized
1956 Nationalising the Life Insurance sector and Life
Insurance Corporation (LIC) came into existence. The
LIC had monopoly till the late 90s when the Insurance
sector was reopened to the private sector.
1969 Nationalization of 14
Private Banks
1972 GIC Act: GIC and its 4 subsidiaries tookover ~107
(private owned) General insurance companies.
1980 Nationalization of 6
Private Banks
Reforms Narasimham ‗91 & Malhotra Committee 1993→ Private insurance
‗98 companies were allowed, FDI was liberalized
Safeguards CRR, SLR, BASEL Investment Pattern, Solvency Margin. E.g. They must
invest minimum ―x%‖ of premium in G-Sec, they can‘t
invest more than ―y%‖ of premium in pvt companies
shares/debentures etc. They must not invest in
companies having less than ―AA‖ credit rating etc.
Financial PSL norms, 25% Rural & Social Obligation Norms: every year ―x‖ number
Inclusion, branches in of policies must be sold in rural areas, PH/backward etc.
Welfarism unbanked rural areas Further Insurance companies required to invest
minimum ―x%‖ in affordable housing projects, State
Govt‘s fire equipment etc. Else IRDAI imposes penalty.
Exact norms not imp.
Delivery Bank branch, Business Insurance Intermediaries:
Channel Correspondence  Agents/brokers.
Agent (BankMitra)  Banks selling insurance (Bancassurance)
 Surveyor/Loss Assessor
 Third Party Administrators (e.g. Hospital where
treatment is given for health insurance)

https://t.me/prelimbits
183

NATIONALIZATION OF BANKS AFTER INDEPENDENCE: REASONS


 NEXUS between Banks and Industrialists: From 1950s to 1960, only 188 elite people
controlled the economy by being in board of top 20 banks, 1452 companies, and
numerous insurances, finance companies. This led to reckless lending to directors and their
firms. So, Banks failed frequently, RBI had to close them.
 Private Banks unwilling to open in rural areas- this did not help in financial inclusion of
poor, farmers, MSME or achievement of FYP targets or reducing regional imbalance.
Nationalization & Merger of Banks after Independence

1948 RBI Transfer of Ownership Act.


48: Op. Polo, Hyd.
51:1st FYP,
53: Air India from Tata
1955 Imperial Bank nationalized and became SBI.
1955-56 LIC Act took over private life insurance cos.
57: 1st Communist Govt in Kerala
60: Op Vijay, Goa
1963 State Bank of Jaipur and Bikaner merged together.
1969 'Banking Companies (Acquisition and Transfer of Undertakings) Ordinance, 1969:
14 Private banks with ₹ 50 cr/> deposits were nationalized e.g. Bank of Baroda,
PNB, Dena, Canara etc.
1972-73 GIC Act- took over private Non-Life (=General) insurance cos. Later GIC was re-
organized with 4 subsidiaries: National Insurance, New India Assurance, Oriental
Insurance and United India Insurance.
1980 6 banks with ₹ 200 cr/> deposits were nationalized e.g. Corporation Bank, Vijaya
Bank, Oriental bank of Commerce etc.
Reforms following Committees were made for reforms in banking sector
M Narasimham-I (1991), M Narasimham-II (1997), Dr. Raghuram Rajan
Committee (2007) and P J Nayak Committee (2014)
BANKING REGULATION ACT, 1949
 The Banking Regulation Act, 1949 is a law. It regulates the functioning of banks and
provides details on several aspects including licensing, management, and operations
of banks in India.
 It had been passed as the Banking Companies Act, 1949 and came into force from March
16, 1949.

Some Provisions of the Banking Regulation Act, 1949

 The Act provides a structure under which commercial banking in India is supervised and
regulated. The Act supplements the Companies Act, 1956. Primary Agricultural Credit
Society and cooperative land mortgage banks are excluded from the Act.
 Several powers are provided by the Act to the Reserve Bank of India →
o to license banks, have regulation over shareholding and voting rights of
shareholders;

https://t.me/prelimbits
184

o to supervise the appointment of the boards and management;


o to regulate the operations of banks;
o to lay down instructions for audits; control moratorium, mergers and liquidation;
o to issue directives in the interests of public good and on banking policy, and impose
penalties

Amendment to the Banking Regulation Act, 1949

 Initially, the law was applicable only to banking companies. But in 1965 the Act was
amended to form it applicable to cooperative banks under its purview by adding Section
56 to the Act.
 Cooperative banks, which operate only in one state, are formed and run by the state
government. But, licensing is controlled by RBI and also regulates the business operations.
The Banking Act was a supplement to the previous acts associated with banking.
 Recently, the Lok Sabha has passed an amendment to the Banking Regulation Act, 1949.
The Banking Regulation (Amendment) ordinance is replaced by the bill and to the same
effect promulgated in June 2020.
 The amendment will bring cooperative banks under the direct supervision of
the RBI and convey them under some of the same governance norms as
commercial banks. Without first imposing a moratorium, it will also allow the RBI to
amalgamate or reconstruct a stressed cooperative bank. These amendments are proposed
so as to guard the interests of the depositors.

132. Consider the following statements: [2009]


1. The Commission for Agricultural Costs and Prices recommends the Minimum Support Prices
for 32 crops.
2. The Union Ministry of Consumer Affairs, Food and Public Distribution has launched the
National Food Security Mission.

Which of the statements given above is/are correct?

A. 1 only
B. 2 only
C. Both 1 and 2
D. Neither 1 nor 2

Ans. D

NATIONAL FOOD SECURITY MISSION


 The National Development Council (NDC) in its 53rd meeting held on 29th May, 2007
adopted a resolution to launch a Food Security Mission consisting of rice, wheat and pulses
and to increase the production of rice by 10 million tons, wheat by 8 million tons and pulses
by 2 million tons by the end of the Eleventh Plan (2011–12).
 The project is under Ministry of Agriculture. The project identifies districts and varieties

https://t.me/prelimbits
185

which would be concentrated on.


 National Food Security Mission (NFSM) to increase production of rice, wheat, pulses,
millets (coarse cereals) and commercial crops & restore soil fertility.
 2018 declared as ―National Year of Millets‖. Further, UN & FAO accepted India‘s proposal
to celebrate 2023 as International Year of Millets. Because, millets are tolerant to
drought, climate change, photo insensitive; need less water, provide nutritious elements in
poor families‘ diet.
 So, Govt create two sub-missions.
o NFSM on Makka and Jau.
o NFSM on Nutri-Cereals- Jowar, Bajra, Ragi and little millets like Kutki, Kodo, Sawa,
Kangni and Cheena.
 NFSM has another sub-mission: National Mission on Oilseeds and Oil Palm (NMOOP) to
augment the availability of vegetable oils and to reduce the import of edible oils.

MINIMUM SUPPORT PRICE (MSP)


 Agro Ministry‘s Commission for Agricultural Costs and Prices (CACP) recommends MSP
(& FRP for sugar) → Cabinet Committee on Economic Affairs chaired by PM approves &
announces MSP

 Under the State APMC Acts, the first sale of agriculture commodity can occur at
Agricultural Produce Market Committee (APMC) Mandis only. However, a farmer may
not get remunerative prices at the Mandi due to following reasons:

o Bumper production /supply which brings down the market prices AND / OR

o Cartelization / price-fixing by the mandi-merchants.

 So, to protect the farmers, Government of India announces MSP BEFORE each crop
sowing season. [fixed twice a year]

 Note: currently MSP does not have any legal backing = EXCEPT

o The only crop where MSP payment has some statutory element is sugarcane. This is
due to its pricing being governed by the Sugarcane (Control) Order, 1966 issued
under the Essential Commodities Act. The responsibility to make FRP payment to
farmers within 14 days of cane purchase lies solely with the sugar mills.

 For 22 crops (MSP) + 1 sugarcane (FRP)

133. Consider the following statements: [2009]


1. MMTC Limited is India‘s largest international trading organization.
2. Neelachal Ispat Nigam Limited has been set up by MMTC jointly with the Government of
Orissa.

Which of the statements given above is/are correct?

A. 1 only
B. 2 only
https://t.me/prelimbits
186

C. Both 1 and 2
D. Neither 1 nor 2

Ans. C

Refer Q. 93

 Neelachal Ispat Nigam Limited (NINL), a company promoted by MMTC Ltd, Industrial
Promotion and Investment Corporation of Orissa Limited (IPICOL) and other government
agencies has set up an 1.1 million ton Integrated Iron and steel plant at Kalinganagar,
Duburi, Orissa.

134. Which one of the following sets of commodities are exported to India by arid and
semi-arid countries in the Middle East? [2009]
A. Raw wool and carpets
B. Fruits and palm oil
C. Precious stones and pearls
D. Perfume and Coffee.

Ans. C

https://tradingeconomics.com/india/imports/saudi-arabia/pearls-precious-stones-metals-
coins#:~:text=Careers-
,India%20Imports%20from%20Saudi%20Arabia,%2C%20precious%20stones%2C%20metals%2C%2
0coins

135. Which one of the following pairs is not correctly matched? [2009]
A. Japan : Nikkei
B. Singapore : Shcomp
C. UK : FTSE
D. USA : Nasdaq

Ans. B

 The Nikkei 225, or the Nikkei Stock Average, more commonly called the Nikkei or the
Nikkei index, is a stock market index for the Tokyo Stock Exchange
 The SSE Composite Index also known as SSE Index is a stock market index of all stocks
that are traded at the Shanghai Stock Exchange.
 The Financial Times Stock Exchange 100 Index, also called the FTSE 100 Index, FTSE 100,
FTSE, or, informally, the "Footsie", is a share index of the 100 companies listed on the
London Stock Exchange with the highest market capitalisation
 SIMEX and Strait Times are indicators for Singapore.
 The Nasdaq Composite is a stock market index that includes almost all stocks listed on the
Nasdaq stock exchange. Along with the Dow Jones Industrial Average and S&P 500, it is one
of the three most-followed stock market indices in the United States.

https://t.me/prelimbits
187

136. Which one of the following statements is an appropriate description of deflation?


[2010]
A. It is a sudden fall in the value of a currency against other currencies
B. It is a persistent recession in both the financial and real sectors of economy
C. It is a persistent fall in the general price level of goods and services
D. It is a fall in the rate of inflation over a period of time
Ans. C

DEFLATION

 It is a persistent fall in the general price level of goods and services.


 When the overall price level decreases so that the inflation rate becomes negative, it is
called deflation. It is the opposite of the often-encountered inflation.
 A reduction in money supply or credit availability is the reason for deflation in most
cases.
o Reduced investment spending by the government or individuals may also lead to this
situation.
o Prices can also fall due to increased productivity and technological
improvements.
 Deflation leads to a problem of increased unemployment due to slack in demand.
ADDITIONAL INFORMATION
 Inflation - It is a persistent rise in the general price level of goods and services.
 Stagflation- A condition of slow economic growth and relatively high unemployment.
 Reflation - A fiscal or monetary policy designated to expand a country's output and curb
the effects of deflation.
o It includes reducing taxes, changing the money supply, and lowering interest rates.
 Disinflation - A slowing in the rate of price inflation.
o Disinflation is used to describe instances when the inflation rate has reduced
marginally over the short term.

https://t.me/prelimbits
188

o Although it is used to describe periods of slowing inflation, disinflation should not be


confused with deflation.
 Currency depreciation - It is a sudden fall in the value of a currency against other
currencies.
137. Which of the following is/are treated as artificial currency? [2010]
A. ADR
B. GDR
C. SDR
D. Both ADR and SDR
Ans. C

SPECIAL DRAWING RIGHTS (SDR)


 Created by the IMF in 1969

 Members would deposit currencies in the proportion of quotas allotted to them (depending
on size of their economy, openness etc) = will earn interest on their deposits.

 IMF would lend this money to a member facing balance of payment crisis.

 IMF would allot an artificial currency / accounting unit called SDR to the to its members on
their deposits. The value of five major currencies

o US DOLLAR

o EURO

o CHINESE RENMINBI (RMB)

o JAPANESE YEN

o BRITISH POUND STERLING

 SDR can be traded among the members to settle their Balance of Payment Transactions /
Crisis.

 The SDR is neither a currency nor a claim on the IMF. Rather, it is a potential claim on the
freely usable currencies of IMF members.

 India is 8th largest quota holder after USA (~18%), Japan (~7%), China (~6%)...
Member's voting power is related directly to their quotas.

 The SDR currency value is calculated daily (except on IMF holidays or whenever the
IMF is closed for business) and the valuation basket is reviewed and adjusted every
five years.

ADR / GDR
 American Depository Receipts: A non-American company wants to mobilize money from
American share market but does not want to go through the lengthy & complex process of
registration with the American sharemarket regulator.

https://t.me/prelimbits
189

o Then such non-American company gives its shares to an American bank.


o Based on those (non-American) shares, the American bank will issue American
Depositary Receipts (ADR) & sell them to American investors. Denomination:
USD.
 Global Depositary Receipt (GDR): If for example, an Indian company which has issued
ADRs in the American market wishes to further extend it to other developed and advanced
countries such as in Europe, then they can sell these ADRs to the public of Europe and the
same would be named as GDR. GDR can be issued in more than one country and can be
denominated in any freely convertible currency.
Recent Development

 Before: Direct listing by the Indian companies on foreign/overseas stock exchanges


was not permitted. (due to danger of money laundering, China/ISI mischief etc.)
 So Indian companies had to use ADR/GDR type mechanisms, But, relatively difficult to
attract investors in through ADR/GDR routes, nowadays. (compared to directly listing
shares.)
 After ATMANIRBHAR: Indian public companies allowed to directly list their shares in
foreign nations stock exchanges. → Foreign capital/dollars can be attracted towards India
→ factory expansion, jobs ⏫ → economic revival.
Note: Foreign Cos cannot be directly listed in india Market.
138. When the Reserve Bank of India announces an increase of the Cash Reserve Rate,
what does it mean? [2010]
A. The commercial banks will have less money to lend
B. The Reserve Bank of India will have less money to lend
C. The Union Government will have less money to lend
D. The commercial banks will have more money to lend
Ans. A

CRR, SLR (FIGHT INFLATION: ↑, DEFLATION: ↓)


 CRR and SLR are collectively known as ―Variable Reserve Ratios‖ or ―Statutory Reserve
Ratios‖
CRR SLR
Banks must keep this much deposits (or Banks must keep with themselves this
balance) with RBI. [only in cash] deposits in liquid assets such as cash, gold,
RBI doesn‘t pay interest on this deposit, except in G-Sec, T-Bills, State Development Loan Bonds
extraordinary circumstances like 1999‘s Banking and other securities notified by RBI.
slowdown.
Bank earns no profit / interest, as such. Some profit may be involved.
CRR: first suggested by the British economist J.M. Mandated under Banking Regulation Act,
Keynes & first introduced in US Federal Reserves 1949
(=Central Bank of USA).
Mandated under RBI Act, 1934

https://t.me/prelimbits
190

RBI can fix any amount of CRR, legally there is Legally, SLR can‘t be more than 40%.
no minimum floor or maximum ceiling. Presently it‘s 18.00% of NDTL.
Presently it‘s 3% of Net Demand & Time
Liabilities (NDTL) of a bank
All Scheduled Commercial Banks (SCB) must Similar
keep CRR.
However, RBI may prescribe separate %
norms/slabs for Regional Rural Banks (RRBs) and
Cooperative Banks.

Role of SLR & CRR

 CRR-SLR are counted on fortnightly basis. If not maintained, bank will have to pay penalty
interest rate to RBI. Penalty rate is linked with Bank Rate.
 CRR-SLR ensure monetary stability of India through two primary functions:
o CRR assists in money multiplier effect,
o CRR-SLR provide buffer/protection during a Bank Run
 While in theory CRR/SLR can be used for inflation control but RBI primarily relies on
REPO Rate (=its Policy Rate) to combat inflation, and not CRR/SLR.
139. With reference to India, consider the following statements: [2010]
1. The Wholesale Price Index (WPI) in India is available on a monthly basis only.
2. As compared to Consumer Price Index for Industrial Workers (CPIIW), the WPI gives less
weight to food articles.
Which of the statements given above is/are correct?

A. 1 only
B. 2 only
C. Both 1 and 2
D. Neither 1 nor 2
Ans. C

 Alter the Abhijit Sen committee‘s proposals in 2004- 05, the government had approved
the proposal to release or wholesale price based inflation data on a monthly basis,
instead of every week. The base year was changed to 2004- 05 from 1993-94. However
data on primary and fuel items was continued to release on a weekly basis.
 Consumer Price Index food group has a weight of 39.1 percent as compared to the
combined weight of 24.4 percent (food articles and Manufactured food products) in
wholesale Price Index food basket.
WHOLESALE PRICE INDEX
 It was launched in 1942
 It measures the changes in the prices of goods [NOT SERVICES] sold and traded in
bulk by wholesale businesses to other businesses.
 Published by the Office of Economic Adviser, Ministry of Commerce and Industry.
 It is the most widely used inflation indicator in India.

https://t.me/prelimbits
191

 It is also used as GDP deflators


 Major criticism for this index is that the general public does not buy products at wholesale
price.
 The base year of All-India WPI has been revised from 2004-05 to 2011-12 in 2017
 Calculated every month in india.
 Presently it covered 697 commodities
Monthly WPI Components in descending order→ Wt.
Manufactured products: Processed Food, Edible Oil, Paper Products, Chemicals, Plastic, ~64%
Cement, Metal Products, Transport Equipments etc.
Primary Articles: ~23%
A. (Unprocessed) food articles, eggs, meat-fishes, oil seeds etc. (~19%)
B. Crude Petroleum (~2%)
C. Minerals (~0.8%)

Fuel & Power: High Speed Diesel (HSD) > Petrol > LPG ~13%
Total 100%
CONSUMER PRICE INDEX
 It measures price changes from the perspective of a retail buyer. It is released by
the National Statistical Office (NSO).
 The CPI calculates the difference in the price of COMMODITIES & SERVICES such as
food, medical care, education, electronics etc, which Indian consumers buy for use.
 The CPI has several sub-groups including food and beverages, fuel and light, housing and
clothing, bedding and footwear.
 Types of CPI are as follows:
o CPI for Industrial Workers (IW). [used to revise dearness allowance (DA)]
o BY – 2016
o CPI for Agricultural Labourer (AL). [used to decide to MGNREGA]
o CPI for Rural Labourer (RL).
o CPI (Rural/Urban/Combined).
o CPI UNME
Of these, the first three are compiled by the Labour Bureau in the Ministry of Labour
and Employment. Fourth & 5th is compiled by the NSO in the Ministry of Statistics
and Programme Implementation.

 Base Year for CPI is 2012.


 The Monetary Policy Committee (MPC) uses CPI data to control inflation. In April 2014,
the Reserve Bank of India (RBI) had adopted the CPI as its key measure of inflation.
Monthly CPI Components in (All India) Index → (decreasing order) Wt.
Food & Beverages 45.86
Services: (Transport & communication > Health > Education > Recreation) 20.62
Housing 10.07
Fuel & Light 6.84
Clothing / footwear 6.53
https://t.me/prelimbits
192

Misc. Personal care (soap etc) 3.89


Household goods & Services 3.80
Pan Masala, Tobacco, Intoxicants 2.38
Total Weight 100
The main difference between WPI vs CPI are:

Parameter CPI WPI


Meaning It reflects the average change in prices It reflects changes in average
paid by consumers at the retail level. wholesale prices for goods sold in
bulk.
Published by National Statistical Office (NSO), Office of Economic Advisor (Ministry
Ministry of Statistics and Programme of Commerce & Industry)
Implementation & Labour Bureau
Base Year 2012 2011-2012
Frequency of Monthly (14th of every month) Primary articles, fuel, and power on
Publishing a Weekly basis (Generally Thursdays)
Overall index on monthly basis.
Source The CPI's item weights are based on The WPI's item weights are based on
average household expenditures production values.
gathered from consumer expenditure
surveys.
Measured at Final stage of transaction First stage of transaction
Price paid by Consumers Wholesale dealers and
manufacturers.
Number of 448 items in rural and 460 items in 697 (including Primary, fuel & power
items urban and manufactured products)
Weightage of The food group has a weight of 39.06%. The food group (Food articles and
food items manufactured food products)
account for 24.4% of the total
weight.
Status of Services are included in the CPI (like Services are not included in the WPI.
services housing, education, medical care etc)
Commodities Education, communication, Minerals, machinery, basic metals,
included transportation, recreation, clothing, and other manufacturing inputs and
foods and beverages, housing and intermediary items etc
medical care etc.

140. In the context of Indian economy, consider the following pairs: [2010]
Term Most appropriate description
1. Meltdown Fall in stock prices
2. Recession Fall in growth rate
3. Slow down Fall in GDP
Which of the pairs given above is/are correctly matched?

A. 1 only
B. 2 and 3 only

https://t.me/prelimbits
193

C. 1 and 3 only
D. 1, 2 and 3
Ans. B

 Recession is a period of temporary economic decline during which trade and industrial
activity are reduced, generally identified by a fall in GDP in two successive quarters.
 Slow down is a period of slow economic growth, especially one that follows a period of
robust growth
 Melt down follows normally a black swan event and quickly results is financial assets
loosing in value, often resulting in liquidity crisis. The stock market in India suffered melt
down from Sensex 20,000 in mid 2008 to 10,000 in mid 2009.
141. In the context of governance, consider the following: [2010]
1. Encouraging Foreign Direct Investment inflows
2. Privatization of higher educational Institutions
3. Down-sizing of bureaucracy
4. Selling/offloading the shares of Public Sector Undertakings
Which of the above can be used as measures to control the fiscal deficit in India?

A. 1, 2 and 3
B. 2, 3 and 4
C. 1, 2 and 4
D. 3 and 4 only
Ans. B

 Statement 1 is incorrect because a Fiscal deficit results in inflation which in turn leads to
increased interest rates and this makes investments unfavorable.
 Statement 2 is correct this will help in reduction in government expenditure and over the
past few decades, we have seen that the private sector has driven capacity-creation in
Indian higher education.
 Statement 3 is correct because this will help in reducing the wage burden of the
government. The erstwhile Planning Commission had recommended to weed out about
2000 senior-level positions in various ministries.
 Statement 4 is correct because reducing the shareholding in the PSUs and selling off the
shares help in revenue generation. The Indian government is keen on this aspect over the
past few years.
142. With reference to‟ Indian economy, consider the following statements: [2010]
1. The Gross Domestic Product (GDP) has increased by four times in the last 10 years.
2. The percentage share of Public Sector in GDP has declined in the last 10 years.
Which of the statements, given above is/ are correct?

A. 1 only
B. 2 only
C. Both 1 and 2

https://t.me/prelimbits
194

D. Neither 1 nor 2
Ans. B

 The economic scenario in India has been pretty stable over the last 5 years.
 Despite the economic downturn two years back the Indian economy has managed to
remain stable.
 The India GDP recorded for the period December 2010 stood at 8.20%. Hence, statement
1 is Not correct.
 However, according to the Centre for Monitoring Indian Economy (CMIE) India will
record a GDP of 9.2% in the year 2011.
 India's GDP growth 2010 - 2011 has not been phenomenal but is certainly encouraging.
 The turnover of central public sector enterprises (CPSEs) for the last three years, 2010-11,
2011-12 and 2012-13 was Rs 14,98,018 crore, Rs 18,22,049 crore and Rs 19,45,777 crore
respectively.
 The contribution of gross value addition of CPSEs in the gross domestic product (GDP) is
6.18%, 6.14%, and 5.85% respectively.
 A percentage-wise increase in GDP at current market prices also witnessed a downward
trajectory. It was reported at 12.25% in 2012-13, down from 15.74% in 2011-12 and
20.17% in 2010-11. Hence, statement 2 is correct.
143. The SEZ Act, 2005 which came, into effect in February 2006 has certain objectives.
In this context, consider the following: [2010]
1. Development of infrastructure facilities.
2. Promotion of investment from foreign sources.
3. Promotion of exports of services only.
Which of the above are the objectives of this Act?

A. 1 and 2 only
B. 3 only
C. 2 and 3 only
D. 1, 2 and 3
Ans. A

SEZ
 An SEZ is a territory within a country that is typically duty-free (Fiscal Concession) and
has different business and commercial laws chiefly to encourage investment and create
employment.
 SEZs are created also to better administer these areas, thereby increasing the ease of
doing business.
SEZs in India →

 Asia‘s first EPZ (Export Processing Zones) was established in 1965 at Kandla, Gujarat.
 While these EPZs had a similar structure to SEZs, the government began to
establish SEZs in 2000 under the Foreign Trade Policy to redress the infrastructural and

https://t.me/prelimbits
195

bureaucratic challenges that were seen to have limited the success of EPZs.
 The Special Economic Zones Act was passed in 2005. The Act came into force along with
the SEZ Rules in 2006.
 However, SEZs were operational in India from 2000 to 2006 (under the Foreign Trade
Policy).
 India‘s SEZs were structured closely with China's successful model.
 Presently, 379 SEZs are notified, out of which 265 are operational. About 64% of the SEZs
are located in five states – Tamil Nadu, Telangana, Karnataka, Andhra Pradesh and
Maharashtra.
 The Board of Approval is the apex body and is headed by the Secretary, Department of
Commerce (Ministry of Commerce and Industry).
 The Baba Kalyani led committee was constituted by the Ministry of Commerce and
Industry to study the existing SEZ policy of India and had submitted its recommendations in
November 2018.
o It was set up with a broad objective to evaluate the SEZ policy towards making
it WTO (World Trade Organisation) -compatible and to bring in global best
practices to maximise capacity utilisation and to maximise potential output of the
SEZs.
Objectives of the SEZ Act →

 To create additional economic activity.


 To boost the export of goods and services.
 To generate employment.
 To boost domestic and foreign investments.
 To develop infrastructure facilities.
Major Incentives and Facilities Available to SEZ →

 Duty free import/domestic procurement of goods for development, operation and


maintenance of SEZ units.
 Exemption from various taxes like Income Tax, minimum alternate tax, etc.
 External commercial borrowing by SEZ units upto US $ 500 million in a year without any
maturity restriction through recognized banking channels.
 Single window clearance for Central and State level approvals.
144. A great deal of Foreign Direct Investment (FDI) to India comes from Mauritius than
from many major and mature economies like UK and France. Why? [2010]
A. India has preference for certain countries as regards receiving FDI
B. India has double taxation avoidance agreement with Mauritius
C. Most citizens of Mauritius have ethnic identity with India and so they feel secure to invest in
India
D. Impending dangers of global climatic change prompt Mauritius to make huge investments
in India
Ans. B

https://t.me/prelimbits
196

DOUBLE TAXATION AVOIDANCE AGREEMENT (DTAA )


 Double taxation avoidance agreement (DTAA ) allowed for aggressive tax saving to
MNCs bringing in Foreign Investments through round-tripping.
 A DTAA is a tax treaty signed between two or more countries.
 Its key objective is that taxpayers in these countries can avoid being taxed twice for the
same income.
 India has comprehensive Double Taxation Avoidance Agreements (DTAA) with 23
countries.
 This means that there are agreed rates of tax and jurisdiction on specified types of income
arising in a country to a tax resident of another country.
 Under the Income Tax Act 1961 of India, there are two specific provisions, Section
90 and Section 91, which provide specific relief to taxpayers to save them from DTAA.
 Section 90 is for taxpayers who have paid the tax in a country with which India has
signed DTAA.
 Section 91 provides relief to taxpayers who have paid tax to a country with which India has
not signed a DTAA.
145. India-based Neutrino Observatory is included by the Planning Commission as a
mega science project under the 11th Five-Year Plan. In this context, consider the
following statements: [2010]
1. Neutrinos are chargeless elementary particles that travel close to the speed of light.
2. Neutrinos are created, in nuclear reactions of beta decay.
3. Neutrinos have a negligible, but non-zero mass
4. Trillions of Neutrinos pass through human body every second.
Which of the statements given above are correct?

A. 1 and 3 only
B. 1, 2 and 3 only
C. 2, 3 and 4
D. 1, 2, 3 and 4
Ans. D

INDIAN NEUTRINO OBSERVATORY (INO) PROJECT


 The neutrino observatory is the most ambitious scientific research facility that India is trying
to build. Neutrinos are tiny particles, almost massless, that travel at near light speeds.

 They are born from violent astrophysical events like exploding stars, nuclear fusion in the
sun and gamma ray bursts.

 Detected for the first time in 1959, though their existence was predicted almost three
decades earlier, in 1931, neutrinos were later found to be omnipresent.

 They are the second most abundant particles in the world, after photons and can move
easily through matter.

https://t.me/prelimbits
197

 These high-energy particles are produced in natural radioactive decays and all
sorts of nuclear reactions happening in nuclear power reactors, particle accelerators or
nuclear bombs.

 But the most common source of neutrinos are celestial phenomena i.e., the birth and
death of stars, collisions and explosions happening in space.

 The core of the Sun is an important source of neutrinos.

 A large number of the neutrinos were produced at the time of the Big Bang, making them
good candidates to extract more information from about the origins of the universe.

 But because they are electrically neutral and almost massless, these neutrinos have an
extremely low tendency to interact with other objects.

 This is the reason why scientists have to go deep underground to set up


special detectors in a bid to catch the faint signals of neutrinos in an
environment that is relatively free from ‗noise‘ and disturbance.

 The proposed INO project primarily aims to study atmospheric neutrinos in a 1,300-m
deep cavern in the Bodi West Hills in Theni district, Tamil Nadu.

 If completed, it would house the largest magnet in the world.

ELEVENTH FIVE YEAR PLAN (2007-2012)

 The Eleventh Plan was significant in its aim to increase enrolment in higher
education and focused on distant education as well as IT institutes. Ex: The Right
to Education Act was introduced in 2009, and came into effect in 2010, making

https://t.me/prelimbits
198

education free and compulsory for children aged between 6-14 years.
 Its main theme was rapid and more inclusive growth.
 It is aimed at environmental sustainability and reduction in gender inequality.
 C.Rangarajan prepared the Eleventh Five Year Plan.
 The focus was also laid on providing clean drinking water for all by 2009.
 The target rate was 9% and the actual growth rate was 8%.
146. Inclusive growth as enunciated in the Eleventh Five Year Plan does not include one
of the following: [2010]
A. Reduction of poverty
B. Extension of employment opportunities
C. Strengthening of capital market
D. Reduction of gender inequality
Ans. C

147. Who of the following shall cause every recommendation made by the finance
Commission to be laid before each House of Parliament? [2010]
A. The President of India
B. The Speaker of Lok Sabha
C. The Prime Minister of India
D. The Union Finance Minister
Ans. A

Article 281: President of India shall cause every recommendation made by the Finance
Commission to be laid before each House of Parliament.

148. Which one of the following is responsible for the preparation and presentation of
Union Budget to the Parliament? [2010]
A. Department of Revenue
B. Department of Economic Affairs
C. Department of Financial Services
D. Department of Expenditure
Ans. B

FINANCE MINISTRY AND ITS DEPARTMENTS


Department of Economic Affairs (DEA)

 Fiscal policy, Preparation and presentation of Union budget including the Railway
component of budget. Budget for union territories without legislature, budget for States
under president rule.
 DEA announces the Interest rates of small saving schemes.
 Maintains a website www.pppinindia.gov.in, to provide information related to Public Pvt
 Partnership (PPP).

https://t.me/prelimbits
199

Organizations under/related to DEA

 Finance Commission (Const. Body)


 Chief Economic Advisor (CEA)
 Financial Stability and Development Council (FSDC): is neither Constitutional nor
statutory body.
o FM is chairman. Members include the chiefs of all financial regulatory bodies- such
as RBI, SEBI, IRDAI, PFRDA and the chief of IBBI
 Govt Company: Security Printing and Minting Corporation of India Ltd. (SPMCIL).
Registered under the Companies Act responsible for printing currency notes, coins,
commemorative coins, cheques, postage stamps, non-judicial stamps, passports/visa and
other travel documents etc.
Department of Expenditure

 Here the Controller General of Account (CGA) from ICAS service) prepares estimate of
how much money to be spent from consolidated fund of India.
 It also deals with Pay Commission reports, Pension Accounting office.
Web Portals under/related to DoE

 Public Financial Management System (PFMS): for disbursing money to various Ministries
and departments at Union and State level
 Bharatkosh - Non Tax Receipts Portal (NTRP): For selling India yearbook Yojana
Kurukshetra mags etc products and services by the government of India
Department of Revenue

Statutory Bodies / Quasi-judicial bodies

 Central Boards of Revenue Act 1963


o Central Board of Direct Taxes (CBDT)→ Department of Income Tax
o Central Board of Indirect Taxes and Customs (CBIC).
 Authorities for Advance rulings under Acts for IT, Customs & Central Excise, GST
 Various Tribunals and appellate bodies related to taxation.
Attached / Subordinate

 Enforcement Directorate (for enforcing PMLA and FEMA Act)


 Central Economic Intelligence Bureau.

https://t.me/prelimbits
200

 Central Bureau of Narcotics Financial Intelligence Unit.


Associated PSU

 Goods and Service Tax Network (GSTN) is a non-profit company.


 Originally its 51% shareholding was with HDFC, ICICI etc.
 But 2018- Government decided to make it 100% owned by Union & State Governments.
Department of Financial Services (DFS)

 Schemes for Financial Inclusion, PSB supervision and recapitalization, Public Sector
Financial Intermediaries, including their regulators (Except EPFO, ESIC etc.)
Organizations under/related to DFS:

 Bank Board Bureau: Neither Constitutional / statutory


o Setup through gazette notification for selection of top officials (MD, CEO, Chairman
and full-time Directors) for PSBs, LIC and other public sector financial institutions.
 PSU: National Credit Guarantee Trustee Company (NCGTC):
o For providing credit guarantee for loans in Mudra, certain MSME loans, Stand up
India, education-skill development related loans.
Department of Investment and Public Asset Management (DIPAM)

 Looks after Disinvestment of CPSE.


Department of Public Enterprises (DPE)

 Before it was under Ministry of Heavy Industries and Public Enterprise


 DPE looks after Government Companies/Central Public Sector Enterprises (CPSEs)‘s
administration, expenditure, financial health, survey/performance monitoring etc.
 DPE gives 'Ratna' status to CPSEs‘ that are making good profits.
 DPE looks after CPSEs‘ employees‘ training, rehabilitation (Voluntary Retirement Scheme:
VRS)
Selection/recruitment of employees in CPSE

Selection/Interview of Managing Director, Who is responsible?


Chairman & other Top-Officials in
Public Sector Banks/NBFCs/AIFI e.g. SBI, PNB, LIC, Financial Services Institutions Bureau
NABARD, NHB etc (FSIB)

Government companies Other than [Public Sector Public Enterprises Selection Board
Bank/NBFC/AIFI] e.g. ONGC, Coal India, Hindustan (PESB) under the Ministry of personnel
Copper, Hindustan Aeronautics Limited (HAL) etc
RBI Governor, SEBI Chief etc regulators Separate Committee headed by
Cabinet Secretary (IAS)

The highest official in each of above 6 depts is called ‗Secretary‘ (usually an IAS) → senior-most
among them is designated as Finance Secretary=signs ₹ 1 note. Finance Secretary usually
heads the Dept of Economic Affairs (DEA)

DPE → National Land Monetization Corporation (NLMC-2022)

https://t.me/prelimbits
201

 The Union Finance Minister had announced in the Union Budget 2021-22 it as a Special
Purpose Vehicle (SPV) to carry out monetisation of government and Surplus Land
holdings of public sector undertakings (PSU)
 This company 100% owned by Govt with paid up capital of ₹150 cr.
 Monetisation Mean : When the government monetises its assets, it essentially means that
it is transferring the revenue rights of the asset to a private player for a specified period
of time.
149. Consider the following actions by the Government: [2010]
1. Cutting the tax rates
2. Increasing the government spending
3. Abolishing the subsidies in the context of economic recession
Which of the above actions can be considered a part of the ―fiscal stimulus‖ package?

A. 1 and 2 only
B. 2 only
C. 1 and 3 only
D. 1, 2 and 3
Ans. A

FISCAL STIMULUS
 A 'Fiscal stimulus' is a set of policies designed by authorities to jump-start a sluggish
economy.
 To encourage consumer spending, the central bank will increase the money supply
or lower the cost of money (interest rates).
 A fiscal stimulus involves the government spending more money from its own coffers or
lowering tax rates to put more money in consumers' hands.
 For instance, during the COVID-19 pandemic, the central government announced a fiscal
stimulus package of Rs. 20 Lakh crore
Features of Fiscal Stimulus

 It emerged as a tool of optimism during the financial crisis and global recession in
advanced economies.
 It could also lead to an increase in the deficit and debt levels of countries, which may
operate as a permanent drag for some countries.
 It can lead to deviation from the path of fiscal consolidation and also fiscal deficit.
 When the fiscal stimulus is used to stimulate consumer demand, rather than to create
income yielding assets through appropriate investment it can cause inflation due to a
high fiscal deficit.
Need for Fiscal Stimulus

 To stimulate economic demand during the unemployment rise, shrinking income and
consumer confidence.

https://t.me/prelimbits
202

 A fiscal stimulus revives business confidence, restarts stalled projects, helps


in job creation and sets off a virtuous cycle of demand and growth.
 Pandemic induced job losses have resulted in increased rates of unemployment across
global economies.
 For better economic growth, wealth creation is essential which can be accomplished by
providing a fiscal stimulus.
Impacts of Fiscal Stimulus Framework

 Fiscal stimulus results in a sudden rise in liquidities and can also cause widespread
bankruptcies, losses of organisational capital, a steep path towards economic recovery.
 It can result in a liquidity trap in which the rate of interest decreases, there is a liquidity
preference as almost everyone prefers holding cash.
 It can result in high inflation.
 Fiscal stimulus involves expenditures on health, food and income support for vulnerable
households which can put a strain on the government exchequer.
 It can cause an increase in gross Public Debt, impact the credit ratings, etc and therefore
cause deterioration of public finances.
Fiscal Stimulus during COVID-19

 During the pandemic there was an increase in job losses that lead to an increase in
unemployment.
 Various sectors of the economy especially the manufacturing sector was badly impacted.
 India offered economic relief packages such as the Pradhan Mantri Garib Kalyan Yojana
worth Rs 1.75 lakh crore or roughly 0.8% of the GDP.
 Atmanirbhar Bharat Abhiyan amounting to Rs. 20 lakh crore was launched for farmers,
cottage industry, MSMEs, labourers, middle class etc.
 Many countries such as Bangladesh and Indonesia, etc resorted to expanding their coverage
of the cash transfer programmes from pre-COVID-19 levels.
 Some such as China, Vietnam, etc adopted a dual strategy of providing relief to workers
who have been laid off and feeding poor families, while also trying to keep firms afloat.
Fiscal Stimulus vs. Monetary Stimulus

Monetary Stimulus Fiscal Stimulus


It is regulated by central banks that focus on It is a Government-Regulated Measure which
low inflation rates to stabilise economic involves change in government spending and
growth by increasing the amount of money taxation to revive the economy.
available.
It is undertaken by central banks to regulate
The government used fiscal stimulus packages to
the supply of money in the country. influence overall supply and demand by →
The main tool of a monetary stimulus is  cutting down on taxes,
Interest Rates.  increasing spending and
 boosting economic growth
It reduces marketing Interest Rates, It is done by the government through direct
increases the money supply by injecting spending and increasing hiring to promote

https://t.me/prelimbits
203

more cash into the economy. employment and growth


It puts extra money into the hands of the They are the last resort to achieve price stability,
people during times of recession steady economic growth and promote
employment

150. In the context of India‟s Five Year Plans, a shift in the pattern of industrialization,
with lower emphasis on heavy industries and more on infrastructure begins in [2010]
A. Fourth Plan
B. Sixth Plan
C. Eighth Plan
D. Tenth Plan
Ans. B

SIXTH FIVE YEAR PLAN (1980-85)


 Under this plan, a shift in the pattern of industrialization, with a lower emphasis on
heavy industries and more on infrastructure began.
 From the sixth five-year plan onwards, there was massive investment in the Social
Services.
 These social services included Education, Health and Family Planning, Housing
& Urban Development, and other services.
 From the 6th Plan onwards, the role & scope of the Planning Commission also increased.
 The plan objectives were poverty alleviation and higher economic growth.
 Special attention was paid to the removal of poverty through rural development schemes
such as Integrated Rural Development Programme (IRDP), National Rural Employment
Programme (NREP), and Rural Landless Employment Guarantee Programme (RLEGP),
etc.
 The poverty was 47% at the beginning of the plan and a target of 30% was fixed to achieve.
 The actual target achieved at that time was 37%.
 The Integrated Rural Development Program (IRDP) was launched by the Government of
India during the financial year of 1978 and implemented during 1980.
 National Rural Employment Programmes (NREP) was launched in 1980 with a view to
significantly increase employment opportunities in rural areas.
 Rural Landless Employment Guarantee Programme was launched on 15th August
1983 to generate additional employment in rural areas, particularly for the rural landless
workers.
FOURTH PLAN
 The Fourth plan when it was introduced after a gap of three years, was an ambitious plan
with an aim of 5.5% growth as the previous plans had a growth target/achievement of
a maximum of 3.5%.
 The Indo-Pakistan war, the liberation of Bangladesh and the problem of Bangladesh
refugees, successive failures of monsoon, Asian Oil Crisis of 1973 marred the objectives

https://t.me/prelimbits
204

of this plan.
 The international economic turmoil due to Oil crisis upset the calculations for Fourth
Plan.
 So only 3.4% growth could be achieved.
EIGHTH PLAN
 The eighth five-year plan can be called a ―Rao and Manmohan Plan‖
 This was the reform period and the following took place during the reform period.
 In 1991, Rupee was once again evaluated.
 Due to the currency devaluation, the Indian Rupee fell from 17.50 per dollar in 1991 to 45
per dollar in 1992.
 The Value of the Rupee was evaluated by 23%.
TENTH PLAN
 The tenth plan was launched by Atal Bihari Vajpayee Government on December 21,
2002.
 This plan was prepared in the background of high expectations arising from the better
growth rate achieved after the liberalization.
 The economy accelerated in the Tenth Plan period (2002–03 to 2006–07) to record an
average growth of 7.7%, the highest in any Plan period so far.
 National Income increased by 7.6% and Per capita income by 6% per annum.
 Industrial production increased at the rate of 8.6% per year.
 In the last year of the plan, double-digit growth was achieved.
151. With reference to the National Investment Fund to which the disinvestment
proceeds are routed, consider the following statements: [2010]
1. The assets in the National Investment Fund are managed by the Union Ministry of Finance.
2. The National Investment Fund is to be maintained within the Consolidated Fund of India.
3. Certain Asset Management companies are appointed as the fund managers.
4. A certain proportion of annual income is used for financing select social sectors.
Which of the statements given above is/are correct ?

A. 1 and 2
B. 2 only
C. 3 and 4
D. 3 only
Ans. C

NATIONAL INVESTMENT FUND (NIF)


 The NIF was constituted by the Government of India in November 2005.
 It was set for channelizing the proceeds from the disinvestment of Central Public
Sector Enterprises are channelized into it.
 The corpus of NIF is of a permanent nature.
 The money with the NIF is permanent in nature and NIF is professionally managed to
https://t.me/prelimbits
205

provide returns to the Government, without depleting its value.


 The NIF is professionally managed by some selected Public Sector Mutual
Funds (like UTI Asset Management Company Ltd., SBI Funds Management Private
Ltd.) and LIC Mutual Fund Asset Management Company Ltd., to provide sustainable
returns to the Government, without depleting the corpus. Hence, statement 1 and 2 is Not
correct.
 Selected Public Sector Mutual Funds, namely UTI Asset Management Company Ltd., SBI
Funds Management Private Ltd., and LIC Mutual Fund Asset Management Company Ltd.
were entrusted with the management of the NIF corpus. Hence, statement 3 is correct.
 The annual income of the NIF is divided into two parts:
o The three-fourth of the annual income of the NIF is used for financing selected
social sector schemes that promote education, health, and employment. Hence,
statement 4 is correct.
o The remaining one-fourth of the annual income of NIF is used to meet the capital
investment requirements of profitable and revivable PSUs.
 Some more purposes where the NIF was to be utilized, including in 2013, were:
o Subscribing to the shares being issued by the CPSE on a rights basis so as to ensure
that 51% ownership of the Government in CPSEs is not diluted.
o Preferential allotment of shares of the CPSE to promoters as per SEBI (Issue of Capital
and Disclosure Requirements) Regulations, 2009 so that Government shareholding
does not go down below 51% in all cases where the CPSEs desire to raise fresh
equity to meet their Capex program.
o Recapitalization of public sector banks and public sector insurance companies so as
to strengthen them by further capital infusion towards achieving the Basel III norms.
NATIONAL INVESTMENT AND INFRASTRUCTURE FUND (NIIF)
 NIIF is a government-backed entity established to provide long-term capital to the
country‘s infrastructure sector.
 The Indian government has a 49% stake in NIIF with the rest held by foreign and domestic
investors.
 With the Centre‘s significant stake, NIIF is considered India‘s quasi-sovereign wealth fund.
 It was set up in December 2015 as a Category-II Alternate Investment Fund, with fund
of Rs. 40000 Crore
 Its registered office is in New Delhi.
 Across its three funds, it manages over USD 4.3 billion of capital.
 Funds that are currently managed by NIIFL →
o Master Fund: Invests primarily in operating assets in core infrastructure sectors
such as roads, ports, airports, power, etc.
o Fund of Funds (FoF) : Managed by fund managers with experience in
infrastructure and related sectors in India. Green Infrastructure, Mid-Income &
Affordable Housing, Infrastructure Services, and allied sectors are some of the areas
of focus.

https://t.me/prelimbits
206

o Strategic Opportunities Fund (SoF): SOF has been established with the objective
to provide long-term capital to high-growth future-ready businesses in
India. The fund‘s strategy is to build a portfolio of large entrepreneur-led or
professionally managed domestic champions and unicorns.
152. In India, the tax proceeds of which one of the following as a percentage of gross tax
revenue has significantly declined in the last five years? [2010]
A. Service tax
B. Personal income tax
C. Excise duty
D. Corporation tax
Ans. C

The excise duty‘s share in the total tax revenue, which was 41.3 percent in 1992-93, declined to
25.1 percent in 2006- 07. The customs duty‘s share in the total tax revenue, which was 31.9 percent
in 1992-93, fell to 17.5 percent in 2006-07, as a result of massive structuring on excise and
customs.

153. Which one of the following authorities makes recommendation to the Governor of a
State as to the principles for determining the taxes and duties which may be
appropriated by the Panchayats in that particular State? [2010]
A. District Planning Committees
B. State Finance Commission
C. Finance ministry of that State
D. Panchayati Raj Ministry of that State
Ans. B

STATE FINANCE COMMISSION


 State Finance Commission governs the distribution between the State and the
Panchayats of the net proceeds of the taxes, duties, tolls and fees leviable by the State,
which may be divided between them under this Part and the allocation between the
Panchayats at all levels of their respective shares of such proceeds; the determination of the
taxes, duties, tolls and fees which may be assigned as, or appropriated by, the Panchayats;
the grants-in-aid to the Panchayats from the Consolidated Fund of the State.
154. Consider the following statements: In India, taxes on transactions in Stock
Exchanges and Futures Markets are [2010]
1. levied by the Union
2. collected by the States
Which of the statements given above is/are correct?

A. 1 only
B. 2 only
C. Both 1 and 2

https://t.me/prelimbits
207

D. Neither 1 nor 2
Ans. A

SECURITIES TRANSACTION TAX


 The securities transaction tax is levied on gains from securities such as equities,
options, and futures done in the domestic stock exchange.
 It is a direct tax. The central government levies and collects this tax. Hence, statement
1 is correct and statement 2 is incorrect.
 Securities transaction tax (STT) was introduced in the year 2004.
 This tax was introduced to avoid tax evasion in the case of capital gains.
 The securities transaction tax is levied on the value of securities (except commodities and
currency).
 In the year 2013, the government was made to cut down the rate of taxation for STT
after a lot of protests by brokers and people from the trading community.
 The rate of taxation is different for different types of securities.
 STT charge is levied on all sell transactions for both options as well as futures.
 For purpose of STT calculation, each futures trade is valued at the actual traded price while
each option trade is valued at a premium.
 The amount STT that a clearing member has to pay is the sum total of all the STT taxes of
trading members under him.
 The most prominent point about securities transaction tax is that STT charge is applicable
only on share transactions made through a recognized stock exchange in the country.
 Off-market share transactions are not covered under STT.
155. Consider the following statements ; [2010]
1. The Union Government fixes the Statutory Minimum Price of sugarcane for each sugar
season
2. Sugar and sugarcane are essential commodities under the Essential Commodities Act.
Which of the statements .given above is/are correct?

A. 1 only
B. 2 only
C. Both 1 and 2
D. Neither 1 nor 2
Ans. C

ESSENTIAL COMMODITIES ACT


 The Act came into force in 1955 and dealt with the commodities or products that are most
required by the people and obstruction in the supply of these could affect the lives of the
common people to a great extent.
 The commodities included are - Fertilizers, Pulses, Edible Oil, Cereals, Oilseeds, Sugarcane
and its products, Petroleum and allied products, Seeds of fruits and vegetables. Hence
Statement 2 is correct.

https://t.me/prelimbits
208

 The government can fix the Minimum Support Price for these commodities.
MINIMUM SUPPORT PRICE
 It is the minimum price decided by the government, for certain agricultural products, below
which the products could not be purchased from the farmers.
 It is decided twice a year on the recommendation of the Commission for Agricultural
Costs and Prices (CACP) for 22 mandated crops.
SUGARCANE PRICE IN INDIA
 The sugarcane price in India is determined by statutory provisions of the Sugarcane
(Control) Order, 1966 issued under the Essential Commodities Act (ECA), 1955.
 From the 2009-10 sugarcane season onwards, the Fair and Remunerative Price of
sugarcane replaced the Statutory Minimum Price (SMP).
 Fair and Remunerative Price (FRP)
o It is the minimum price that is determined by the Government of India on the
recommendation of the Commission for Agricultural Costs and Prices (CACP).
o Sugar mills can't purchase sugarcane from farmers below this price. Hence
Statement 1 is correct.
 State Advised Price (SAP)
o It is the price announced by the states having a major share in sugarcane
production.
o It is generally higher than the FRP.
ADDITIONAL INFORMATION
 The sugar industry is an important agro-based industry that impacts the rural livelihood
of about 50 million sugarcane farmers and around 5 lakh workers directly employed in
sugar mills.
 Employment is also generated in various ancillary activities relating to transport, trade
servicing of machinery, and supply of agriculture inputs.
 India has emerged as the world's largest producer and consumer of sugar as well as the
world's second largest exporter of sugar
 Today Indian sugar industry‘s annual output is worth approximately Rs.80,000 crores.
 With the amendment of the Sugarcane (Control) Order, 1966 on 22.10.2009 and the
concept of Statutory Minimum Price (SMP) of sugarcane was replaced with the ‗Fair and
Remunerative Price (FRP) of sugarcane for 2009-10 and subsequent sugar seasons.
 The cane price announced by the Central Government is decided on the basis of the
recommendations of the Commission for Agricultural Costs and Prices (CACP) after
consulting the State Governments and associations of the sugar industry.
156. Which one of the following is not a feature of Limited Liability Partnership firm?
[2010]
A. Partners should be less than 20
B. Partnership and management need not be separate

https://t.me/prelimbits
209

C. Internal governance may be decided by mutual agreement among partners


D. It is corporate body with perpetual succession
Ans. A

 The Indian Partnership Act 1932 enacts all the provisions relating to partnership.
THE INDIAN PARTNERSHIP ACT 1932
 It enforces the conduct of business and agreements between the partners to settle
disputes between them.
 These Acts are useful acts to control partnership business.
 In the Indian economy, either a single business or a partnership business is being done.
 Limited liability partnership means some or all partners have limited liability.
 Each partner is not responsible for another partner's misconduct.
 It is a corporate body with perpetual succession.
 To start this, a minimum of two members are required and there is no cap on the
maximum number of partners.
 Partnership and management need not be separate.
 Internal governance can be decided by mutual agreement among partners.
157. In India, which of the following, is regulated by the Forward Markets Commission?
[2010]
A. Currency Futures Trading
B. Commodities Futures Trading
C. Equity Futures Trading
D. Both Commodities Futures and Financial Futures Trading
Ans. B

FORWARD MARKETS COMMISSION (FMC)


 The Forward Markets Commission (FMC) is the regulatory body for the commodity
market and futures market in India.
 It is a division of the Securities and Exchange Board of India, Ministry of Finance,
Government of India.
 It is headquartered in Mumbai and this financial regulatory agency is overseen by the
Ministry of Finance. The Commission allows commodity trading in 22 exchanges in India, of
which 6 are national.
 On 28 September 2015 the FMC was merged with the Securities and Exchange Board of
India (SEBI) to make the regulation of commodity futures market strong.
158. Which of the following terms indicates a mechanism used by commercial banks for
providing credit to the government? [2010]
A. Cash Credit Ratio
B. Debt Service Obligation
C. Liquidity Adjustment Facility

https://t.me/prelimbits
210

D. Statutory Liquidity Ratio


Ans. D

STATUTORY LIQUIDITY RATIO(SLR)


 Commercial banks provide long-term credit to the government by investing their funds in
government securities and short-term finance by purchasing Treasury Bills. This comes
under SLR.
 Statutory Liquidity Ratio or SLR is the minimum percentage of deposits that a
commercial bank has to maintain in the form of liquid cash, gold or other securities. It
is basically the reserve requirement that banks are expected to keep before offering credit
to customers. The SLR is fixed by the RBI and is a form of control over the credit growth in
India.
 The government uses the SLR to regulate inflation and fuel growth. Increasing the SLR
will control inflation in the economy while decreasing the statutory liquidity rate will cause
growth in the economy. The SLR was prescribed by Section 24 (2A) of Banking Regulation
Act, 1949.
CASH RESERVE RATIO (CRR)
 CRR is a specific part of the total deposit that is held as a reserve by the commercial banks
mandated by RBI.
 This reserve must be reserved in cash or cash equivalent.
CRR SLR
Banks must keep this much deposits (or Banks must keep with themselves this
balance) with RBI. [only in cash] deposits in liquid assets such as cash, gold,
RBI doesn‘t pay interest on this deposit, except G-Sec, T-Bills, State Development Loan Bonds
in extraordinary circumstances like 1999‘s and other securities notified by RBI.
Banking slowdown.
Bank earns no profit / interest, as such. Some profit may be involved.
CRR: first suggested by the British economist Mandated under Banking Regulation Act,
J.M. Keynes & first introduced in US Federal 1949
Reserves (=Central Bank of USA).
Mandated under RBI Act, 1934

RBI can fix any amount of CRR, legally there is Legally, SLR can‘t be more than 40%.
no minimum floor or maximum ceiling. Presently it‘s 18.00% of NDTL.
Presently it‘s 4.50% of Net Demand & Time
Liabilities (NDTL) of a bank
All Scheduled Commercial Banks (SCB) must Similar
keep CRR.
However, RBI may prescribe separate %
norms/slabs for Regional Rural Banks (RRBs) and
Cooperative Banks.

https://t.me/prelimbits
211

LIQUIDITY ADJUSTMENT FACILITY (2000)


 A liquidity adjustment facility (LAF) is a tool used in monetary policy, primarily by the
Reserve Bank of India (RBI) that allows banks to borrow money through repurchase
agreements or to make loans to the RBI through reverse repo agreements
 RBI‘s Liquidity Adjustment Facility (LAF): has two windows:
Repo Rate [RR] Reverse Repo Rate [RRR]
The Interest rate at which RBI lends short- It‘s the interest rate that clients [Banks] earn
term loans to its clients [Banks or govt.], when parking their surplus funds with the
keeping their G-Sec as collaterals. RBI for short periods, to earn interest. [or when
RBI buy money form Commercial Banks]
Clients enter into an agreement with RBI to Mechanism similar to Repo, RBI gives its G-Sec
repurchase their G-sec at a future date at a as collateral.
(higher) pre-determined price.
Banks can‘t pledge their SLR-quota-G-Secs
for this borrowing.
Repo Rate is RBI‘s Policy Rate to control RR is higher than RRR
inflation. If RBI decreases the Reverse Repo rate, it means
Inflation - Increase RBI wants expansion of credit.
Deflation - Decrease With this fewer banks will supply their surplus
to central bank because earning is less.
This will affect the financial market as supply of
money in financial market will increase.
Due to increase in the supply of credit in the
market, inflation rate will increase.
DEBT SERVICE OBLIGATION
 Debt service is the cash that is required to cover the repayment of interest and principal on
a debt for a particular period.
159. With reference to the Non-banking Financial Companies (NBFCs) in India, consider
the following statements: [2010]
1. They cannot engage in the acquisition of securities issued by the government.
2. They cannot accept demand deposits like Savings Account
Which of the statement given above is/ are correct?

A. 1 only
B. 2 only
C. Both 1 and 2
D. Neither 1 nor 2
Ans. B

NBFCS
Parameter Commercial Banks NBFCs
Registration Banking Regulation Act Companies Act
Supervision RBI Varies: Mutual funds-SEBI, Insurance

https://t.me/prelimbits
212

Company: IRDAI etc.


Entry Capital 500 Cr. 5cr for MicroFin, 2 cr for others; 200 cr. For
reinsurer etc.
Can accept Deposits? Can accept Time & Demand Only NBFC-Deposit-Taking (NBFC-D) & even
[chequable deposit]** they can accept only Time Deposits. E.g. Bajaj
Their deposits are insured under Finance.
DICGCI Act. Can’t issue their own chequebook,
debit/credit card.
Deposits are not insured under DICGCI Act.
NBFC is engaged in the business of loans and
advances, acquisition of
shares/stock/bonds/debentures/ securities
issued by a government or local authority or
other securities of like marketable nature,
leasing, hire-purchase, insurance business, chit
business.
Prudential Norms CRR, SLR, applicable NBFC-D: SLR required but RBI can prescribe
different slabs / norms.
CRR not applicable on any type of NBFC.

BASEL Capital Yes, Applicable. Applicable on 108 NBFC-D and


Adequacy Norms, Applicable on 276 NBFCs – ND – SI (non-
LCR-HQLA norms deposit taking Systematically Important with
assets over ₹ 500 crores) e.g. L&T Finance,
Cholamandalam etc.
But RBI can prescribe different slabs / norms
/deadlines.
Investment They can keep depositor’s Can invest clients’ money in share market.
money in RBI approved E.g. Mutual Funds, Insurance Companies.
securities. But can’t invest in
share market per se, directly.
Loan Rate Decided as per RBI’s Varies & depends on nature of biz.
methodology from time to time
(BPLR, MCLR, External
Benchmark etc.)
Recovery Loan recovery powers under Housing Finance Companies have SARFAESI
SARFAESI Act. powers. But, all types of NBFCs don’t have it.
E.g.
Gold Loan company can auction gold.
Mutual Fund / Insurance Company may have
to wait till liquidation of bankrupt company
where they invested clients’ ₹.
Consumer Complaints RBI’s Ombudsman, Bank’s RBI’s separate Ombudsman for NBFCs starting
Internal Ombudsman the NBFC-D since 2018

160. In the parlance of financial investments, the term „bear‟ denotes [2010]
A. An investor who feels that the price of a particular security is going to fall
B. An investor who expects the price of particular shares to rise
C. A shareholder or a bondholder who, has an interest in a company, financial or otherwise
D. Any lender whether by making a loan or buying a bond

https://t.me/prelimbits
213

Ans. A

Bear is a speculator who sells shares in anticipation of fall operator prices to buy them back and
thus make a profit.

BEAR MARKET
 A bear market refers to a market where share prices are continuously declining.
 Its downward trend makes investors believe that the trend will continue, which, in turn,
perpetuates the downward spiral.
 It is considered riskier to invest in a bear market, as many equities lose value.
 Thus, most investors withdraw their money from the markets.
 During a bear market, the economy slows down and unemployment rises as companies
begin laying off workers.

BULL MARKET
 A bull market refers to a market that experiences a sustained increase in market share
prices.
 It ensures investors that the uptrend will continue over the long term.
 It signifies that the country's economy is strong and employment levels are high.
161. In India, the interest rate on savings accounts in all the nationalized commercial
banks is fixed by [2010]
A. Union Ministry of Finance
B. Union Finance Commission
C. Indian Banks‘ Association
D. None of the above.
Ans. D

 The following rates are fixed by banks themselves and are not considered to be key policy
rates
https://t.me/prelimbits
214

o Base Rate
o Interest Rates on Saving Accounts
o Interest Rates on Current Accounts
 In 2011, RBI permitted the commercial banks to fix interest rate on saving account
independently.
162. With reference to the institution of Banking Ombudsman in, India, which one of the
statements is not correct? [2010]
A. The Banking Ombudsman is appointed by the Reserve Bank of India.
B. The Banking Ombudsman can, consider complaints from Non Resident Indians having
accounts in India.
C. The orders passed by the Banking Ombudsman are final and binding on the parties
concerned.
D. The service provided by the Banking Ombudsman is free of any fee.
Ans. C

INSTITUTION OF BANKING OMBUDSMAN


 It is an autonomous independent organization that monitors the services provided by
banks.
 The Banking Ombudsman Scheme was implemented in 1995, amendments were made
in 2002 and 2006 to expand the scope of the scheme to provide clean, transparent, non-
discriminatory, and responsible banking services by banks.
 Under this, a 'Banking Ombudsman' is appointed by the Reserve Bank of India who is a
quasi-judicial authority.
 The customer can lodge a complaint by postal, e-mail, online to the Banking
Ombudsman on the complaint of any officer and employee of the bank and for not
receiving the services in time.
 This free complaint is resolved within thirty days. Non-Resident Indians having accounts in
India can also lodge a complaint with him.
 The orders passed by the Banking Ombudsman are not final and are then shared with
Appellate Authority who gives the decision according to appeal.
 Appellate Authority is the Deputy Governor in charge of the Department of the Reserve
Bank implementing the Scheme.
RBI INTEGRATED OMBUDSMAN SCHEME
 It amalgamates three ombudsman schemes of RBI - banking ombudsman scheme of
2006, ombudsman scheme for NBFCs of 2018 and ombudsman scheme of digital
transactions of 2019.
 The unified ombudsman scheme will provide redress of customer complaints involving
deficiency in services rendered by RBI regulated entities viz. banks, NBFCs (Non
banking Financial Companies) and pre-paid instrument players if the grievance is not
resolved to the satisfaction of the customers or not replied within a period of 30

https://t.me/prelimbits
215

days by the regulated entity.


 It also includes non-scheduled primary co-operative banks with a deposit size of Rs 50
crore and above. The integrated scheme makes it a ―One Nation One
Ombudsman‘ approach and jurisdiction neutral.
Features

 The Scheme defines ‗deficiency in service‘ as the ground for filing a complaint, with a
specified list of exclusions.
 Therefore, the complaints would no longer be rejected simply on account of ―not covered
under the grounds listed in the scheme‖.
 The scheme is jurisdiction neutral and a centralised receipt and processing centre has
been set-up in Chandigarh for initial handling of complaints in any language.
 RBI had created a provision for the use of Artificial Intelligence tools so that banks and
investigating agencies could coordinate in a better way in the fastest time possible.
 The bank customers will be able to file complaints, submit documents, track status, and give
feedback through a single email address.
 There will also be a multilingual toll-free number that will provide all relevant information
on grievance redress.
 The regulated entity will not have any right to appeal in cases where an award is issued
by the ombudsman against it for not furnishing satisfactory and timely information.
Appellate Authority:

 RBI‘s Executive Director-in charge of Consumer Education and Protection Department


would be the Appellate Authority under the integrated scheme.
Significance:

 This will help in improving the grievance redress mechanism for resolving customer
complaints against RBI's regulated entities.
 It is expected to ensure uniformity and streamlined user-friendly mechanisms which
will add value to the scheme and bring customer delight and financial inclusion.
 44 crore loan account holders and 220 crore deposit account holders would directly benefit
from the single ombudsman, they would now be able to lodge a complaint and track their
complaints on the same platform.
163. With reference to India, consider the following: [2010]
1. Nationalization of Banks
2. Formation of Regional Rural Banks
3. Adoption of villages by Bank Branches
Which of the above can be considered as steps taken to achieve the ―financial inclusion‖ in India?

A. 1 and 2 only
B. 2 and 3 only
C. 3 only
D. 1, 2 and 3

https://t.me/prelimbits
216

Ans. D

FINANCIAL INCLUSION
 Financial inclusion is a way of offering banking services and financial solutions to each
and every individual in the society without having any discrimination.
 The aim is that every person should get basic financial services irrespective of a person's
income or savings.
 The main focus was to provide and engage economically unprivileged sections of the
country.
 Under this scheme in 2014, Pradhan Mantri Jan Dhan Yojana was launched to give
banking services to each and every household.
o Nationalization of Banks - It is a kind of action through which an entity of
the private sector is transferred to the public sector. Hence the statement 1 is
correct.
o Formation of Regional Rural Banks - Regional Rural Banks (RRBs) was established
under the provisions of the Ordinance of 26 September 1975 and the RRB Act of
1976. Hence the statement 2 is correct.
 It was done so that banking and loan services can be provided for
agriculture and other rural areas. It was started on 2 October 1975.
o Adoption of villages by Bank Branches - In India, the village adoption scheme
was launched by the banks in 1960. Hence the statement 3 is correct.
 The aim is to grow the rural economy of the country and slow down urban
migration.
 Banks adopt villages with the active participation of village panchayats.
164. Consider the following statements: [2010]
The functions of commercial banks in India include

1. Purchase and sale of shares and securities on behalf of customers


2. Acting as executors and trustees of wills
Which of the statements given above is/are correct?

A. 1 only
B. 2 only
C. Both 1 and 2
D. Neither 1 nor 2
Ans. C

COMMERCIAL BANK
 A commercial bank is a financial institution that accepts deposits from the public and gives
loans for the purposes of consumption and investment to make a profit.
 The functions of commercial banks are broadly classified into two categories:-
o Primary functions

https://t.me/prelimbits
217

o Secondary functions
 Primary functions include:-
o Accepts deposit
o Provides loan and advances
o Credit cash
 Secondary functions include:-
o Discounting bills of exchange
o Overdraft facility
o Paying and gathering the credit
o Purchasing and selling of securities. Hence, statement 1 is correct.
 A bank is an Agent, Trustee, Executor, Administrator for Customers. Hence, statement
2 is correct.
 Trustee
o A person may desire that after his death, a part or whole of his property be held in
trust for the benefit of various beneficiaries named in the will.
o In such a case he may create a trust under his will directing a certain person to hold
the property to such persons after a specified time.
o When the bankers take the liability to administrate this type of property he will be
called the trustee.
 Executor
o A person may make will expressing his intention regarding the disposal of his
property after his death.
o A will has to be in writing, signed by the person making the will which called trusted
and attested by two witnesses.
o The person appointed as an administrator of the deceased is known as the executor.
o The bank may appoint as an executor for such service.
 Administrator:-
o In case a person dies without making a valid will, the property of the deceased may
be administered according to law.
o The bank may be appointed for the administration of this property and then the
banker will be called the administrator.
165. The International Development Association, a lending agency, is administered by
the [2010]
A. International Bank for Reconstruction and Development
B. International Fund for Agricultural Development
C. United Nations Development Programme.
D. United Nations Industrial Development Organization
Ans. A

INTERNATIONAL DEVELOPMENT ASSOCIATION (IDA)


 International Development Association (IDA) is a financial institution that offers loans

https://t.me/prelimbits
218

and funds to the poorest developing countries of the world.


 It was developed in 1960 headquartered in Washington DC, United States and works in
cooperation with International Bank for Reconstruction and Development (IBRD) which
was developed in 1944 to help rebuild Europe after World War II.
 IBRD and IDA together are called the World Bank.
 Both the organizations have the same headquarters and staff and evaluate projects
based on the same standards.
 IDA emphasizes those countries which have the lowest gross national income and lowest
per capita income.
 The aim of the World Bank is to reduce poverty and provide affordable development
facilities to poor countries.
 These two organizations work collectively for the same.
INTERNATIONAL FUND FOR DEVELOPMENT
 The International Fund for Agricultural Development is an international financial institution
and a specialised agency of the United Nations that works to address poverty and hunger in
rural areas of developing countries. It is the only multilateral development organization that
focuses solely on rural economies and food security.
 IFAD was established as an international financial institution in 1977
 Headquartered in Rome,
UNITED NATIONS DEVELOPMENT PROGRAMME
 The United Nations Development Programme (UNDP) is a United Nations agency tasked
with helping countries eliminate poverty and achieve sustainable economic growth and
human development.
 The UNDP emphasizes developing local capacity towards long-term self-sufficiency and
prosperity
 Headquartered in New York City, it is the largest UN development aid agency, with offices in
177 countries
 The UNDP is funded entirely by voluntary contributions from UN member states.
 Formation : 22 November 1965
UNITED NATIONS INDUSTRIAL DEVELOPMENT ORGANIZATION
 The United Nations Industrial Development Organization is a specialized agency of the
United Nations that assists countries in economic and industrial development.
 It is headquartered at the UN Office in Vienna, Austria, with a permanent presence in
over 60 countries.
 As of April 2019, UNIDO comprises 170 member states, which together set the
organization's policies, programs, and principles through the biannual General Conference.
 UNIDO was established in 1966 by the UN General Assembly to promote and accelerate
the industrialization of developing countries, which were emerging from
decolonization in record numbers and with little to no industrial base.
 In 1979 it became one of the 15 specialized agencies of the UN, with its new constitution

https://t.me/prelimbits
219

coming into force in 1985.


166. India is regarded as a country with “Demographic Dividend‟‟. This is due to [2011 –
I]
A. Its high population in the age group below 15 years.
B. Its high population in the age group of 15–64 years.
C. Its high population in the age group above 65 years.
D. Its high total population.
Ans. B

DEMOGRAPHIC DIVIDEND
 The demographic dividend is the economic growth potential that can result from shifts in a
population‘s age structure, mainly when the share of the working-age population (15 to
64) is larger than the non-working-age share of the population (14 and younger, and 65
and older).
 Demographic dividend in India :
o As populations in countries such as China, US, and Japan is getting older, India‘s
population is getting younger.
o India‘s working-age population is now increasing because of rapidly declining
birth and death rates.
o India‘s age dependency ratio, the ratio of dependents (children and the elderly) to
the working-age population (14- to 65-year-olds), is expected to only start rising in
2040, as per UN estimates. This presents a golden opportunity for economic growth.
o However, this growth will depend on those in the working-age population actually
working.
o India‘s labour force participation rate is declining, especially among rural youth (15-
to 29-year-olds) and women.
o For India to harness the power of its favourable demographics, India‘s labour
force needs to be empowered with the right skills for the modern economy.
167. Which one of the following is not a feature of “Value Added Tax” ? [2011 - I]
A. It is multi-point destination-based system of taxation.
B. It is a tax levied on value addition at each stage of transaction in the production distribution
chain.
C. It is a tax on the final consumption of goods or services and must ultimately be borne by
the consumer.
D. It is basically a subject of the central government and the state governments are only a
facilitator for its successful implementation.
Ans. D

VALUE-ADDED TAX (VAT)


 It is a type of indirect tax levied on goods and services for value-added at every point of
the production or distribution cycle, starting from raw materials and going all the way to

https://t.me/prelimbits
220

the final retail purchase.


 VAT was introduced on April 1, 2005.
 Under it, the amount of value addition is first identified at each stage, and then tax is
levied on the same.
 Ultimately, the end consumer has to pay the complete VAT while buying goods; buyers
at earlier stages of production receive reimbursements for tax they have paid. Because the
consumer bears the entire tax, VAT is also a consumption tax.
 Each state has its own VAT laws for proper implementation and levying. Different states
apply different VAT rates according to their implied laws.
o VAT, being a 'tax on sale or purchase of goods within a State' is a State Subject
by virtue of Entry 54 of State List of the Seventh Schedule of the Constitution of
India. Since VAT/Sales tax is a State subject, the Central Government has been
playing the role of a facilitator for the successful implementation of
VAT. Hence, option 4 is not correct.
 The main aim behind the introduction of VAT was to eliminate the presence of double
taxation and the cascading effect of the then-existing sales tax structure.
o A cascading effect is when there is a tax levied on a product at every step of the
sale.
o The tax is levied on a value that includes tax paid by the previous buyer, so the
consumer ends up paying tax on already-paid tax.
 Has GST totally subsumed VAT?
o To completely eliminate the cascading effect of taxes and to make the indirect tax
structure simpler, the union government introduced the Goods and Services Tax
(GST) in July 2017.
o Though GST replaced VAT on most goods, some goods are still not covered under
the new regime. VAT continues to be the tax levied on such goods.
168. Karl Marx explained the process of class struggle with the help of which one of the
following theories? [2011 - I]
A. Empirical liberalism
B. Existentialism
C. Darwin‘s theory of evolution
D. Dialectical materialism
Ans. D

DIALECTICAL MATERIALISM
 Dialectical materialism, a philosophical approach to reality derived from the writings
of Karl Marx and Friedrich Engels.
 For Marx and Engels, materialism meant that the material world, perceptible to the
senses, has an objective reality independent of mind or spirit.
 They did not deny the reality of mental or spiritual processes but affirmed that ideas could
arise, therefore, only as products and reflections of material conditions.

https://t.me/prelimbits
221

 Marx and Engels understood materialism as the opposite of idealism, by which they meant
any theory that treats the matter as dependent on mind or spirit, or mind or spirit as
capable of existing independently of matter.
 For them, the materialist and idealist views were irreconcilably opposed throughout the
historical development of philosophy.
 They adopted a thoroughgoing materialist approach, holding that any attempt to combine
or reconcile materialism with idealism must result in confusion and inconsistency.
 The concept of dialectical materialism—which is a theoretical basis for a method of
reasoning—should not be confused with ―historical materialism,‖ which is the Marxist
interpretation of history in terms of the class struggle. However, it is based on dialectical
materialism.
Additional Information

 Dialectical materialism is the world outlook of the Marxist-Leninist party.


 It is called dialectical materialism because its approach to the phenomena of nature, its
method of studying and apprehending them, is dialectical, while its interpretation of the
phenomena of nature, its conception of these phenomena, its theory, is materialistic.
 Historical materialism is the extension of the principles of dialectical materialism to
the study of social life, an application of the principles of dialectical materialism to
the phenomena of the life of society, to the study of society and of its history.
 It posits that history is made as a result of the struggle between different social
classes rooted in the underlying economic base.
169. All revenues received by the Union Government by way of taxes and other receipts
for the conduct of Government business are credited to the ? [2011 - I]
A. Contingency Fund of India
B. Public Account
C. Consolidated Fund of India
D. Deposits and Advances Fund
Ans. C

CONSOLIDATED FUND OF INDIA


 Consolidated Fund is constituted under Article 266 (1) of the Constitution of India.
 All expenditures of the government are incurred from this fund.
 All revenues received by the Union Government by way of taxes (Income Tax, Central
Excise, Customs and other receipts) and other receipts flowing to the Government in
connection with the conduct of Government business i.e. Non-Tax Revenues are
credited into the Consolidated Fund.
 All loans raised by the Government by the issue of Public notifications, treasury bills
(internal debt), and loans obtained from foreign governments and international institutions
(external debt) are credited into this fund.
 No amount can be withdrawn from the Consolidated Fund without authorization from the
Parliament.
https://t.me/prelimbits
222

CONTINGENCY FUND OF INDIA


 Art 267(I) of the constitution provides that "Parliament may by law establish a Contingency
Fund in the nature of an imprest to be entitled the Contingency Fund of India into which
shall be paid from time to time such sums as may be determined by such law, and the said
Fund shall be placed at the disposal of the President to enable advances to be made by him
out of such Fund for the purposes of meeting unforeseen expenditure pending
authorisation of such expenditure by Parliament by law under Article 115 or Article 116".
PUBLIC ACCOUNT
 Public Account constituted under Article 266 (2) of the Constitution.
 The transactions relate to debt other than those included in the Consolidated Fund of India
are part of this fund.
 The transactions under Debt, Deposits, and Advances in this part are those in respect of
which Government incurs a liability to repay the money received or has a claim to recover
the amounts paid.
 The receipts under Public Account do not constitute normal receipts of Government.
 Parliamentary authorization for payments from the Public Account is therefore not required.
170. A rapid increase in the rate of inflation is sometimes attributed to the “base effect”.
What is “base effect”? [2011 - I]
A. It is the impact of drastic deficiency in supply due to failure of crops
B. It is the impact of the surge in demand due to rapid economic growth
C. It is the impact of the price levels of previous year on the calculation of inflation rate
D. None of the statements (a), (b) and (c) given above is correct in this context
Ans. C

BASE EFFECT
 The base effect refers to the impact of the rise in the price level (i.e. last year‘s
inflation) in the previous year over the corresponding rise in price levels in the current
year (i.e., current inflation).
 If the price index had risen at a high rate in the corresponding period of the previous
year leading to a high inflation rate, some of the potential rise is already factored in,
therefore a similar absolute increase in the Price index in the current year will lead to
relatively lower inflation rates.
 On the other hand, if the inflation rate was too low in the corresponding period of the
previous year, even a relatively smaller rise in the Price Index will arithmetically give a
high rate of current inflation.
Year Price Index Inflation
2008 120 20
2009 140 16.67
2010 160 14.29
 The index has increased by 20 points in all three years – 2008, 2009, 2010. However, the

https://t.me/prelimbits
223

inflation rate (calculated on a year-on-year basis) tends to decline over the three years from
20% in 2008 to 14.29% in 2010.
 This is because the absolute increase of 20 points in the price index in each year increases
the base year price index by an equivalent amount, while the absolute increase in price
index remains the same. Remember, year-on-year inflation is calculated as:

171. In the context of Indian economy, consider the following statements: [2011 - I]
1. The growth rate of GDP has steadily increased in the last five years.
2. The growth rate in per capita income has steadily increased in the last five years.
Which of the statements given above is/are correct ?

A. 1 only
B. 2 only
C. Both 1 and 2
D. Neither 1 nor 2
Ans. D

GROSS DOMESTIC PRODUCT (GDP)


 Gross domestic product (GDP) is the standard measure of the value-added created
through the production of goods and services in a country during a certain period.
 It is the sum of gross value added by all resident producers in the economy plus any
product taxes and minus any subsidies not included in the value of the products.
 The growth rate of GDP
o 2016- 8.26%
o 2017- 7.04%
o 2018- 6.12%
o 2019- 4.18%
o 2020 -10.29%
o 2021 - 8.7 %
o 2022-23 – 7 %
 As we can see, it has not been increasing for five years. Hence, statement 1 is not correct.
 GDP per capita is the gross domestic product divided by the midyear population of the
country.
 The growth rate in per capita income-
o 2016- 7.4%

https://t.me/prelimbits
224

o 2017- 9.7%
o 2018- 8.6%
o 2019- 10.0%
o 2020- 4%
 As we can see, it has not been increasing for five years. Hence, statement 2 is not correct.
172. In India, which of the following have the highest share in the disbursement of credit
to agriculture and allied activities? [2011 - I]
A. Commercial Banks
B. Cooperative Banks
C. Regional Rural Banks
D. Microfinance Institutions
Ans. A

 Commercial Banks – 60%


 Co-operative Banks – 30%
 RRB and others – 10%
SHARE IN THE DISBURSEMENT OF CREDIT TO AGRICULTURE AND ALLIED ACTIVITIES
 Within the agriculture sector the allied sectors like horticulture, animal husbandry, dairy and
fisheries, have an important bearing on the overall economic growth.
 Public sector lending (PSL) is mandated by the Reserve Bank of India (RBI), making it
a requirement for domestic and foreign banks to offer loans to specific sectors and
sub-sectors within the nation‘s economy.
 Priority Sector includes the following categories:
o Agriculture
o Micro, Small and Medium Enterprises
o Export Credit
o Education
o Housing
o Social Infrastructure
o Renewable Energy
o Others
 At present, institutional agricultural credit is mainly disbursed by commercial banks,
cooperative banks and Regional Rural Banks (RRBs) under the multi-agency approach.
 The commercial banks disburse around 60% credit followed by cooperative banks
around 30% and RRB and others.
173. India has experienced persistent and high food inflation in the recent past. What
could be the reasons? [2011 - I]
1. Due to a gradual switchover to the cultivation of commercial crops, the area under the
cultivation of food grains has steadily decreased in the last five years by about 30%.
2. As a consequence of increasing incomes, the consumption patterns of the people have

https://t.me/prelimbits
225

undergone a significant change.


3. The food supply chain has structural constraints.
Which of the statements given above are correct ?

A. 1 and 2 only
B. 2 and 3 only
C. 1 and 3 only
D. 1, 2 and 3
Ans. B

 Increasing incomes has led to changes in consumption patterns, there has been switch over
to processed foods, milk and milk products and so on.
 This increase in demand for different products has led to increase in cost of high end
products.
 Eg Organic products Structural constraints like cold chain infrastructure, transportation
(rising fuel prices) etc. increases the cost of supply contributing to inflation.
174. In terms of economy, the visit by foreign nationals to witness the XIX
commonwealth games in India amounted to [2011 - I]
A. Export
B. Import
C. Production
D. Consumption
Ans. A

Income from ―tourism‖ is an invisible export and brings foreign exchange in the country.

 Exports are goods and services that are produced in one country and sold to buyers in
another.
o The visit of foreign nationals to India amounted to export as
 Foreign nationals will buy Indian goods and use Indian services.
 Also, India will get foreign currency.
 An import is a good or service bought in one country that was produced in another.
o Suppose a citizen of the US buying goods and services in India, then it amounted to
Import for The US. Because its citizen is buying goods and services from another
country.
 Production - the process of making or growing goods to be sold.
 Consumption - the amount used or eaten.
175. Which one of the following statements appropriately describes the “fiscal stimulus”
? [2011 - I]
A. It is a massive investment by the government in manufacturing sector to ensure the supply
of goods to meet the demand surge caused by rapid economic growth.
B. It is an intense affirmative action of the government to boost economic activity in the
country.

https://t.me/prelimbits
226

C. It is government‘s intensive action on financial institutions to ensure disbursement of loans


to agriculture and allied sectors to promote greater food production and contain food
inflation.
D. It is an extreme affirmative action by the government to pursue its policy of financial
inclusion.
Ans. B

Refer Q. 149

Governments use fiscal policy to influence the level of aggregate demand in the economy. It is an
effort to achieve economic objectives of price stability, full employment, and economic growth.

176. Consider the following actions which the government can take: [2011 - I]
1. Devaluing the domestic currency.
2. Reduction in the export subsidy.
3. Adopting suitable policies which attract greater FDI and more funds from FIIs.
Which of the above action/actions can help in reducing the current account deficit ?

A. 1 and 2
B. 2 and 3
C. 3 only
D. 1 and 3
Ans. D

 CAD exists due to a host of factors including existing exchange rate, consumer spending
level, capital inflow, inflation level, and prevailing interest rate.
 For the Current Account Deficit in India, crude oil and gold imports are
the primary reasons behind high CAD.
 The Current Account Deficit could be reduced by boosting exports and curbing
non-essential imports such as gold, mobiles, and electronics. Hence,
statement 3 is correct.
 Currency hedging and bringing easier rules for manufacturing entities to raise foreign
funds could also help.
 The government and RBI could also look to review debt investment limits for FPIs, among
other measures.
o Devaluing the domestic currency makes the export of a country competitive in
the international market, which eventually helps to increase export and to earn
foreign currency. This can help to reduce CAD. Hence, statement 1 is correct.
o Reduction in the export subsidy will discourage export and export from India will
become expensive at the international market vis-a-vis other countries. Thus, CAD
will increase. Hence, statement 2 is not correct.
CURRENT ACCOUNT DEFICIT (CAD)
 It is a measurement of a country‘s trade where the value of the goods and services it

https://t.me/prelimbits
227

imports exceeds the value of the products it exports.


 To understand CAD in detail, it is essential to learn about the Current Account.
o A nation‘s Current Account maintains a record of the country‘s transactions with
other nations, in terms of trade of goods and services, net earnings on overseas
investments and net transfer of payments over a period of time, such
as remittances.
o This account goes into a deficit when money sent outward exceeds that coming
inward.
 Current Account Deficit is slightly different from Balance of Trade, which measures only
the gap in earnings and expenditure on exports and imports of goods and services.
o Whereas, the current account also factors in the payments from domestic capital
deployed overseas.
o For example, rental income from an Indian owning a house in the UK would be
computed in the Current Account, but not in the Balance of Trade.
How to calculate the Current Account Deficit?

 The current account constitutes net income, interest and dividends and transfers such as
foreign aid, remittances, donations among others. It is measured as a percentage of GDP.
 Trade gap = Exports – Imports
 Current Account = Trade gap + Net current transfers + Net income abroad
Is the Current Account Deficit something to worry about?

 Current Account Deficit may be a positive or negative indicator for an economy


depending upon why it is running a deficit.
 Foreign capital is seen to have been used to finance investments in many economies.
 Current Account Deficit may help a debtor nation in the short-term, but it may worry in
the long-term as investors begin raising concerns over adequate return
on their investments.
177. Both Foreign Direct Investment (FDI) and Foreign Institutional Investor (FII) are
related to investment in a country. Which one of the following statements best
represents an important difference between the two ? [2011 - I]
A. FII helps bring better management skills and technology, while FDI only brings in capital.
B. FII helps in increasing capital availability in general, while FDI only targets specific sectors.
C. FDI flows only into the secondary market while FII targets primary market
D. FII is considered to be more stable than FDI.
Ans. B

FOREIGN DIRECT INVESTMENT (FDI) & FOREIGN INSTITUTIONAL INVESTMENT (FII)

 Foreign Direct Investment (FDI) is defined as the type of investment into production
or business in a country, by an enterprise based in another country. It is often
contrasted with Foreign Institutional Investment (FII), which is an investment fund,

https://t.me/prelimbits
228

based in the country, other than the country, in which investment is made.
 Both are the forms of investment made in a foreign country. FDI is made to acquire
controlling ownership in an enterprise but FII tends to invest in the foreign
financial market. In most cases, the former is given preference over the latter because it
benefits the whole economy.
 FDI targets a particular company, but FII does not target a particular company.
 FII flows only into the secondary markets, while FDI targets the primary market.
 FDI is considered to the more stable than FII.
 FDI helps bring better management skills and technology, while FII only brings in the
capital.
FDI FII
Meaning When a company situated in one FII is when foreign companies make
country makes an investment in a investments in the stock market of a
company situated abroad, it is country
known as FDI.
Entry and Exit Difficult Easy
It brings Long term capital Long/Short term capital
Transfer of Funds, resources, technology, Funds only.
strategies, know-how, etc
Economic Yes No
Growth
Consequences Increase in country's Gross Domestic Increase in the capital of the country.
Product (GDP).
Target Specific Company No such target, investment flows into
the financial market.
Control over a Yes No
company

178. With reference to “Aam Admi Bima Yojana‟‟, consider the following statements ?
[2011 - I]
1. The member insured under the scheme must be the head of the family or an earning
member of the family in a rural landless house-hold.
2. The member insured must be in the age group of 30 to 65 years.
3. There is a provision for free scholarship for up to two children of the insured who are
studying between classes 9 and 12.
Which of the statements given above is/are correct?

A. 1 only
B. 2 and 3 only
C. 1 and 3 only
D. 1, 2 and 3
Ans. C

https://t.me/prelimbits
229

AAM ADMI BIMA YOJANA (AABY)


 Aam Admi Bima Yojana has been launched under the Ministry of Finance. It is a social
security scheme that was launched on 2nd October 2007 to provide insurance cover
to individuals belonging to specific groups against natural and accidental
deaths and disabilities.
 AABY Scheme will increase the coverage of social security schemes in India. Life Insurance
Corporation of India (LIC) is the nodal agency for this scheme.
Objectives →

 To extend the coverage of social security in India.


 To provide benefits to BPL families.
 To provide insurance coverage against both death and disability.
 To put more money supply into the economy.
Salient Features

 It has been merged with the Janashree Bima Yojana to provide a wider scope of reach to
eligible individuals.
 Natural death, accidental death, permanent or partial disability suffered due to an
accident, and permanent total disability suffered due to an accident are covered under
this scheme.
 This scheme covers the members of 48 groups that have been identified for the purpose of
coverage.
 The coverage amount in this scheme is fixed and the amount depends on the
contingency suffered.
 The lump-sum amount of coverage is paid in case of a claim.
 The scheme is implemented through a nodal agency. A nodal agency can be a State
Government, A Department of Central Ministry, Union Territory, or a registered NGO that
has been appointed for implementing the scheme.
 Life Insurance Corporation of India (LIC) is the only insurance company that is offering
the scheme presently.
Eligibility

 Only one member from each family can be covered.


 Coverage is allowed either for the earning family member or the family head as chosen
by the family.
 The age of the beneficiary should be between the age of 18 to 59 years.
 Individuals must belong to a Below Poverty Line (BPL) family or a family above the poverty
line belonging to the vocational group identified under the scheme, or belonging to the
rural community.
 48 vocational groups are identified under this scheme and the members belonging to
those groups will be covered.
Aam Aadmi Bima Yojana: Beneficiary Groups under the Scheme

https://t.me/prelimbits
230

Beedi workers Brick kiln workers Cobblers Carpenters


Fishermen Handloom weavers Hamals Handicrafts artisans
Handloom and Khadi Leather and tannery Lady tailors Physically handicapped
weavers workers people who are self-
employed
Papad workers who Primary milk Auto drivers or Slat growers
are associated with producers rickshaw pullers
SEWA
Safai karamcharis Tendu leaf collectors Forest workers Scheme for the urban poor
Sericulture Forest worker Powerloom Toddy tappers
workers
Woman of hilly areas Textile Manufacture of Manufacture of leather
wood products products
Food stuffs like sugar Manufacture of Rubber and coal Printing
or khandsari paper products products
Mineral products like Chemical products Transport drivers Agriculturists
earthen toys like candle association
manufacture manufacture
Rural poor Transport Firecrackers Construction worker
karamcharis worker
Anganwadi teachers Coconut processors Plantation Kotwal
workers
Sheep breeders Women associated Overseas Indian Unorganized workers
with self-help groups workers covered under Rashtriya
Swasthya Bima Yojana
Implementation

 The Aam Admi Bima Yojana covers the following →


o Natural death
o Accidental death
o Accidental permanent or partial disability wherein the insured loses one limb or sight
of one eye.
o Accidental permanent total disability wherein the insured loses both limbs or sight or
both eyes or one limb and sight of one eye.
 Besides these inbuilt coverage benefits, a free additional coverage benefit is also provided
which is called Scholarship Benefit and is available for up to two children of the
beneficiary of the scheme. The children should be studying in Classes 9th to 12th.
 The coverage amount for each coverage benefit is as follows-
Insured suffered Sum insured payable
Natural death INR 30,000
Accidental death INR 75,000
Accidental permanent total disability INR 75,000
Accidental permanent partial disability INR 37,500
Scholarship benefit INR 100 per month per child
 However, some of the elements have been excluded under this scheme. These include

https://t.me/prelimbits
231

death or disability due to the following:-


o Self-inflicted injuries or attempted suicide
o Insanity or mental disorders
o Pregnancy or childbirth
o War or related perils
o Chemical, radioactive or biological weapons
o Participating in dangerous and adventurous sports
o Criminal acts or activities which breach the law
Moreover, medical expenses which are incurred due to an accident are also not
covered
 Premium : Rs.200 per year. (out of that Rs. 100 by Union Government. And remaining
Rs.100 by either State Government / NGO).
 Administrative control of this AABY scheme : Previous with DFS, now with Labour
Ministry (2017)

179. Microfinance is the provision of financial services to people of low-income groups.


This includes both the consumers and the self-employed. The service/services
rendered under microfinance is/are: [2011 - I]
1. Credit facilities
2. Savings facilities
3. Insurance facilities
4. Fund Transfer facilities
Select the correct answer using the codes given below the lists ?

A. 1 only
B. 1 and 4 only
C. 2 and 3 only
D. 1, 2, 3 and 4
Ans. D

Microfinance is a movement whose object is "a world in which as many poor and near-poor
households as possible have permanent access to an appropriate range of high quality financial
services, including not just credit but also savings, insurance, and fund transfers."

MICROFINANCE INSTITUTIONS
 MFI is an organization that offers financial services to low income populations.
o These services include microloans, microsavings and microinsurance.
 MFIs are financial companies that provide small loans to people who do not have any
access to banking facilities.
 The definition of ―small loans‖ varies between countries. In India, all loans that are below
Rs.1 lakh can be considered as microloans.
 In most cases the so-called interest rates are lower than those charged by normal

https://t.me/prelimbits
232

banks, certain rivals of this concept accuse microfinance entities of creating gain by
manipulating the poor people‘s money.
 Microfinance sector has grown rapidly over the past few decades and currently it is
serving around 102 million accounts (including banks and small finance banks) of the
poor population of India.
 Different types of financial services providers for poor people have emerged - non-
government organizations (NGOs); cooperatives; community-based development
institutions like self-help groups and credit unions; commercial and state banks; insurance
and credit card companies; telecommunications and wire services; post offices; and other
points of sale - offering new possibilities.
 Non Banking Finance Company (NBFC)-MFIs in India are regulated by The Non-Banking
Financial Company -Micro Finance Institutions (Reserve Bank) Directions, 2011 of
the Reserve Bank of India (RBI).
Major Business Models

 Joint Liability Group:


o This is usually an informal group that consists of 4-10 individuals who seek loans
against mutual guarantee.
o The loans are usually taken for agricultural purposes or associated activities.
 Self Help Group:
o It is a group of individuals with similar socio-economic backgrounds.
o These small entrepreneurs come together for a short duration and create a common
fund for their business needs. These groups are classified as non-profit organisations.
 The National Bank for Agriculture and Rural Development
(NABARD) SHG linkage programme is noteworthy in this regard, as several
Self Help Groups are able to borrow money from banks if they are able to
present a track record of diligent repayments.
 Grameen Model Bank:
o It was the brainchild of Nobel Laureate Prof. Muhammad Yunus in Bangladesh in
the 1970s.
o It has inspired the creation of Regional Rural Banks (RRBs) in India. The primary
motive of this system is the end-to-end development of the rural economy.
 Rural Cooperatives:
o They were established in India at the time of Indian independence.
o However, this system had complex monitoring structures and was beneficial only
to the creditworthy borrowers in rural India. Hence, this system did not find the
success that it sought initially.
Benefits →

 They provide easy credit and offer small loans to customers, without any collateral.
 It makes more money available to the poor sections of the economy, leading
to increased income and employment of poor households.

https://t.me/prelimbits
233

 Serving the under-financed section such as women, unemployed people and those
with disabilities.
 It helps the poor and marginalised section of the society by making them aware of the
financial instruments available for their help and also helps in developing a culture of
saving.
 Families benefiting from microloans are more likely to provide better and continued
education for their children.
180. Among the following who are eligible to benefit from the “Mahatma Gandhi National
Rural Employment Guarantee Act”? [2011 - I]
A. Adult members of only the scheduled caste and scheduled tribe households
B. Adult members of below poverty line (BPL) households
C. Adult members of households of all backward communities
D. Adult members of any household
Ans. D

MGNREGA SCHEME
 The Mahatma Gandhi National Rural Employment Guarantee Act, earlier known as the
National Rural Employment Guarantee Act was passed in 2005 to augment employment
generation and social security in India.
 The scheme is a DEMAND-DRIVEN wage employment scheme, which functions under
the Ministry of Rural Development.
 EVERY ADULT MEMBER of a household in a rural area with a job card is eligible for a job
under the scheme.
 The scheme envisages providing 100 days of guaranteed wage employment in a
financial year to adult member volunteers for UNSKILLED MANUAL WORK.
 It covers all districts of India except the ones with 100% urban population.
 There is also a provision for ADDITIONAL 50 DAYS of unskilled wage employment in
drought/natural calamity notified rural areas.
 As per Section 3(4) of the MGNREGA, the States may make provisions for providing
additional days beyond the period guaranteed under the Act from their own funds.
181. With reference to the Finance Commission of India, which of the following
statements is correct ? [2011 - I]
A. It encourages the inflow of foreign capital for infrastructure development
B. It facilitates the proper distribution of finances among the Public Sector Undertakings
C. It ensures transparency in financial administration
D. None of the statements (a), (b) and (c) given above is correct in his context
Ans. D

FINANCE COMMISSION:
 Finance Commission is a CONSTITUTIONAL BODY.

https://t.me/prelimbits
234

 Finance Commission was constituted to recommend the distribution of the net


proceeds of taxes between the Centre and states, and among the states.
 It was established under Article 280 of the Indian Constitution.
 The Commission is constituted for every five years [or earlier] by the President.
 The Commission consists of five members including the chairperson.
 The Chairman of the Commission shall be selected from among persons who have had
experience in public affairs, and the four other members shall be selected from among
persons who →
o are, or have been, or are qualified to be appointed as Judges of a High Court; or
o have special knowledge of the finances and accounts of Government, or
o have had wide experience in financial matters and in administration, or
o have special knowledge of economics.
Functions of the Finance Commission of India

 The net tax proceeds distribution to be divided between the Centre and the states, and
the allocation of the same between states. (not among the Public Sector Undertaking).
 The principles governing the grants-in-aid to the states by the Centre out of the
consolidated fund of India.
 The steps required to extend the consolidated fund of a state to boost the resources of
the panchayats and the municipalities of the state on the basis of the recommendations
made by the state Finance Commission.
182. What is the difference between “vote-on-account” and interim budget” ? [2011 - I]
1. The provision of a ―vote-on-account‘‘ is used by a regular Government, while an ―interim
budget‘‘ is a provision used by a caretaker Government
2. A ―vote-on-account‘‘ only deals with the expenditure in Government budget, while an
―interim budget‘‘ includes both expenditure and receipts
Which of the statements given above is/are correct?

A. 1 only
B. 2 only
C. Both 1 and 2
D. Neither 1 nor 2
Ans. B

DIFFERENCE BETWEEN INTERIM BUDGET AND VOTE ON ACCOUNT


Interim Budget Vote on Account
During the time of approaching elections, it is Vote-on-Account is a provision by which the
not practical to present a full budget, therefore the government seeks Parliament‘s approval for
government presents an interim budget. funds that are sufficient to bear the
expenditure till the formation of a new
government takes place
Interim Budget consists of both expenditure and A vote-on-account lists only the
receipts expenditure beared by the government

https://t.me/prelimbits
235

It has to be discussed in the Lok Sabha and then A Vote-On-Account is treated as a formal
passed. matter so it can be passed by the Lok
Sabha without discussion
The Government of India has the power to make Vote on Account cannot change the Direct
changes in the tax regime in the interim budget Taxes at any cost. Any alteration in direct
also. taxes can only be brought about by passing
of the Finance Bill.
It is like a budget for the transition period ( when The vote-on-account can be passed through
there are a few months of the Government the interim budget.
remaining in power)
An interim budget is valid for an entire year. A vote on account is usually valid for 2
months.
NO TO ‗VOTE ON ACCOUNT‘
 Vote on account had become an integral part of our budgetary exercise. The concept entails
a special provision given to the Indian Government to obtain the vote of parliament to
withdraw money when the budget for the new financial year is not released or
elections are underway, and the caretaker government is in place.
 In simple terms, Article 266 of the Constitution makes it mandatory for the Indian
Government to seek approval from the parliament before raising any funds from the
consolidated funds of India.
 The vote on account represents the expenditure side of the government‘s budget; i.e.
government gives the estimate of the funds required during the first two months of
the financial year. It stays valid for two months which can be extended if the year is an
election year and it is anticipated that the main demand and the appropriation bill will take
longer to be passed by the House.
 From the above definition that vote on account should be applicable only in an
election year i.e. once in five years or when there is a caretaker government in place.
 However, the practice of vote on account became a regular feature in the
normal years too. Additionally, the period of vote on account sometimes extended even
beyond two months.
New System from 2017

 As proposed by the new Government, the budget for 2017-18 will be presented in the
Parliament on February 1, 2017.
 It will be given 30-40 days scrutiny by the Department-related Parliamentary Standing
Committees which examine the individual demands for grants thread-bare and give their
reports to the Parliament.
 Earlier this process used to spill over to April end and sometimes early May and hence
the vote on account was needed.
 Once the entire budget exercise gets over by March 31, there will be no
need for a vote on account and the government can straightway start

https://t.me/prelimbits
236

implementing projects from April 1.


 This will also not leave any excuse to the bureaucracy for delaying the project. On an
average the efficiency of the government was 75 per cent and an equal proportion of
revenue was collected. Now the time available for implementation of all the projects will be
the full 12 months and the revenue collection will also be 100 per cent
183. Why is the offering of “teaser loans‟‟ by commercial banks a cause of economic
concern ? [2011 - I]
1. The teaser loans are considered to be an aspect of sub-prime lending and banks may be
exposed to the risk of defaulters in future.
2. In India, the teaser loans are mostly given to inexperienced entrepreneurs to set up
manufacturing or export units.
Which of the statements given above is/are correct?

A. 1 only
B. 2 only
C. Both 1 and 2
D. Neither 1 nor 2
Ans. A

TEASER LOANS
 Teaser loans are loans that offer a lower rate of interest in the first few years after which
the rates are increased.
 SBI pioneered the teaser rate concept in home loans in November 2009.
 The teaser loans are considered to be an aspect of sub-prime lending. Hence, statement
1 is correct.
 Subprime lending is the practice of lending to borrowers with a low credit
rating that may be exposed to the risk of defaulters in future.
 It has nothing to do with the experience of the entrepreneurs as the teaser loans are
given to entrepreneurs as well as home loans. Hence, statement 2 is incorrect.
Prime Borrower He has the capacity to repay loans.
Subprime Such person doesn’t have the capacity to repay loan. Giving teaser rate home loans
Borrower to sub-prime borrowers was among the reasons for Sub-prime Crisis in USA (2007-08),
(Individual) which ultimately led to Global Financial Crisis (GFC).
Overleveraged Such company has borrowed too much money than its ability to pay it back. An
Borrower Overleveraged company has high ratio of Debt (Bonds/loans) to Equity (Shares).
(Company)
Zombie Lending When a weak bank keeps giving new loans to a subprime / overleveraged borrower

184. Which of the following can aid in furthering the Government‟s objective of inclusive
growth ? [2011 - I]
1. Promoting Self-Help Groups.
2. Promoting Micro Small and Medium Enterprises.
3. Implementing the Right to Education Act.

https://t.me/prelimbits
237

Select the correct answer using the codes given below:

A. 1 only
B. 1 and 2 only
C. 2 and 3 only
D. 1,2 and 3 only
Ans. D

INCLUSIVE GROWTH
 As per OECD (Organisation for Economic Co-operation and Development), inclusive growth
is economic growth that is distributed fairly across society and creates
opportunities for all.
 Good quality infrastructure is the most critical physical requirement for attaining faster
growth in a competitive world and also for ensuring investment in backward regions.
 Rural electrification is an important instrument too to bring about inclusive growth by
making electricity available to farmers and in rural areas.
 Presently a large number of Self Help Groups (SHGs) are working in India at the grass-
roots level especially in rural India on the principles of mutual help, solidarity and joint
responsibility. The more the Self Help Groups, the better the financial inclusion. Hence
statement 1 is correct.
 The Government of India has introduced MSME or Micro, Small, and Medium Enterprises in
agreement with the Micro, Small and Medium Enterprises Development (MSMED) Act of
2006.
 MSMEs are an important sector for the Indian economy and have contributed
immensely to the country‘s socio-economic development. It not only generates
employment opportunities but also works hand-in-hand towards the development of the
nation‘s backward and rural areas. It can help in financial inclusion. Hence statement 2 is
correct.
 The RTE Act aims to provide primary education to all children aged 6 to 14 years. The
act mandates 25% reservation for disadvantaged sections of the society where
disadvantaged groups include: SCs and STs. Education can help uplift the backward and
downtrodden society. Hence statement 3 is correct.
185. Why is the Government of India disinvesting its equity in the Central Public Sector
Enterprises (CPSEs) ? [2011 - I]
1. The Government intends to use the revenue earned from the disinvestment mainly to pay
back the external debt.
2. The Government no longer intends to retain the management control of the CPSEs
Which the correct statements given above is/are correct ?

A. 1 only
B. 2 only
C. Both 1 and 2

https://t.me/prelimbits
238

D. Neither 1 nor 2
Ans. D

When we say ―Mainly‖ the money is to be used to pay the external debt, it is not correct to say so.
Government though ―may intend‖ to retain the management control, but there are fiscal
constraints, that induce the government to go for disinvestment. Both statements are NOT correct.

DISINVESTMENT
 Disinvestment means sale or liquidation of assets by the government, usually Central
and state public sector enterprises, projects, or other fixed assets.
 The government undertakes disinvestment to reduce the fiscal burden on the
exchequer, or to raise money for meeting specific needs, such as to bridge the revenue
shortfall from other regular sources.
o Money goes to National Investment Fund.
 STRATEGIC DISINVESTMENT is the transfer of the ownership and control of a public
sector entity to some other entity (mostly to a private sector entity).
o Unlike the simple disinvestment, strategic sale implies a kind of privatization.
 Nodal: Department of Investment and Public Asset Management (DIPAM) under the
Ministry of Finance.
 Strategic disinvestment in India has been guided by the basic economic principle that
the government should not be in the business to engage itself in
manufacturing/producing goods and services in sectors where competitive markets
have come of age.
o The economic potential of such entities may be better discovered in the hands of the
strategic investors due to various factors, e.g. infusion of capital, technology up-
gradation and efficient management practices etc.
 The Government does not necessarily intend to use the revenue earned from the
disinvestment mainly to pay back the external debt.
 Also From time to time, the Government takes the decision to disinvest the CPSEs but
largely it decides to retain the management control on case to case basis.
FY 24 Disinvestment Target

 The government has set a disinvestment target at ₹51,000 crore for FY24, lower than the
previous financial year, according to the Union Budget for 2023-24
 The estimates for FY23 has also been revised to ₹50,000 crore from the earlier target
of ₹65,000 crore.
 Since it had missed targets set for prior financial years, the most recent being ₹1.75 trillion
target for FY22 which was revised downwards to ₹78,000 crore in the revised estimates in
the Union Budget last year. The actual proceeds for FY22 were at a dismal ₹13,627 crore,
primarily owing to the disruptions caused by covid-19 pandemic.
 The government outlined plans to sell its shares in IDBI Bank, Shipping Corporation of India,
BEML and Container Corporation of India. It aims to invite financial bids for the stake sale in
IDBI Bank, which will spill over to FY24.

https://t.me/prelimbits
239

186. Economic growth is usually coupled with [2011 - I]


A. Deflation
B. Inflation
C. Stagflation
D. Hyperinflation
Ans. B

DEFLATION
 It occurs when asset and consumer prices fall over time.
 It is a negative inflation rate.
 It is often used to express a declining economy.
 Real interest rates become too high. [Will cause an increase in the real value of debt.]
 It increases the burden of debt and reduces the disposable income of indebted people.
INFLATION
 It refers to the rise in the prices of most goods and services of daily or common use, such
as food, clothing, housing etc.
 A certain level of inflation is required in the economy to ensure that
expenditure is promoted and hoarding money through savings is demotivated.
 As the spending increases, the demand also increases which leads to inflation.
 Hence, Economic growth is usually coupled with inflation.
STAGFLATION
 It is an economic event in which the INFLATION RATE IS HIGH, the ECONOMIC GROWTH
RATE SLOWS, and UNEMPLOYMENT REMAINS STEADILY HIGH.
 It can also be alternatively defined as a period of inflation combined with a decline in the
gross domestic product (GDP).
HYPERINFLATION

 It is a term to describe rapid, excessive, and out-of-control general price


increases in an economy.
 Hyperinflation refers to rapid and unrestrained price increases in an economy, typically at
rates exceeding 50% each month over time.
 It has two main causes: an increase in the money supply and demand-pull inflation.
o Excessive Money Supply
 Central banks generally control the circulating supply of money. In
circumstances that historically warrant an increase in the money supply—like
a recession or depression—central banks can increase the amount of money
circulating. The intent behind this action is to encourage banks to lend and
consumers and businesses to borrow and spend.
 However, if the increase in money supply is not supported by economic
growth—as measured by gross domestic product (GDP)—hyperinflation can

https://t.me/prelimbits
240

result. If GDP—the measure of an economy's production—isn't growing,


businesses raise prices to boost profits and stay afloat.
 Because consumers have more money, they pay higher prices and feed
inflation. If economic output continues to stagnate or shrink and inflation
keeps rising, companies charge more, consumers pay more, and the central
bank prints more money. A cycle of increasing inflation rates occurs, leading
to hyperinflation.
o Demand-Pull Inflation
 Demand-pull inflation is a scenario in which aggregate demand becomes too
high for aggregate supply. This increases prices rapidly because there are not
enough goods and services available to meet the increase in overall demand
from consumers and businesses.
 It can occur in times of war and economic turmoil in the underlying production economy,
in conjunction with a central bank printing an excessive amount of money.
187. The lowering of Bank Rate by the Reserve Bank of India leads to [2011 - I]
A. more liquidity in the market
B. less liquidity in the market
C. no change in the liquidity in the market
D. mobilization of more deposits by commercial banks
Ans. A

BANK RATE
 Introduced under RBI act 34
 Bank rate is a rate at which the Reserve Bank of India (RBI) provides the loan to
commercial banks without keeping any security. [For Longer Period]
 There is no agreement on repurchase that will be drawn up or agreed upon with no
collateral as well.
 The RBI allows short-term loans with the presence of collateral. This is known as Repo Rate.
 Bank Rates in India is determined by the RBI. It is usually higher than a Repo Rate on
account of its ability to regulate liquidity.
 The lowering of the Bank Rate makes the domestic banks borrow money at a
cheap rate which in turn increased the liquidity in the market.
188. With reference to micro-irrigation, which of the following statements is/are correct
? [2011 - I]
1. Fertilizer/nutrient loss can be reduced.
2. It is the only means of irrigation in dryland farming.
3. In some areas of farming, receding of ground water table can be checked.
Select the correct answer using the codes given below:

A. 1 only
B. 2 and 3 only

https://t.me/prelimbits
241

C. 1 and 3 only
D. 1, 2 and 3
Ans. C

MICRO-IRRIGATION

 In the micro-irrigation technique water is directly applied to the root zone, through
drippers, sprinklers, foggers, and other emitters.
 Micro-irrigation is a scientific method of irrigation carrying desired water and nutrients
directly to the root zone of the plant, drop by drop. The system has an extensive
network of pipes operated at low pressure.
 Dryland farming is defined as the production of crops without irrigation in regions that
receive rainfall of less than 500 mm annually. Hence statement 2 is incorrect.
 Micro-irrigation benefits →
o Increase in crop yield.
o Reduces the loss of water.
o Fertilizer saving.
o Nutrient leaching can be reduced. Hence, statement 1 is correct.
o Reduces pest problems
o Reduces soil erosion
o Help in reducing ground water depletion. Hence, statement 3 is correct.

189. Which of the following are the methods of Parliamentary control over public
finance in India? [2012 - I]
1. Placing Annual Financial Statement before the Parliament
2. Withdrawal of moneys from Consolidated Fund of India only after passing the
Appropriation Bill
3. Provisions of supplementary grants and vote-on-account
4. A periodic or at least a mid-year review of programme of the Government against
macroeconomic forecasts and expenditure by a Parliamentary Budget Office
5. Introducing Finance Bill in the Parliament

Select the correct answer using the codes given below:

A. 1, 2, 3 and 5 only

https://t.me/prelimbits
242

B. 1, 2 and 4 only
C. 3, 4 and 5 only
D. 1, 2, 3, 4 and 5

Ans. A

METHODS OF PARLIAMENTARY CONTROL OVER PUBLIC FINANCE IN INDIA


 Article 112 of the Indian Constitution deals with placing annual financial statements in
parliament. Hence statement 1 is correct.
 An Appropriation Bill is a money bill that allows the government to withdraw funds from
the Consolidated Fund of India to meet its expenses during the course of a financial year.
 As per Article 114 of the Constitution, the government can withdraw money from it only
after receiving approval from Parliament. Hence statement 2 is correct.
 Votes on account, votes of credit and exceptional grants are specified in Article 116 of the
constitution. Hence statement 3 is correct.
 There is no parliamentary budget office to review programs of Government.
 Though there is an Estimate Committee of parliament to review policies and continuous
examination of estimates from time to time throughout the year. Hence statement 4 is
incorrect.
 The Finance Bill is a part of the Union Budget, stipulating all the legal amendments required
for the changes in taxation proposed by the Finance Minister. Hence statement 5 is
correct.

190. The Reserve Bank of India (RBI) acts as a bankers‟ bank. This would imply which of
the following? [2012 - I]
1. Other banks retain their deposits with the RBI.
2. The RBI lends funds to the commercial banks in times of need.
3. The RBI advises the commercial banks on monetary matters.

Select the correct answer using the codes given below:

A. 2 and 3 only
B. 1 and 2 only
C. 1 and 3 only
D. 1, 2 and 3

Ans. D

RBI FUNCTIONS
 The ten vital functions of RBI are enumerated as follows-
o Regulating the issue of currency notes and coins.
o Keeping a watch on the financial markets.
o Banker to the Central and State Government.
o Managing the foreign exchange.

https://t.me/prelimbits
243

o Looking after the payment and settlement system.


o Debt manager to the Government.
o Regulating different kinds of banks.
o Focusing on financial inclusion and development.
o Banker to the Banks.
o Ensuring consumer education and protection
 Out of all the functions of RBI, one of the most important ones is that of a banker‘s bank.
This can be understood by followings given reasons →
o Management of rates →
 There are different rates that RBI administers for all the banks. The RBI decides
the rates, and that is mandatory for every bank to follow. The rates can be the
policy rates which include bank rate, the repo rate, the reverse repo rate, and
marginal standing facility rate. The percentage of reserves that every bank has
to maintain with the RBI is also decided by the RBI. It includes the cash reserve
ratio (CRR) and the statutory liquidity ratio (SLR). RBI also decides the foreign
exchange rates. Also, RBI is responsible for setting the benchmark rate such as
MIBOR, commonly known as Mumbai Interbank Offer Rate. The lending rates
and deposit rates are also pre-decided by RBI. No bank can charge any rate
lower than the rates fixed by the RBI. It also facilitates the rate in the money
market and call market.
o Maintaining Reserves
 The cash reserve ratio is the amount of cash every bank has to keep as a
reserve with the RBI. It is calculated as a percentage of NET demand and time
liability. The NDTL is arrived at by adding up the savings account, current
account, and fixed deposit balances held by the particular bank. RBI maintains
this reserve to ensure liquidity is maintained in banks and the country‘s
economy
 Similarly, the Statutory Liquidity ratio is the reserve that is to be maintained by
all the banks in addition to the CRR. It means that a specific percentage of
reserves need to be held in cash, gold, bonds, or securities approved by RBI.
The bank itself maintains these reserves for liquidity purposes. RBI revises the
rates of CRR and SLR.
o Account for inter-bank obligations
 It is imperative for every bank to maintain an account with the RBI. All the
dealings that happen between two banks are done through this account. RBI
keeps a watch over the dealings. Many times it happens that any bank may
need a loan, so it takes it from some other bank. The rate at which one bank
can lend to the other bank is also decided by RBI. And the transfer of funds
takes place from the account for inter-bank obligations. Any discrepancy if
found by the RBI, gives them the right to stop the transaction from
happening.

https://t.me/prelimbits
244

o Monitoring operations and defaults


 RBI lays down various policies for the working of the bank and revises them as
and when necessary. It also releases a set of operational instructions. RBI
records all the information related to credit. Rules and regulations for the
functioning of a bank are also clearly defined. If RBI feels that any transaction
is unethical, doubtful or any suspicious activity is undertaken, they can
interrogate the bank.
o Facilitate cash pooling
 Cash pooling refers to the process in which all the transactions move to the
parent branch or company. The cash is transferred based on surplus or deficit
from the main branch to other branches. Suppose, a bank named ABC has two
other branches, A and B. Branch A is facing a deficit of cash, and B has a
surplus. So, ABC can transfer funds from B to A; RBI facilitates this process.
o Fund management
 RBI helps the bank to manage its funds. When the need arises, RBI provides
short term loans and advances to the banks against any collateral. It has a
mechanism for managing the funds of banks known as the Centralised Fund
Management System. Banks can collect information about their balances,
enquire, or transfer funds from the same.
o Risk management
 RBI must manage the various risks faced by banks such as liquidity risk,
interest rate risk, credit risk, market risk, and operational risk. RBI has specified
the norms concerning three major areas under the BASEL accord. To manage
risk in a bank, it focuses on three areas: a minimum capital requirement,
supervisory review processes, and the market discipline requirements.
o Lender of last resort
 When nobody extends credit to a particular bank then RBI acts as a lender of
last resort. This means that RBI will provide loans to the bank at the prevailing
rates. Also, RBI will take adequate steps to manage the liquidity of the bank
and make sure they don‘t prove to be insolvent. This helps in protecting the
interest of the consumers and saves the bank from failure or collapse.
o Management of NPA‘s
 RBI issues circulars and guidelines for the management of the Non-
Performing assets of banks. A system is established where the NPAs should be
identified on an on-going basis and classified accordingly. Any defaults need
to be recognised within 30 days by the bank. It may take strict action if the
NPAs are large in number and investigate the same.
o Asset Liability management
 RBI has to manage the risk and liquidity of the bank, or there is a high
possibility that the bank may collapse. So various tools are undertaken by RBI
to manage the liquidity such as the creation of the ALCO committee. By

https://t.me/prelimbits
245

managing the assets and liabilities of the bank, liquidity can be ensured, and
risk can be mitigated. Various tools for managing the asset-liability mismatch
are used like maturity gap analysis, duration gap analysis, etc.

191. How do District Rural Development Agencies (DRDAs) help in the reduction of rural
poverty in India? [2012 - I]
1. DRDAs act as Panchayati Raj Institutions in certain specified backward regions of the
country.
2. DRDAs undertake area-specific scientific study of the causes of poverty and malnutrition
and prepare detailed remedial measures.
3. DRDAs secure inter-sectoral and inter-departmental coordination and cooperation for
effective implementation of anti-poverty programmes.
4. DRDAs watch over and ensure effective utilization of the funds intended for anti-poverty
programmes.

Which of the statements given above is/are correct?

A. 1, 2 and 3 only
B. 3 and 4 only
C. 4 only
D. 1, 2, 3 and 4

Ans. B

DRDA
 DRDA has traditionally been the principal organ at the district level to oversee the
implementation of anti-poverty programmes of the Ministry of Rural Development.
 From April 1999 a separate DRDA Administration has been introduced to take care of the
administrative costs.
 Expenses of DRDAs are funded on a 75:25 basis between Centre and State Governments.
 However, from 2008-09 the funding pattern for N.E. States has been changed from 75: 25
to 90: 10. In the case of UTs, the Centre provides entire (100%) funds under the Scheme.

Duties

 DRDA is a supporting and facilitating organisation and needs to play a very effective
role as a catalyst in the development process.
o They are expected to coordinate with the line departments, the Panchayati Raj
Institutions, the banks and other financial institutions, the NGOs as well as the
technical institutions.
o The DRDAs are expected to coordinate effectively with the Panchayati Raj
institutions. Under no circumstances will they perform the functions of PRIs. Hence
statement 1 is not correct.
 Thus the role of the DRDA is in terms of planning for effective implementation of
anti-poverty programmes; coordinating with other agencies- Governmental, non-

https://t.me/prelimbits
246

Governmental, technical and financial for successful programme implementation; enabling


the community and the rural poor to participate in the decision making process, overseeing
the implementation to ensure adherence to guidelines, quality, equity and efficiently;
reporting to the prescribed authorities on the implementation; and promoting transparency
in decision making and implementation. Hence statement 3 is correct.
 DRDA will continue to watch over and ensure effective utilisation of the funds intended for
anti-poverty programmes, it will need to develop a far greater understanding of the
processes necessary for poverty alleviation/ eradication. Hence statement 4 is correct.
 The DRDAs shall take the necessary step to improve the awareness regarding rural
development and poverty alleviation particularly among the rural poor.
 The DRDAs will strive to promote transparency in the implementation of different anti-
poverty programmes. Towards this end, they shall publish periodically, the details of the
different programmes and their implementation.
 No such provisions are there for DRDAs to undertake an area-specific scientific study of the
causes of poverty and malnutrition and prepare detailed remedial measures. Hence
statement 2 is not correct.

192. The Multi-dimensional Poverty Index developed by Oxford Poverty and Human
Development Initiative with UNDP support covers which of the following? [2012 - I]
1. Deprivation of education, health, assets and services at household level
2. Purchasing power parity at national level
3. Extent of budget deficit and GDP growth rate at national level

Select the correct answer using the codes given below:

A. 1 only
B. 2 and 3 only
C. 1 and 3 only
D. 1, 2 and 3

Ans. A

For more refer Q. 40

https://t.me/prelimbits
247

193. Which of the following is /are among the noticeable features of the
recommendations of the Thirteenth Finance Commission? [2012 - I]
1. A design for the Goods and Services Tax, and a compensation package linked to adherence
to the proposed design
2. A design for the creation of lakhs of jobs in the next ten years in consonance with India‘s
demographic dividend
3. Devolution of a specified share of central taxes to local bodies as grants

Select the correct answer using the codes given below:

A. 1 only
B. 2 and 3 only
C. 1 and 3 only
D. 1, 2 and 3

Ans. C

RECOMMENDATIONS OF THIRTEENTH FINANCIAL COMMISSION


 It recommended that both Center and the states should conclude a 'Grand Bargain' to
implement model GST.
 To incentivize implementation of the Grand Bargain, this Commission recommends sanction
of a grant of Rs. 50,000 crore.
 A design for the Goods and Services Tax, and a compensation package linked to
adherence to the proposed design.
 Devolution of a specified share of central taxes to local bodies as grants.
 Fiscal Responsibility and Budget Management Act, 2003 need to be amended to mention
the nature of shocks which shall require targets relaxation.
 Fiscal deficit to be reduced to 3% of the gross domestic product (GDP) by 2014–2015.

https://t.me/prelimbits
248

 No such mentions for a design for the creation of lakhs of jobs in the next ten years in
consonance with India's demographic dividend. Hence statement 2 is not correct.

Thirteenth Financial Commission-

 Constituted on 13th November 2007 under the chairmanship of Vijay L Kelkar.


 Members of the Commission were:
o Dr. Vijay L. Kelkar, Chairman
o B.K. Chaturvedi, Part-time
o Dr. Indira Rajaraman
o Prof. Atul Sarma
o Dr. Sanjiv Misra
o Shri Sumit Bose, Secretary
 The 1st Finance Commission was constituted on 22nd November 1951 under the
chairmanship of K.C. Neogi.

194. Consider the following specific stages of demographic transition associated with
economic development: [2012 - I]
1. Low birthrate with low death rate
2. High birthrate with high death rate
3. High birthrate with low death rate

Select the correct order of the above stages using the codes given below:

A. 1, 2, 3
B. 2, 1, 3
C. 2, 3, 1
D. 3, 2, 1

Ans. C

THEORY OF DEMOGRAPHIC TRANSITION-


 It suggests that population growth is linked to overall levels of economic development and
that every society follows a typical pattern of development-related population growth.
 There are three basic phases of population growth.
o The First Stage is that of low population growth in a society that is underdeveloped
and technologically backward. Growth rates are low because both the death rate and
the birth rate are very high so that the difference between the two (or the net growth
rate) is low.
o Stage 2 of the Demographic Transition Model (DTM) is characterized by a rapid
decrease in a country's death rate while the birth rate remains high.
o The Third (And Last) Stage is also one of low growth in a developed society where
both the death rate and birth rate have been reduced considerably and the
difference between them is again small.

Population Explosion happens because death rates are brought down relatively quickly through
https://t.me/prelimbits
249

advanced methods of disease control, public health, and better nutrition.

195. Under which of the following circumstances may „capital gains‟ arise? [2012 - I]
1. When there is an increase in the sales of a product
2. When there is a natural increase in the value of the property owned
3. When you purchase a painting and there is a growth in its value due to increase in its
popularity

Select the correct answer using the codes given below:

A. 1 only
B. 2 and 3 only
C. 2 only
D. 1, 2 and 3

Ans. B

CAPITAL GAINS TAX

 Any profit or gain realised during the year as a result of the transfer of a
capital asset is taxed under the heading "Capital Gains."
 Divided into →
o long-term capital gains tax (LTCG)
o short-term capital gains tax (STCG)
 According to the IT Act, if a person inherits property and does not sell it, no capital
gains tax is required.
 However, if →
o the person who inherited the property decides to sell it, he or she will have to pay
tax on the earnings.
 Capital assets include jewellery, machinery, paintings, Bonds, Stocks trademarks, patents,
autos, real estate, buildings, and land.
 For instance →
o A trader invests Rs. 1 lakh in equity and after 5 years when he sells it for Rs. 11 lakhs.
o The capital gained here is Rs. 10 lakhs.
o Since the equity was held for 5 years it is considered as Long term capital gain and
is taxed at 10% of the gain.
o Therefore Rs. 1 lakh is paid as capital gains tax.
Issues with CGT

 The rate changes from asset to asset. LTCG tax on stocks and equity mutual funds is 10%
but on debt mutual funds is 20% with indexation.
 Holding period changes from asset to asset. The holding period for LTCG tax is two years
in real estate, one year for stocks, and three years for debt mutual funds and gold.
 Exemptions available against it come with their own complex conditions.
o For instance, buying a house after selling one can get you an exemption, but the new

https://t.me/prelimbits
250

house must be bought in two years or built in three years of the sale.

196. Which of the following measures would result in an increase in the money supply in
the economy? [2012 - I]
1. Purchase of government securities from the public by the Central Bank
2. Deposit of currency in commercial banks by the public
3. Borrowing by the government from the Central Bank
4. Sale of government securities to the public by the Central Bank

Select the correct answer using the codes given below:

A. 1 only
B. 2 and 4 only
C. 1 and 3
D. 2, 3 and 4

Ans. C

WHEN MONEY SUPPLY INCREASE & DECREASE


 When the RBI wants to increase the money supply in the economy, it purchases
government securities from the market which led to the flow of more money in the
market. Hence statement 1 is correct.
 Deposit of currency in commercial banks by the public does not increase instead
decreases the supply of money in the market. Hence statement 2 is not correct.
 Borrowing by the government from the Central Bank led to the release of money to the
Central Bank which eventually releases money in the market via loans etc. Hence
statement 3 is correct.
 The sale of government securities to the public by the Central Bank leads to reduce the
money supply in the market. Hence statement 4 is not correct.

197. Which of the following would include Foreign Direct Investment in India? [2012 - I]
1. Subsidiaries of foreign companies in India
2. Majority foreign equity holding in Indian companies
3. Companies exclusively financed by foreign companies
4. Portfolio investment

Select the correct answer using the codes given below:

A. 1, 2, 3 and 4
B. 2 and 4 only
C. 1 and 3 only
D. 1, 2 and 3 only

Ans. D

https://t.me/prelimbits
251

FOREIGN DIRECT INVESTMENT


 A foreign direct investment (FDI) is an investment made by a firm or individual in one
country into business interests located in another country.
 The following include FDIs →
o Subsidiaries of foreign companies in India.
o Majority of foreign equity holding in Indian companies.
o Day to Day operations of the company.
o It is a major source of nondebt financial resources.
o Companies exclusively financed by foreign companies.
 FDI is different from foreign portfolio investment where the foreign entity merely
buys equity shares of a company. Hence statement 4 is not correct.
198. Government of India encourages the cultivation of „sea buckthorn‟. What is the
importance of this plant? [2012- I]
1. It helps in controlling soil erosion and in preventing desertification.
2. It is a rich source of biodiesel.
3. It has nutritional value and is well-adapted to live in cold areas of high altitudes.
4. Its timber is of great commercial value.

Which of the statements given above is/are correct?

A. 1 only
B. 2, 3 and 4 only
C. 1 and 3 only
D. 1, 2, 3 and 4

Ans. C

SEABUCKTHORN
The question is "Why Government of India encourages", not in general.

 It is also known as Chharma in Himachal Pradesh.


 Seabuckthorn is known for its immunity-boosting and medical properties.
 It is found in the cold desert and dry temperate regions of Himachal Pradesh, Ladhak,
Uttrakhand, Sikkim, and Arunachal Pradesh.
 Seabuckthorn can play a crucial role in preventing soil erosion and help nitrogen
fixation in cold and desert areas.
 The fruit and leaves of seabuckthorn are quite rich in a variety of vitamins and
antioxidants i.e. vitamin C, A, E, K, carotenoids, polyphenols, and sterols, etc.
 Seabuckthorn timber is of great commercial value because of its diverse use in medicine,
herbal etc.
 It was studied as a good option/candidate for the manufacture of biodiesel. The major
use of seabuckthorn oil is in medicinal and food use.

https://t.me/prelimbits
252

 The plant has an extensive root system that can fix atmospheric nitrogen, making it
ideal for controlling soil erosion and preventing desertification. Hence statement 1 is
correct.
 The Seabuckthorn berry, also called the ―Wonderberry‖, ―Leh berry‖ and ―Ladakh gold‖,
is among the most nutritious fruits. Hence statement 3 is correct.

199. What is/are the recent policy initiative(s) of Government of India to promote the
growth of manufacturing sector? [2012 - I]
1. Setting up of National Investment and Manufacturing Zones
2. Providing the benefit of ‗single window clearance‘
3. Establishing the Technology Acquisition and Development Fund

Select the correct answer using the codes given below:

A. 1 only
B. 2 and 3 only
C. 1 and 3 only
D. 1, 2 and 3

Ans. D

NATIONAL MANUFACTURING POLICY, 2011


 The National Manufacturing Policy was approved by the Government in October 2011.
 The major objectives of the policy are enhancing the share of manufacturing in the
gross domestic product (GDP) to 25 per cent and creating an additional 100 million
jobs over a decade or so.
 The NMP provides for the promotion of clusters and aggregation, especially through the
creation of National Investment and Manufacturing Zones (NIMZs).
o Out of twelve NIMZs so far announced, eight are along the DMIC. Hence statement
1 is correct.
 Advisory to States on simplification & rationalization of business regulations and skill
development.
 Constitution of the approval/monitoring mechanism under the policy.
 Scheme prepared under NMP on Technology Acquisition and Development Fund
(TADF). Hence statement 3 is correct.
 Initiating the process of simplification and rationalization of state-level business regulations
and developing single-window clearance. Hence statement 2 is correct.

200. In India, in the overall Index of Industrial Production, the Indices of Eight Core
Industries have a combined weight of 37.90%. Which of the following are among those
Eight Core Industries? [2012 - I]
1. Cement
2. Fertilizers
3. Natural gas

https://t.me/prelimbits
253

4. Refinery products
5. Textiles

Select the correct answer using the codes given below:

A. 1 and 5 only
B. 2, 3 and 4 only
C. 1, 2, 3 and 4 only
D. 1, 2, 3, 4 and 5

Ans. C

8 CORE SECTOR

 The eight core sector industries include COAL, CRUDE OIL, NATURAL GAS, REFINERY
PRODUCTS, FERTILISER, STEEL, CEMENT and ELECTRICITY
 The eight core industries comprise nearly 40% of the weight of items included in the
Index of Industrial Production (IIP).
 Weightage (in decreasing): Refinery Products> Electricity> Steel> Coal> Crude Oil>
Natural Gas> Cement> Fertilizers.

201. Despite having large reserves of coal, why does India import millions of tonnes of
coal? [2012 - I]
1. It is the policy of India to save its own coal reserves for future, and import it from other
countries for the present use.
2. Most of the power plants in India are coal-based and they are not able to get sufficient
supplies of coal from within the country.
3. Steel companies need large quantity of coking coal which has to be imported.

Which of the statements given above is/are correct?

A. 1 only
B. 2 and 3 only
C. 1 and 3 only
D. 1, 2 and 3

Ans. B

WHY DOES INDIA IMPORT MILLIONS OF TONNES OF COAL?


 The lack of coking coal reserves that are used as a raw material in steelmaking and allied
industries is the main reason India imports millions of tonnes of coal.
 Coal in India is used either from domestic sources, mostly mined by coal India, or is
imported. The imports are mainly to compensate for the lack of good quality coal,
especially coking coal from the mining sources in the country.
 Coking coal is imported by the state-run Steel Authority of India Limited (SAIL) and other
steel manufacturing units mainly to bridge the gap between the requirement and

https://t.me/prelimbits
254

indigenous availability and to improve the quality. Hence, statement 3 is correct.


 India does not have enough reserves for good quality coking coal and most of it
is imported from Indonesia, South Africa, Russia, and Australia. Hence, statement 2 is
correct.
 Experts say, it is this requirement of coking coal added with power plants whose boilers are
designed to run only on imported coal, which is likely to continue importing coal in the
coming years.
 The coal requirement will not end any time soon.
o Älso, we have power plants at the coasts which are based on imported coal.
o Their boilers are designed only for imported coal. They will continue to use imported
coal.
o So, it is only the third category of thermal power plants which were using imported
coal as a substitute for domestic coal because if its scarcity, which we can do
something about.
 Thus, India imports millions of tonnes of coal due to a lack of availability of good
coal. Hence, statement 1 is not correct.

202. The basic aim of Lead Bank Scheme is that [2012 - I]


A. big banks should try to open offices in each district
B. there should be stiff competition among the various nationalized banks
C. individual banks should adopt particular districts for intensive development
D. all the banks should make intensive efforts to mobilize deposits

Ans. C

LEAD BANK SCHEME


 The Lead Bank Scheme, introduced towards the end of 1969, envisages the assignment of
lead roles to individual banks (both in the public sector and private sector) for the districts
allotted to them.
 A bank has a relatively large network of branches in the rural areas of a given district and
endowed with adequate financial and manpower resources has generally been
entrusted with the lead responsibility for that district.

Objectives of the Lead Bank Scheme:

 Eradication of unemployment and underemployment.


 The appreciable rise in the standard of living for the poorest of the poor.
 Provision of some of the basic needs of the people who belong to poor sections of the
society.

https://t.me/prelimbits
255

Prelims 2013
203. The balance of payments of a country is a systematic record of
A. all import and transactions of a during a given period normally a year
B. goods exported from a country during a year
C. economic transaction between the government of one country to another
D. capital movements from one country to another
Ans. A

BALANCE OF PAYMENTS
 The balance of payments (BOP) of a country is the record of all economic transactions
between the residents of a country and the rest of the world in a particular period (over a
quarter of a year or more commonly over a year).
o Balance of Payment (BoP) of a country can be defined as a systematic statement
of all economic transactions of a country with the rest of the world during a
specific period usually one year.
o It indicates whether the country has a surplus or a deficit in trade.
 When exports exceed imports, there is a trade surplus and when imports
exceed exports there is a trade deficit
204. The Reserve Bank of India regulates the commercial banks in matters of
A. liquidity of assets
B. branch expansion
C. merger of banks
D. winding-up of banks
Select the correct answer using the codes given below.

A. 1 and 4 only
B. 2, 3 and 4 only
C. 1, 2 and 3 only
D. 1, 2, 3 and 4
Ans. D

 RBI is called the banker's bank and regulates the banking sector in India.
 By using mechanisms like CRR, SLR, etc, it keeps a check on the liquidity of assets of the
banks.
 Moreover, RBI also sets rules and regulations concerning the merger of banks, their
winding-up operations, and branch expansion.
205. An increase in the Bank Rate generally indicates that the
A. market rate of interest is likely to fall
B. Central Bank is no longer making loans to commercial banks
C. Central Bank is following an easy money policy
D. Central Bank is following a tight money policy

https://t.me/prelimbits
256

Ans. D

BANK RATE
 Introduced under RBI act 34
 Bank rate is a rate at which the Reserve Bank of India (RBI) provides the loan to
commercial banks without keeping any security. [For Longer Period]
 There is no agreement on repurchase that will be drawn up or agreed upon with no
collateral as well.
 The RBI allows short-term loans with the presence of collateral. This is known as Repo Rate.
 Bank Rates in India is determined by the RBI. It is usually higher than a Repo Rate on
account of its ability to regulate liquidity.
 The lowering of the Bank Rate makes the domestic banks borrow money at a
cheap rate which in turn increased the liquidity in the market == > Easy
Monetary Policy
 An increase in this rate means that the Central bank is following a Tight Monetary
Policy as an increase in rates will lead to a decrease in money supply thereby leading to a
decrease in inflation and a reduction in investment.
206. In India, deficit financing is used for raising resources for
A. economic development
B. redemption of public debt
C. adjusting the balance of payments
D. reducing the foreign debt
Ans. A

DEFICIT FINANCING
 Deficit financing is a necessary evil in a welfare state as the states often fail to generate
tax revenue that is sufficient enough to take care of the expenditure of the state.
 The basic intention behind deficit financing is to provide the necessary impetus to
economic growth by Artificial Means.
 Deficit financing is defined as ―borrowings from the Reserve Bank of
India against the issue of Treasury Bills and running down of accumulated cash
balances‖.
 When the government borrows from the Reserve Bank of India, it merely transfers its
securities to the Bank.
o On the basis of these securities the bank issues more currency and puts
them into circulation on behalf of the government. This amounts to the
creation of money.
The rationale for Deficit Financing →

 Sometimes the government fails to mobilize adequate resources. In this situation, the
option of deficit financing is required to meet fiscal deficit targets.

https://t.me/prelimbits
257

 If the option of deficit financing is not utilized the government ends up compromising
on growth targets.
207. Which of the following constitute Capital Account?
A. Foreign Loans
B. Foreign Direct Investment
C. Private Remittances
D. Portfolio Investment
Select the correct answer using the codes given below.

A. 1, 2 and 3
B. 1, 2 and 4
C. 2, 3 and 4
D. 1, 3 and 4
Ans. B

BALANCE OF PAYMENTS
 It summarizes all transactions that a country‘s individuals, companies, and government
bodies complete with individuals, companies, and government bodies outside the
country.
 These transactions consist of Imports and Exports of goods, services, and capital, as
well as transfer payments, such as foreign aid and remittances.
 The sum of all transactions recorded in the balance of payments must be zero, as
long as the capital account is defined broadly.
 Balance of Payments is made up of 3 components →
o Current Account – Deals with inflow and outflow of goods and services between
countries.
o Capital Account – Deals with foreign exchange reserves, investments, loans &
borrowings.
o Financial Account – Deals with investments in real estates, business ventures,
Foreign Direct Investments (FDI).
RBI‘s (Actual) Method Of Classifying BOP

Current Account Capital & Financial Account


Goods and services  Direct Investment (FDI)
 Primary Income: wages, dividend,  Portfolio Investment (FPI)
interest  Loans / ECB
 Secondary income: remittance, gift,  Non-resident‘s investment in Bank,
donation Insurance, Pension schemes.
 RBI‘s foreign exchange reserve

https://t.me/prelimbits
258

208. Consider the following statements :


1. Inflation benefits the debtors.
2. Inflation benefits the bond-holders.
Which of the statements given above is/are correct?

A. 1 only
B. 2 only
C. Both 1 and 2
D. Neither 1 nor 2
Ans. A

LIKELY OUTCOMES OF INFLATIONS

Effect During Inflation During Deflation


Businessman, They make huge profits because the They make losses because prices of
Borrowers price of final product is rising at a much final products fall faster than the cost
faster speed than the price of raw of production→ lay-off workers to cut
materials. salary bill.
 Salaried individual, pensions
Fixed Income While they will benefit because the
suffer.
Groups, value (=purchasing power) of money
 Lenders suffer because even if
will increase, but some workers /
Lenders borrowed money is returned their
employees will lose their jobs during
‗real Purchasing Power‘ would
deflation as per the Philip Curve.
have declined due to the fall in
Real Interest Rate.
Currency itself Since rupee‘s purchasing power will Reverse will happen.
decline, its exchange rate value will
weaken against foreign currencies, as
foreigners get less keen to buy from

https://t.me/prelimbits
259

India.

209. Disguised unemployment generally means


A. large number of people remain unemployed
B. alternative employment is not available
C. marginal productivity of labour is zero
D. productivity of workers is low
Ans. C

UNEMPLOYMENT
 Unemployment occurs when a person who is actively searching for employment is
unable to find work.
 Unemployment is often used as a measure of the health of the economy.
 The most frequent measure of unemployment is the UNEMPLOYMENT RATE, which is the
number of unemployed people divided by the number of people in the labour force.
 National Sample Survey Organisation (NSSO) defines employment and unemployment
on the following activity statuses of an individual →
o Working (engaged in an economic activity) i.e. 'Employed'.
o Seeking or available for work i.e. 'Unemployed'.
o Neither seeking nor available for work.
o The first two constitute the labour force and unemployment rate is the percent of the
labour force that is without work
o Unemployment rate = (Unemployed Workers / Total labour force) × 100
UNEMPLOYMENT (INVOLUNTARY TYPES)
Types Features
Cyclical  Economy goes through boom-bust cycles.
 During bust / recession / depression when workers are laid off on
mass scale.
 Cyclical unemployment figures in India are negligible. It is a
phenomenon that is mostly found in capitalist economies.
Frictional  When a person is out of one job and is searching for another job.
 During this transition time, he‘s deemed frictionally unemployed.
Disguised  Unemployment which is not visible because person seems visibly
Unemployment working but his marginal productivity / contribution is zero.
E.g. Farming family of 4 persons produces 200 kgs of grapes, but
even if you remove 3 persons still production remains at 200 kgs.
Seasonal  Labourers in Agriculture, Salt-pans, Sugar Mills, Ice-factory, Tourist
spots, Marriage Catering-Orchestra etc.
Underemployment  Person is employed but not in a befitting position or salary
or Educated corresponding to his qualification.
unemployment  e.g. M.Com working as Swiggy delivery boy, M. Tech working as
Bank clerk etc.

https://t.me/prelimbits
260

Technological  Technological unemployment occurs when men are replaced with


machines e.g. Textile / Automobile.
Open / Structural  Lack of jobs when person‘s skill/qualification is insufficient for
the jobs available in the market
 e.g. An IT Graduate knows C++ but demand is for Python/JAVA
computer language experts

210. Consider the following liquid assets:


1. Demand deposits with the banks
2. Time deposits with the banks
3. Savings deposits with the banks
4. Currency
The correct sequence of these decreasing order of Liquidity is

A. 1-4-3-2
B. 4-3-2-1
C. 2-3-1-4
D. 4-1-3-2
Ans. D

 Currency is most liquid because you can use it as and when you want.
 A time deposit with the bank (e.g. fixed deposit), are the least liquid compared to a
savings/demand deposit with banks and currency.
 That means decreasing order of liquidity will be 4-*-*-2.
o and Demand deposit with bank >more liquid> savings deposit
Liquidity trap

 The liquidity trap is the situation in which prevailing interest rates are low and savings
rates are high, making monetary policy ineffective.
o In a liquidity trap, consumers choose to avoid bonds and keep their funds in
savings, because of the prevailing belief that interest rates will soon rise.
o This reduces the Bond yield and makes them less attractive.
 A liquidity trap is a situation when expansionary monetary policy (increase in the money
supply) does not increase the interest rate, income and hence does not stimulate economic
growth
211. In the context of Indian economy, Open Market Operations‟ refers to
A. borrowing by scheduled banks from the RBI
B. lending by commercial banks to industry and trade
C. purchase and sale of government securities by the RBI
D. None of the above
Ans. C

OPEN MARKET OPERATIONS (OMOS)


 Open Market Operations (OMOs) are market operations conducted by RBI by way
https://t.me/prelimbits
261

of sale/purchase of government securities to/from the market with an objective to


adjust the rupee liquidity conditions in the market on a durable basis.
o If there is excess liquidity, RBI resorts to the sale of securities and sucks out the rupee
liquidity.
o It is one of the quantitative (to regulate or control the total volume of money)
monetary policy tools that are employed by the central bank of a country to control
the money supply in the economy.
SIMILAR TOPIC → OPERATION TWIST
 ‗Operation Twist‘ is RBI‘s simultaneous selling of short-term securities and buying of
long term securities through open market operations (OMO).
 Under this mechanism, the short-term securities are transitioned into long-term securities.
 How does RBI manage ‗Operation Twist‘?
 This operation involves buying and selling government securities simultaneously in order to
bring down long-term interest rates and bolster short-term rates.
 There is an inverse relationship between the bond prices and their yields. As the central
bank buys long-term securities (bonds), their demand rise which in turn pushes up their
prices. [Bond Yield is the effective rate of return that a bond earns]
o However, the bond yield comes down with an increase in prices. Yield is the return
an investor gets on his (bond) holding/investment.
 The interest rate in an economy is determined by yield. Thus, lower long-term interest
rates mean people can avail long-term loans (such as buying houses, cars or financing
projects) at lower rates.
 This also results in a dip in the expected returns from long-term savings which tilts the
balance from saving towards spending. Hence, cheaper retail loans can help encourage
consumption spending which is the largest GDP component in the economy.
How does it affect investors?

 Fixed income investors with higher exposure to long term debt will benefit from easing
yield of long-term bonds.
 Consumers/borrowers will also profit from ‗Operation Twist‘ as the retail loans will now get
cheaper.
 Previously banks were forced to price their retail loans at higher rates owing to high yields
on long-term government borrowings. Cheaper retail loans mean a boost in consumption
and spending in the economy which in turn will revive growth.
Bond & Yield Inversion

 The Bond Yield is the EFFECTIVE RATE OF RETURN that a bond earns. But the rate of
return is not fixed as it changes with the price of the bond.
 Suppose a Rs. 100 bond (this is called issue price or Face Value) is issued @ interest rate
10% (which means that interest in the market is around 10% ).
 Now if the interest rate in the market decreases (the interest rate is decreasing in the
market but the bond which has been issued @10% interest rate → This interest rate is fixed
and it will never change) then the new bonds which will be issued at less interest rate
(say 8%).
 Now if you want to purchase a previously issued bond of Rs. 100 face value→ the

https://t.me/prelimbits
262

holder of the bond will not give you in Rs. 100, rather he will ask for more Rupees → i.e.
the price of the previously issued bond will increase and suppose you purchase this bond in
Rs. 125 then for you annual RETURN will be = (Rs. 10/Rs. 125)*100 = 8% . This 8% is yield.
 So if bond Price goes up Yield will fall [opposite if price of bond fall down than Yield will
rise]
Yield inversion
 Yield inversion happens when the yield on a longer tenure bond becomes less than the
yield for a shorter tenure bond
 A yield inversion typically portends a recession. An inverted yield curve shows that
investors expect the future growth to fall sharply; in other words, the demand for money
would be much lower than what it is today and hence the yields are also lower.
212. Priority Sector Lending by banks in India constitutes the lending to
A. agriculture
B. micro and small enterprises
C. weaker sections
D. All of the above
Ans. D

PRIORITY SECTOR LENDING


 It means those sectors which the Government of India and Reserve Bank of India consider
as important for the development of the basic needs of the country and are to be
given priority over other sectors.
 All scheduled commercial banks and foreign banks (with a sizable presence in India) are
mandated to set aside 40% of their Adjusted Net Bank Credit (ANDC) for lending to these
sectors.
 The origins of Priority Sector Lending is traced back to 1966

 The then government felt the need for increasing credit to agriculture and small
industries.

 However, the definition for Priority Sector was only formalized based on a Reserve
Bank of India (RBI) report in the National Credit Council in 1972.

 After bank nationalization, the Priority Sector formulation also allowed the government to
focus on different sectors by making credit available, through direct lending.

 Later, Deposit Insurance and Credit Guarantee Corporation (DICGC) was setup to
facilitate bank lending to the priority sectors.
Priority Sector Loans norms updated in 2015 Min. Quota
(List is imp for UPSC so remember each sector)
Weaker Sections : SC, ST, Women, PH, Minorities, Manual scavengers, Artisans, 10 %
PMJDY Overdrafts upto Rs.10k, NRLM/NULM beneficiaries
Agriculture : (all farmers small and big) 10%

https://t.me/prelimbits
263

Agriculture: Marginal Farmer (upto 1ht); Small farmer (>1 upto 2ht) 8%
Micro Enterprises, Khadi-Village industries 7.50%
+ 4.50%
Small & Medium Enterprises, Affordable housing loans to beneficiaries under
Pradhan Mantri Awas Yojana, food processing companies, Vermi compost,
biofertilizer, seed production, Exporters, Student-Education loans (upto Rs.10l),
Social Infrastructure (schools, health care, drinking water, sanitation facilities);
Renewable Energy Projects (wind mills, biomass generators, solar street light,
micro-hydel plants etc.) & Latest Start Ups
Total PSL for SCB and (Foreign Banks with 20 or more branches). 40%
 If foreign bank has less than 20 branches, they‘ll also have to give 40% PSL-Quota loans
WEF 31/3/2020, BUT no internal loan-quotas for weaker section, small farmers, khadi
enterprise etc.
 If Regional Rural Bank (RRB) or Small Finance Bank (SCB:) & UCBs, then above 40%
quota + extra 35% quota (in any PSL-sectors as per given bank‘s wish) = 75% PSL loan-
quota.
 PSL quotas are ‗minimum‘ & not maximum. So, if bank wishes, it can EVEN give even 60% of
its loans to weaker sections, instead of just 10% ―minimum quota‖ meant for weaker
section.
 Bank‘s Loans given to Non-Banking Financial Company (NBFC) who are lending to above
PSL categories = such ‗indirect loan‘ to PSL sectors will also be counted for bank‘s quota.
 Bank + NBFC‘s joint lending / co-origination loans to PSL categories are also eligible
Priority Sector Lending Certificates (PSLCs)

 Priority Sector Lending Certificates (PSLCs) are a mechanism to enable banks to achieve
the priority sector lending target and sub-targets by purchase of these instruments in
the event of shortfall.
 This also incentivizes surplus banks as it allows them to sell their excess achievement
over targets thereby enhancing lending to the categories under priority sector.
213. To obtain full benefits of demographic dividend, what should India do?
A. Promoting skill development
B. Introducing more social security schemes
C. Reducing infant mortality rate
D. Privatization of higher education
Ans. A

 The Economic survey 2011-12 says that India did not have as much growth in capital
per worker as other countries such as China, South Korea but had comparatively
stronger growth in TFP.
 China grew because of more capital deployed as well as strong increases in TFP.
Underpinnings for continued strong Chinese growth in the years beyond the second decade
after takeoff are a robust investment rate as well as substantial increases in the intrinsic
productivity of jobs.
https://t.me/prelimbits
264

 If India were to follow a similar path, it would need to increase savings and investment, both
of which will follow from the demographic transformation.
o But it will also have to increase the intrinsic productivity of jobs, that is total
factor productivity (TFP).
 In summary, the survey says that the country can reap the demographic dividend by
improving the Total Factor Productivity, a transition from agriculture to non-agriculture
sectors such as manufacturing/service sector, and from unorganized sector to the
organized.
 This can be done via skill development and that is why skill development is key to
reaping the demographic dividend for a country like India.
 Hence the correct option is 1.
214. A rise in general level of prices may be caused by
1. an increase in the money supply
2. a decrease in the aggregate level of output
3. an increase in the effective demand
Select the correct answer using the codes given below.

A. 1 only
B. 1 and 2 only
C. 2 and 3 only
D. 1, 2 and 3
Ans. D

GENERAL LEVEL OF PRICES MAY BE CAUSED BY


 The increase in the general level of prices may be caused by many factors like an increase
in the money supply, a decrease in the aggregate level of output, an increase in the
effective demand, an increase in income, the rapid growth of population, etc.
o Increase in the money supply →
 The increase in the money supply will generate inflation and increase more
customer spending. The rate of interest will become low. Hence statement 1
is correct.
o The decrease in the Aggregate level of output →
 The aggregate supply of goods and services decreases with the rise in input
prices. The changes in the level of prices don't affect aggregate supply. It
creates Cost-push inflation.
o Increase in the Effective demand →
 Effective demand refers to a condition in which it shows the consumer's
willingness and ability to buy a particular product and services at different
rates. An increase in the effective demand results in rising in employment
opportunities with rising in prices.

https://t.me/prelimbits
265

215. Which one of the following groups of items is included in India‟s foreign-exchange
reserves?
A. Foreign-currency assets, Special Drawing Rights (SDRs) and loans from foreign countries
B. Foreign-currency assets, gold holdings of the RBI and SDRs
C. Foreign-currency assets, loans from the World Bank and SDRs
D. Foreign-currency assets, gold holdings of the RBI and loans from the World Bank
Ans. B

FOREIGN EXCHANGE RESERVES


 These are assets denominated in a foreign currency that is held by a central bank.

 RBI Act and the Foreign Exchange Management Act, 1999 set the legal provisions for
governing the foreign exchange reserves.

 Foreign exchange reserves of India act as a cushion against rupee volatility once
global interest rates start rising

 Forex Reserve component(s) in decreasing order of size →

o FOREIGN CURRENCY ASSETS (includes foreign currencies & G-Sec/bonds of


foreign Govts

o GOLD

o RESERVE TRANCHE Position (RTP) in the IMF.

o SDRS

216. Which one of the following is likely to be the most inflationary in its effect?
A. Repayment of public debt
B. Borrowing from the public to finance a budget deficit
C. Borrowing from banks to finance a budget deficit
D. Creating new money to finance a budget deficit
Ans. D

https://t.me/prelimbits
266

 The chances of inflation will increase more when the government plans to print new
currency to finance a budget deficit.
 When the supply of money increases in the market, the customer demand automatically
increases, resulting in the rise of Aggregate Demand.
DEFICIT FINANCING
 A budget deficit refers to a situation where total expenditure exceeds the total revenue.
 Deficit financing is a tool that is used to generate funds in order to come out with a
budget deficit.
 There are three ways to finance a budget:
o Creating new currency
o Borrowing from internal sources like RBI, issuing bonds, etc.
o Borrowing from External sources like WB, IMF, etc.
 Borrowing from the public by issuing bonds at a lower interest rate will not create
inflation.
 Similarly, borrowings from banks will not generate inflation.
217. Supply of money remaining the same when there is an increase in demand for
money, there will be
A. a fall in the level of prices
B. an increase in the rate of interest
C. a decrease in the rate of interest
D. an increase in the level of income and employment
Ans. B

 The supply of money remaining the same when there is an increase in demand for money,
there will be an increase in the rate of interest. This occurs in order to attract customers
and initiate deposits among them by the banks.
 As we know, when the supply of onion in the market is the same but the demand for it
rises, so in such a case the price of onion is increased in order to maintain stability in the
market.
 It will definitely develop a fall in Aggregate Demand.
INFLATION AND INTEREST RATE, BOTH HAVE AN INVERSE RELATIONSHIP.
 Increased interest rates →
o It will slow down economic growth and control the inflationary pressure.
o It will reduce disposable income and increase the cost of borrowing.
o It will bring appreciation to the exchange rate.
o It will lead to more savings than spending.
o May give rise to unemployment.
 Decreased interest rate →
o It will boost economic growth and inflation increases.
o It will bring depreciation to the exchange rate.

https://t.me/prelimbits
267

o The cost of borrowing will become cheaper.


o It will promote more spending than saving.
o The situation of unemployment decreases.
218. Economic growth in country X will necessarily have to occur if
A. there is technical progress in the world economy
B. there is population growth in X
C. there is capital formation in X
D. the volume of trade grows in the world economy
Ans. C

ECONOMIC GROWTH
 The economic growth of any country results in a healthy and wealthy nation.
 Economic Growth means the increase in the level of GDP of the country.
 It can be possible only when all the factors of production are applied in the right manner
i.e. land, labor, capital, and entrepreneurship.
CAPITAL FORMATION
 The capital formation in any country boosts its development as investment activities
increases resulting in high production, increase in per-capita income, employment
generation, the standard of living, poverty reduction, etc. Hence option 3 is correct.
 The increase in population has both pros and cons for the economy. If the population is
skilled then it will generate more output for the economy. But in case if the economy itself
is not in a position to stand still then in such case it will prove a burden to the country.
 Technical progress refers to the introduction of new and improved technologies for the
production of goods and services. Despite having their advantages, when these
technologies enter the market it requires a heavy investment in training workers and
sometimes these technologies replace human beings resulting in unemployment.
 The volume of trade may be favourable or non- favourable depending upon the overall
trade surplus or deficit.
219. The national income of a country for a given period is equal to the
A. total value of goods and services produced by the nationals
B. sum of total consumption and investment expenditure
C. sum of personal income of all individuals
D. money value of final goods and services produced
Ans. D

 National Income is the money value of all the final goods and services produced by a
country during a period of one year.
 National Income consists of a collection of different types of goods and services of different
types.

https://t.me/prelimbits
268

220. Which of the following grants/ grant direct credit assistance to rural households?
1. Regional Rural Banks
2. National Bank for Agriculture and Rural Development
3. Land Development Banks
Select the correct answer using the codes given below:

A. 1 and 2 only
B. 2 only
C. 1 and 3 only
D. 1, 2 and 3
Ans. C

REGIONAL RURAL BANKS (RRB)


 Based on M.Narasimham‘s Committee on Financial Inclusion in 1970s
 Setup under the provisions of RRB act 1976 & its amendment in 2015.
 Stakeholders: Central government + State Government + Sponsor Bank = 50:15:35
 Subjected to CRR, SLR norms but RBI could prescribe separate norms.
 PSL: 75%.
 Main objectives of RRBs are →
o To Provide Credit and other facilities to the small and marginal farmers,
agricultural labourers, artisans and small entrepreneurs in rural areas. Their loan
interest rates can‘t be more than prevailing lending rates of Cooperative Banks in the
area.
o To check the outflow of rural deposits to urban areas and reduce regional imbalances
and increase rural employment generation
 After the reforms in the 1990s, the government in 2005-06 initiated a consolidation
program that resulted in the number of RRBs declining from 196 in 2005 to 43 in FY21,
and 30 of the 43 RRBs reported net profits.
 Ultimate Regulator: RBI but immediate regulator NABARD.
NATIONAL BANK FOR AGRICULTURE AND RURAL DEVELOPMENT (NABARD)
 NABARD is the country's premier financial institution for agricultural finance and rural
development.
 It was formed in July 1982 by combining the Reserve Bank of India's Agriculture Credit
Department, Rural Planning and Credit Cell, and the entire Agriculture Refinance and
Development Corporation.
 It was established to meet the credit needs of agriculture and rural development.
 NABARD was envisioned as the national apex institution for the entire rural credit system,
providing supplemental funding to all rural credit institutions and
coordinating their operations. [Not directly to Farmers]

https://t.me/prelimbits
269

LAND DEVELOPMENT BANK


 Land Development Bank was earlier known as Land Mortgage Bank.
 They are cooperative institutions registered under Cooperative Societies Act.
 Objective: To provide long-term loans facility to farmers to borrow equipment such as
tractors, pump sets, etc.
 The first Land Mortgage Bank was set up in Jhind, Punjab in 1920. But its real beginning
was marked by the establishment in 1929 in Madras.

Prelims 2014
221. The sales tax you pay while purchasing a toothpaste is a
A. tax imposed by the Central Government
B. tax imposed by the Central Government but collected by the State Government
C. tax imposed by the State Government but collected by the Central Government
D. tax imposed and collected by the State Government
Ans. D

 Taxes on toothpaste come under CST Act, which is administered by the State Government.
 Sales Tax is paid to the Sales Tax Authority in the state from where the goods are
moved.
 Now the taxes on toothpaste comes under GST.
222. What does venture capital mean?
A. A short-term capital provided to industries
B. A long-term start-up capital provided to new entrepreneurs
C. Funds provided to industries at times of incurring losses
D. Funds provided for replacement and renovation of industries
Ans. B

VENTURE CAPITAL
 It is a type of private equity provided by wealthy investors to small business companies that
are believed to have long-term growth potential.
 The amount of the venture capital raised varies greatly depending on the stage a startup is
at.
 Venture capital is a long-term risk capital to finance high technology projects, which
involve risk, but at the same time has a high potential for growth.
223. The main objective of the 12th Five-Year Plan is
A. inclusive growth and poverty reductions
B. inclusive and sustainable growth
C. sustainable and inclusive growth to reduce unemployment
D. faster, sustainable and more inclusive growth.

https://t.me/prelimbits
270

Ans. D

TWELFTH FIVE YEAR PLAN (2012-17)

 The last Five Year Plan had "Faster, More Inclusive and Sustainable Growth" as
its theme.
 The plan aimed at strengthening infrastructure projects, and providing
electricity supply in all villages.
 It also aimed at removing the gender and social gap in admissions at school and improved
access to higher education.
 Further, it aspired to enhance the green cover by 1 million hectares each year and to create
new opportunities in the non-farming sector.
 The target growth rate was 9% but in 2012, National Development Council approved a
growth rate of 8% for this twelfth plan.
224. With reference to Balance of Payments, which of the following
constitutes/constitute the Current Account?
1. Balance of trade
2. Foreign assets
3. Balance of invisibles
4. Special Drawing Rights
Select the correct answer using the code given below.

A. 1 only
B. 2 and 3
C. 1 and 3
D. 1, 2 and 4
Ans. C

BALANCE OF PAYMENT

 The balance of payment is a record of all monetary transactions made between


the residents of one country and the rest of the world.
 A balance of payments deficit means the nation imports are more than it exports.
 The two components of the Balance of Payments are the current account and capital
account.
 The Current Account includes trade in goods and services (Invisibles) and transfer
payments etc.
 The Capital Account includes Foreign Direct Investment, Foreign Portfolio
Investment, External Commercial Borrowings, SDR.
o The capital account records all international purchases and sales of assets such as
money, stocks, bonds, etc.

https://t.me/prelimbits
271

225. The terms „Marginal Standing Facility Rate‟ and „Net Demand and Time Liabilities‟,
sometimes appearing in news, are used in relation to
A. banking operations
B. communication networking
C. military strategies
D. supply and demand of agricultural products
Ans. A

MSF (2011)
 Marginal Standing Facility (MSF) is the Interest rate at which RBI lends short-term
loans to Scheduled Commercial Banks (SCB) with their SLR-quota G-Sec as collaterals.
 MSF higher than higher than Repo Rate. MSF = Repo% + 0.25%.
 Under MSF, banks can borrow funds up to one percent of their net demand and time
liabilities (NDTL).

 Banks can borrow through MSF on all working days except Saturdays.
 The minimum amount for which RBI receives application is Rs.1 Crore, and afterward in
multiples of Rs.1 Crore.
226. What is/are the facility/facilities the beneficiaries can get from the services of
Business Correspondent (Bank Saathi) in branchless areas?
1. It enables the beneficiaries to draw their subsidies and social security benefits in their
villages.
2. It enables the beneficiaries in the rural areas to make deposits and withdrawals.
Select the correct answer using the code given below.

A. 1 only
B. 2 only
C. Both 1 and 2
D. Neither 1 nor 2
Ans. C

BUSINESS CORRESPONDENT (BC)


 The Business Correspondent (BC) Model was initiated by the Reserve Bank of India
(RBI) in 2006 to upgrade financial inclusion in India.
 Business Correspondents are agents who represent banks and are responsible for delivering
banking services at locations other than a bank branch/ATM.
 It enables government subsidies and social security benefits to be directly credited to the
accounts of the beneficiaries, enabling them to draw the money from the bank saathi or
business correspondents in their village itself.

https://t.me/prelimbits
272

227. In the context of Indian economy; which of the following is/are the
purpose/purposes of „Statutory Reserve Requirements‟?
1. To enable the Central Bank to control the amount of advances the banks can create
2. To make the people‘s deposits with banks safe and liquid
3. To prevent the commercial banks from making excessive profits
4. To force the banks to have sufficient vault cash to meet their day-to-day requirements
Select the correct answer using the code given below.

A. 1 only
B. 1 and 2 only
C. 2 and 3 only
D. 1, 2, 3 and 4
Ans. B

STATUTORY RESERVE REQUIREMENT (CRR & SLR)


 Statutory Reserve Requirement is an instrument to manage liquidity.
 RBI requires commercial banks to keep reserves in order to ensure that banks have
sufficient assets to draw on when account holders want to be paid. Hence statement 2
is correct.
 The first instrument is the Cash Reserve Ratio in which is the fraction of their deposits that
banks must keep with RBI.
 Another tool is the Statutory Liquidity Ratio which requires the banks to maintain a given
fraction of their total demand and time deposits in the form of specified liquid assets.
 When the central bank wants to increase the money supply in the economy, it lowers
the reserve ratio. Hence it enables the Central Bank to control the number of
advances the banks can create. Hence statement 1 is correct.
 Reserve requirements are designed as ―precautionary measures‖ to control the economy
and not to stop banks from ―excessive‖ profit and not to force the banks to have
sufficient cash to meet their day-to-day requirements. Hence statement 3 and 4 is
incorrect. [As Per NCERT]
228. If the interest rate is decreased in an economy, it will
A. decrease the consumption expenditure in the economy
B. increase the tax collection of the Government
C. increase the investment expenditure in the economy
D. increase the total savings in the economy
Ans. C

DECREASE IN INTEREST RATE


 When the interest rates decrease, citizens do not want to keep their currency in banks since
they would not get a high return. Option 4 can be eliminated.
 In such times, citizens keep the cash flowing in the economy.

https://t.me/prelimbits
273

 They either invest in productive things or can spend the money on a day-to-day basis.
Option 1 can be eliminated.
 Hence, the interest rate is decreased in an economy, it will increase the investment
expenditure in the economy.
o Decreased interest rates would ensure the availability of capital for investment
expenditure. Hence Option 3 is correct.
 The relationship between the interest rate and investment Expenditure is also illustrated by
the investment curve of the economy.
o The curve has a downward slope, indicating that a drop in interest rate, causes the
investment-spending to rise.
229. Which of the following organizations brings out the publication known as „World
Economic Outlook‟?
A. The International Monetary Fund
B. The United Nations Development Programme
C. The World Economic Forum
D. The World Bank
Ans. A

WORLD ECONOMIC OUTLOOK (WEO)


 The World Economic Outlook is published by the International Monetary Fund twice a
year.
 It analyses and predicts near and medium-term growth prospects.
 Countries are compared based on GDP growth rate, or absolute GDP.
 Recently, the International Monetary Fund (IMF) has released its World Economic
Outlook (WEO) Update, which has marginally improved the forecast for global growth in
2023.
 Takeaways of WEO →
o Global Growth will Bottom Out:
o The global growth, which was estimated at 3.4% in 2022, is now projected to fall to
2.9% in 2023 before rising to 3.1% in 2024.
o The IMF effectively rules out a global recession.
o Negative growth in global GDP (Gross Domestic Product) or global GDP per
capita—which often happens when there is a global recession—is not expected.
o Instead, it expects global growth to bottom out in 2023 before starting to gather
speed in 2024.
230. With reference to Union Budget, which of the following is/are covered under Non-
Plan Expenditure?
1. Defence expenditure
2. Interest payments
3. Salaries and pensions

https://t.me/prelimbits
274

4. Subsidies,
Select the correct answer using the code given below.

A. 1 only
B. 2 and 3 only
C. 1, 2, 3 and 4
D. None
Ans. C

PLAN & NON PLAN EX.


 The Government of India had announced in 2016, that the classification of plan expenditure
and non-plan expenditure will be abolished.
 The plan and non-plan classification was removed from 2017-18.
 Henceforth the expenditures of the Government are reclassified as Capital and Revenue
spending.

Prelims 2015
231. „Basel III Accord‟ or simply „Basel III‟, often seen in the news, seeks to
A. develop national strategies for the conservation and sustainable use of biological diversity
B. improve banking sector‘s ability to deal with financial and economic stress and improve risk
management
C. reduce the greenhouse gas emissions but places a heavier burden on developed countries
D. transfer technology from developed countries to poor countries to enable them to replace
the use of chlorofluorocarbons in refrigeration with harmless chemicals
Ans. B

BASEL-III NORMS
 Bank for International Settlements (BIS) is an international institution made up of 60
countries‘ Central Banks.
 HQ @ BASEL, Switzerland.

 Its Committee on Banking Supervision set norms in 1988 (I), 2004 (II), 2011(III) to
ensure Global Financial Stability.
 From 1/4/2013, RBI began phased implementation of BASEL-III norms in India
 First, a bank needs to calculate its Risk-Weighted Assets (RWA)
 Against these RWAs banks must keep →
o Minimum Capital to Risk Weighted Assets Ratio (CRAR) at 8% (by BASEL
committee)
 But In India, scheduled commercial banks are required to maintain a CAR of
9% while Indian public sector banks are emphasized to maintain a CAR of
12% as per RBI norms.

https://t.me/prelimbits
275

 It is arrived at by dividing the capital of the bank with aggregated risk-


weighted assets for credit risk, market risk, and operational risk.
o 2.5% Capital Conversation Buffer (CCB) of RWA
o X % Counter Cyclic Capital Buffer (CCCB): whenever RBI notifies.
o Y % High quality liquid assets (HQLA)
o Z % Liquidity Coverage Ratio (LCR)
o Leverage Ratio: The ratio of a company's loan capital (debt) to the value of its
ordinary shares (equity). High leverage is bad for Banks or Companies.
 Each member country‘s Central Bank can prescribe different %, ratios depending on
their country‘s situation.
 As Bad loans (NPA) ↑ bank‘s asset quality degrades → its Risk-weighted assets (RWA)
↑→ bank must arrange more capital to comply with these ratios, norms and
buffers.
 If a bank can‘t comply with BASEL norms → RBI puts it in PCA list. In worst case,
bank will have to merge/ sell off its business to another bank or shut down.
 BASEL Norms also apply on Differential Commercial Banks (LAB, RRB, SFB, PyB),
Cooperative Banks, AIFI (EXIM, NABARD, NHB, SIDBI) and certain category of NBFCs, but
RBI can prescribe separate norms / limits / deadlines for them.
Basel Committee on Banking Supervision

 BCBS is the primary global standard setter for the prudential regulation of banks.

 It was established by the Central Bank governors of the Group of Ten countries in
1974.

 The committee expanded its membership in 2009 and then again in 2014.

 The BCBS now has 45 members from 28 Jurisdictions, consisting of Central Banks and
authorities with responsibility of banking regulation.

 Its Objective is to enhance understanding of key supervisory issues and improve the
quality of banking supervision worldwide.
232. With reference to the Indian economy, consider the following statements:
1. The rate of growth of Real Gross Domestic Product has steadily increased in the last decade.
2. The Gross Domestic Product at market prices (in rupees) has steadily increased in the last
decade.
Which of the statements given above is/are correct?

A. 1 only
B. 2 only
C. Both 1 and 2
D. Neither 1 nor 2

https://t.me/prelimbits
276

Ans. B

Due to recession in 2008, the growth rate of the Indian economy had declined for the next few
years from 8-9% to 5-6%. Even though the growth rate had declined, it never became negative. So,
the GDP at market prices has always increased year on year since last one decade

233. With reference to the Indian economy, consider the following


A. Bank rate
B. Open market operations
C. Public debt
D. Public revenue
Which of the above is/are component/ components of Monetary Policy?

a. 1 only
b. 2, 3 and 4
c. 1 and 2
d. 1, 3 and 4
Ans. C

MONETARY POLICY TOOL


 The monetary policy tool is implemented by the RBI through open market
operations, bank rate, CRR, SLR, the repo rate, reverse repo rate.
 The bank rate is the rate of interest at which RBI charges while lending loans to a
commercial bank.
 The Open market operations are conducted by the RBI by the way of sale and purchase of
Government Securities (G-Secs) to adjust liquidity in the market.
ADDITIONAL INFORMATION
 Public debt is the total amount that is borrowed by the government to meet its
development budget.
o It includes the total liabilities of the Union government that have to be paid from the
Consolidated Fund of India.
 The income of the government through taxes, fees, sale of public goods and services, fines,
donations, etc are known as public revenue.
234. reference to inflation in India, which of the following statements is correct?
A. Controlling the inflation in India is the responsibility of the Government of India only
B. The Reserve Bank of India has no role in controlling the inflation
C. Decreased money circulation helps in controlling the inflation
D. Increased money circulation helps in controlling the inflation
Ans. C

INFLATION CONTROL
 Inflation refers to the general increase in the prices of goods and services in an economy

https://t.me/prelimbits
277

over a period of time.


 If the money supply increases than the economy's ability to produce goods and services,
then inflation will also increase.
 During the high inflation period, the government can reduce the spending to decrease
the money circulation in the country.
 Therefore decreased money circulation helps in controlling inflation.
 Sterilization is the process under which the RBI absorbs excess liquidity in the economy.
 Reserve Bank of India is the authority to control inflation through monetary policies
which it does by increasing bank rates, repo rates, cash reserve ratio, buying dollars,
regulating money supply and availability of credit.
235. The problem of international liquidity is related to the non-availability of
A. goods and services
B. gold and silver
C. dollars and other hard currencies
D. exportable surplus
Ans. C

INTERNATIONAL LIQUIDITY
 The concept of international liquidity is associated with international payments that arise
out of international trade in goods and services.
 International liquidity consists of all the resources that are available to the monetary
authorities of countries for the purpose of meeting balance of payments deficits.
 Such liquidity ranges from assets readily available to resources that become available only
after extensive negotiation.
 The primary medium of international liquidity are gold and those foreign currencies
which are universally acceptable in the settlement of international transactions.
 The problem of international liquidity exists essentially for developing countries.
236. There has been a persistent deficit budget year after year. Which of the following
actions can be taken by the government to reduce the deficit?
1. Reducing revenue expenditure
2. Introducing new welfare schemes
3. Rationalizing subsidies
4. Expanding industries
Select the correct answer using the code given below.

A. 1 and 3 only
B. 2 and 3 only
C. 1 only
D. 1,2,3 and 4
Ans. A

https://t.me/prelimbits
278

ACTIONS THAT CAN BE TAKEN BY THE GOVERNMENT TO REDUCE THE DEFICIT


 A budget deficit occurs when government spending is more than its revenues.
 Reducing revenue expenditure and Rationalizing subsidies will reduce the deficit. Hence
statement 1 and 3 are correct
 The increase of taxes is also a way to reduce the deficit i.e higher taxes increase revenue and
help to reduce the budget deficit.
 Introducing new welfare schemes will further enhance the budget deficit. Hence
statement 2 is wrong
 Expanding industries will also increase the budget deficit as in the short run, it will not add
anything to the tax revenues of the government, and thus will increase the budget
deficit. Hence statement 4 is wrong
237. A decrease in tax to GDP ratio of a country indicates which of the following?
1. Slowing economic growth rate
2. Less equitable distribution of national income
Select the correct answer using the code given below.

A. 1 only
B. 2 only
C. Both 1 and 2
D. Neither 1 nor 2
Ans. A

TAX-TO-GDP
 A tax-to-GDP ratio is a gauge of a nation's tax revenue relative to the size of its
economy as measured by gross domestic product (GDP).
 The tax-to-GDP ratio is a measure of a nation's tax revenue relative to the size of its
economy.
 It determines how well a nation's government use its economic resources via taxation.
 Developed nations typically have higher tax-to-GDP ratios than developing nations.
 If the tax to GDP ratio is low it shows a slow economic growth rate.
 The ratio represents that the government can finance its expenditure.
 A higher tax to GDP ratio means that an economy's tax buoyancy is strong.
 A lower tax-to-GDP ratio puts pressure on the government to meet its fiscal deficit
targets.
Reasons behind low India's tax-GDP ratio:

 The excise rate was 16 per cent in 2007-08, whereas it dropped to 12 per cent in
consequent years. An expanding economy needs to maintain a 16% rate recovery.
 The drop in customs revenue, from 2.02 per cent of GDP to 1.50 per cent.
 The drop in corporate tax revenue, with GDP.
 Excessive subsidies and revenue expenditure.
It only shows growth in the economy not the distribution of national income. Hence

https://t.me/prelimbits
279

statement 2 is incorrect

238. In the „Index of Eight Core Industries‟, which one of the following is given the
highest weight?
A. Coal production
B. Electricity generation
C. Fertilizer production
D. Steel production
Ans. B

8 CORE SECTOR

 The eight core sector industries include COAL, CRUDE OIL, NATURAL GAS, REFINERY
PRODUCTS, FERTILISER, STEEL, CEMENT and ELECTRICITY
 The eight core industries comprise nearly 40% of the weight of items included in the
Index of Industrial Production (IIP).
 Weightage (in decreasing): Refinery Products> Electricity> Steel> Coal> Crude Oil>
Natural Gas> Cement> Fertilizers.
239. The Fair and Remunerative Price (FRP) of sugarcane is approved by the
A. Cabinet Committee on Economic Affairs
B. Commission for Agricultural Costs and Prices
C. Directorate of Marketing and Inspection, Ministry of Agriculture
D. Agricultural Produce Market Committee
Ans. A

SUGARCANE PRICE IN INDIA


 The sugarcane price in India is determined by statutory provisions of the Sugarcane
(Control) Order, 1966 issued under the Essential Commodities Act (ECA), 1955.
 From the 2009-10 sugarcane season onwards, the Fair and Remunerative Price of
sugarcane replaced the Statutory Minimum Price (SMP).
 Fair and Remunerative Price (FRP)
o It is the minimum price that is determined by the Government of India on the
recommendation of the Commission for Agricultural Costs and Prices (CACP).
o Sugar mills can't purchase sugarcane from farmers below this price.
 State Advised Price (SAP)
o It is the price announced by the states having a major share in sugarcane
production.
240. Which one of the following issues the „Global Economic Prospects‟ report
periodically?
A. The Asian Development Bank
B. The European Bank for Reconstruction and Development
C. The US Federal Reserve Bank

https://t.me/prelimbits
280

D. The World Bank


Ans. D

241. When the Reserve Bank of India reduces the Statutory Liquidity Ratio by 50 basis
points, which of the following is likely to happen?
A. India‘s GDP growth rate increases drastically
B. Foreign Institutional Investors may bring more capital into our country
C. Scheduled Commercial Banks may cut their lending rates
D. It may drastically reduce the liquidity to the banking system
Ans. C

INCREASING OR DECREASING THE STATUTORY LIQUIDITY RATIO


 When RBI reduce the statutory liquidity ratio by 50 basis points, then Scheduled
commercial banks will cut their lending rates.
 SLR is a mechanism used by RBI to regulate the liquidity of assets and requires the
banks to invest a certain portion of their deposits in RBI-approved securities or gold.
 When SLR is reduced, banks have more money to lend which may lead to a decrease in
lending rates.
 By changing the level of SLR, the Reserve Bank of India can increase or decrease bank
credit expansion. Ensuring the solvency of commercial banks.
 By reducing the level of SLR, the RBI can increase liquidity with the commercial banks,
resulting in increased investment. This is done to fuel growth and demand.
242. In India, markets in agricultural products are regulated under the
A. Essential Commodities Act, 1955
B. Agricultural Produce Market Committee Act enacted by States
C. Agricultural Produce (Grading and Marking) Act, 1937
D. Food Products Order, 1956 and Meat and Food Products Order, 1973
Ans. B

AGRICULTURAL PRODUCE MARKET COMMITTEE


 Agricultural Produce Market Committee (APMC) under the State Government regulates
the notified agricultural produce and livestock.
 Agriculture is a State subject under Schedule 7 of the Indian Constitution.
 The whole geographical area in the State is divided and declared as a market
area (Yard Mandis) wherein the markets are managed by the Market Committees
constituted by the State Governments.
 Once a particular area is declared a market area and falls under the jurisdiction of a Market
Committee, no person or agency is allowed freely to carry on wholesale marketing
activities.
 Buyers, too, need to obtain individual licenses from each APMC to transact.
 It is a part of government policy toward food security, remunerative prices to farmers,

https://t.me/prelimbits
281

and fair prices to consumers.


ESSENTIAL COMMODITIES ACT (ECA), 1955
 The ECA has been used by the Government to regulate the production, supply, and
distribution of a whole host of commodities it declares ‗essential to make them
available to consumers at fair prices.
243. Which of the following brings out the „Consumer Price Index Number for Industrial
Workers‟?
A. The Reserve Bank of India
B. The Department of Economic Affairs
C. The Labour Bureau
D. The Department of Personnel and Training
Ans. C

CONSUMER PRICE INDEX


 It measures price changes from the perspective of a retail buyer. It is released by
the National Statistical Office (NSO).
 The CPI calculates the difference in the price of COMMODITIES & SERVICES such as
food, medical care, education, electronics etc, which Indian consumers buy for use.
 The CPI has several sub-groups including food and beverages, fuel and light, housing and
clothing, bedding and footwear.
 Types of CPI are as follows:
o CPI for Industrial Workers (IW). [used to revise dearness allowance (DA)]
o BY – 2016
o CPI for Agricultural Labourer (AL). [used to decide to MGNREGA]
o CPI for Rural Labourer (RL).
o CPI (Rural/Urban/Combined).
o CPI UNME
Of these, the first three are compiled by the Labour Bureau in the Ministry of Labour
and Employment. Fourth & 5th is compiled by the NSO in the Ministry of Statistics
and Programme Implementation.

 Base Year for CPI is 2012.


 The Monetary Policy Committee (MPC) uses CPI data to control inflation. In April 2014,
the Reserve Bank of India (RBI) had adopted the CPI as its key measure of inflation.
244. Convertibility of rupee implies
A. being able to convert rupee notes into gold
B. allowing the value of the rupee to be fixed by market forces
C. freely permitting the conversion of rupee to other currencies and vice versa
D. developing an international market for currencies in India
Ans. C

https://t.me/prelimbits
282

Refer Q. 37 & or Q.17

Prelims 2016
245. The term „Core Banking Solutions‟ is sometimes seen in the news. Which of the
following statements best describes/describe this term?
1. It is a networking of a bank‘s branches that enables customers to operate their accounts
from any branch of the bank on its network regardless of where they open their accounts.
2. It is an effort to increase RBI‘s control over commercial banks through computerization.
3. It is a detailed procedure by which a bank with huge non-performing assets is taken over by
another bank.
Select the correct answer using the code given below.

A. 1 only
B. 2 and 3 only
C. 1 and 3 only
D. 1, 2 and 3
Ans. A

CORE BANKING SOLUTION (CBS)


 Core Banking Solution (CBS) is a networking of bank branches, which allows customers
to manage their accounts, and use various banking facilities from any part of the
world. Hence statement 1 is correct.
 It is a centralized back-end system that efficiently processes banking transactions across the
various branches of a bank.
 Core Banking Solution (CBS) helps in speeding up banking transactions and to expand the
presence of banking in rural & remote areas.
246. With reference to pre-packaged items in India, it is mandatory to the manufacturer
to put which of the following information on the main label, as per the Food Safety and
Standards (Packaging and Labelling) Regulations, 2011?
1. List of ingredients including additives
2. Nutrition information
3. Recommendations, if any, made by the medical profession about the possibility of any
allergic reactions
4. Vegetarian/non-vegetarian
Select the correct answer using the code given below.

A. 1, 2 and 3
B. 2, 3 and 4
C. 1, 2 and 4
D. 1 and 4 only
Ans. C

https://t.me/prelimbits
283

FOOD SAFETY AND STANDARDS (PACKAGING AND LABELLING) REGULATIONS, 2011?


 Every package of food shall carry the following information on the label, namely-
o The Name of Food
o List of Ingredients including additives (Hence the statement 1 is correct)
o Nutritional information (Hence the statement 2 is correct)
o Declaration regarding Veg or Non-veg (Hence the statement 4 is correct)
o Declaration regarding Food Additives
o Name and complete address of the manufacturer
o Net quantity
o Date of manufacture or packing
o Lot/Code/Batch identification
o Best Before and Use By Date
o Country of origin for imported food
o Instructions for use
 Information about Allergic reactions made by the medical profession is not mandatory on
the food packets. (Hence the statement 3 is incorrect)
247. India‟s ranking in the „Ease of Doing Business Index‟ is sometimes seen in the news.
Which of the following has declared that ranking?
A. Organization for Economic Cooperation and Development (OECD)
B. World Economic Forum
C. World Bank
D. World Trade Organization (WTO)
Ans. C

EASE OF DOING BUSINESS REPORT


 Report was introduced in 2003
 is an index by the World Bank to measure how easy or difficult it is to run a business
organisation in a given country, based on simple average (equally weighted) of 10
parameters- such as no. of documents, time & cost involved in registering a property,
getting an electricity connection, paying taxes etc.
 As such no specific themes are given in these reports.
Report 2018 → #1: NZ>Singapore>Denmark> India (100) >... Somalia (190)
Rank2019 #1: NZ>Singapore>Denmark> India (77) >... Somalia (190)
(released
in 2018-Oct) →
Rank2020 #1: NZ>Singapore>Hongkong> India (63) >... Somalia (190)
(released Report acknowledges India as one of the top 10 improvers, third, time in a
in 2019-Oct) → row. India‘s rank in 2017 (#130) to 2020 (#63) = shows a jump of 67 steps-
this is highest by any large country since 2011.
India‘s ranking on 10 parameters

https://t.me/prelimbits
284

Ease of Doing Rank- Rank- Rank- Improved Rank- Improved 2020


Biz Parameters 2017 2018 2019 2019 over 2020 over 2019
2018
Overall 130 100 77 100 -77=23 63 77 Minus 63 =14
Starting a 155 156 137 19 136 137 Minus 136= 1
Business
Construction 185 181 52 129 27 25
Permits (highest
jump)
Getting 26 29 24 5 22 2
Electricity
Registering 138 154 166 -12 154 12
Property
-3 (getting tougher
Getting Credit 44 29 22 7 25 to get loans, thanks
(loan) to NPA, ILFS-NBFC
crisis)
Protecting 13 4 7 -3 13 -6 (corporate scams
Minority in ILFS etc)
Investors
Paying Taxes -2 (despite
172 119 121 apps & 115 6
portals!)
Trading across 143 146 80 66 68 12
Borders
Enforcing 172 164 163 1 163 0
Contracts
Resolving 136 103 108 -5 (despite 52 56
Insolvency I&B Code)
World Bank Stops ‗Ease of Doing Business‘ Report

 The World Bank would discontinue the practice of issuing ‗Doing Business report‘ following
an investigation reported ―data irregularities‖ in its 2018 and 2020 editions (released in 2017
and 2019, respectively) and possible ―ethical matters‖ involving bank staff.
 It will be working on a new approach to assessing the business and investment climate.
 The World Bank announced in December 2022 that it would be releasing the methodology
for the replacement to the index in the second quarter of 2023.
248. What is/are the purpose/purposes of the `Marginal Cost of Funds based Lending
Rate (MCLR)‟ announced by RBI?
1. These guidelines help improve the transparency in the methodology followed by banks for
determining the interest rates on advances.
2. These guidelines help ensure availability of bank credit at interest rates which are fair to the
borrowers as well as the banks.
Select the correct answer using the code given below.

https://t.me/prelimbits
285

A. 1 only
B. 2 only
C. Both 1 and 2
D. Neither 1 nor 2
Ans. C

BANKS‘ LENDING RATES


Lending rate or interest rate is the AMOUNT CHARGED BY LENDERS for a certain period as a
percentage of the amount lent or deposited. The total interest on the amount or the principal
sum is determined by the duration of time over which the amount is deposited or lent.
1969 Government began nationalization of private banks, and ‗Administered interest rates‘
on them i.e. Government would decide how much loan interest rate the banks
should charge on borrowers.
1991 M. Narsimham suggested deregulation: Govt should not dictate / administer
individual banks‘ interest rates. RBI should only give methodology to banks.
2003 RBI introduced BENCHMARK PRIME LENDING RATE (BPLR) system
2010 RBI introduced BASE Rate + Spread system;
Update frequency on individual banks‘ discretion. So, it did not help transmission of
monetary policy much. Even IF RBI reduced repo rate, the banks would not update their
formula figures regularly.
2016 MCLR system
2019 External Benchmark
MCLR system (2016)

 In 2016, RBI ordered banks to link their loan interest rate = ―Marginal Cost of Funds based
Lending Rate (MCLR) + Spread‖ system.
 Banks to calculate on monthly basis. It consists of CRR Cost, Operating Cost,
Marginal cost of funds (Repo Rate, Deposit Interest) etc.
 Benefits? Better Transmission Of Monetary Policy; transparency & accountability to
borrowers.
o Transmission of Monetary Policy: The transmission of monetary policy describes
how changes made by the Reserve Bank of India (RBI) to the policy rate flow
through to economic activity (like lending) and inflation.
 Limitation? From January to Oct 2019, RBI has reduced its repo rate by 135 bps but
banks reduced their loan interest rates by merely 40-47 bps. Thus, even though the RBI
reduces its repo rate, banks are not quickly reducing their loan interest rates.
External Benchmark

 To ensure complete transparency and standardization, RBI mandated the banks to adopt
a uniform external benchmark within a loan category, effective 1st October, 2019.

 Unlike MCLR which was internal system for each bank, RBI has offered banks the options
to choose from 4 external benchmarking mechanisms:

https://t.me/prelimbits
286

1. RBI repo rate or


2. 91-day T-bill yield or
3. 182-day T-bill yield or
4. any other benchmarks by Financial Benchmarks India Ltd.
 Banks must revise its lending rate once every THREE MONTHS.
Benefits
 Banks are free to decide the Spread over the external benchmark.

 However, the interest rate must be reset as per the external benchmark at least once
every three months.

 Being an external system, this means any policy rate cut decision will reach
borrowers faster.
 The adoption of external benchmarking will make the interest rates transparent.
o The borrower will also know the spread or profit margin for each bank over the fixed
interest rate making loan comparisons easier and more transparent.

There are some segments of borrowers, who were issued loans before October 2019, continuing
with the old MCLR regime.

249. With reference to „Financial Stability and Development Council‟, consider the
following statements :
1. It is an organ of NITI Aayog.
2. It is headed by the Union Finance Minister.
3. It monitors macroprudential supervision of the economy.
Which of the statements given above is/are correct?

A. 1 and 2 only
B. 3 only
C. 2 and 3 only
D. 1, 2 and 3
Ans. C

FSDC
 It is a non-statutory apex council under the Ministry of Finance constituted by
the Executive Order in 2010.
 The Raghuram Rajan committee (2008) on financial sector reforms first proposed the
creation of FSDC.
Composition →

 It is chaired by the Finance Minister and its members include the heads of all Financial
Sector Regulators (RBI, SEBI, PFRDA & IRDA), Finance Secretary, Secretary of
Department of Economic Affairs (DEA), Secretary of Department of Financial Services (DFS),
https://t.me/prelimbits
287

and Chief Economic Adviser.


o In 2018, the government reconstituted FSDC to include the Minister of
State responsible for the Department of Economic Affairs (DEA), Secretary of
Department of Electronics and Information Technology, Chairperson of
the Insolvency and Bankruptcy Board of India (IBBI) and the Revenue Secretary.
 FSDC sub-committee is headed by the Governor of RBI.
 The Council can invite experts to its meeting if required.
Functions →

 To strengthen and institutionalize the mechanism for maintaining financial stability,


enhancing inter-regulatory coordination and promoting financial sector development.
 To monitor macro-prudential supervision of the economy. It assesses the functioning of
large financial conglomerates.
250. The term „Base Erosion and Profit Shifting‟ is sometimes seen in the news in the
context of
A. mining operation by multinational companies in resource-rich but backward areas
B. curbing of the tax evasion by multinational companies
C. exploitation of genetic resources of a country by multinational companies
D. lack of consideration of environmental costs in the planning and implementation of
developmental projects
Ans. B

BASE EROSION AND PROFIT SHIFTING (BEPS)


 OECD & G20
 Multinational Corporation (MNC) ―M‖ opens fast food outlets in India & makes ₹ 50 crores
profit. By default, it should be subjected to 40% Corporation tax in India.

 But then MNC shows its Indian outlets had taken loan / raw material / patented technology
from MNC‘s shell firm in Bahamas (where Corporation tax is 0-2%). So, after deducting
these operating costs, it has zero profit, so in India, it will pay only 18.5% Minimum
Alternative Tax (MAT), instead of 40% Corporation tax.

 Thus, when MNCs shift profit from its source country to a tax-haven to
avoid / reduce paying taxes, its known as ―BEPS‖.
 2019-July: India ratified the OECD‘s & G20 joint Multilateral Convention to Implement Tax
Treaty Related Measures to Prevent Base Erosion and Profit Shifting (commonly referred to
as MLI. MLI will be applied alongside existing tax treaties, modifying their application in
order to implement the BEPS measures.

https://t.me/prelimbits
288

251. Recently, India‟s first „National Investment and Manufacturing Zone‟ was proposed
to be set up in [From Current Affairs of 2015-16]
A. Andhra Pradesh
B. Gujarat
C. Maharashtra
D. Uttar Pradesh
Ans. A

MFG POLICY → NATIONAL MFG POLICY 2011


 Boss? Commerce ministry → DIPP / DPIIT [The DPIIT is the nodal Department for
formulation of policy of the Government on Foreign Direct Investment (FDI).]
 Target? To increase manufacturing‘s share in GDP to 25% by 2022, & create 100 million
jobs. [ presently it is 27.5 % ]

 For this target, Govt will pursue ease of doing business, skill upgradation for young
workforce, funding for innovation & green Technologies

 Creating National Investment and Manufacturing Zone (NIMZ).

NIMZ

 NIMZ is an ‗industrial township‘ containing Special Economic Zones, Industrial Parks &
Warehousing Zones, Export Oriented Units etc.

 NIMZ are given additional support by government e.g.

o Tax incentives, Relaxed norms for FDI approval

o Providing Rail, Road, energy, communication connectivity, schools-hospitals & other


social infrastructure for the workers, etc. in a time bound manner.

o relaxations in the labour laws e.g. women allowed to work in night shift, easier hiring-
firing norms:

 NIMZ will be treated as self-governing bodies under Article 243(Q-c) of the Constitution.
So the traditional norms related to Municipality, its functions, election of ward members etc.
will not apply for this township area.

 We have more than 15 NIMZ such as Ahmedabad-Dholera Investment Region @Gujarat,


Dadri- Noida-Ghaziabad investment Region @Uttar Pradesh, Manesar-Bawal Investment
Region @Haryana etc

 Previously, Delhi Mumbai Industrial Corridor had setup Special Investment Regions (SIR) in
its region. They‘re converted into NIMZ.

 2017: Commerce ministry launched Industrial Information System (IIS), a GIS-enabled


database of industrial areas across the country. This helps entrepreneurs to find out raw
material, distance from key transport hubs etc.

Zero Defect and Zero Effect (ZED)


https://t.me/prelimbits
289

 The objective of the scheme is the promotion of Zero Defect and Zero Effect (ZED)
manufacturing in industry with special focus on micro, small and medium enterprises
(MSMEs).
 ―Zero defects‖ means manufacturing of high quality goods without any defects
and ―zero effect‖ means to ensure that the production of goods do not have any impact
on environment.
 This initiative is applicable to all sectors of manufacturing industries with focus on
MSMEs.
The following are some of the objectives of the scheme
 Enable MSMEs for manufacturing quality products.
 Encourage MSMEs to constantly upgrade their quality standards in products and processes.
 Drive manufacturing with adoption of Zero Defect production processes and without
impacting the environment.
 Support Make in India campaign.

252. With reference to „Bitcoins‟, sometimes seen in the news, which of the following
statements is/are correct?
1. Bitcoins are tracked by the Central Banks of the countries.
2. Anyone with a Bitcoin address can send and receive Bitcoins from anyone else with a Bitcoin
address.
3. Online payments can be sent without either side knowing the identity of the other.
Select the correct answer using the code given below.

A. 1 and 2 only
B. 2 and 3 only
C. 3 only
D. 1, 2 and 3
Ans. B

BITCOINS
 Bitcoin is a type of digital currency that enables instant payments to anyone. Bitcoin
was introduced in 2009. Bitcoin is based on an open-source protocol and is not issued by
any central authority.
 The origin of Bitcoin is unclear, as is who founded it. A person, or a group of people, who
went by the identity of Satoshi Nakamoto are said to have conceptualised an accounting
system in the aftermath of the 2008 financial crisis.
 Originally, Bitcoin was intended to provide an alternative to fiat money and become a
universally accepted medium of exchange directly between two involved parties.
o Fiat money is a government-issued currency that is not backed by a commodity
such as gold.
o It gives central banks greater control over the economy because they can control
how much money is printed.
o Most modern paper currencies, such as the US dollar and Indian Rupee are fiat

https://t.me/prelimbits
290

currencies.
Record of Bitcoins (Blockchain)

 All the transactions ever made are contained in a publicly available, open
ledger, although in an anonymous and an encrypted form called a blockchain.
 Transactions can be denominated in sub-units of a Bitcoin.
 Satoshi is the smallest fraction of a Bitcoin.
 Blockchain is a shared, immutable ledger that facilitates the process of recording
transactions and tracking assets in a business network.
 An asset can be tangible (a house, car, cash, land) or intangible (intellectual property,
patents, copyrights, branding).
 Virtually anything of value can be tracked and traded on a blockchain
network, reducing risk and cutting costs for all involved.
o A simple analogy for understanding blockchain technology is a Google Doc.
o When one creates a document and shares it with a group of people, the document is
distributed instead of copied or transferred.
o This creates a decentralized distribution chain that gives everyone access to the
document at the same time.
 It needs to be noted that other usage and applications of Blockchain technology have
emerged in the last few years.
o The government of Andhra Pradesh and Telangana have put the land records on
the blockchain technology owing to its easy traceability feature.
o Election Commission (EC) officials are exploring the potential of using blockchain
technology to enable remote voting.
Acquiring Bitcoins

 One can either mine a new Bitcoin if they have the computing capacity, purchase them
via exchanges, or acquire them in over-the-counter, person-to-person transactions.
 Miners are the people who validate a Bitcoin transaction and secure the network with their
hardware.
o The Bitcoin protocol is designed in such a way that new Bitcoins are created at a
fixed rate.
o No developer has the power to manipulate the system to increase their profits.
o One unique aspect of Bitcoin is that only 21 million units will ever be created.
 A Bitcoin exchange functions like a bank where a person buys and sells Bitcoins with
traditional currency. Depending on the demand and supply, the price of a Bitcoin keeps
fluctuating.
Bitcoin Regulation

 The supply of bitcoins is regulated by software and the agreement of users of the
system and cannot be manipulated by any government, bank, organisation or individual.

https://t.me/prelimbits
291

 Bitcoin was intended to come across as a global decentralised currency, any central
authority regulating it would effectively defeat that purpose.
 It needs to be noted that multiple governments across the world are investing in
developing Central Bank Digital Currencies (CBDCs), which are digital versions of
national currencies.
Legitimacy of Bitcoins (or cryptocurrencies) in India

 At the moment, there is no legislature that covers cryptocurrencies in India. In India,


owning cryptocurrencies is still not illegal. In 2020, the Supreme Court had struck down a
ban on trading of crypto currency in India, which was imposed by the Reserve Bank of India
(RBI).
 Central bank has been cautioning people against the use of virtual currencies since as far
back as 2013.
 In April 2018, the RBI had prohibited regulated entities from dealing in virtual
currencies or providing services for facilitating any person or entity in dealing with or
settling them. The directive was set aside by the Supreme Court in March 2020.
 Subsequently, in May 2021, the central bank advised its regulated entities to continue
to carry out customer due diligence processes for transactions in such currencies, in line
with regulations governing standards for Know Your Customer (KYC), Anti-Money
Laundering, Combating of Financing of Terrorism, obligations under Prevention of
Money Laundering Act, 2002, etc. and Foreign Exchange Management Act (FEMA) Norms
for overseas remittances.
 The Union Budget 2022-2023 also proposed to introduce a digital currency in the
coming financial year.
RBIs Concerns

 Not a Fiat Money : Cryptocurrencies are not a currency because every modern currency
needs to be issued by the central bank or Government.

 Speculative and Destabilising: The value of fiat currencies is anchored by monetary


policy and their status as legal tender, however the value of cryptocurrencies rests solely
on the speculation and expectations of high returns that are not well anchored, so it will
have a destabilising effect on the monetary and fiscal stability of a country.

CENTRAL BANK DIGITAL CURRENCIES (CBDCS)

https://indianexpress.com/article/explained/explained-economics/cbdc-the-digital-rupee-that-rbi-
could-introduce-this-year-8105208/

https://t.me/prelimbits
292

 Central Bank (RBI) Digital Currency is a digital version of so-called ―fiat money,‖ or the
regular currency a country uses, as established and regulated by its government.

 It is a digital payment instrument that is denominated in a national currency and issued


by a central bank.

 Unlike private virtual currencies whose value is based on its ownership, distribution and
trading on exchanges, a CBDC‘s intrinsic value is equivalent to any other form of money
issued by the central bank.

 It will be based on Blockchain Technology

 CBDCs allow users to perform domestic and cross-border transactions without the
involvement of a third party or a bank.

CBDCs around the world

 Bank of Thailand

 Bank of Lithuania

 Riksbank, Sweden‘s central bank

 Central Bank of Bahamas launched ‗Sand Dollar‘

 Central Bank of Venezuela launched Petro

253. What is/are the purpose/purposes of the Government‟s „Sovereign Gold Bond
Scheme‟ and „Gold Monetization Scheme‟?
1. To bring the idle gold lying with Indian households into the economy
2. To promote FDI in the gold and jewellery sector
3. To reduce India‘s dependence on gold imports
Select the correct answer using the code given below.

A. 1 only
B. 2 and 3 only
C. 1 and 3 only
D. 1, 2 and 3
Ans. C

SOVEREIGN GOLD BONDS


 Government securities denominated in gold. They are substitutes for holding physical
gold.
 Investors have to pay the issue price and the bonds will be redeemed in cash on maturity.
 Issued by Reserve Bank on behalf of Government of India.
 SGBs are sold through banks, Stock Holding Corporation of India Limited, Clearing
Corporation of India Limited, designated post offices, National Stock Exchange of
India Limited and the BSE

https://t.me/prelimbits
293

 The quantity of gold for which the investor pays is protected, since he receives the
ongoing Market Price at the time of redemption/ premature redemption.
 The risks and costs of storage are eliminated. Investors are assured of the market value of
gold at the time of maturity and periodical interest. SGB is free from issues like making
charges and purity in the case of gold in jewellery form.
 The minimum limit 1 gram of gold, while the maximum limit is 4 kg for individual, 4 kg for
HUF and 20 kg for trusts and similar entities, as notified by the government from time to
time.
 Tenor: 8 years with exit option after completion of five years.
 Rate : 2.5% per annum.
 Bonds can be used as collateral for loans.
 No capital gains tax

254. „Global Financial Stability Report‟ is prepared by the


A. European Central Bank
B. International Monetary Fund
C. International Bank for Reconstruction and Development
D. Organization for Economic Cooperation and Development
Ans. B

https://t.me/prelimbits
294

IMF (1945)
 Helps in global currency EXCHANGE STABILITY, HELPS AGAINST BALANCE OF
PAYMENT CRISIS. [ primary purpose]

The International Monetary Fund (IMF) is an organization of 190 countries, working to foster
global monetary cooperation, secure financial stability, facilitate international trade,
promote high employment and sustainable economic growth, and reduce poverty around the
world. [https://www.imf.org/en/About]

 RESERVOIR OF THE CURRENCIES & giver these money to other nations- using the Special
Drawing Rights (SDR) mechanism.
 Notable reports? Global Financial Stability Report, World
Economic Outlook.
 Last added Country = Andorra [situated between France
and Spain]
 Headquartered in Washington, D.C.
India & IMF
 India is a founder member of the IMF.
 India‘s Union Finance Minister is the Ex Officio Governor on the IMF‘s Board of
Governors. Each member country also has an alternate governor. The alternate governor
for India is the Governor of the RBI. There is also an Executive Director for India who
represents the country at the IMF.
 India‘s quota in the IMF is SDR 13,114.4 million that gives India a shareholding of 2.75%.
[8th]

QUOTAS determine the size of contingency funds at the disposal of IMF to lend to countries in
need of help, as well as the power of individual countries to influence lending decisions and tap
into the funds themselves.
The IMF‘s Board of Governors conducts general quota reviews at regular intervals (usually every
five years). Any changes in quotas must be approved by an 85 percent majority of the total voting
power, and a member‘s quota cannot be changed without its consent.
 In 2000, India completed the repayment of all the loans it had taken from the IMF. Now,
India is a contributor to the IMF.
Rapid Financing Instrument (RFI)

For higher income countries

 The Rapid Financing Instrument (RFI) provides rapid financial assistance, which is
available to all member countries facing an urgent balance of payments need.

 The RFI was created as part of a broader reform to make the IMF‘s financial support
more flexible to address the diverse needs of member countries.

 The RFI replaced the IMF‘s previous emergency assistance policy and can be used in a wide

https://t.me/prelimbits
295

range of circumstances.

https://www.imf.org/en/About/Factsheets/Sheets/2016/08/02/19/55/Rapid-Financing-Instrument

Rapid Credit Facility (RCF)

For low-income countries (LICs)

 The Rapid Credit Facility (RCF) provides rapid concessional financial assistance to low-
income countries (LICs) facing an urgent balance of payments (BoP) need with no ex post
conditionality where a full-fledged economic program is neither necessary nor
feasible.

 The RCF was created under the Poverty Reduction and Growth Trust (PRGT) as part of a
broader reform to make the Fund‘s financial support more flexible and better tailored to the
diverse needs of LICs, including in times of crisis.

https://www.imf.org/en/About/Factsheets/Sheets/2016/08/02/21/08/Rapid-Credit-Facility

ORGANISATION FOR ECONOMIC CO-OPERATION AND DEVELOPMENT


 Organisation for Economic Co-operation and Development (OECD) is a Paris-based
international organisation of developed countries that that discuss and develop
economic and social policy.

 The OECD brings together Member countries and a range of partners that collaborate on
key global issues at national, regional and local levels.

 OECD is a group of 38 member countries and India is not a member.

Crypto-Asset Reporting Framework of OECD

Recently OECD has released a global tax reporting framework for cryptocurrencies as a respond
to a G20 request.

 In 2021, the G20 mandated the OECD to develop a framework providing for the
automatic exchange of tax-relevant information on Crypto-Assets.

 The crypto assets can be used for tax evasion because of a lack of any regulations.

 The crypto market has also given rise to new intermediaries and service providers, such as
crypto-asset exchanges and wallet providers.

 The crypto-assets and related transactions are not comprehensively covered by the
OECD/G20 Common Reporting Standard (CRS).

o The OECD/G20 Common Reporting Standard (CRS) is a set of guidelines


developed by the OECD for automatic exchange of information.

 In 2022, the OECD approved the Crypto-Asset Reporting Framework (CARF) with a view to
automatically exchanging such information.

 CARF provides for the reporting of tax information on transactions in Crypto-Assets in


a standardised manner.

https://t.me/prelimbits
296

 The information, according to CARF, will be shared on an annual basis and accounting
will be done on the lines of CRS.

 The CARF will target any digital representation of value that relies on a cryptographically
secured distributed ledger or a similar technology to validate and secure transactions.

 The CARF contains model rules that can be transposed into domestic legislation, and
commentary to help administrations with implementation.

INTERNATIONAL BANK FOR RECONSTRUCTION AND DEVELOPMENT (IBRD)


 The International Bank for Reconstruction and Development (IBRD) is an international
financial institution, established in 1944 and headquartered in Washington, D.C., United
States, that is the lending arm of World Bank Group.
 The IBRD offers loans to middle-income developing countries.
 The IBRD is the first of five member institutions that compose the World Bank Group. The
initial mission of the IBRD in 1944, was to finance the reconstruction of European
nations devastated by World War II.
 The IBRD and its concessional lending arm, the International Development Association
(IDA), are collectively known as the World Bank as they share the same leadership and
staff.
255. The term „Regional Comprehensive Economic Partnership‟ often appears in the
news in the context of the affairs of a group of countries known as
A. G20
B. ASEAN
C. SCO
D. SAARC
Ans. B

INDIA DID NOT JOIN RCEP


 Regional Comprehensive Economic Partnership (RCEP) is a free-trade agreement between
the 10 ASEAN countries and their five Free-Trade Agreements partners → Australia,
China, Japan, New Zealand and S. Korea.
 The 15 member countries account for about 30% of the world's population (2.2 billion
people) and 30% of global GDP ($26.2 trillion) as of 2020,

 RCEP requires them to reduce the tariff and non-tariff barriers (About 90%) against each
other, encourage investments, economic and technical cooperation, protect Intellectual
Property Rights.

 RCEP was signed on 15 November 2020 at a virtual ASEAN Summit hosted by Vietnam
and and will take effect 60 days after it has been ratified by at least six ASEAN and three
non-ASEAN signatories. [ till now it has been ratified by china & Thailand]

 India did not join in Nov 2019. [due the worries of the agriculture sector and the fear that
Chinese goods would flood Indian markets]

https://t.me/prelimbits
297

256. On which of the following can you find the Bureau of Energy Efficiency Star Label?
1. Ceiling fans
2. Electric geysers
3. Tubular fluorescent lamps
Select the correct answer using the code given below.

A. 1 and 2 only
B. 3 only
C. 2 and 3 only
D. 1, 2 and 3
Ans. D

STAR LABELING PROGRAMME


 Bureau of Energy Efficiency (BEE) has been set up under the provisions of the Energy
Conservation Act, 2001 on 1st March 2002.
 The Star Labeling Programme has been formulated by the Bureau of Energy Efficiency.
 BEE sets all the standards and norms that need to be followed while rating an appliance.
 Appliances that need to have an energy rating label mandatorily are Frost-free
refrigerator, Tubular Fluorescent Lamps, Room Air-Conditioners, Distribution Transformer,
Colour TV, CST AC, Direct Cool Refrigerator and Electric Geyser.
 There are two variants of these labels, a big label and a smaller label:
o Products with a big label include Refrigerators, air-conditioners, geysers and washing
machines.
o Products with a small label include Ceiling fans, tube lights, computers/laptops and
televisions.
257. „European Stability Mechanism‟, sometimes seen in the news, is an
A. agency created by EU to deal with the impact of millions of refugees arriving from Middle
East
B. agency of EU that provides financial assistance to eurozone countries
C. agency of EU to deal with all the bilateral and multilateral agreements on trade
D. agency of EU to deal with the conflicts arising among the member countries
Ans. B

EUROPEAN STABILITY MECHANISM (ESM)


 European Stability Mechanism (ESM) was an international financial institution set up by the
euro area Member States to help euro area countries in severe financial distress.
 The European Stability Mechanism (ESM) was set up in October 2012 as a successor to the
European Financial Stability Facility (EFSF).
 It provides financial assistance, in the form of loans, to eurozone
countries or as new capital to banks in difficulty.
 This assistance is granted only if it is proven necessary to safeguard the financial stability of

https://t.me/prelimbits
298

the euro area as a whole and of ESM Members.


EURO AREA
 The euro area, commonly called eurozone (EZ), is a currency union of 20 member states of
the European Union (EU) that have adopted the euro (€) as their primary currency and
sole legal tender, and have thus fully implemented EMU policies.
 The 20 eurozone members are Austria, Belgium, Croatia, Cyprus, Estonia, Finland, France,
Germany, Greece, Ireland, Italy, Latvia, Lithuania, Luxembourg, Malta, the Netherlands,
Portugal, Slovakia, Slovenia, and Spain.
 The seven non-eurozone members of the EU are Bulgaria, Czech Republic, Denmark,
Hungary, Poland, Romania, and Sweden. They continue to use their own national currencies,
although all but Denmark are obliged to join once they meet the euro convergence criteria

258. There has been a persistent deficit budget year after year. Which action/actions of
the following can be taken by the Government to reduce the deficit?
1. Reducing revenue expenditure
2. Introducing new welfare schemes
3. Rationalizing subsidies
4. Reducing import duty
Select the correct answer using the code given below.

A. 1 only
B. 2 and 3 only
C. 1 and 3 only
D. 1, 2, 3 and 4

https://t.me/prelimbits
299

Ans. C

Similar Q was asked in 2015 as well except 4th option was different. [Q. 236]

259. The establishment of „Payment Banks‟ is being allowed in India to promote financial
inclusion. Which of the following statements is/are correct in this context?
1. Mobile telephone companies and supermarket chains that are owned and controlled by
residents are eligible to be promoters of Payment Banks.
2. Payment Banks can issue both credit cards and debit cards.
3. Payment Banks cannot undertake lending activities.
Select the correct answer using the code given below.

A. 1 and 2 only
B. 1 and 3 only
C. 2 only
D. 1, 2 and 3
Ans. B

PAYMENT BANKS
https://cleartax.in/g/terms/payment-
banks#:~:text=Payment%20banks%20can%20take%20deposits,of%20savings%20and%20current%
20accounts.

 A payments bank (Airtel Payments Bank, India Post Payments Bank, etc.) is like any other
bank, but operating on a smaller or restricted scale.
 Credit risk is not involved (cannot advance loans or issue credit cards.)
 Demand deposits only (not time deposits)
 Cannot set up subsidiaries to undertake non-banking financial services activities.
 Can take deposits upto ₹2 lakh per account.
 Est. based on Dr. NACHIKET MOR committee
 Benefits: Expansion of rural banking, access to diversified services, social & financial
inclusion are some of the benefits.
 Challenges: Lack of customer awareness, lack of incentives for agents, lack of infrastructure,
technological issues are some of the challenges.
Note

There are two kinds of banking licences that are granted by the Reserve Bank of India - universal
bank licence and differentiated bank licence.
Payments bank comes under a DIFFERENTIATED BANK LICENCE since it cannot offer all the
services that a commercial bank offers. In particular, a payments bank cannot lend.
Objectives

 Financial inclusion by providing small savings accounts and payments/remittance

https://t.me/prelimbits
300

services to migrant labour workforce, low income households, small businesses, other
unorganised sector entities and other users.
Scope of Activities

 Allowed to invest into government securities such as SLR / T-Bilss. (minimum 75%)
 Need to hold maximum 25% in SCBs as deposit in Current or Fixed.
 They CANNOT accept NRI deposits.
Eligible Promoters

 Existing non-bank Pre-paid Payment Instrument (PPI) issuers;


 Other entities such as : individuals/professionals , NBFCs , telephone companies,
Supermarket chains, companies, Public sector, real sector cooperatives (Asked in pre)
 A promoter/promoter group can have a joint venture with an existing scheduled
commercial bank to set up a payments bank.
 Scheduled commercial banks can take equity stake in a payments bank to the extent
permitted under the Banking Regulation Act, 1949. [The BR Act, 1949 is a legislation in India
that regulates all banking firms in India]
Regulation

 PB are registered as a public limited company under the Cos Act, 2013. It is governed by
the provisions of the BR Act, 1949; RBI Act, 1934; FEMA, 1999, Payment and Settlement
Systems Act, 2007, other relevant Statutes and Directives.
 They need to maintain a Cash Reserve Ratio (CRR).
260. With reference to `IFC Masala Bonds‟, sometimes seen in the news, which of the
statements given below is/are correct?
1. The International Finance Corporation, which offers these bonds, is an arm of the World
Bank.
2. They are the rupee-denominated bonds and are a source of debt financing for the public
and private sector.
Select the correct answer using the code given below.

A. 1 only
B. 2 only
C. Both 1 and 2
D. Neither 1 nor 2
Ans. C

MASALA BONDS
 They are bonds issued outside India by an Indian entity or corporate.
 These bonds are issued in Indian currency than local currency.
 Indian corporates usually issue Masala Bonds to raise funds from foreign investors.
 As it is pegged into Indian currency, if the rupee rates fall, investors bear the risk. The first

https://t.me/prelimbits
301

Masala bond was issued in 2014 by IFC for the infrastructure projects in India.
 Investors from outside of India who would like to invest in Indian assets can invest in Masala
bonds.
 Indian entities like HDFC, NTPC and Indiabulls Housing have raised funds via Masala Bonds.
261. Which of the following is/are included in the capital budget of the Government of
India?
1. Expenditure on acquisition of assets like roads, buildings, machinery, etc.
2. Loans received from foreign governments
3. Loans and advances granted to the States and Union Territories
Select the correct answer using the code given below.

A. 1 only
B. 2 and 3 only
C. 1 and 3 only
D. 1, 2 and 3
Ans. D

 The capital budget refers to an account of assets and liabilities of the government that
includes capital receipts and capital expenditures.
 Capital expenditure components are,
o Examples: Expenditure on the acquisition of land, building, machinery, equipment,
creating assets such as roads and hospitals, repayment of government
borrowings, loans, and advances by the central government to state and union
territory governments, etc. Hence statement 1 and 3 is correct.
 Capital receipts components are,
o Examples: Loan borrowings, disinvestments, funds received from the issue of shares
or debentures, etc. Hence statement 2 is correct.
262. Recently, which one of the following currencies has been proposed to be added to
the basket of the IMF‟s SDR? [From Current Affairs 2015-16]
A. Rouble
B. Rand
C. Indian Rupee
D. Renminbi
Ans. D

SPECIAL DRAWING RIGHTS (SDR)


 Created by the IMF in 1969

 Members would deposit currencies in the proportion of quotas allotted to them (depending
on size of their economy, openness etc) = will earn interest on their deposits.

 IMF would lend this money to a member facing balance of payment crisis.

https://t.me/prelimbits
302

 IMF would allot an artificial currency / accounting unit called SDR to the to its members on
their deposits. The value of five major currencies

o US DOLLAR

o EURO

o CHINESE RENMINBI (RMB)

o JAPANESE YEN

o BRITISH POUND STERLING

 SDR can be traded among the members to settle their Balance of Payment Transactions /
Crisis.

 The SDR is neither a currency nor a claim on the IMF. Rather, it is a potential claim on the
freely usable currencies of IMF members.

 India is 8th largest quota holder after USA (~18%), Japan (~7%), China (~6%)...
Member's voting power is related directly to their quotas.

 The SDR currency value is calculated daily (except on IMF holidays or whenever the
IMF is closed for business) and the valuation basket is reviewed and adjusted every
five years.

263. With reference to the International Monetary and Financial Committee (IMFC),
consider the following statements :
1. IMFC discusses matters of concern affecting the global economy and advises the
International Monetary Fund (IMF) on the direction of its work.
2. The World Bank participates as an observer in IMFC‘s meetings.
Which of the statements given above is/are correct?

A. 1 only
B. 2 only
C. Both 1 and 2
D. Neither 1 nor 2
Ans. C

IMFC
 The IMFC advises and reports to the IMF Board of Governors on the supervision and
management of the international monetary and financial system, including on responses to
unfolding events that may disrupt the system.
 The IMFC usually meets twice a year.
 A number of international institutions, including the World Bank, participate as observers
in the IMFC's meetings.

https://t.me/prelimbits
303

264. Which of the following best describes the term „import cover‟, sometimes seen in
the news?
A. It is the ratio of value of imports to the Gross Domestic Product of a country
B. It is the total value of imports of a country in a year
C. It is the ratio between the value of exports and that of imports between two countries
D. It is the number of months of imports that could be paid for by a country‘s international
reserves
Ans. D

IMPORT COVER
 It measures the number of months of imports that could be paid for by a country's
international reserves.
 It is an important indicator of the stability of the currency and a minimum of eight to ten
months of import cover is essential for the stability of a currency.
 Import cover avoids a BoP crisis by taking early preventive action.
 India Foreign Exchange Reserves equaled 8.3 Months of Import in Dec 2022, compared
with the ratio of 8.4 in the previous month.

Prelims 2017
265. Which of the following has/have occurred in India after its liberalization of
economic policies in 1991?
1. Share of agriculture in GDP increased enormously.
2. Share of India's exports in world trade increased.
3. FDI inflows increased.
4. India's foreign exchange reserves increased enormously.
Select the correct answer using the codes given below:

A. 1 and 4 only
B. 2, 3 and 4 only
C. 2 and 3 only
D. 1, 2, 3 and 4
Ans. B

INDIA AFTER ITS LIBERALIZATION


 India's agriculture sector has shown a gradual decline in contribution to the Indian
economy post-reform.
o Agriculture, India‘s traditional occupation, now contributes only about 15% to the
GDP, from 29% in 1991. Hence, statement 1 is not correct.
o The opening up of the economy has led to a rapid increase in foreign direct
investment (FDI -> by 150%) and foreign exchange reserves (by 20 times).
 The foreign investment, which includes foreign direct investment and foreign institutional

https://t.me/prelimbits
304

investment, has increased from about US $ 100 million in 1990-91 to the US $ 150 billion
in 2003-04. Hence, statements 3 and 4 are correct.
 Reforms in Industry:
o India is seen as a successful exporter of IT software, auto parts, textiles,
and engineering goods in the reform period. Hence, statement 2 is correct.
266. Consider the following statements:
1. The Standard Mark of Bureau of Indian Standards (BIS) is mandatory for automotive tyres
and tubes.
2. AGMARK is a quality Certification Mark issued by the Food and Agriculture Organisation
(FAO).
Which of the statements given above is/are correct?

A. 1 only
B. 2 only
C. Both 1 and 2
D. Neither 1 nor 2
Ans. A

THE STANDARD MARK OF BUREAU OF INDIAN STANDARDS (BIS) IS COMPULSORY FOR


 Certain types of electronics and IT goods
 Cement
 Household electrical products
 Food products
 Steel materials.
 All types of automotive tyres and tubes. So statement 1 is correct.
AGMARK VS FSSAI
 AGMARK is a quality Certification Mark issued by the Directorate of Marketing and
Inspection, an agency of the Government of India. So statement 2 is incorrect.
 AGMARK is a certification mark employed on agricultural products.
 FSSAI is a body under the Food safety and standard act of 2006 for inspecting the quality
of food products.
 FSSAI's work is the inspection of Agency whereas AGMARK's role is the certification.
267. With reference to 'Quality Council of India (QCI)', consider the following statements:
[2017-I]
1. QCI was set up jointly by the Government of India and the Indian Industry.
2. Chairman of QCI is appointed by the Prime Minister on the recommendations of the
industry to the Government.
Which of the above statements is/are correct?

A. 1 only
B. 2 only

https://t.me/prelimbits
305

C. Both 1 and 2
D. Neither 1 nor 2
Ans. C

QUALITY COUNCIL OF INDIA (QCI)


 Quality Council of India (QCI) was set up jointly by the Government of India and the
Indian Industry represented by the three premier industry associations i.e. Associated
Chambers of Commerce and Industry of India (ASSOCHAM), Confederation of Indian
Industry (CII) and Federation of Indian Chambers of Commerce and Industry (FICCI), to
establish and operate national accreditation structure and promote
quality through National Quality Campaign. (Hence statement 1 is correct)
 QCI is registered as a non-profit society with its own Memorandum of Association.
 QCI is governed by a Council comprising of 38 members including the Chairman and
Secretary-General.
 The Council has an equal representation of Government, Industry, and other Stakeholders.
 The Chairman of QCI is appointed by the Prime Minister on the recommendation of the
industry to the government. (Hence statement 2 is correct)
 The Department of Industrial Policy & Promotion, Ministry of Commerce & Industry, is the
nodal ministry for QCI.
268. Consider the following statements: [2017-I]
1. National Payments Corporation of India (NPCI) helps in promoting the financial inclusion in
the country.
2. NPCI has launched RuPay, a card payment scheme.
Which of the statements given above is/are correct?

A. 1 only
B. 2 only
C. Both 1 and 2
D. Neither 1 nor 2
Ans. C

NATIONAL PAYMENTS CORPORATION OF INDIA


 It is an umbrella organisation for all retail payments systems in India.
 It was set up [2008] with the guidance and support of the Reserve Bank of India (RBI)
and Indian Banks‘ Association (IBA) under the P&SS Act 2007, for creating a robust
Payment & Settlement Infrastructure in India
 NPCI has been incorporated as a ―Not for Profit‖ Company under the provisions of Section
25 of Companies Act 1956 (now Section 8 of Companies Act 2013),
 Objectives:
o To consolidate and integrate the existing multiple systems into a nation-wide
uniform and standard business process for all retail payment systems.

https://t.me/prelimbits
306

o To facilitate an affordable payment mechanism to benefit the common man across


the country and propel financial inclusion.
 Some of the payment services of NPCI are as follows- NFS, IMPS, AePS, CTS, RuPay, UPI,
Bharat BillPay, NETC, BHIM, BharatQR, BHIM Aadhaar Pay.
NPCI Products
BBPS Bharat Bill Payment system for automated monthly payments of utility bills (gas,
electricity etc.)
NACH National Automated Clearing House (NACH) for Banks, Financial Institutions,
Corporates and Government a web based solution to facilitate interbank, high
volume, electronic transactions which are repetitive and periodic in nature. NACH
System can be used for making bulk transactions towards distribution of subsidies,
dividends, interest, salary, pension etc.
NFS National Financial switch runs the ATM network, IMPS, UPI and BHIM
UPI (2016) Full Form: Unified Payment Interface
It’s a technology for building digital payment apps based on IMPS with following
features:
 QR Scan & Pay to merchants.
 Invoice in the inbox.
 QR authenticity checker – whether merchant verified or not?
 We can link Current Account (CA) Savings Account (SA) & Overdraft Account for
direct transfer of money without storing in ‘wallet’ first. (unlike PayTM)
BHIM (2016) Full form: Bharat Interface for Money
 It works on Android, iOS/Apple mobile phones as an APP, and even on non-smart
(=basic feature) phones using *99# USSD- Unstructured Supplementary Service
Data.
 Bank to bank / peer to peer transaction using mobile phone (xyz@upi).
 App has 3 factor authentication system.
BharatQR While Mobikwick QR code may not work with Phonepe app (interoperability issue),
(2016) But the BharatQR Quick Response code works with all UPI based apps & BHIM app.
RuPay Rupee + Payment = RuPay card is world’s 7th payment gateway
(2016)
AEPS  Useful for Direct Benefit Transfer into beneficiary’s account for MNREGA
wages, LPG subsidy, scholarship etc.
 It’s also required for Bankmitra-MicroATM system.
NETC National Electronic Toll Collection provides backend technological support to the
FASTag toll collection.

269. Which of the following is a most likely consequence of implementing the 'Unified
Payments Interface (UPI)'? [2017-I]
A. Mobile wallets will not be necessary for online payments.
B. Digital currency will totally replace the physical currency in about two decades.
C. FDI inflows will drastically increase.
D. Direct transfer of subsidies to poor people will become very effective.
Ans. A

https://t.me/prelimbits
307

UPI

 UPI allows you to pay directly from your bank account to different
merchants without the hassle of typing your card details, or net banking/wallet
password. Hence option 1 is correct.
 UPI is a payment system that allows money transfer between any two bank accounts by
using a smartphone.
 Unified Payments Interface (UPI) is an instant real-time payment system developed by NPCI
to facilitating inter-bank transactions through mobile phone.
270. Which of the following statements is/are correct regarding the Monetary Policy
Committee (MPC)? [2017-I]
1. It decides the RBI's benchmark interest rates.
2. It is a 12-member body including the Governor of RBI and is reconstituted every year.
3. It functions under the chairmanship of the Union Finance Minister.
Select the correct answer using the code given below:

A. 1 only
B. 1 and 2 only
C. 3 only
D. 2 and 3 only
Ans. A

MONETARY POLICY

 Monetary policy refers to the policy of the central bank in matters of interest rates,
money supply and availability of credit.
 It is through the monetary policy, RBI controls inflation in the country.
 RBI uses various monetary instruments like REPO rate, Reverse RERO rate, SLR, CRR etc to
achieve its purpose.
 This responsibility is explicitly mandated under the Reserve Bank of
India Act, 1934.
Inflation Targeting

 Inflation targeting is a monetary policy in which a central bank


sets a specific target for Medium-Term Inflation Rate and
declares the target for inflation to the public.

 Mostly, the monetary policy can support the economy's long-


term growth by conserving price stability.

 The central bank uses its primary short-term monetary weapon,


the interest rates.

 An inflation-targeting central bank can boost or lower interest


rates, respectively, based on above-target or under-target

https://t.me/prelimbits
308

inflation.

 The conventional ideology is that rising interest rates typically cools the economy to
reign during inflation; lowering interest rates generally speeds up the economy and
thus increases inflation.

 Canada, New Zealand, and the UK were the first three countries to introduce full-
fledged inflation targeting in the early 1990s. However, Germany had introduced many
elements of inflation targeting elements earlier.

Inflation Targeting in India

 In India, the Reserve Bank of India undertook inflation targeting in August 2016. The
decision was taken after India had ~10% inflation rate for around five years.

 The amendment to Reserve Bank of India Act, 1934, effective in June 2016, made way for
a Flexible Inflation Targeting (FIT) framework in India by defining the primary objective
of monetary policy as Maintaining Price Stability together with the objective of
growth.

 To operationalise this command, the Government of India notified a medium-term


inflation target of 4%, with a band of +/- 2% between August 2016 and March 2021 &
again extended this for another 5 Years till 2026.

 The inflation target is fixed in terms of all-India CPI-Combined issued by the Central
Statistics Office (CSO) or NSO.

Monetary Policy Committee‘s Inflation report

https://www.thehindu.com/business/Economy/for-first-time-rbis-rate-setting-panel-to-discuss-
inflation-report-this-week/article66072848.ece

 Failure to keep inflation between 2-6% for three straight quarters will require RBI
Governor to explain to FinMin why target was missed, and steps MPC will take to
achieve it.

 For the first time since the implementation of the monetary policy framework in 2016, the
Reserve Bank will submit an inflation report to the government.

 For three consecutive quarters beginning January 2022, RBI has failed to keep the retail
inflation rate below 6%.

Failure to Maintain Inflation Target: (Section 45ZN)

 Factors that constitute failure to achieve the inflation target (set by Govt.)

o The average inflation is more than the upper tolerance level of the inflation target
for any three consecutive quarters; or

o The average inflation is less than the lower tolerance level for any three
consecutive quarters.

https://t.me/prelimbits
309

 When the Bank fails to meet the inflation target, it shall summit a report to the Central
Government stating→

o The reasons for failure to achieve the inflation target.

o Remedial actions proposed to be taken by the Bank and

o An estimate of the time-period within which the inflation target shall be achieved
pursuant to timely implementation of proposed remedial actions.

Main Objectives of Fiscal Policy in India


 To maintain and achieve full employment.
 To stabilize the price level.
 To stabilize the growth rate of the economy.
 To maintain equilibrium in the Balance of Payments.
 To promote the economic development of underdeveloped countries
MONETARY POLICY COMMITTEE
 The RBI has a Government-Constituted Monetary Policy Committee (MPC) which is
tasked with framing monetary policy (policy rates) to achieve the inflation
target.
o Policy (Repo) Rate : repo rate is used to policy rate bcuz its changes transmit
through the money market to the entire financial system, which, in turn, influences
aggregate demand. (key determinant of inflation and growth)

 It has been instituted (in 2016) by the Central Government of India under Section 45ZB
of the RBI Act.
 Decision of the Monetary Policy Committee shall be binding on the Bank.

Selection and term of members

 Selection: The government nominees to the MPC will be selected by a Search-cum-


Selection Committee under Cabinet Secretary with RBI Governor and Economic Affairs
Secretary and three experts in the field of economics or banking or finance or monetary
policy as its members.

https://t.me/prelimbits
310

 Term: 4 yrs & NO reappointment.


How decisions are made?

 Decisions will be taken by majority


 RBI governor will have a casting vote in case of a tie.
Quorum
 Meeting quorum 4 members, incl. Governor.
 Legally required to hold minimum 4 meetings in a year

 In practice, they meet every two months to decide bi-monthly monetary policy
updates.
Various Policy Stances of RB

Accommodative  An accommodative stance means the CENTRAL BANK is prepared to


expand the money supply to boost economic growth.
 The central bank, during an accommodative policy period, is willing to
cut the interest rates. A rate hike is ruled out.
 The central bank typically adopts an accommodative policy when
growth needs policy support and inflation is not the immediate
concern.
Neutral  A ‗neutral stance‘ suggests that the central bank can either cut rate or
increase rate.
 This stance is typically adopted when the policy priority is equal on
both inflation and growth.
Hawkish Stance  A hawkish stance indicates that the central bank‘s top priority is to
keep the inflation low.
 During such a phase, the central bank is willing to hike interest rates
to curb money supply and thus reduce the demand.
 A hawkish policy also indicates tight monetary policy.
Calibrated  This means the central bank may not go for a rate increase in every
Tightening policy meeting but the overall policy stance is tilted towards a rate
hike.
 This can happen outside the policy meetings as well if the situation
warrants.

271. What is the purpose of setting up of Small Finance Banks (SFBs) in India? [2017-I]
1. To supply credit to small business units
2. To supply credit to small and marginal farmers
3. To encourage young entrepreneurs to set up business particularly in rural areas.
Select the correct answer using the code given below:

A. 1 and 2 only
B. 2 and 3 only
C. 1 and 3 only
D. 1, 2 and 3

https://t.me/prelimbits
311

Ans. A

SMALL FINANCE BANK SFB


The small finance bank will primarily undertake basic banking activities of acceptance of deposits
and lending to unserved and underserved sections including small business units, small and
marginal farmers, micro and small industries and unorganised sector entities.
What they can do? What they cannot do?
 Take small deposits and disburse  Lend to big corporates and groups.
loans.  Cannot open branches with prior RBI
 Distribute mutual funds, insurance approval for first five years.
products and other simple third-party  Other financial activities of the
financial products. promoter must not mingle with the
 Lend 75% of their total adjusted net bank.
bank credit to priority sector.  It cannot set up subsidiaries to undertake
 Maximum loan size would be 10% of non-banking financial services activities.
capital funds to single borrower, 15% to  Cannot be a business correspondent of
a group. any bank.
 Minimum 50% of loans should be up to
25 lakhs
The guidelines they need to follow

 Promoter must contribute minimum 40% equity capital and should be brought down to
30% in 10 years.
 Minimum paid-up capital would be Rs 100 cr.
 Capital adequacy ratio should be 15% of risk weighted assets, Tier-I should be 7.5%.
 Foreign shareholding capped at 74% of paid capital, FPIs cannot hold more than 24%.
 Priority sector lending requirement of 75% of total adjusted net bank credit.
 50% of loans must be up to Rs 25 lakh.
Eligibility conditions to get SFB license ‗On Tap [No need to wait for notification]

 Minimum 200 crore capital.


 resident individuals/professionals with ten years of experience in banking and finance;
 5 year/> old companies owned by Indian residents
 Existing NBFCs, MFIs, local area banks and payments banks.
 Urban cooperative banks (UCB) allowed to convert into SFB but capital norms slightly
different.
 2019-Dec: Even Payment banks can convert into SFB, after 5 years of operation
272. Who among the following can join the National Pension System (NPS)? [2017-I]
A. Resident Indian citizens only
B. Persons of age from 21 to 55 only
C. All State Government employees joining the services after the date of notification by the

https://t.me/prelimbits
312

respective State Governments


D. All Central Government employees including those of Armed Forces joining the services on
or after 1st April, 2004
Ans. C

PENSION FOR GOVT EMPLOYEES & MIDDLE CLASS → NPS


 NPS is a government-sponsored pension scheme. It was launched in January 2004 for
government employees.
 It was extended to all citizens of Indian on a voluntary basis from May 2009
and to corporates in December 2011 and to Non-Resident Indians in
October 2015.
 PFRDA is the statutory authority established by an enactment of the Parliament, to
regulate, promote and ensure orderly growth of the NPS and pension schemes to which
this Act applies.
 The scheme allows subscribers to contribute regularly in a pension account during their
working life.
 On retirement, subscribers can withdraw a part of the corpus in a lump sum and use the
remaining corpus to buy an annuity to secure a regular income after retirement.
 NPS subscribers have PRAN: Permanent Retirement Account Number, Just like EPFO
subscriber has UAN Number.
 ES20: in NPS: # of State Govt employees >> union govt employees. And within State govt
employees: UP > Madhya Pradesh >Raj > Maharashtra.
Who can join NPS?

 Any Indian citizen between 18 and 60 years can join NPS.


 The only condition is that the person must comply with know your customer (KYC) norms.
 An NRI can join NPS. However, the account will be closed if there is a change in the
citizenship status of the NRI.
 Now, any Indian citizen, resident or non-resident and OCIs are eligible to join NPS till the
age of 65 years.
 Hindu Undivided Family (HUFs) are not eligible for opening of NPS accounts.
273. What is/are the most likely advantages of implementing 'Goods and Services Tax
(GST)'? [2017-I]
1. It will replace multiple taxes collected by multiple authorities and will thus create a single
market in India.
2. It will drastically reduce the 'Current Account Deficit' of India and will enable it to increase
its foreign exchange reserves.
3. It will enormously increase the growth and size of economy of India and will enable it to
overtake China in the near future.
Select the correct answer using the code given below:

A. 1 only

https://t.me/prelimbits
313

B. 2 and 3 only
C. 1 and 3 only
D. 1, 2 and 3
Ans. A

CASCADING EFFECT OF TAXES


 Cascading effect is when there is a tax on tax levied on a product at every step of the
sale.
 The tax is levied on a value that includes tax paid by the previous buyer, thus, making the
end consumer pay ―tax on already paid tax‖.

 Breakdown the ₹132 paid by the final customer: 132=100+10+10+11+1


 132 = 100 (price of original product)+10 (tax paid by retailer to wholesaler)+10 (as retailer‘s
profit margin)+11 (tax paid by customer to buy from retailer) + 1**. –
o 1** this one rupee is 10% of 10(tax paid by retailer to wholesaler).
 So, it‘s ―TAX on TAX paid at previous stage‖ / cascading effect of tax on the end-
customer.
 Then, both buyer and seller will prefer to do transaction without bills, to entirely avoid
tax liability and its cascading effect → Govt.‘s revenue collection ↓, Fiscal deficit ↑, black
money ↑
VAT

 Value Added Tax (VAT) is an indirect value added tax which was introduced into Indian
taxation system on 1st April 2005.
 A value-added tax (VAT) is a consumption tax levied on a commodity whenever it adds
value at any point in the supply chain, from production to sale. The amount of VAT that
the consumer pays is based on the cost of the product, minus any previously taxable costs
of products used in the product.
 As a taxation concept, VAT replaced Sales Tax. VAT was introduced to make India a
single integrated market. However, it was introduced at state-level. On 2nd June 2014,
VAT was implemented in all states and union territories of India, except Andaman and
Nicobar Islands and Lakshadweep Islands.
 For interstate supplies, CST or Central Sales Tax was imposed. CST applies on the sale of
goods levied by the Central Government. It is collected and retained by the state where the
tax is collected.
Disadvantages of VAT

 CASCADING EFFECT of taxes


 It was not possible to claim Input Tax Credit (ITC) on service under VAT
 DIFFERENT VAT RATES in different states

https://t.me/prelimbits
314

 Different VAT laws in every states


 Input of CST cannot be adjusted against VAT and vice versa
To remove the Cascading Effect GST was introduced in 2016.

GST IMPLEMENTATION
 GST is a value-added tax levied on most goods and services sold for domestic
consumption. The GST is paid by consumers, but it is remitted to the government by the
businesses selling the goods and services.
Main Features of GST

 Applicable On supply side: GST is applicable on ‗supply‘ of goods or services as against


the old concept on the manufacture of goods or on sale of goods or on provision of
services.
 Destination based Taxation: GST is based on the principle of destination-based
consumption taxation as against the earlier principle of origin-based taxation.
 Dual GST: It is a dual GST with the Centre and the States simultaneously levying tax on
a common base. GST to be levied by the Centre is called Central GST (CGST) and that to
be levied by the States is called State GST (SGST).
o Import of goods or services would be treated as inter-state supplies and would be
subject to Integrated Goods & Services Tax (IGST) in addition to the applicable
customs duties.
 GST rates to be mutually decided: CGST, SGST & IGST are levied at rates to be mutually
agreed upon by the Centre and the States.
o The rates are notified on the recommendation of the GST Council.
 Multiple Rates: Initially GST was levied at four rates viz. 5%, 12%, 16% and 28%. The
schedule or list of items that would fall under these multiple slabs are worked out by the
GST council.
Legislative Basis Of GST

 Bill was first introduced in 2014 as The Constitution 122nd Amendment) Bill.
 The bil was passed as The Constitution (101st Amendment) Act, 2016
 Its provisions
o Central GST to cover Excise duty, Service tax etc, State GST to cover VAT, luxury tax
etc.
o Integrated GST to cover inter-state trade. IGST per se is not a tax but a system to
coordinate state and union taxes.
o Article 246A – States have power to tax goods and services.
GST Council

 Article 279A - GST Council to be formed by the President to administer & govern GST.
 It's Chairman is Union Finance Minister of India with ministers nominated by the state
governments as its members.
 The council is devised in such a way that the centre will have 1/3rd voting power and the

https://t.me/prelimbits
315

states have 2/3rd.


 The decisions are taken by 3/4th majority.
GST Compensation Scheme

The Centre has extended the time for levy of GST compensation cess by almost four years till
March 31, 2026.
As per the Goods and Services Tax (Period of Levy and Collection of Cess) Rules, 2022, notified by
the Finance Ministry, the compensation cess would continue to be levied from July 1, 2022, to
March 31, 2026
 The GST Compensation Act, 2017 guaranteed States that they would be compensated
for any loss of revenue in the first five years of GST implementation, until 2022, using a
cess levied on sin and luxury goods.
 This Act assumed that the GST revenue of each State would grow at 14% every year,
from the amount collected in 2015-16, through all taxes subsumed by the GST.
 A State that had collected tax less than this amount in any year would be
compensated for the shortfall.
 The amount would be paid every two months based on provisional accounts, and adjusted
every year after the State‘s accounts were audited by the Comptroller and Auditor General.
 This scheme is valid for five years, i.e., March 31, 2026.
Exemptions under GST

 Custom duty will be still collected along with the levy of IGST on imported goods.
 Petroleum and tobacco products are currently exempted.
 Excise duty on liquor, stamp duty and electricity taxes are also exempted.

274. With reference to the 'Prohibition of Benami Property Transactions Act, 1988 (PBPT
Act)', consider the following statements: [2017-I]
1. A property transaction is not treated as a benami transaction if the owner of the property is
not aware of the transaction.
2. Properties held benami are liable for confiscation by the Government.
3. The Act provides for three authorities for investigations but does not provide for any
appellate mechanism.
Which of the statements given above is/are correct?

A. 1 only
B. 2 only
C. 1 and 3 only
D. 2 and 3 only
Ans. B

BENAMI TRANSACTIONS (PROHIBITION) AMENDMENT ACT 2016


 The Act amended the Original Act (Benami Transaction (Prohibition) Act 1988) and

https://t.me/prelimbits
316

renamed it as Prohibition of Benami Property Transaction Act, 1988.

 The Act defines a benami transaction as a transaction where →

o a property is held by or transferred to a person, but has been provided for or


paid by another person.

o the transaction is made in a fictitious name

o the owner is not aware of denies knowledge of the ownership of the property,

o the person providing the consideration for the property is not traceable.

Appellate Tribunal

 The Act provides for an Appellate Tribunal to hear appeals against any orders passed by
the Adjudicating Authority.

o Appeals against the orders of the Appellate Tribunal will lie to the high court.

 The special court should conclude the trial within six months from the date of filing of
the complaint.

Authorities

 The Act established four authorities to conduct inquiries or investigations regarding


benami transactions →

o Initiating Officer

o Approving Authority

o Administrator

o Adjudicating Authority

 If an Initiating Officer believes that a person is a benamidar, he may issue a notice to that
person.

o The Initiating Officer may hold the property for 90 days from the date of issue of
the notice, subject to permission from the Approving Authority.

o At the end of the notice period, the Initiating Officer may pass an order to
continue holding the property

 If an order is passed to continue holding the property, the Initiating Officer will refer the
case to the Adjudicating Authority

o The Adjudicating Authority will examine all documents and evidence relating to the
matter and then pass an order on whether or not to hold the property as
benami.

o Based on an order to confiscate the Benami property, the Administrator will


receive and manage the property in a manner and subject to conditions as
prescribed.

https://t.me/prelimbits
317

 The amended law empowers the specified authorities to provisionally attach benami
properties which can eventually be confiscated.

Penalty

 If a person is found guilty of the offence of the Benami transaction by the competent
court, he shall be punishable with rigorous imprisonment for a term not less than one
year but which may extend to 7 years.

 He shall also be liable to a fine which may extend to 25% of the fair market value of the
property.

275. Consider the following statements: [2017-I]


1. Tax revenue as a per cent of GDP of India has steadily increased in the last decade.
2. Fiscal deficit as a per cent of GDP of India has steadily increased in the last decade.
Which of the statements given above is/are correct?

A. 1 only
B. 2 only
C. Both 1 and 2
D. Neither 1 nor 2
Ans. D

276. What is the aim of the programme 'Unnat Bharat Abhiyan'? [2017-I]
A. Achieving 100% literacy by promoting collaboration between voluntary organizations and
government's education system and local communities.
B. Connecting institutions of higher education with local communities to address development
challenges through appropriate technologies.
C. Strengthening India's scientific research institutions in order to make India a scientific and
technological power.
D. Developing human capital by allocating special funds for health care and education of rural
and urban poor, and organizing skill development programmes and vocational training for
them.
Ans. B

UNNAT BHARAT ABHIYAN


 Unnat Bharat Abhiyan is a flagship program of the Ministry for Human Resource
Development (MHRD).
 It aims to connect the Higher Education Institutions with a set of at least five villages
or local communities to address development challenges through appropriate
technologies so that these institutions can contribute to the economic and social
betterment of these village communities using their knowledge base. Hence option 2 is
correct.
Unnat Bharat Abhiyan 2.0

https://t.me/prelimbits
318

 It is the upgraded version of Unnat Bharat Abhiyan 1.0. It was launched in 2018.
 The scheme is extended to all educational institutes; however, under Unnat Bharat
Abhiyan 2.0 participating institutes are selected based on the fulfilment of certain criteria.
 The technological interventions under the Unnat Bharat Abhiyan cover different subjects
broadly categorized like in the area of sustainable agriculture; water resource management;
artisans, industries and livelihood; basic amenities (infrastructure & services) and rural
energy system.
 This has transformed the living conditions in villages and has been beneficial for rural India.
277. With reference to 'National Investment and Infrastructure Fund', which of the
following statements is/are correct? [2017-I]
1. It is an organ of NITI Aayog.
2. It has a corpus of ` 4, 00,000 crore at present.
Select the correct answer using the code given below:

A. 1 only
B. 2 only
C. Both 1 and
D. Neither 1 nor 2
Ans. D

NIIF is under Department of Economic Affairs

NATIONAL INVESTMENT AND INFRASTRUCTURE FUND (NIIF)


 NIIF is a government-backed entity established to provide long-term capital to the
country‘s infrastructure sector.
 The Indian government has a 49% stake in NIIF with the rest held by foreign and domestic
investors.
 With the Centre‘s significant stake, NIIF is considered India‘s quasi-sovereign wealth fund.
 It was set up in December 2015 as a Category-II Alternate Investment Fund, with fund
of Rs. 40000 Crore
 Its registered office is in New Delhi.
 Across its three funds, it manages over USD 4.3 billion of capital.
 Funds that are currently managed by NIIFL →
o Master Fund: Invests primarily in operating assets in core infrastructure sectors
such as roads, ports, airports, power, etc.
o Fund of Funds (FoF) : Managed by fund managers with experience in
infrastructure and related sectors in India. Green Infrastructure, Mid-Income &
Affordable Housing, Infrastructure Services, and allied sectors are some of the areas
of focus.
o Strategic Opportunities Fund (SoF): SOF has been established with the objective
to provide long-term capital to high-growth future-ready businesses in
India. The fund‘s strategy is to build a portfolio of large entrepreneur-led or

https://t.me/prelimbits
319

professionally managed domestic champions and unicorns.


278. Which of the following are the objectives of 'National Nutrition Mission'? [2017-I]
1. To create awareness relating to malnutrition among pregnant women and lactating
mothers.
2. To reduce the incidence of anemia among young children, adolescent girls and women.
3. To promote the consumption of millets, coarse cereals and unpolished rice.
4. To promote the consumption of poultry eggs.
Select the correct answer using the code given below:

A. 1 and 2 only
B. 1, 2 and 3 only
C. 1, 2 and 4 only
D. 3 and 4 only
Ans. A

NATIONAL NUTRITION MISSION


 National Nutrition Mission is also known as the POSHAN Abhiyaan.
 POSHAN stands for Prime Minister‘s Overarching Scheme for Holistic Nutrition.
 It launched on the occasion of the International Women‘s Day on 8 March 2018 from
Jhunjhunu in Rajasthan.
 Objectives of 'National Nutrition Mission →
o To create awareness relating to malnutrition among pregnant women and lactating
mothers. So statement 1 is correct.
o To reduce the incidence of anaemia among young children, adolescent girls, and
women. So statement 2 is correct.
o To improve maternal and child under-nutrition (children below 3 years) in 200 high
burden districts.
 There is no mention in the report to promote the consumption of millets, coarse cereals,
and unpolished rice. So statement 3 is incorrect.
 Also, there is no mention in the report to promote the consumption of poultry eggs. So
statement 4 is incorrect.
279. What is the purpose of Vidyanjali Yojana'? [2017-I]
1. To enable the famous foreign educational institutions to open their campuses in India.
2. To increase the quality of education provided in government schools by taking help from
the private sector and the community.
3. To encourage voluntary monetary contributions from private individuals and organizations
so as to improve the infrastructure facilities for primary and secondary schools.
Select the correct answer using the code given below:

A. 2 only
B. 3 only
C. 1 and 2 only

https://t.me/prelimbits
320

D. 2 and 3 only
Ans. A

VIDYANJALI YOJANA
 Vidyanjali Yojana is a school volunteer program.
 It is an initiative of the Ministry of Education.
 It was launched to increase the quality of education provided in
government schools by taking help from the private sector and the
community. Hence option 2 is correct.
 Considering the problems related to teaching quality out of various issues faced by
government public schools, Vidyanjali Yojana‘s purpose is to meet those resources
requirements.
 Yojana allows an individual to take part in government reforms and help the government in
tackling issues on quality education and scarcity of teachers across India.
 National Council for Teacher Education (NCTE) established by statute, involves four regional
committees which are responsible to monitor the quality of teacher passing institutes.
 The program being piloted across 21 states which includes Indian states like Assam, Andhra
Pradesh, Bihar, Chhattisgarh, Delhi, Haryana, Himachal Pradesh, Gujarat, Goa, Jammu and
Kashmir, Karnataka, Madhya Pradesh, Maharashtra, Manipur, Odisha, Punjab, Rajasthan,
Telangana, Tripura, Uttarakhand, and Uttar Pradesh.

Prelims 2018
280. Which one of the following best describes the term “Merchant Discount Rate”
sometimes seen in news ?
A. The incentive given by a bank to a merchant for accepting payments through debit cards
pertaining to that bank.
B. The amount paid back by banks to their customers when they use debit cards for financial
transactions for purchasing goods or services.
C. The charge to a merchant by a bank for accepting payments from his customers through
the bank‘s debit cards.
D. The incentive given by the Government, to merchants for promoting digital payments by
their customers through Point of Sale (PoS) machines and debit cards.
Ans. C

MDR
 MDR is the fees that a merchant must pay to his (acquirer) bank for every credit / debit
card transaction.
 MDR fees is shared among 3 parties 1) customer‘s card issuing bank 2) merchant‘s acquiring
bank 3) payment gateway provider.

https://t.me/prelimbits
321

 MDR hurts merchants‘ profit margin, discourages them from adopting Point of Sale (PoS)
terminals (=card swiping machine) → obstacle to digital economy.
Recent Developments

 Recently [2020], the government has mandated that neither the customers nor the
merchants will have to pay the Merchant Discount Rate(MDR) while transacting
digital payments using BHIM UPI, UPI QR Code, Aadhaar Pay, Debit Cards, NEFT,
RTGS, among others.
 RBI and Banks will absorb this burden.
281. With reference to the governance of public sector banking in India, consider the
following statements
1. Capital infusion into public sector banks by the Government of India has steadily increased
in the last decade.
2. To put the public sector banks in order, the merger of associate banks with the parent State
Bank of India has been affected.
Which of the statements given above is/are correct ?

A. 1 only
B. 2 only
C. Both 1 and 2
D. Neither 1 nor 2
Ans. B

 Capital infusion into public sector banks by the Government of India has not steadily
increased in the last decade, there has been a fall in between. Hence, statement 1 is not
correct.

 To put the public sector banks in order, the merger of associate banks with the parent State
Bank of India has been affected.

https://t.me/prelimbits
322

 Merging of banks would help in strengthening the bargaining power of the banks,
reduce operational expenditure, enhance capital efficiency, streamline banking
operations and reduce their NPA burden. Hence, statement 2 is correct.
282. With reference to digital payments, consider the following statements:
1. BHIM app allows the user to transfer money to anyone with a UPI-enabled bank account.
2. While a chip-pin debit card has four factors of authentication, BHIM app has only two
factors of authentication.
Which of the statements given above is/are correct?

A. 1 only
B. 2 only
C. Both 1 and 2
D. Neither 1 nor 2
Ans. A

BHARAT INTERFACE FOR MONEY


 Bharat Interface for Money (BHIM) is a UPI enabled initiative to make easy and quick
payment transactions. (Hence statement 1 is correct)
 It is an Indian mobile payment App developed by the National Payments Corporation of
India (NPCI).
 Chip pin debit card does not have four factor authorization while BHIM app has three
factor authentication. (Hence statement 2 is incorrect)
283. Consider the following statements:
1. Capital Adequacy Ratio (CAR) is the amount that banks have to maintain in the form of their
own funds to offset any loss that banks incur if the account-holders fail to repay dues.
2. CAR is decided by each individual bank.
Which of the statements given above is/are correct?

A. 1 only
B. 2 only
C. Both 1 and 2
D. Neither 1 nor 2
Ans. A

For more refer Q. 231

 The Capital Adequacy Ratio (CAR) is a measurement of a bank's available capital


expressed as a percentage of a bank's risk-weighted credit exposures.
 The Capital Adequacy Ratio, also known as capital-to-risk weighted assets ratio (CRAR),
is used to protect depositors and promote the stability and efficiency of financial
systems around the world.
 It is decided by central banks and bank regulators to prevent commercial banks from
taking excess leverage and becoming insolvent in the process.

https://t.me/prelimbits
323

284. Consider the following statements:


1. The Reserve Bank of India manages and services Government of India Securities but not any
State Government Securities.
2. Treasury bills are issued by the Government of India and there are no treasury bills issued
by the State Governments.
3. Treasury bills offer are issued at a discount from the par value.
Which of the statements given above is/are correct?

A. 1 and 2 only
B. 3 Only
C. 2 and 3 only
D. 1, 2 and 3
Ans. C

GOVERNMENT SECURITY
 Government security (G-Sec) is a tradeable instrument issued by the central government
or state governments. It acknowledges the government‘s debt obligations.
 The G-Secs issuances are managed by the RBI, who on behalf of the Centre, regularly
conducts G-Sec auctions every Friday.
 State Government transactions are carried out by RBI in terms of the agreement entered
into with the State Governments.
 Treasury bills →
o They are short-term debt instruments issued by the Central government. State
Government don't issue treasury bills.
o Treasury bills play a vital role in cash management of the Government.
o Being risk-free, their yields at varied maturities serve as short term benchmarks and
help pricing varied floating-rate products in the market.
 Treasury bills are issued at a discount and redeemed at the face value at maturity.
285. Which one of the following statements correctly describes the meaning of legal
tender money ?
1. The money which is tendered in courts of law to defray the fee of legal cases
2. The money which a creditor is under compulsion to accept in settlement of his claims
3. The bank money in the form of cheques, drafts, bills of exchange, etc.
4. The metallic money in circulation in a country
Ans. B

LEGAL TENDER MONEY


This money cannot be denied in the settlement of the monetary obligation
Limited Legal Tender Money:

 It is compulsory to accept up to a certain limit = COINS


 Example – A sum of 10 can be paid in denominations of 50 paisa coins and the recipient has
https://t.me/prelimbits
324

to legally accept it.


Unlimited Legal Tender Money:

 This money can be used to make any amount of payment


 E.g = BANK NOTES.
The Indian currency system was converted into decimal system by Indian coinage act, 1955,
which was brought into force from 1st April, 1957.
The old system of Rupee, Annas and Paisa was replaced by rupee and paise system. I Rupee = 16
Annas, 1 Annas = 12 Paise

NON-LEGAL TENDER MONEY


 There is no legal compulsion to accept this money. It is also called optional money or
Fiduciary Money (on the basis of trust).
 E.g. – Nepalese currency at India – Nepal border may be used as but the recipient is not
legally bound to accept it.
FIDUCIARY MONEY refers to money backed up by trust between the payer and payee.
Example: Cheques are fiduciary money as these are accepted as a means of payment on the basis
of trust but not on the basis of any order of the government.

286. Which one of the following links all the ATMs in India ? (Pre18 Set-D)
A. Indian banks‘ Association
B. National Securities Depository Limited
C. National Payments Corporation of India
D. Reserve Bank of India
Ans. C

NATIONAL PAYMENTS CORPORATION OF INDIA (NPCI)


 National Payments Corporation of India (NPCI) is the apex authority for maintaining
a nationwide link of all the ATMs in India.
 The NPCI was founded on 2008 for the operation of retail payments in India.
 NPCI exercises this role through its wing IDRBT (Institute for Development of Research
in Banking Technology).
 IDRBT created a massive network called National Financial Switch in the year 2004 for
linking all the ATMs in India.
 The NPCI provides a variety of services those are Rupay: A domestic card scheme, BHIM:
Unified payment interface, Check truncation system and Aadhar enabled payment
system.
287. Consider the following statements
1. The Fiscal Responsibility and Budget Management ( FRBM) Review Committee Report has
recommended a debt to GDP ratio of 60% for the general (combined) government by 2023,
comprising 40% for the Central Government and 20% for the State Governments.
2. The Central Government has domestic liabilities of 21% of GDP as compared to that of 49%

https://t.me/prelimbits
325

of GDP of the State Governments.


3. As per the Constitution of India, it is mandatory for a State to take the Central Government‘s
consent for raising any loan if the former owes any outstanding liabilities to the latter.
Which of the statements given above is/are correct?

A. 1 only
B. 2 and 3 only
C. 1 and 3 only
D. 1, 2 and 3
Ans. C

FISCAL RESPONSIBILITY AND BUDGET MANAGEMENT


 The Fiscal Responsibility and Budget Management (FRBM) Review Committee chaired
by Mr. N.K. Singh submitted its report in January 2017.
 The Committee suggested using debt as the primary target for fiscal policy.
 Debt to GDP ratio of 60% should be targeted with a 40% limit for the center and 20%
limit for the states.
 The targeted debt to GDP ratio should be achieved by 2023. Hence, statement 1 is
correct.
 The Central Government has domestic liabilities of 46.1% of GDP (2016-17) and as a
percent of GDP, whereas States‘ Government liabilities increased to 23.2 % at end-March
2016. Hence, statement 2 is not correct.
 Article 293(1) empowers State Governments to borrow only from domestic sources.
 Further, according to Article 293(3) as long as a State has outstanding borrowings from
the Central Government, it is required to obtain Central Government's prior approval
before incurring debt. Hence, statement 3 is correct.
288. Consider the following items:
items is/are exempted under GST (Goods and Services Tax)?

A. 1 only
B. 2 and 3 only
C. 1, 2 and 4 only
D. 1, 2, 3 and 4
Ans. C

 Previously GST council imposed a 5% rate of GST on hulled grains. However, later on 11th
June 2017, the rate of GST applicable on hulled grains is 0% (nil rate) fixed by GST council
at the introduction of GST in July 2017 is 0%
 Cereal grains hulled falls under GST HSN code chapter 1104.
 Cooked eggs is exempted from paying GST, the rate of GST payable on cooked eggs is
nil rate.
 Fish, crustaceans, molluscs & other aquatic invertebrates in processed, cured or frozen state
are taxable at 5% rate under GST.

https://t.me/prelimbits
326

 Newspapers containing advertising material has 0% GST rate.


 However, ads attract 5% GST, not the newspapers publishing them.
 Newspapers with or without advertisements are exempted from GST.
289. Regarding Money Bill, which of the following statements is not correct?
A. A bill shall be deemed to be a Money Bill if it contains only provisions relating to imposition,
abolition, remission, alteration or regulation of any tax.
B. A Money Bill has provisions for the custody of the Consolidated Fund of India or the
Contingency Fund of India.
C. A Money Bill is concerned with the appropriation of moneys out of the Contingency Fund of
India.
D. A Money Bill deals with the regulation of borrowing of money or giving of any guarantee by
the Government of India.
Ans. C

MONEY BILL
 A money bill cannot be introduced in Rajya Sabha.
 Rajya Sabha can neither reject a Money Bill nor amend it
 The Speaker of the Lok Sabha decides whether the Bill is a Money Bill or not.
 Also, the Speaker‘s decision shall be deemed to be final.
 Article 110 of the constitution deals with Money Bill in India.
 A Money Bill may only be introduced in Lok Sabha, on the recommendation of the
President.
 It must be passed in Lok Sabha by a simple majority
 It may be sent to the Rajya Sabha for its recommendations, which Lok Sabha may reject if it
chooses to.
 If such recommendations are not given within 14 days, it will be deemed to be passed by
Parliament.
 A Bill is said to be a Money Bill if it only contains provisions related to taxation, borrowing
of money by the government, expenditure from or receipt to the Consolidated Fund or
Contingency Fund of India, (Hence option 1, 2 and 4 is correct) and the appropriation of
money out of the consolidated Fund of India (Option 3 is incorrect.).
290. With reference to India‟s decision to levy an equalization tax of 6% on online
advertisement services offered by non-resident entities, which of the following
statements is/are correct?
1. It is introduced as a part of the Income Tax Act.
2. Non-resident entities that offer advertisement services in India can claim a tax credit in their
home country under the ―Double Taxation Avoidance Agreements‖.
Select the correct answer using the code given below:

A. 1 only
B. 2 only

https://t.me/prelimbits
327

C. Both 1 and 2
D. Neither 1 nor 2
Ans. D

DIGITAL SERVICE TAX

 The DST is aimed at ensuring that non-resident, digital service providers pay their fair
share of tax on revenues generated in the Indian digital market.
 India‘s 2% DST is levied on revenues generated from digital services offered in India,
including digital platform services, digital content sales, and data-related services.
 India was one of the first countries in the world to introduce a 6% equalization levy in
2016, but the levy was restricted to online advertisement services (commonly known as
―digital advertising taxes‖ or ―DATs‖).
 In March 2020, it expanded the scope of the existing equalization levy to a range of
digital services that includes e-commerce platforms. Any payment made by non-
residents in connection with an Indian user will now attract a 2% levy.
SIGNIFICANT ECONOMIC PRESENCE
 Indian lawmakers have created scenarios where such foreign corporations are deemed to
have ―business connection‖ in India, thus bringing their Indian operations within the
ambit of the Indian tax system.
 In the Finance Act, 2018, the Indian income tax law was amended to widen the scope of
the existing term ‗business connection‘ to include SEP.
 SEP was defined to mean, inter-alia, transaction of goods and services with any
person in India, including provision of download of data or software in
India, if one of two conditions are satisfied:
o the aggregate of payments arising from such transactions exceeds a specified limit,
or
o the engagement with Indian consumers exceeds a specified number.
GAFA TAX
 The French government‘s ―GAFA‖ tax‖ named after Google, Apple, Facebook, Amazon,
is being introduced to combat attempts by the firms to avoid paying what is considered a
―fair share‖ of taxes in the country.
 OECD used a phrase ‗Tax challenges of digitization‘ to denote problems where digital
services type Multinational Corporation (MNC) are avoiding taxes
291. India enacted The Geographical Indications of Goods (Registration and Protection)
Act, 1999 in order to comply with the obligations to
A. ILO
B. IMF
C. UNCTAD
D. WTO

https://t.me/prelimbits
328

Ans. D

GEOGRAPHICAL INDICATIONS
 Geographical Indications (GI) are one of the eight intellectual property items coming
under WTO‘s TRIPs (Trade-Related Intellectual Property Rights).
 According to the WTO, ―Geographical indications are indications which identify a good as
originating in the territory of a Member, or a region or locality in that territory, where a
given quality, reputation or other characteristics of the good is essentially attributable to its
geographical origin.‖
 India enacted the Geographical Indication of Goods (Registration and Protection) Act,
1999 which came into force with effect from September 15, 2003, to comply with
India‘s obligations under the TRIPS agreement.
 The Controller General of Patents, Designs & Trade Marks (CGPDT), (under the Dept of
Industrial Policy and Promotion of Ministry of Commerce and Industry) is the ‗Registrar of
Geographical indications‘.
 The CGPDT directs and supervises the functioning of the Geographical Indications Registry
(GIR).
292. Consider the following countries :
1. Australia
2. Canada
3. China
4. India
5. Japan
6. USA
Which of the above are among the ‗free-trade partners‘ of ASEAN ?

A. 1, 2, 4 and 5
B. 3, 4, 5 and 6
C. 1, 3, 4 and 5
D. 2, 3, 4 and 6
Ans. C

ASSOCIATION OF SOUTHEAST ASIAN NATIONS


 ASEAN has five free trade agreements (FTAs) with six Dialogue Partners,
namely China, Japan, Republic of Korea, India, and Australia and New Zealand.
 The Association of Southeast Asian Nations (ASEAN) is a regional grouping that promotes
economic, political, and security cooperation among its ten members: Brunei, Cambodia,
Indonesia, Laos, Malaysia, Myanmar, the Philippines, Singapore, Thailand, and Vietnam.
 ASEAN was established on 8 August 1967 in Bangkok.

https://t.me/prelimbits
329

293. As per the NSSO 70th Round “Situation Assessment Survey of Agricultural
Households”, consider the following statements
1. Rajasthan has the highest percentage share of agricultural households among its rural
households.
2. Out of the total agricultural households in the country, a little over 60 percent belong to
OBCs.
3. In Kerala, a little over 60 percent of agricultural households reported to have received
maximum income from sources other than agricultural activities.
Which of the statements given above is/are correct?

A. 2 and 3 only
B. 2 only
C. 1 and 3 only
D. 1, 2 and 3
Ans. C

 According to the NSSO 70th Round "Situation Assessment Survey of Agricultural


Households", Rajasthan had the highest percentage of agricultural households (78.4
percent) among its rural households followed by Uttar Pradesh (74.8 percent) and Madhya
Pradesh (70.8 percent). Hence, statement 1 is correct.
 As per the survey, about 45 percent out of the total agricultural households in the
country belonged to Other Backward Classes (OBC). About 16 percent of agricultural
households were from Scheduled Castes (SC) and 13 percent were from Scheduled
Tribes (ST). Hence, statement 2 is not correct.
 Agricultural activity (cultivation, livestock, and other agricultural activities) was reported to
be the principal source of income for the majority of the households in all the major States,
except Kerala where about 61 percent of the agricultural households reported to
have earned maximum income from sources other than agricultural
activities. Hence, statement 3 is correct.
294. Consider the following:
1. Areca nut
2. Barley
3. Coffee
4. Finger millet
5. Groundnut
6. Sesamum
7. Turmeric
The Cabinet Committee on Economic Affair, has announced the Minimum Support Price for which
of the above?

A. 1, 2, 3 and 7 only
B. 2, 4, 5 and 6 only

https://t.me/prelimbits
330

C. 1, 3, 4, 5 and 6 only
D. 1, 2, 3, 4, 5, 6 and 7
Ans. B

CROPS UNDER MSP

14 kharif crops  Kharif: Grown In Summer / Monsoon Season.


 Paddy, Jowar, Bajra, Maize, Ragi, Arhar, Moong, Urad, Groundnut-
In-Shell, Soyabean, Sunflower, Sesamum, Nigerseed And Cotton;
6 rabi crops  Rabi: grown in winter season.
 Wheat, Barley, Gram, Masur(Lentil), Rapeseed/Mustard And
Safflower
3 commercial  Jute, Copra and Sugarcane.
/cash crops  For Sugarcane, mechanism is different: It requires the sugar mill
companies to pay the minimum Fair and Remunerative Price (FRP)
 fixed by Govt. (Whereas for other 22, Govt itself procures @MSP)

295. Consider the following statements


1. The quantity of imported edible oils is more than the domestic production of edible oils in
the last five years.
2. The Government does not impose any customs duty on all the imported edible oils a special
case.
Which of the statements given above is/are correct

A. 1 only
B. 2 only
C. Both 1 and 2
D. Neither 1 nor 2
Ans. A

 India occupies a prominent position in the world oilseeds industry with a contribution of
around 10% in worldwide production.
 But the demand for edible oils (extracted from oilseeds in addition to palm oil)
is significantly higher than the domestic production, leading to dependence on
imports (60% of requirement). Hence, statement 1 is correct.
 The government imposes customs duty on edible oils to safeguard the interests of the
domestic oil crushing industry.
 The duty on two major edible oils, namely crude sunflower seed oil and crude
canola/rapeseed/mustard is 25 percent, while crude soybean oil attracts 30 percent duty.
Hence, statement 2 is not correct.
296. If a commodity is provided free to the public by the Government, then
A. the opportunity cost is zero.

https://t.me/prelimbits
331

B. the opportunity cost is ignored.


C. the opportunity cost is transferred from the consumers of the product to the tax-paying
public.
D. the opportunity cost is transferred from the consumers of the product to the Government.
Ans. C

OPPORTUNITY COST
 Opportunity cost is what a business owner misses out on when selecting
one option over another.
 Opportunity cost represents the benefits an individual, investor, or business misses out on
when choosing one alternative over another.
 If a commodity is provided free to the public by the Government, then the opportunity cost
is transferred from the consumers of the product to the tax-paying public.
 As per microeconomics, the opportunity cost is zero for free goods such as air and
common goods such as fish/grazing land.
 For public goods such as street lights and defense, the opportunity cost is involved (The
government could have spent that much money on street lights rather than on the military).
So, the opportunity cost is not zero.
297. Despite being a high saving economy, capital formation may not result in significant
increase in output due to
A. weak administrative machinery
B. illiteracy
C. high population density
D. high capital-output ratio
Ans. D

CAPITAL OUTPUT RATIO (COR)


 Capital Output Ratio (COR) is a measure of the percentage increase in capital formation
required to obtain a percentage increase in GDP.
 The capital-output ratio is the relationship between investment and
resulting output over a period of time.
 COR is a measure of capital required for producing one unit of output.
 If capital to output ratio is high then capital formation may not result in a significant
increase in the output.
298. Which of the following is/are the aim/aims of “Digital India” Plan of the Government
of India?
1. Formation of India‘s own Internet companies like China did.
2. Establish a policy framework to encourage overseas multinational corporations that collect
Big Data to build their large data centres within our national geographical boundaries.
3. Connect many of our villages to the Internet and bring Wi-Fi to many of our school, public
places and major tourist centres.

https://t.me/prelimbits
332

Select the correct answer using the code given below

A. 1 and 2 only
B. 3 only
C. 2 and 3 only
D. 1, 2 and 3
Ans. B

DIGITAL INDIA‘
 The ‗Digital India‘ Programme was launched by the Prime Minister on 1st July 2015.
 Digital India aims to provide the much-needed thrust to the nine pillars of growth areas,
namely Broadband Highways, Universal Access to Mobile Connectivity, Public Internet
Access Programme, e-Governance: Reforming Government through Technology, e-Kranti -
Electronic Delivery of Services, Information for All, Electronics Manufacturing, IT for Jobs and
Early Harvest Programmes.
 The programme is centred around three key areas, namely, Digital Infrastructure to every
citizen, Digital services & governance on demand and Digital empowerment of citizens.
 The Government‘s ambitious ―Digital India‖ plan aims to digitally connect all of India's
villages and gram panchayats by broadband internet, promote e-governance and transform
India into a connected knowledge economy. (Hence statement 3 is correct)
299. Increase in absolute and per capita real GNP do not connote a higher level of
economic development, if
A. industrial output fails to keep pace with agricultural output.
B. agricultural output fails to keep pace with industrial output.
C. poverty and unemployment increase.
D. imports grow faster than exports.
Ans. C

 Economic growth means an increase in real national income/national output.


 Economic development means an improvement in the quality of life and living
standards, e.g. measures of literacy, life-expectancy and health care.
 If with economic growth, a country experiences various economic changes such as a
reduction in poverty and unemployment, reduction in income and wealth inequality, an
increase in literacy rate, improvement in health and hygiene, etc, that improve the quality of
life then that is economic development.
300. With reference to the provisions made under the National Food Security Act, 2013
consider the following statements:
1. The families coming under the category of ‗below poverty line (BPL)‘ only are eligible to
receive subsidised grains.
2. The eldest woman in a household, of age 18 years or above, shall be the head of the
household for the purpose of issuance of a ration card.
3. Pregnant women and lactating mothers are entitled to a take-home ration‘ of 1600 calories

https://t.me/prelimbits
333

per day during pregnancy and for six months thereafter.


Which of the statements given above is/are correct?

A. 1 and 2
B. 2 only
C. 1 and 3
D. 3 only
Ans. B

NATIONAL FOOD SECURITY ACT (NFSA), 2013


 Objective → adequate quantity of quality food at affordable prices to people to live a
life with dignity
Key features

 Coverage under TPDS: 50 & 75% of the Urban & rural population respectively.
 Uniform entitlement of 5 kg per person per month. However, the poorest of the poor
households will continue to receive 35 kg per household per month under Antyodaya
Anna Yojana (AAY).
o How wide is NFSA coverage?
 The NFSA covers 67.21% of India‘s population (75% in rural India, 50% in
urban). Out of 81.35 crore accepted (upper limit) beneficiaries, 79.73 crore
(98.01%) have been identified as on June 2.
 There are two type of beneficiaries:
 Antyodaya Anna Yojana households (entitled to 35 kg foodgrains per
household per month) and
 Priority Households (5 kg per person per month). Rice is provided at
Rs 3 per kg, wheat at Rs 2 per kg and coarse grains at Rs 1 per kg.
 Subsidised prices under TPDS and their revision: For a period of three years from the
date of commencement of the Act, Food grains under TPDS will be made available at
subsidised prices of Rs. 3/2/1 per kg for rice, wheat and coarse grains.
 Identification: The identification of eligible households is to be done by States/UTs under
TDPS determined for each State.
 Nutritional Support to women and children: Children in the age group of 6 months to 14
years and pregnant women and lactating mothers will be entitled to meals as per
prescribed nutritional norms under Integrated Child Development Services (ICDS) and
Mid-Day Meal (MDM) schemes. Malnourished children up to the age of 6 have been
prescribed for higher nutritional norms.
o Schedule II of the National Food Security Act lays down nutritional standards for
government food safety programmes like mid-day meal, PM Poshan and
Integrated Child Development Services scheme.

https://t.me/prelimbits
334

o Currently, it quantifies nutrition per meal in terms of calories and protein only,
but the inter-ministerial panel has called for micronutrients to also be taken into
account.
 Maternity Benefit: Pregnant women and lactating mothers will also be receiving
maternity benefit of Rs. 6,000.
 Women Empowerment: For the purpose of issuing of ration cards, eldest woman of the
household of age 18 years or above is to be the head of the household.
 Grievance Redressal Mechanism: Grievance redressal mechanism available at the District
and State levels.
 Transparency and Accountability: Thru social audits and setting up of Vigilance
Committees.
 Food Security Allowance: In case of non-supply of entitled food grains or meals, there is a
provision for food security allowance to entitled beneficiaries.
 Penalty: If the public servant or authority fails to comply with the relief recommended by
the District Grievance Redressal Officer, penalty will be imposed by the State Food
Commission according to the provision.

301. Consider the following statements :


1. As per the Right to Education (RTE) Act, to be eligible for appointment as a teacher in a
State, a person would be required to possess the minimum qualification laid down by the
concerned State Council of Teacher Education.
2. As per the RTE Act, for teaching primary classes, a candidate is required to pass a Teacher
Eligibility Test conducted in accordance with the National Council of Teacher Education
guidelines.
3. In India, more than 90% of teacher‘s education institutions are directly under the State
Governments.
Which of the statements given above is/are correct?

A. 1 and 2
B. 2 only
C. 1 and 3
D. 3 only
Ans. B

RTE ACT, 2009


 In accordance with the provisions of sub-section (1) of section 23 of the Right of
Children to Free and Compulsory Education (RTE) Act, 2009, the National Council for
Teacher Education (NCTE) has laid down the minimum qualifications for a person to be
eligible for appointment as a teacher in class I to VIII, vide its Notification dated August 23,
2010. Hence, statement 1 is not correct.
 One of the essential qualifications for a person to be eligible for appointment as a

https://t.me/prelimbits
335

teacher in any of the primary schools is that he/she should pass the Teacher Eligibility
Test (TET) which will be conducted by the appropriate Government. Hence, statement 2 is
correct.
 From the mid-1960s to 1993, the number of Teacher Education Institutions(TEIs) in
India went up from about 1,200 to about 1,500. After the NCTE was set up, the number of
TEIs exploded to about 16,000 by 2011, of which over 90% are private. So in India, more
than 90% of teacher education institutions are private institutes. Hence, statement 3 is not
correct.
302. With reference to Pradhan Mantri Kaushal Vikas Yojana, consider the following
statements
1. It is the flagship scheme of the Ministry of Labour and Employment.
2. It, among other things, will also impart training in soft skills, entrepreneurship, financial and
digital literacy.
3. It aims to align the competencies of the unregulated workforce of the country to the
National Skill Qualification Framework.
Which of the statements given above is/are correct ?

A. 1 and 3 only
B. 2 only
C. 2 and 3 only
D. 1, 2 and 3
Ans. C

PRADHAN MANTRI KAUSHAL VIKAS YOJANA


 It is the flagship scheme of the Ministry of Skill Development & Entrepreneurship
(MSDE).
 It is implemented by National Skill Development Corporation.
 Under this Scheme, Training and Assessment fees are completely paid by the Government.
 The objective of this Skill Certification Scheme is to enable a large number of Indian youth
to take up industry-relevant skill training that will help them in securing a better livelihood.
 Individuals with prior learning experience or skills will also be assessed and certified under
Recognition of Prior Learning (RPL).
 Apart from providing training according to the National Skills Qualification
Framework (NSQF), Training Centres shall also impart training in Soft Skills,
Entrepreneurship, Financial and Digital Literacy.
 Individuals with prior learning experience or skills shall be assessed and certified under the
Recognition of Prior Learning (RPL) component of the Scheme
 RPL aims to align the competencies of the unregulated workforce of the country to
the NSQF.

https://t.me/prelimbits
336

303. Consider the following statements:


Human capital formation as a concept is better explained in terms of a process, which
enables
1. individuals of a country to accumulate more capital.
2. increasing the knowledge, skill levels and capacities of the people of the country.
3. accumulation of tangible wealth.
4. accumulation of intangible wealth.
Which of the statements given above is/are correct?

A. 1 and 2
B. 2 only
C. 2 and 4
D. 1, 3 and 4
Ans. C

HUMAN CAPITAL FORMATION


 According to the OECD, human capital is defined as: ―the knowledge, skills,
competencies and other attributes embodied in individuals or groups of individuals
acquired during their life and used to produce goods, services or ideas in market
circumstances‖. Hence statement 2 is correct.
 Intangible wealth of a nation comprises of the skilled population, human resource base,
culture, arts etc. Hence statement 4 is correct.
 Human capital formation is the outcome of investments in education, health, on-the-job
training, migration and information.
 As per RBI, Gross capital formation refers to the 'aggregate of gross additions to fixed
assets (that is fixed capital formation) plus change in stocks during the counting
period. Hence statement 1 is incorrect.
 Tangible capital is also GCF is it involves largely infrastructural components. Hence
statement 3 is incorrect
304. Consider the following statements
1. The Food Safety and Standards Act, 2006 replaced the Prevention of Food Adulteration Act,
1954.
2. The Food Safety and Standards Authority of India (FSSAI) is under the charge of Director
General of Health Services in the Union Ministry of Health and Family Welfare.
Which of the statements given above is/are correct?

A. 1 only
B. 2 only
C. Both 1 and 2
D. Neither 1 nor 2
Ans. A

https://t.me/prelimbits
337

FOOD SAFETY AND STANDARDS


 The Food Safety and Standards(FSS) Act, 2006 consolidates various acts & orders that
had earlier handled food-related issues in various Ministries and Departments.
 The acts that were repealed after commencement of FSS Act, 2006 are as follows:
o Prevention of Food Adulteration Act, 1954 Hence, statement 1 is correct.
o Fruit Products Order, 1955
o Meat Food Products Order, 1973
o Vegetable Oil Products (Control) Order, 1947
o Edible Oils Packaging (Regulation) Order 1988
o Milk and Milk Products Order, 1992
 Food Safety and Standards Authority of India (FSSAI) is an autonomous statutory
body established under the Food Safety and Standards Act, 2006.
 The Ministry of Health & Family Welfare, Government of India is the Administrative
Ministry for the implementation of FSSAI.
 Before the formation of FSSAI, from 1954 onwards Prevention of Food Adulteration was part
of the Directorate General of Health Services.
 After having detached from the direct administrative control of the Health Ministry, this
authority is holding independent authority and has attained a special status.
Hence, statement 2 is not correct.
Prelims 2019
305. Which of the following is not included in the assets of a commercial bank in India?
A. Advances
B. Deposits
C. Investments
D. Money at call and short notice
Ans. B

 The key business of the banks is to accept different types of deposits from the public and
then lend these funds to the borrowers.
o This is called Financial intermediation.
 In terms of the banks, the deposits represent the ―liabilities‖ of the banks while loans
advanced and investments made by banks represent their ―assets‖.
 The deposit itself is a liability owed by the bank to the depositor.
 Bank deposits refer to this liability rather than to the actual funds that have been
deposited.
306. Money multiplier in an economy increases with which one of the following?
A. Increase in the cash reserve ratio
B. Increase in the banking habit of the population
C. Increase in the statutory liquidity
D. Increase in the population of the country

https://t.me/prelimbits
338

Ans. B

MONEY MULTIPLIER
 Refers to how an initial deposit can lead to a bigger final increase in the total money
supply.
 For example, if the commercial banks gain deposits of $1 million and this leads to a final
money supply of $10 million. The money multiplier is 10.
 The money multiplier is a key element of the fractional banking system.
o There is an initial increase in bank deposits (monetary base)

o The bank holds a fraction of this deposit in reserves and then lends out the rest.

o This bank loan will, in turn, be re-deposited in banks allowing a further increase
in bank lending and a further increase in the money supply.

 If the Reserve Ratio is higher, then the money multiplier will be lower and the banks
need to keep more reserves. As a result, they will not be able to lend more money to
individuals and businesses.
 Similarly, a lower reserve ratio results in a higher money multiplier which allows a lesser
amount of money to be kept as a reserve and more lending opportunities to the public.
FACTORS AFFECTING MONEY SUPPLY [M1, M3]
M1, M3 Money Supply will increase when:
 When Money multiplier and / or Velocity of money increases.
 When RBI‘s asset side increases e.g. Government borrowing more from RBI using G-sec or
increase in RBI‘s foreign securities.
 With the increase in banking penetration, financial inclusion, formalization of economy,
Boom period, whenever loan demand increases.
 When RBI adopts Cheap / Easy / Dovish / Expansionary monetary policy to combat
deflation.
 Currency Deposit Ratio (CDR) - ratio of (money held by the public) divided by (public‘s
deposit in banks).
o For example, CDR increases during the festive season as people convert deposits to
cash balance for meeting extra expenditure.
 Reserve Deposit Ratio (RDR) - (A commercial bank‘s vault cash) divided by (its deposits
with RBI such as CRR).
o Value cash= banks keep some money with themselves for meeting day to day
withdrawal by depositors & for misc. biz expenses.
307. What was the purpose of the Inter-Creditor Agreement signed by Indian banks and
financial institutions recently?
A. To lessen the Government of India‘s perennial burden of fiscal deficit and current account
deficit

https://t.me/prelimbits
339

B. To support the infrastructure projects of Central and State Governments


C. To act as independent regulator in case of applications for loans of Rs 50 crore or more
D. To aim at faster resolution of stressed assets of Rs 50 crore or more which are under
consortium lending.
Ans. D

PROJECT SASHAKT BY FINANCE MINISTRY (2018)


2018-Jul: Finmin‘s Project Sashakt for PSB-NPA on report by Sunil Mehta [for reco on AMC /
ARC] (PNB CEO).
5 Pronged approach to resolve the NPA problem in a timebound manner:
 Small sized bad loans upto ₹50 cr: SME-resolution within 90 days by bank itself without
approaching NCLT/IP.
 Mid-sized bad loans ₹ 50-500cr: Inter-Creditor Agreement (ICA), 180 days. Banks
themselves should work it out, without approaching NCLT/IP.
 Large size above 500 cr: (Proposed) independent Asset Management Company (AMC)
to buy off bad loans from banks. AMC will not be funded by Government.
 Online asset trading platform.
 NCLT/IBC legal-technical reforms
308. The Chairman of public sector banks are selected by the
A. Banks Board Bureau
B. Reserve Bank of India
C. Union Ministry of Finance
D. Management of concerned bank
Ans. A

FINANCIAL SERVICES INSTITUTIONS BUREAU (FSIB)

 The Appointments Committee of the Cabinet (ACC) has approved a government


resolution for establishing the Financial Services Institutions Bureau (FSIB) in place of
the Banks Board Bureau (BBB).
 The FSIB will now select the chiefs of public sector banks and insurance companies.
 The selection process of top officials of public sector insurance companies was in limbo in
the wake of the Delhi High Court decision to strike down the power of BBB to select
directors and chiefs of PSU insurers.
 The ACC has also approved the appointment of Bhanu Pratap Sharma, former
Chairman, BBB, as initial Chairperson of FSlB for a term of two years from the date of
notification of government resolution or until further orders, according to a note issued by
the Department of Personnel & Training to the Department of Financial Services (DFS).

https://t.me/prelimbits
340

309. The Reserve Bank of India‟s recent directives relating to „Storage of Payment
System Data‟, popularly known as data diktat command the payment system providers
that :
1. They shall ensure that entire data relating to payment systems operated by them are stored
in a system only in India.
2. They shall ensure that the systems are owned and operated by public sector enterprises.
3. They shall submit the consolidated system audit report to the comptroller and Auditor
General of India by the end of the calendar year.
Which of the statements given above is/are correct

A. 1 only
B. 1 and 2 only
C. 3 only
D. 1,2 and 3 only
Ans. A

RBI HAS GIVEN THE FOLLOWING DIRECTIVES RELATING TO STORAGE OF PAYMENT SYSTEM DATA
 All system providers shall ensure that the entire data relating to payment systems
operated by them are stored in a system only in India (Hence statement 1 is correct).
o This data should include the full end-to-end transaction details/information
collected/carried/processed as part of the message/payment instruction.
o For the foreign leg of the transaction, if any, the data can also be stored in a foreign
country, if required.
 System providers shall ensure compliance of above rule within a period of six months and
report compliance of the same to the Reserve Bank latest by October 15, 2018.
 System providers shall submit the System Audit Report (SAR) on completion of the
requirement.
o The audit should be conducted by CERT-IN empanelled auditors certifying
completion of the activity.
o The SAR duly approved by the Board of the system providers should be
submitted to the Reserve Bank not later than December 31, 2018.
o CAG is not involved here.
310. The Service Area Approach was implemented under the purview of
A. Integrated Rural Development Programme
B. Lead Bank Scheme
C. Mahatma Gandhi National Rural Employment Guarantee Scheme
D. National Skill Development Mission
Ans. B

LEAD BANK SCHEME.


 Service area approach (SAA) is a developed version of the 'area approach' structure of

https://t.me/prelimbits
341

the Lead Bank Scheme.


 Under SAA plan each commercial bank / RRB branch in a rural and semi-urban area is
designated to serve 15 to 25 villages for the planned and orderly development of the
areas.
 Under the Lead bank Scheme, the service area approach was introduced in 1989 for the
planned and orderly development of rural and semi-urban areas.
311. Which of the following is issued by registered foreign portfolio investors to
overseas investors who want to be part of the Indian stock market without registering
themselves directly?
A. Certificate of Deposit
B. Commercial Paper
C. Promissory Note
D. Participatory Note
Ans. D

PARTICIPATORY NOTES (P-NOTES)


 Participatory Notes or P-Notes (PNs) are financial instruments issued by a registered
foreign institutional investor (FII) to an overseas investor who wishes to invest in
Indian stock markets without registering themselves with the market
regulator, the Securities and Exchange Board of India (SEBI).
 SEBI permitted FIIs to register and participate in the Indian stock market in 1992.
 P-Notes are Offshore Derivative Investments (ODIs) with equity shares or debt securities
as underlying assets.
 They provide liquidity to the investors as they can transfer the ownership by endorsement
and delivery.
 While the FIIs have to report all such investments each quarter to SEBI, they need not
disclose the identity of the actual investors.
What are govt & regulator‘s concerns?

 The primary reason why P-Notes are worrying is because of the anonymous nature of the
instrument as these investors could be beyond the reach of Indian regulators.
 Further, there is a view that it is being used in money laundering with wealthy Indians, like
the promoters of companies, using it to bring back unaccounted funds and to manipulate
their stock prices.
312. In India, which of the following review the independent regulators in sectors like
telecommunications, insurance, electricity etc. ?
1. Ad Hoc Committees set up by the Parliament.
2. Parliamentary Department Related Standing Committees
3. Finance Commission
4. Financial Sector Legislative Reforms Commission
5. NITI Aayog
Select the correct answer using the code given below.

A. 1 and 2

https://t.me/prelimbits
342

B. 1 , 3 and 4
C. 3, 4 and 5
D. 2 and 5
Ans. A

DEPARTMENT RELATED STANDING COMMITTEES


 In India, there are 24 Department Related Standing Committees that comprise members
from both Houses of Parliament.
o These committees are ministry specific and may review the working of
regulators within their respective departments.
o For example, in August 2012, the Standing Committee on Energy presented a report
on the functioning of the Central Electricity Regulatory Commission‘.
o In 2011, the Standing Committee on Information Technology had listed the
functioning of TRAI for examination.
 Parliament may also establish ad-hoc committees which may examine the working of
regulators.
o For instance, the terms of reference of the Joint Parliamentary Committee (JPC) on
the allocation of the 2G spectrum include the review of the policy on spectrum
pricing and grant of telecom licences.
o Another example of parliamentary oversight through ad-hoc committees is the
scrutiny of the working of SEBI and RBI by the JPC on the stock market scam.
 Finance Commission and NITI Aayog are of advisory nature, they do not review the
functioning of any regulator.
 The Financial Sector Legislative Reforms Commission was set up once to review financial
legislation and not regulators in the country.
313. Among the following, which one of the following is the largest exporter of rice in
the world in the last five years?
A. China
B. India
C. Myanmar
D. Vietnam
Ans. B

314. Among the agricultural commodities imported by India, which one of the following
accounts for the highest imports in terms of value in the last five years?
A. Spices
B. Fresh fruits
C. Pulses
D. Vegetable oils
Ans. D

 India is the world's second-largest consumer and number one importer of vegetable oil,

https://t.me/prelimbits
343

and it meets 55-60 per cent of its need through imports


315. Consider the following statements:
1. Most of India‘s external debt is owed by government entities.
2. All of India‘s external debt is denominated in US dollars.
Which of the statements given above is/are correct?

A. 1 only
B. 2 only
C. Both 1 and 2
D. Neither 1 nor 2
Ans. D

Check latest data here

https://www.rbi.org.in/scripts/BS_PressReleaseDisplay.aspx?prid=54457#:~:text=At%20end
%2DJune%202022%2C%20India's,March%202022%20(Table%201).

 Commercial borrowings continued to be the largest component of external debt with a


share of 37.4%, followed by NRI deposits (24.1%) and short term trade credit (19.9%).
Hence Statement 1 is Not Correct.
 US dollar-denominated debt continued to be the largest component of India's external
debt with a share of 45.9% at end December 2018, followed by the Indian rupee (24.8%),
SDR (5.1%), yen (4.9%) and euro (3.1%).
316. In the context of India, which of the following factors is/are
contributor/contributors to reducing the risk of a currency crisis?
1. The foreign currency earnings of India‘s IT sector.
2. Increasing the government expenditure.
3. Remittances from Indians abroad.
Select the correct answer using the code given below.

A. 1 only
B. 1 and 3 only
C. 2 only
D. 1,2 and 3 only
Ans. B

 A currency crisis involves the sudden and steep decline in the value of a nation's currency,
which causes negative ripple effects throughout the economy.
 Central banks and governments can intervene to help stabilize a currency by selling off
reserves of foreign currency or gold, or by intervening in the forex markets.
 This decline in value negatively affects an economy by creating instabilities in exchange
rates, meaning that one unit of a certain currency no longer buys as much as it used to in
another currency.
 Foreign currency earnings and Remittances contribute to the strengthening of the rupee.

https://t.me/prelimbits
344

Hence Statements 1 and 3 are Correct.


 Increasing government Expenditure will have no effect on the value of the currency.
Hence Statement 2 is Not Correct.
317. Which one of the following is not the most likely measure the Government/RBI
takes to stop the slide of Indian rupee?
A. Curbing imports of non-essential goods and promoting exports
B. Encouraging indian borrowers to issue rupee denominated Masala bonds
C. Easing conditions relating to external commercial borrowing
D. Following an expansionary monetary policy
Ans. D

MOST LIKELY MEASURE THE GOVERNMENT/RBI TAKES TO STOP THE SLIDE OF INDIAN RUPEE
 Curbing imports of non-essential goods and promoting exports would help control
imports and thus the depreciation of the rupee by increasing the growth with promoting
exports. Hence statement 1 is correct.
 Encouraging Indian borrowers to issue rupee-denominated Masala Bonds is a measure
of the RBI/government to stop the slide of Indian rupee as it does not put pressure on our
currency through borrowing dollars as the bond issue would be rupee denominated. Hence
statement 2 is correct.
 Easing conditions relating to external commercial borrowing will lead to higher
borrowing abroad and would temporarily reduce the deficit of forex in India preventing the
slide of rupee. Hence statement 3 is correct.
 Following an expansionary monetary policy may lead to lower interest rates thereby
increasing the inflation with higher imports through higher spending of the
government and therefore the slide of rupee takes place. Hence Option 4 is not a measure
taken by the government/RBI to stop the slide of Indian Rupee. Hence statement 4 is not
correct.
318. With reference to Asian Infrastructure Investment Bank (AIIB), consider the
following statements:
1. AIIB has more than 80 member nations.
2. India is the largest shareholder in AIIB.
3. AIIB does not have any members from outside Asia.
Which of the statements given above is/are correct?

A. 1 only
B. 2 and 3 only
C. 1 and 3 only
D. 1, 2 and 3 only
Ans. A

https://t.me/prelimbits
345

AIIB: ASIAN INFRASTRUCTURE INVESTMENT BANK


 2015-16
 China, India, UK, Switzerland, 103 nations as of 2021. [Non Asian countries is also members]
 Voting Power = Based on share capital provided. China ~27%, India ~7%. Asian countries
control about 75% voting.
 Invest around the word
 HQ = Beijing, China
319. Considering the following statements :
1. Purchasing Power Parity (PPP) exchange rates are calculated by the prices of the same
basket of goods and services in different countries.
2. In terms of PPP dollars, India is the sixth largest economy in the world.
Which of the statements given above is/ are correct?

A. 1 only
B. 2 only
C. Both 1 and 2
D. Neither 1 nor 2
Ans. A

PPP
 PPP is an economic theory that compares currencies of different countries through a
similar "basket of goods and services".
 The PPP between two countries measures the amount of one country's currency required to
purchase a basket of goods and services in the country that as compared to the amount of
other country's currency in order to purchase a similar basket of goods and services in the
other country. Hence statement 1 is correct.
 In terms of PPP dollars, China is the world‘s largest economy in 2018, followed by
the United States and India at second and third positions respectively. Hence statement
2 is not correct.
320. The economic cost of food grains to the Food Corporation Of India is Minimum
Support Price and bonus (if any) paid to the farmers plus
A. Transportation cost only
B. Interest cost only
C. Procurement incidentals and distribution costs
D. Procurement incidentals and charges for godowns
Ans. C

ECONOMIC COST OF FOOD GRAINS


 The economic cost of food grains procured by the Food Corporation of India (FCI) is a total
of Minimum Support Price and bonus (if any) paid to the farmers plus the procurement
incidentals and distribution cost.

https://t.me/prelimbits
346

 FCI's economic cost has three main components - procurement cost, procurement price,
and distribution cost.
 The procurement incidentals are the initial costs incurred during the procurement of
foodgrains.
 The distribution costs include freight, handling charges, storage charges, losses during
transit and establishment cost.
321. The Global Competitiveness Report is published by the
A. International Monetary Fund
B. United Nations Conference on Trade and Development
C. World Economic Forum
D. World Bank
Ans. C

322. Which one of the following is not a sub-index of the World Bank‟s „Ease of Doing
Business Index?
A. Maintenance of law and order
B. Paying taxes
C. Registering property
D. Dealing with construction permits
Ans. A

Note : World Bank Stops ‗Ease of Doing Business‘ Report due to data irregularities. The World
Bank announced in December 2022 that it would be releasing the methodology for the
replacement to the index in the second quarter of 2023

INDIA‘S RANKING ON 10 PARAMETERS


Ease of Doing Rank- Rank- Rank- Improved Rank- Improved 2020
Biz Parameters 2017 2018 2019 2019 over 2020 over 2019
2018
Overall 130 100 77 100 -77=23 63 77 Minus 63 =14
Starting a 155 156 137 19 136 137 Minus 136= 1
Business
Construction 185 181 52 129 27 25
Permits (highest
jump)
Getting 26 29 24 5 22 2
Electricity
Registering 138 154 166 -12 154 12
Property
-3 (getting tougher
Getting Credit 44 29 22 7 25 to get loans, thanks
(loan) to NPA, ILFS-NBFC
crisis)

https://t.me/prelimbits
347

Protecting 13 4 7 -3 13 -6 (corporate scams


Minority in ILFS etc)
Investors
Paying Taxes -2 (despite
172 119 121 apps & 115 6
portals!)
Trading across 143 146 80 66 68 12
Borders
Enforcing 172 164 163 1 163 0
Contracts
Resolving 136 103 108 -5 (despite 52 56
Insolvency I&B Code)

323. Atal innovation mission is set up under the


A. Department of science of technology
B. Ministry of labour and employment
C. NITI Ayog
D. Ministry of skill development and entrepreneurship
Ans. C

ATAL INNOVATION MISSION


 The Atal Innovation Mission (AIM) is a flagship initiative of NITI Aayog.
 It is launched to promote innovation and entrepreneurship across the country.
 Its objective is to serve as a platform for the promotion of world-class Innovation Hubs,
Grand Challenges, Start-up businesses, and other self-employment activities,
particularly in technology-driven areas.
324. Consider the following statements:
1. According to the Indian Patents Act, a biological process to create a seed can be patented
in India.
2. In India, there is no Intellectual Property Appellate Board.
3. Plant Varieties are not eligible to the patented in India.
Which of the statements given above is/ are correct?

A. 1 and 3 only
B. 2 and 3 only
C. 3 only
D. 1,2, and 3
Ans. C

 Article 3(J) of Indian Patent Act, excludes from patentability ―plants and animals in whole or
in any part thereof other than microorganisms, including seeds, varieties, and species, and
essentially biological processes for production or propagation of plants and animals‖.
Hence Statement 1 is Not Correct.
 The Intellectual Property Appellate Board (IPAB) was constituted on 2003 by the

https://t.me/prelimbits
348

Government of India to hear and resolve the appeals against the decisions of the registrar
under the Indian Trademarks Act, 1999 and the Geographical Indications of Goods
(Registration and Protection) Act, 1999. Hence statement 2 is Not Correct.
 Plant variety protection provides legal protection of a plant variety to a breeder in the form
of Plant Breeder‘s Rights (PBRs).
 In India, the Plant Variety Protection And Farmers Rights (PPVFR) Act, 2001 is a sui
generis system that aims to provide for the establishment of an effective system for the
protection of plant varieties and the rights of plant breeders and farmers.
 There are no laws in India that allows for patenting plants.Hence Statement 3 is Correct.
325. As per the Industrial Employment (Standing Orders) Central (Amendment)
Rules,2018:
1. If rules for fixed-term employment are implemented, it becomes easier for the
firms/companies to lay off workers
2. No notice of termination of employment shall be necessary in the case of temporary
workman
Which of the following statements given above is/are correct

A. 1 only
B. 2 only
C. Both 1 and 2
D. Neither 1 nor 2
Ans. C

INDUSTRIAL EMPLOYMENT (STANDING ORDERS) CENTRAL RULES, 1946.


 The government has notified fixed-term employment for all sectors through an amendment
to the Industrial Employment (Standing Orders) Central Rules, 1946.
 Fixed-term employment for all sectors will make it easier for companies to hire-and-fire
workers along with reducing the role of middlemen. Hence, Statement 1 is Correct.
 As per the Industrial Employment (Standing Orders) Central (Amendment) Rules, 2018, a
―fixed-term employment workman is a workman who has been engaged on the basis of a
written contract of employment for a fixed period‖.
 No notice of termination of employment shall be necessary in the case of a temporary
workman whether monthly rated, weekly rated or piece rated and probationers or badli
workmen. Hence, Statement 2 is Correct.
326. With reference to India‟s Five Year Plans, which of the following statements is/are
correct?
1. From the Second Five-Year Plan, there was a determined thrust towards substitution of
basic and capital good industries.
2. The Fourth Five-Year Plan adopted the objective of correcting the earlier trend of increased
concentration of wealth and economic power.
3. In the Fifth Five-Year Plan, for the first time, the financial sector was included as an integral

https://t.me/prelimbits
349

part of the Plan.


Select the correct answer using the code given below

A. 1 and 2 only
B. 2 only
C. 3 only
D. 1,2 and 3
Ans. A

SECOND FIVE-YEAR PLAN


 The stated objective of the Second five-year plan was rapid industrialisation with
particular emphasis on the development of basic and heavy industries - aimed at
establishing the Socialistic Pattern of Society. Hence Statement 1 is Correct.
FOURTH FIVE-YEAR PLAN
 The fourth five-year plan emphasised the reduction of the concentration of incomes, wealth
and economic power to achieve social equality and justice. Hence Statement 2 is Correct.
 The fifth five-year plan aims at the removal of poverty and the achievement of self-reliance.
Hence Statement 3 is Not Correct.
FIFTH FIVE YEAR PLAN
 Its duration was 1974 to 1978.
 This plan focused on Garibi Hatao, employment, justice, agricultural production and
defence.
 The Electricity Supply Act was amended in 1975, the Twenty-point programme was
launched in 1975, the Minimum Needs Programme (MNP) and the Indian National Highway
System was introduced.
 Overall this plan was successful which achieved a growth of 4.8% against the target of 4.4%.
 This plan was terminated in 1978 by the newly elected Morarji Desai government.
327. Which of the following statements is/are correct regarding the Maternity Benefit
(Amendment) Act, 2017?
1. Pregnant women are entitled for three months pre-delivery and three months post-delivery
paid leave
2. Enterprises with creches must allow the mother minimum six creche visits daily
3. Women with two children get reduced entitlements.
Select the correct answer using the code given below

A. 1 and 2 only
B. 2 only
C. 3 only
D. 1,2 and 3
Ans. C

 The Maternity Benefit (Amendment) Act 2017 provides for 26 weeks paid maternity leave

https://t.me/prelimbits
350

for women employees. (Hence statement 1 is wrong)


 Any establishment with more than 50 employees shall have the facility of crèche for working
mothers and the enterprises shall allow four visits a day for the mother to look after and
feed the child in the crèche.
 For women who are expecting a child after already having 2 children, the duration of paid
maternity leave shall be 12 weeks. (Hence Statement 3 is correct).
MATERNITY BENEFIT (AMENDMENT) ACT 2017
 The Maternity Benefit (Amendment) Act 2017 had received Presidential assent on 27 March
2017 after being passed by the Parliament. The Act has made amendments to the Maternity
Benefits Act, 1961. The majority of the provisions of the Maternity Benefit (Amendment) Act
has come into force with effect from 1st April, 2017.
 The main aim of the Act is to regulate the employment of women during the period of child
birth. It has amended the provisions related to the duration and applicability of maternity
leave, and other facilities.
Major changes

 The Maternity Benefit (Amendment) Act 2017 has increased the duration of paid maternity
leave available for women employees to 26 weeks from 12 weeks. However for those
women who are expecting after having 2 children, the duration of the leave remains
unaltered at 12 weeks.
 The paid maternity leave can be availed 8 weeks before the expected date of delivery.
Before the amendment, it was 6 weeks.
 The Maternity Benefit (Amendment) Act 2017 has extended the benefits applicable to the
adoptive and commissioning mothers and provides that woman who adopts a child will be
given 12 weeks of maternity leave from the date of adoption.
 The Act has introduced an enabling provision relating to ―work from home‖ that can be
exercised after the expiry of 26 weeks‘ leave period. Depending upon the nature of work, a
woman can avail of this provision on such terms that are mutually agreed with the
employer.
 The amended Act has mandated crèche facility for every establishment employing 50 or
more employees. The women employees should be permitted to visit the facility 4 times
during the day.
 The amended act makes it compulsory for the employers to educate women about the
maternity benefits available to them at the time of their appointment.
Other provisions

 The act is applicable to all those women employed in factories, mines and shops or
commercial establishments employing 10 or more employees.
328. Consider the following statements :
1. Petroleum and Natural Gas Regulatory Board (PNGRB) is the first regulatory body set up by
the Government of India.

https://t.me/prelimbits
351

2. One of the tasks of PNGRB is to ensure competitive markets for gas.


3. Appeals against the decisions of PNGRB go before the Appellate Tribunals of Electricity.
Which of the statements given above is/are correct?

A. 1 and 2 only
B. 2 and 3 only
C. 1 and 3 only
D. 1,2 and 3
Ans. B

 The Petroleum and Natural Gas Regulatory Board (PNGRB) was constituted under
the Petroleum and Natural Gas Regulatory Board Act, 2006.
o The independent regulator TRAI is the first independent regulator in India.
Hence Statement 1 is Not Correct.
 The Act provides for the establishment of Petroleum and Natural Gas Regulatory Board
to protect the interests of consumers and entities engaged in specified activities relating
to petroleum, petroleum products and natural gas and to promote competitive markets and
for matters connected therewith or incidental thereto.
o One of the tasks of PNGRB is to ensure competitive markets for gas.
Hence Statement 2 is Correct.
 The board has also been mandated to regulate the refining, processing, storage,
transportation, distribution, marketing and sale of petroleum, petroleum products and
natural gas excluding production of crude oil and natural gas so as and to ensure an
uninterrupted and adequate supply of petroleum, petroleum products and natural gas in all
parts of the country.
 The PNGRB will have the same powers as a civil court to settle disputes.
 The Appellate Tribunal established under section 110 of the Electricity Act, 2003 (36 of
2003) is the Appellate Tribunal for the purposes of Petroleum and Natural Gas Regulatory
Board Act, 2006. Hence Statement 3 is Correct.
329. Consider the following statements:
1. Coal sector was nationalized by the Government of India under Indira Gandhi.
2. Now, coal blocks are allocated on lottery basis.
3. Till recently, India imported coal to meet the shortages of domestic supply, but now india is
self-sufficient in coal production.
Which one of the following statements given above is/ are correct?

A. 1 only
B. 2 and 3 only
C. 3 only
D. 1,2 and 3
Ans. A

https://t.me/prelimbits
352

COAL SECTOR
 Coal sector was nationalised under Indira Gandhi Government in 1972 in two phases.
Hence Statement 1 is Correct.
 The coal Blocks are allocated through auctions and not on lottery
basis. Hence Statement 2 is Not Correct.
 The coal sector is the monopolistic sector in India.
 India holds 5th biggest coal reserves in the world but due to incapacity of coal production
by monopolistic firms, it imports coals to meet the shortages of domestic supply.
 But, still the country is not self-sufficient of coal production.Hence Statement 3 is Not
Correct.
330. With reference to the management of minor minerals in India, consider the
following statements :
1. Sand is a ‗minor mineral‘ according to the prevailing law in the country.
2. State Governments have the power to grant mining leases of minor minerals, but the
powers regarding the formation of rules related to the grant of minor minerals lie with the
Centre Government.
3. State Governments have the power to frame rules to prevent illegal mining of minor
minerals.
Which of the statements given above is/are correct?

A. 1 and 3 only
B. 2 and 3 only
C. 3 only
D. 1, 2 and 3 only
Ans. A

MINOR MINERALS
 Sand is a minor mineral, as defined under section 3 (e) of the Mines and Minerals
(Development and Regulation) Act, 1957 (MMDR Act). Hence Statement 1 is Correct.
 Section 15 of the MMDR Act empowers state governments to make rules for regulating the
grant of mineral concessions in respect of minor minerals and for purposes connected
therewith.
 The regulation of grant of mineral concessions for minor minerals is, therefore, within the
legislative and administrative domain of the state governments.
 Under the power granted to them by section 15 of the MMDR Act, State Governments have
framed their own minor minerals concession rules. Hence Statement 2 is Not Correct.
 Section 23C of the MMDR Act, 1957 empowers state governments to frame rules to
prevent illegal mining, transportation and storage of minerals and for purposes connected
therewith.
 Control of illegal mining is, therefore, under the legislative and administrative jurisdiction of
state governments. Hence Statement 3 is correct.

https://t.me/prelimbits
353

331. In a given year in India, official poverty lines are higher in some states than in other
because
A. Poverty rates vary from state to state
B. Price levels vary from state to state
C. Gross state product varies from state to state
D. Quality of public distribution varies from state to state
Ans. B

POVERTY LINES
 Poverty lines would vary from State to State because of inter-state price differentials.
 According to the Planning Commission, in 2011-12 for rural areas, the national poverty
line by using the Tendulkar methodology is estimated at Rs 816 per capita per month in
villages and Rs 1,000 per capita per month in cities.
 The poverty line depends on the income profile of the population and on the basis of
consumer expenditure.
 The poverty line is a function of the cost of consumption basket which varies from state to
state.
332. In the context of any country which one of the following would be considered as
part of its social capital?
A. The proportion of literates in the population.
B. The stock of its buildings, other infrastructure and machines.
C. The size of the population in the working age group.
D. The level of mutual trust and harmony in the society.
Ans. D

The stock of its buildings, other infrastructure, and machines implies a physical capital.

SOCIAL CAPITAL
 Social capital is the values, beliefs, and attitudes that govern the nature of social
interactions.
 Social capital refers to connections among individuals – social networks and the norms of
reciprocity and trustworthiness that arise from them i.e the level of mutual trust and
harmony in the society.
Prelims 2020
333. With reference to “Blockchain Technology” consider the following statements:
1. It is a public ledger that everyone can inspect, but which no single user controls.
2. The structure and design of blockchain is such that all the data in it are about
cryptocurrency only
3. Applications that depend on basic features of blockchain can be developed without
anybody‘s permission.

https://t.me/prelimbits
354

Which of the statements given above is/are correct?

A. 1 only
B. 1 and 2 only
C. 2 only
D. 1 and 3 only
Ans. D

BLOCKCHAIN TECHNOLOGY
 It is referred to as Distributed Ledger Technology.
 It facilitates the process of recording transactions and tracking assets in a business network.
 It is ideal for delivering the information in business because it provides immediate, shared,
and completely transparent information stored on an immutable ledger that can be
accessed only by permission network members.
 With the shared ledger, transactions are recorded only once, eliminating the duplication of
effort.
 No participant can change or tamper with a transaction after it‘s been recorded to the
shared ledger.
 The blocks form a chain of data as the ownership changes hands. Hence, it has no single
user control. Hence statement 1 is correct.
 The blocks confirm the exact time and sequence of transactions, and the blocks link securely
together to prevent any block from being altered.
 Each additional block strengthens the verification of the previous block and hence the entire
blockchain is formed.
 The verification of each block make the blockchain tamper-evident and builds a ledger of
transactions that network members can trust.
 The applications that depend on the basic features of the blockchain can be developed
without asking anybody for permission or paying anyone. Hence statement 3 is correct.
 Since blockchain operates through a decentralized platform requiring no central
supervision, it is used in voting, banking, messaging app, internet advertising, etc. Hence, it
is not restricted to cryptocurrency. Hence statement 2 is not correct.
334. If you withdraw Rs. 1,00,000 in cash from your Demand Deposit Account at your
bank, the immediate effect on aggregate money supply in the economy will be
A. to reduce it by ₹ 1,00,000
B. to increase it by ₹ 1,00,000
C. to increase it by more than ₹ 1,00,000
D. to leave it unchanged
Ans. D

Also Refer Q. 36

https://t.me/prelimbits
355

DEMAND DEPOSIT ACCOUNT


 It consists of funds held in a bank account from which deposited funds can be withdrawn at
any time while a term deposit account restricts access for a predetermined time.
 Current Account and Savings Account are Demand Deposit Accounts.
 Some banks ask their customers to maintain a minimum balance in the account.
 The money supply is the total value of money available in an economy at a point of time.
 Money supply consists of total currency circulating in the public plus the demand deposits
of the public with banks.
 Hence, we can write Money Supply = Currency with public + Currency in the bank.
 If we look at the equation; when you draw cash from the bank, it goes to the currency in
hand but it does not change the value of the money supply.
 Hence, there will be no change in the aggregate money supply.
335. If the RBI decides to adopt an expansionist monetary policy, which of the following
would it not do ?
1. Cut and optimize the Statutory Liquidity Ratio
2. Increase the Marginal Standing Facility Rate
3. Cut the Bank Rate and Repo Rate
Select the correct answer using the code given below :

A. 1 and 2 only
B. 2 only
C. 1 and 3 only
D. 1, 2 and 3
Ans. B

MONETARY POLICY
 Expansionary policy is proceeded by RBI in order to relax interest norms and to facilitate
more liquidity in the market to boost the economy.
 It increased the purchasing power of employees. It is also referred to as 'easy-moderate
policy'.
 Contractionary Monetary Policy- it slows the rate of growth in the money supply or
outright decreases the money supply in order to control inflation.
 When a central bank tries to expand the overall money supply to boost the economy, then
it is called as Accommodative Monetary Policy.
Tool Contractionary Policy Expansionary Policy
Cash Reserve Ratio Increase Decrease
Repo Rate Increase Decrease
SLR Increase Decrease
MSFR Increase Decrease

336. Consider the following statements :


1. In terms of short-term credit delivery to the agriculture sector, District Central Cooperative

https://t.me/prelimbits
356

Banks (DCCBs) deliver more credit in comparison to Scheduled Commercial Banks and
Regional Rural Banks
2. One of the most important functions of DCCBs is to provide funds to the Primary
Agricultural Credit Societies.
Which of the statements given above is/are correct ?

A. 1 only
B. 2 only
C. Both 1 and 2
D. Neither It nor 2
Ans. B

 Schedules bank contributes to about 70-80% share in agricultural and allied credit.
Though co-operative institutions also play a significant role in providing finance to the
agriculture sector which stands at 15-16%, hence not as much as Scheduled banks, hence
statement 1 is incorrect.
 The co-operative banking sector thrives either as a three-tier or two-tier structure.
o The three-tier structure includes STCB, DCCB, and PACS.
o The two-tier structure only STCB and PACS are present.
o In the three-tier structure, the lower level tiers, i.e. DCCB and PACS extend credit to
individual borrowers using their own funds/deposits and claim to refinance from the
upper tier, i.e. PACS from DCCB/StCB and DCCB from CB.
 In the two-tier structure, PACS provides credit to individual borrowers and claim to
refinance from StCB. In some cases, the StCBs also extend credit to the individuals through
its branches across the state. Hence statement 2 is correct.
DCCBS
 The Board of the DCCB comprises elected Chairmen of PACS, representative of the State
Government and the State Cooperative Bank apart from the CEO of the DCCB who would
be the member secretary.
 DCCBs provide funds directly either to PACs or to the farmers for excavation of wells,
purchase of pump sets to horticulture, animal husbandry, and even rural transport like
tractors and other farm equipment, etc. Hence statement 2 is correct.
337. What is the importance of the term “Interest Coverage Ratio” of a firm in India?
1. It helps in understanding the present risk of a firm that a bank is going to give a loan to.
2. It helps in evaluating the emerging risk of a firm that a bank is going to give a loan to.
3. The higher a borrowing firm‘s level of Interest Coverage Ratio, the worse is its ability to
service its debt.
Select the correct answer using the code given below:

A. 1 and 2 only
B. 2 only
C. 1 and 3 only

https://t.me/prelimbits
357

D. 1, 2 and 3
Ans. A

INTEREST COVERAGE RATIO


 It indicates the ability of a firm to take the loan or debt and repay it within the tenure of the
loan. It helps in understanding and evaluating the present risk of a firm that a bank is going
to give a loan to. Hence statement 1 is true.
 It is calculated by dividing a company's earnings before interest and taxes by the company's
interest expenses for a given period. Hence 2 is correct.
 The lower the ratio, the more the company is burdened by debt expense. When a
company's interest coverage ratio is only 1.5 or lower, its ability to meet interest expenses
may be questionable. Hence statement 3 is incorrect.
Key Points

 The interest coverage ratio is a good assessment of a company‘s short-term financial


health.
 Moreover, an interest coverage ratio below 1 indicates the company is not generating
sufficient revenues to satisfy its interest expenses.
 A good interest coverage ratio would serve as a good indicator of this circumstance and
potentially as an indicator of the company‘s ability to pay off the debt itself as well.
338. In the context of the Indian economy, non-financial debt includes which of the
following ?
1. Housing loans owed by households
2. Amounts outstanding on credit cards
3. Treasury bills
Select the correct answer using the code given below :

A. 1 only
B. 1 and 2 only
C. 3 only
D. 1, 2 and 3
Ans. D

DEBT FINANCING
 When an economy borrows money to be paid back at future date along with interest, it is
known as Debt Financing.
 Debt Financing mainly includes two types of debts-
o Public Debt-
 It is the total amount borrowed by the government of a country.
 It is further divided into two types-
 Internal Public Debt
 External Public Debt

https://t.me/prelimbits
358

 Non-Financial Debt-
o It consists of credit instruments issued by government entities, households, and
business which are not covered in the financial sector.
o Housing loans owed by households, Amounts outstanding on credit cards, Treasury
bills, Credit Card balance etc. are types of nonfinancial debts. Hence 1 2 and 3 are
correct.
PUBLIC DEBT AND PRIVATE DEBT
 The money owed by the union government comes under Public Debt.
 All the loans, debts etc. raised by private companies, the corporate sector and individuals
such as home loans, auto loans, personal loans come under Private Debt.
339. Which of the following phrases defines the nature of the „Hundi‟ generally referred
to in the sources of the post-Harsha period?
A. An advisory issued by the king to his subordinates
B. A diary to be maintained for daily accounts
C. A bill of exchange
D. An order from the feudal lord to his subordinates
Ans. C

Hundi/ Bill of exchange are short term financial instruments.

340. With reference to the Indian economy, consider the following statements :
1. Commercial Paper‘ is a short-term unsecured promissory note.
2. Certificate of Deposit‘ is a long-term instrument issued by the Reserve Bank of India to a
corporation.
3. Call Money‘ is a short-term finance used for interbank transactions.
4. Zero-Coupon Bonds‘ are the interest bearing short-term bonds issued by the Scheduled
Commercial Banks to corporations.
Which of the statements given above is/are correct

A. 1 and 2 only
B. 4 only
C. 1 and 3 only
D. 2, 3 and 4 only
Ans. C

THE MONEY MARKET


 The money market is the centre for dealing mainly of short character, in monetary assets; it
meets the short-term requirements of borrowers and provides liquidity or cash to the
lenders.
 Money market deals in financial assets whose period of maturity is up to one year.
 Short term financial instruments are -
o Call money

https://t.me/prelimbits
359

o Treasury Bill
o Commercial Paper
o Deposit Certificates
o Trade bill
THE CAPITAL MARKET
 The capital market is the market for medium and long-term funds.
 Capital Market deals in financial assets whose period of maturity more than one year.
 Long term financial instruments are-
 Security Market- Equity, Debt, IPOs, Commodity Markets etc. Hence 4 is incorrect.
 Non-Securit Market - Mutual Funds, Fixed Deposits, Saving Deposits etc.
ADDITIONAL INFORMATION
 Call money deals with day to day cash requirement of banks. Its is a short-term financial
instrument. Hence 3 is correct.
o Participants in the call money market are banks and related entities specified by the
RBI.
 Notice money deals with loans for 2-14 days to banks, it is also a short-term financial
instrument.
341. Under the Kisan Credit Card scheme, short-term credit support is given to farmers
for which of the following purposes ?
1. Working capital for maintenance of farm assets
2. Purchase of combine harvesters, tractors and mini trucks
3. Consumption requirements of farm households
4. Post-harvest expenses
5. Construction of family house and setting up of village cold storage facility
Select the correct answer using the code given below :

A. 1, 2 and 5 only
B. 1, 3 and 4 only
C. 2, 3, 4 and 5 only
D. 1, 2, 3, 4 and 5
Ans. B

 The Kisan Credit Card scheme aims at providing adequate and timely credit support from
the banking system under a single window with the flexible and simplified procedure to the
farmers for their cultivation and other needs as indicated below:
o To meet the short term credit requirements for the cultivation of crops
o Post-harvest expenses; Produce marketing loan
o Consumption requirements of farmer household
o Working capital for maintenance of farm assets and activities allied to agriculture
o Investment credit requirement for agriculture and allied activities.
 The Kisan Credit Card Scheme detailed in the ensuing paragraphs is to be implemented by

https://t.me/prelimbits
360

Commercial Banks, RRBs, Small Finance Banks, and Cooperatives.


KISAN CREDIT CARD (KCC)
 The Kisan Credit Card (KCC) scheme was introduced in 1998 for the issue of Kisan
Credit Cards to farmers on the basis of their holdings for uniform adoption by the banks so
that farmers may use them to readily purchase agricultural inputs such as seeds, fertilizers,
pesticides, etc. and draw cash for their production needs.
 Eligible Farmers -
o individual/joint borrowers who are owner cultivators
o Tenant farmers, oral lessees & sharecroppers
o Self Help Groups (SHGs) or Joint Liability Groups (JLGs) of farmers including tenant
farmers, sharecroppers, etc.
 The short term loan limit arrived for the 5th year plus the estimated long term loan
requirement will be the Maximum Permissible Limit (MPL) and is to be treated as the
Kisan Credit Card limit.
342. In India, under cyber insurance for individuals, which of the following benefits are
generally covered, in addition to payment for the loss of funds and other benefits ?
1. Cost of restoration of the computer system in case of malware disrupting access to one‘s
computer
2. Cost of a new computer if some miscreant wilfully damages it, if proved so
3. Cost of hiring a specialized consultant to minimize the loss in case of cyber extortion
4. Cost of defence in the Court of Law if any third party files a suit
Select the correct answer using the code given below :

A. 1, 2 and 4 only
B. 1, 3 and 4 only
C. 2 and 3 only
D. 1, 2, 3 and 4
Ans. B

CYBER INSURANCE
 Most of the cybersecurity policies not only offer a comprehensive cover against cyber risks
and frauds but also pay for the legal costs and expenses. Hence statement 1 and 4 are
correct.
 It also covers the expenses incurred on counselling sessions with a psychologist due to any
traumatic stress one may face as a result of cyber-bullying or harassment. Hence statement
3 is correct.
 But the cost of a new computer if some miscreant wilful damages it, is not covered under
insurance. Hence statement 2 is incorrect.
Additional Information

 The Insurance Regulatory and Development Authority of India (IRDAI) is keen on


evolving a basic Standard Cyber Liability Insurance product.

https://t.me/prelimbits
361

 The nine-member group, with Consultant-Liability Insurance P. Umesh as chairman, has


been tasked to explore the possibility of developing standard coverages, exclusions, and
optional extensions for various categories.
343. Along with the Budget, the Finance Minister also places other documents before
the Parliament which include „The Macro Economic Framework Statement‟. The
aforesaid document is presented because this is mandated by
A. Long standing parliamentary convention
B. Article 112 and Article 110(1) of the Constitution of India
C. Article 113 of the Constitution of India
D. Provisions of the Fiscal Responsibility and Budget Management Act, 2003
Ans. D

Refer Q. 116

344. With reference to the international trade of India at present, which of the following
statements is/are correct?
1. India‘s merchandise exports are less than its merchandise imports.
2. India‘s imports of iron and steel, chemicals, fertilisers and machinery have decreased in
recent years.
3. India‘s exports of services are more than its imports of services.
4. India suffers from an overall trade/current account deficit.
Select the correct answer using the code given below :

A. 1 and 2 only
B. 2 and 4 only
C. 3 only
D. 1, 3 and 4 only
Ans. D

This Question was cancelled by UPSC. [Q.52 (Set A), UPSC CSP 2020]

 India is a net exporter in services. Observing the recent trends, there was a surplus of $6.84
billion in June, with exports standing at $16.48 billion and imports at $9.64 billion. Hence
3 is correct.
 As per RBI‘s data, India‘s merchandise exports during April-August 2019-20 was 133 bn USD
as compared to 210 bn of imports during the same period. Hence statement 1 is correct.
 India suffers from an overall trade deficit. For instance-
Year 2015 2016 2017 2018 2019
Trade Balance - -108.9 - -182.3 -153.5
(in USD bn) 117.3 158.6
Hence 4 is correct.

 India's trade deficit narrowed sharply to USD 6.77 billion in August of 2020 from USD
13.86 billion in the same month last year.

https://t.me/prelimbits
362

 In 2018, major countries to which India Exported include the United States, United Arab
Emirates, China, Hong Kong and Singapore.
Additional Information

 Export of iron and steel products witnessed a sharp rise of more than 100% in June
2020. Hence 2 is incorrect.
 Chemicals include dyes and dye intermediates, organic chemicals, inorganic chemicals,
agro-chemicals, cosmetics & toiletries, and castor oil.
 From April 2019 to January 2020, the export of dyes increased by 9.12% y-o-y to US$ 2.27
billion. Cosmetics and toiletries increased by 5.62%. Hence 2 is incorrect.
345. The term „West Texas Intermediate‟, sometimes found in news, refers to a grade of
A. Crude oil
B. Bullion
C. Rare earth elements
D. Uranium
Ans. A

CRUDE OILS → CHARACTERISTICS & QUALITY


 There are about 160 crude oils that are traded internationally. They vary in terms of their
characteristics & quality.
 West Texas Intermediate and Brent Crude oil are two major types of crude oil
benchmarks at the international arena.
 Comparison of WTI and Brent Crude oil is as follows-
WTI Bent
Benchmark for oil extracted from America Benchmark for crude oil obtained from the North
Sea near Norway, Sweden, and the United
Kingdom
Benchmark used by US Oil Prices Benchmark used for OPEC oil prices
Traded on New York Mercantile Exchange Traded on International Exchange, London
Low share at international trade but futuristic Two-third of the world's crude contracts are
opportunities signed in Brent oil benchmark
OTHER IMPORTANT BENCHMARKS ARE AS FOLLOWS-
 NYMEX Futures-
o represents the market-determined value of a futures contract to either buy or sell
1,000 barrels(@ per barrel rate) of WTI at a specified time.
 Opec basket price-
o OPEC collects pricing data on a "basket" of seven crude oils:
 Algeria's Saharan Blend
 Indonesia's Minas
 Nigeria's Bonny Light
 Saudi Arabia's Arab Light
 Dubai's Fateh

https://t.me/prelimbits
363

 Venezuela's Tia Juana Light and


 Mexico's Isthmus
346. With reference to Foreign Direct Investment in India, which one of the following is
considered its major characteristics ?
A. It is the investment through capital instruments essentially in a listed company.
B. It is largely non-debt creating capital flow.
C. It is the investment which involves debt-servicing.
D. It is the investment made by foreign institutional investors in the Government securities.
Ans. B

 Debt service is the cash that is required to cover the repayment of interest and principal on
a debt for a particular period. FDIs have no link with this concept, Hence option 3 is
incorrect.
 The investment is done through capital instruments in (1) an unlisted Indian company; or (2)
10% or more of the post issue paid-up equity capital on a fully diluted basis of a listed
Indian company. Hence, option 1 is incorrect.
 FDI investment can be made in equities or equity-linked instruments or debt instruments
issued by the company and they are not directly related to government securities. Hence
option 4 is incorrect.
 Foreign Institutional Investment does the investment made by foreign institutional
investors in Government Securities.
 FII refers to the group of investors who helps to bring the FPI into a country. In other words,
Foreign Institutional Investors is the way through Foreign Portfolio Investment is done.
 Foreign Direct Investment (FDI) is the investment by a non-resident entity/person
resident outside India in the capital of an Indian company under Foreign Exchange
Management (Transfer or Issue of Security by a Person Resident Outside India) Regulations,
2017.
 FDI‘s are long term investment. It is also nondebt creating capital flow as it has no direct
repayment obligation for the residents.
DEBT FLOW AND NON DEBT FLOW
 Debt Flow-
o A debt flow is a type of foreign capital where there is an obligation for the residents
to repay it.
 Non Debt Flow-
o A non-debt flow is the one where there is no direct repayment obligation for the
residents.
o For example-FDI, FPI, and Depository Receipts are non-debt flows.ECBs, FCCBs,
Rupee Denominated Bonds, NRI deposits, and banking capital are debt creating
flows. Hence option 2 is correct.

https://t.me/prelimbits
364

347. “Gold Tranche” (Reserve Tranche) refers to


A. A loan system of the World bank
B. One of the operations of a central bank
C. A credit system granted by WTO to its members
D. A credit system granted by IMF to its members
Ans. D

RESERVE TRANCHE
 The IMF is funded through its members and their quota contributions.
 The reserve tranche is basically an emergency account that IMF members can access at any
time without agreeing to conditions or paying a service fee.
 It is a portion of a member country‘s quota that can be withdrawn free of charge at its own
discretion.
 The first 25% reserve tranche portion charges no interest.
 Anything beyond that could require permission and be subject to a service fee.
The International Monetary Fund (IMF) is an organization of 190 countries, working to foster
global monetary cooperation, secure financial stability, facilitate international trade,
promote high employment and sustainable economic growth, and reduce poverty around the
world.

348. If another global financial crisis happens in the near future, which of the following
actions/policies are most likely to give some immunity to India ?
1. Not depending on short-term foreign borrowings
2. Opening up to more foreign banks
3. Maintaining full capital account convertibility
Select the correct answer using the code given below :

A. 1 only
B. 1 and 2 only
C. 3 only
D. 1, 2 and 3
Ans. A

 Both international creditors and the debtor nations suffer significant hardships due to
a global financial crisis and are forced into default.
o Both economic and political considerations leave it difficult to resist coming to the
aid of a distressed nation or region.
o So, India should not depend on short-term foreign borrowings. It will
surely provide immunity to India.
o Hence, Statement A is correct.
 The financial crisis will most likely increase the NPA (Non-performing Assets) burden on
banks making the banks inefficient in NPA recovery and declaring themselves

https://t.me/prelimbits
365

insolvent in near future.


o So, opening up to more foreign banks would be a huge risk for India.
o Hence, Statement B is incorrect.
 Capital account convertibility means the freedom to conduct investment transactions
without any constraints.
o Typically, it would mean no restrictions on the amount you can convert into
foreign currency to enable you to acquire any foreign asset and vice versa.
o During a financial crisis, it will be a huge failure and mistake.
o Hence, statement C is not correct.
349. With reference to Trade-Related Investment Measures (TRIMS), which of the
following statements is/are correct ?
1. Quantitative restrictions on imports by foreign investors are prohibited.
2. They apply to investment measures related to trade in both goods and services.
3. They are not concerned with the regulation of foreign investment.
Select the correct answer using the code given below :

A. 1 and 2 only
B. 2 only
C. 1 and 3 only
D. 1, 2 and 3
Ans. C

TRADE-RELATED INVESTMENT MEASURES


 Trade-Related Investment Measures provide quantitative restrictions on imports by
foreign investors are prohibited. Hence statement 1 is correct.
 TRIMS agreement applies only to investment measures related to trade in goods and not in
services and it is mentioned in Article 1 of the TRIMS agreement, hence statement 2 is
incorrect.
 As per the TRIMs Agreement, members are required to notify the WTO Council for Trade in
Goods of their existing TRIMs that are inconsistent with the agreement.
 TRIMs are rules that restrict the preference of domestic firms and thereby enable
international firms to operate more easily within foreign markets.
 They don‘t regulate foreign investment as such, which is governed by FEMA and DIPP
guidelines/regulations. Hence statement 3 is correct.
Features of TRIMS are →

 Offering equal rights to the foreign investor on par with domestic players
 Abolition of restrictions imposed on foreign goods.
 Permission grant to import raw material without restrictions.
 Abolition of restrictions on any area of investment
Article. 5.1 of the TRIMs Agreement

 As per the provisions of Article. 5.1 of the TRIMs Agreement India had notified three

https://t.me/prelimbits
366

trade-related investment measures which are as follows-


o Local content (mixing) requirements in the production of News Print,
o Dividend balancing requirement in the case of investment in 22 categories of
consumer goods.
o Local content requirement in the production of Rifampicin and Penicillin – G
350. Consider the following statements:
1. The value of Indo-Sri Lanka trade has consistently increased in the last decade.
2. Textile and textile articles constitute an important item of the trade between India and
Bangladesh.
3. In the last five years, Nepal has been the largest trading partner of India in South Asia.
Which of the statements given above is/are correct?

A. 1 and 2 only
B. 2 only
C. 3 only
D. 1, 2 and 3
Ans. B

 The Indo-Srilanka trade reached its peak in 2013-14 in the last decade. Hence statement 1
is incorrect.
 Bangladesh is India‘s largest trading partner in South Asia, followed by Nepal, Sri Lanka,
Pakistan, Bhutan, Afghanistan, and the Maldives. Hence statement 3 is incorrect.
 Cotton is the largest export item to Bangladesh from India, accounting for a fifth of India‘s
supplies to Bangladesh, followed by mineral fuels, automobiles and capital goods. Hence
statement 2 is correct.
 India‘s imports from Bangladesh jumped 22% to $1.2 billion. Garment and textile products
make up for around 40% of the imports
The top four export countries of India are as follows-

 China- trade deficit of 51 bn USD.


 The USA - trade surplus of 25 bn USD.
 UAE - trade surplus of 12 bn USD.
 Saudi Arabia trade deficit of 15 bn USD.
351. In which one of the following groups are all the four countries members of G20 ?
A. Argentina, Mexico, South Africa and Turkey
B. Australia, Canada, Malaysia and New Zealand
C. Brazil, Saudi Arabia and Vietnam
D. Indonesia, Japan, Singapore and South Korea
Ans. A

G20
 G20 is an international forum for global economic cooperation.

https://t.me/prelimbits
367

 It has 20 members, and these countries are as follows-


 Argentina, Australia, Brazil, Canada, China, France, Germany, India, Indonesia, Italy, Japan,
Republic of Korea, Mexico, Russia, Saudi Arabia, South Africa, Turkey, United Kingdom,
United States, and the European Union. Hence option 1 is correct.
 G20 members account for 85 per cent of the world economy, 75 per cent of global trade,
and two-thirds of the world's population.
 G20 was formed in 1999 and its present chairman is Joko Widodo.
 Canadian finance minister Paul Martin was chosen as the first chairman and German finance
minister Hans Eichel hosted the inaugural meeting.
2023 G20 summit

 The G20 Summit is formally known as the ―Summit on Financial Markets and the World
Economy‖.
 India is hosting the G-20 Leaders‘ Summit in 2023.
LIST OF SOME IMPORTANT INTERNATIONAL GROUPS AND MEMBER COUNTRIES ARE AS FOLLOWS-
 ADB - Asian Development Bank Established on 19 December 1966 Nd ADB now has 68
members.
 G7 - Canada, France, Germany, Italy, Japan, UK and USA.
 SCO- founded in 2001, f China, Kazakhstan, Kyrgyzstan, Russia, Tajikistan, Pakistan, India
and Uzbekistan.
 BBIN Initiative- Bangladesh, Bhutan, India and Nepal, it was founded in 2016.
 BIMSTEC- Bangladesh, Bhutan, India, Myanmar, Nepal, Sri Lanka and Thailand.
352. In India, which of the following can be considered as public investment in
agriculture?
1. Fixing Minimum Support Price for agriculture produce of all crops
2. Computerization of Primary Agriculture Credit Societies
3. Social Capital development
4. Free electricity supply of farmers
5. Waiver of agriculture loans by the banking system
6. Setting up of cold storage facilities by the governments
Select the correct answer using the code given below:

A. 1, 2 and 5 only
B. 1, 3, 4 and 5 only
C. 2, 3 and 6 only
D. 1, 2, 3, 4, 5 and 6
Ans. C

 Fixing Minimum Support Price for Agriculture for agricultural products of all crops and
free electricity supply to farmers are subsidies, hence they are not categorized as public
investment in agriculture.
 The waiver of agricultural loans by the banking system is a concession or indirect support.

https://t.me/prelimbits
368

hence it is not considered as public investment in agriculture.


 Subsidies and incentives are not considered as public investment, hence statement 2 3,
and 6 are correct codes.
In May 2020, the government announced Aatam Nirbhar Bharat Abhiyan and invested heavily in
agriculture, These include-

 The Agri Infra Fund of Rs 1 lakh crore for post-harvest infrastructure, a scheme
for 10,000 farmer producer organizations.
 Special drive to include 25 million farmers who still do not have the Kisan Credit Cards
(KCC)
 developing a digital agri-stack which will be a key enabler for online marketplaces and
smart agriculture.
353. Which of the following factors/policies were affecting the price of rice in India in
the recent past?
1. Minimum Support Price
2. Government‘s trading
3. Government‘s stockpiling
4. Consumer subsidies
Select the correct answer using the code given below:

A. 1,2 and 4 only


B. 1,3 and 4 only
C. 2 and 3 only
D. 1,2, 3 and 4
Ans. D

FACTORS/POLICIES WERE AFFECTING THE PRICE OF RICE


 The domestic factors which affect prices of rice in India are as follows-
o The intensity of seasonal monsoon across India and production of Basmati and non-
basmati rice.
o The ratio of full and broken kernels in milling the paddy.
o The government's trading and Minimum support price of rice- it results in the
diversion of stocks from the open market hence driving up the price for the ultimate
consumers. Hence 1 is correct.
o Rice tenders by Food Corporation of India and other organizations
o Consumer subsidies ( it lowers the prices for the ultimate consumer) Hence 3 is
correct.
o The government's stockpiling (low stock levels constraint that ability to buffer the
price rise) Hence 4 is correct.
o Climatic shocks determine rice production and prices in the short run due to
increasing demand. Hence 2 is correct.
 Some international factors which affect Indian rice pricing are as follows-

https://t.me/prelimbits
369

o Global tenders for procurement of rice


o Appreciation or depreciation of USD and Indian Rupees.
o Crude Oil Prices
ADDITIONAL INFORMATION
 The Manipur black rice, called ‗Chakhao‘ is the most recent addition to GI tagged rice
varieties of India.
 West Bengal is the largest rice producer in India.
 In 1965, the ICAR launched the AICRIP or All India Coordinated Rice Improvement Project
for R&D for improving rice production.
354. Consider the following statements
1. In the case of all cereals, pulses and oil-seeds, the procurement at Minimum Support Price
(MSP) is unlimited in any State/UT of India.
2. In the case of cereals and pulses, the MSP is fixed in any State/UT at a level to which the
market price will never rise.
Which of the statements given above is/are correct?

A. 1 only
B. 2 only
C. Both 1 and 2
D. Neither 1 nor 2
Ans. D

MSP
 While procurement is open-ended, it is not unlimited since our buffer stock is
limited. Hence statement 1 is incorrect.
 Market price has no link with Minimum Support Prices and it can go below or above of MSP
depends upon the demand of crop in the market, hence statement 2 is incorrect.
 The chief objectives of setting up MSP are:
o Support farmers from distress sales
o To procure food grains for public distribution
 Issue Price is the price at which the procured and buffer stock food grains are provided
through the PDS.
 The MSP is recommended by the Commission for Agricultural Costs and Prices and
announced by Cabinet Committee on Economic Affairs.
 MSP for sugarcane is known as Fair and Remunerative Prices.
 The Centre currently fixes MSPs for 23 crops and these are as follows →
o 7 cereals – paddy, wheat, maize, bajra, jowar, ragi and barley
o 5 pulses – chana, arhar/tur, urad, moong and masur.
o 7 oilseeds – rapeseed-mustard, groundnut, soya bean, sunflower, sesamum,
safflower and nigerseed.
o 4 commercial crops – cotton, sugarcane, copra and raw jute.

https://t.me/prelimbits
370

355. With reference to the Indian economy after the 1991 economic liberalization,
consider the following statements :
1. Worker productivity per worker (at 2004 — 05 prices) increased in urban areas while it
decreased in rural areas.
2. The percentage share of rural areas in the workforce steadily increased.
3. In rural areas, the growth in the non-farm economy increased.
4. The growth rate in rural employment decreased.
Which of the statements given above is/are correct ?

A. 1 and 2 only
B. 3 and 4 only
C. 3 only
D. 1, 2 and 4
Ans. B

 As per NITI Ayog's report, worker productivity has increased for both rural and urban areas.
 Consider the data shown below-For rural areas it was Rs. 37273 in 2004 -05 and Rs. 101755
in 2011 -12, while for urban areas it was Rs. 120419 in 2004 -05 and Rs. 282515 in 2011 -12.
Hence statement 1 is incorrect.
 Steady transition to urbanization over the years has led to a decline in the rural share
in the workforce, from 77.8% in 1993 -94 to 70.9% in 2011 -12. Hence statement 2 is
incorrect.
 The rural share in the total workforce declined steadily from 76.1 per cent in 1999-00 to 70.9
per cent in 2011-12.
 As evident from the table below, the share of the non-farm economy increased in rural
areas.
Share of rural areas in total NDP and workforce (per cent):

Year Economy Workforce


2011-12 46.9 70.9
2004-05 48.1 74.6
1999-00 48.1 76.1
1993-94 54.3 77.8
 Hence 3 is correct.
 Also, the growth rate of rural employment decreased (becoming negative in 2005-
12). Hence 4 is correct.
356. Consider the following statements
1. The weightage of food in Consumer Price Index (CPI) is higher than that in Wholesale Price
Index (WPI).
2. The WPI does not capture changes in the prices of services, which CPI does.
3. Reserve Bank of India has now adopted WPI as its key measure of inflation and to decide on
changing the key policy rates.
Which of the statements given- above is/are correct ?

https://t.me/prelimbits
371

A. 1 and 2 only
B. 2 only
C. 3 only 4,
D. 1, 2 and 3
Ans. A

THE WEIGHTAGE OF ITEMS IN WPI IS AS FOLLOWS


 Primary article - 22.62 %
 Fuel and Power - 13.51 %
 Manufactured goods - 64.23 %
WEIGHTAGE OF ITEMS IN CPI IS AS FOLLOWS
 Food and Beverages – 45.86
 Housing – 10.07
 Fuel and Light – 6.84
 Clothing and Footwear – 6.53
 Pan, tobacco and intoxicants – 2.38
 Miscellaneous – 28.32
WPI CPI
Weights of items are based on production Weight is based on average household
values. expenditure taken from consumer
expenditure data.
Weightage of food group - 24.4% Weightage of food group is 39.06%
It does not include services It includes services, hence statement 2 is
correct.
It reflects a change in averages prices for bulk It reflect the average change in prices at
sale of commodities at the wholesale level retail level paid by consumer

357. With reference to the funds under Members of Parliament Local Area Development
Scheme(MPLADS), which of the following statements are correct?
1. MPLADS funds must be used to create durable assets like physical infrastructure for health,
education etc.,
2. A specified portion of each MP‘s fund must benefit SC/ST populations
3. MPLADS funds are sanctioned on yearly basis and the unused funds cannot be carried
forward to the next year.
4. The district authority must inspect at least 10% of all works under implementation every
year
Select the correct answer using the code given below:

A. 1 and 2 only
B. 3 and 4 only
C. 1, 2 and 3 only
D. 1, 2 and 4 only
Ans. D

https://t.me/prelimbits
372

MPLAD SCHEME
 Launched in December, 1993.

 Seeks to provide a mechanism for the Members of Parliament to recommend works of


developmental nature for creation of durable community assets and for provision of
basic facilities including community infrastructure, based on locally felt needs.

 The MPLADS is a PLAN SCHEME fully funded by Government of India.

 The annual MPLADS fund entitlement per MP constituency is Rs. 5 crore.

Special focus

 MPs are to recommend every year, works costing at least 15 per cent of the MPLADS
entitlement for the year for areas inhabited by Scheduled Caste population and 7.5 per
cent for areas inhabited by S.T. population.

 In order to encourage trusts and societies for the betterment of tribal people, a ceiling of
Rs. 75 lakh is stipulated for building assets by trusts and societies subject to conditions
prescribed in the scheme guidelines.

Release of Funds

 Funds are released in the form of GRANTS IN-AID directly to the district authorities.

 The funds released under the scheme are NON-LAPSABLE.

 The liability of funds not released in a particular year is carried forward to the subsequent
years, subject to eligibility.

 The MPs have a recommendatory role under the scheme.

 The district authority is empowered to examine the eligibility of works, sanction funds
and select the implementing agencies, prioritise works, supervise overall execution, and
monitor the scheme at the ground level.

 At least 10% of the projects under implementation in the district are to be inspected
every year by the district authority.

Recommendation of works:

 The Lok Sabha Members can recommend works in their respective constituencies.

 The elected members of the Rajya Sabha can recommend works anywhere in the state
from which they are elected.

 Nominated members of the Lok Sabha and Rajya Sabha may select works for
implementation anywhere in the country.

Prelims 2021
358. The money multiplier in an economy increases with which one of the following?
A. Increase in the Cash Reserve Ratio in the banks.

https://t.me/prelimbits
373

B. Increase in the Statutory Liquidity Ratio in the banks


C. Increase in the banking habit of the people
D. Increase in the population of the country
Ans. C

Same Q was asked in 2019 [Q.306]

359. With reference to the Indian economy, demand-pull inflation can be


caused/increased by which of the following?
1. Expansionary policies
2. Fiscal stimulus
3. Inflation-indexing wages
4. Higher purchasing power
5. Rising interest rates
Select the correct answer using the code given below.

A. 1, 2 and 4 only
B. 3, 4 and 5 only
C. 1, 2, 3 and 5 only
D. 1, 2, 3, 4 and 5
Ans. A

 Inflation is the rate of increase in prices over a given period of time.


 Inflation is typically a broad measure, such as the overall increase in prices or the increase in
the cost of living in a country.
 Inflation measures how much more expensive a set of goods and services has become over
a certain period, usually a year.
 There are mainly two types of inflation →
o Demand-pull inflation - This occurs due to an increase in aggregate demand in the
economy.
o Cost-push inflation - This occurs when there is a rise in the price of raw materials,
higher taxes, e.t.c
DEMAND-PULL INFLATION IS MAINLY CAUSED DUE TO
 Depreciation of rupee.
 Low unemployment rate.
 Increased borrowing.
 Due to fiscal stimulus - It includes increased government consumption or lowering of
taxes. Hence 2 is correct.
 When the government spends more freely, prices go up. Expansionary policies lead to
more economic activity via low-interest rates, more money with the public etc. Hence
1 is correct.
 Asset inflation or Increase in Forex reserves– A sudden rise in exports forces a depreciation
of the currencies involved.

https://t.me/prelimbits
374

 Higher purchasing Power - When consumers feel confident, they spend more and take on
more debt. This leads to a steady increase in demand, which means higher prices. Hence
4 is correct.
 The rising interest rate - It decreases the money supply in the economy. This may result in
a credit crunch in the economy. It is costlier to borrow money in the economy and it leads
to a decreased money supply. So, it can not cause demand-pull inflation in the
economy Hence 5 is not correct.
 Inflation-indexing wages - Inflation indexing wages, wages in the economy is linked to
inflation which means wage moves as inflation changes in the economy. Such indexing is
provided to reduce the effect of inflation on wages. For example - a worker is getting 100 rs
as a wage and inflation in the economy increases to 5%, so the wage of the worker
increases by 5% i.e. 105. So effective change in the wages is zero and it does not
increase/decrease purchasing power. So, it can not lead to a demand to pull inflation in the
economy. Hence 3 is not correct.
360. With reference to India, consider the following statements:
1. Retail investors through Demat account can invest in Treasury Bills and Government of India
Debt Bonds in the primary market
2. The ―Negotiated Dealing System-Ordering Matching‖ is a government securities trading
platform of the Reserve Bank of India.
3. The ―Central Depository Services Ltd‖ is jointly promoted by the Reserve Bank of India and
the Bombay Stock Exchange.
Which of the statements given above is/are correct?

A. 1 only
B. 1 and 2
C. 3 only
D. 2 and 3
Ans. B

 Retail investors have multiple channels to invest in treasury bills (T-Bills) and Government
bonds. It is mandatory to open demat account for a retail investor to invest in ‗Treasury
Bills‘ and ‗Government of India Debt Bonds‘ in primary market.
 The Negotiated Dealing System Order Matching is an electronic trading platform
operated by the Reserve Bank of India to facilitate the issuing and exchange of
government securities and other types of money market instruments.
 CDSL was promoted by BSE Ltd. jointly with leading banks such as State Bank of India, Bank
of India, Bank of Baroda, HDFC Bank, Standard Chartered Bank and Union Bank of India.
361. In India, the Central Bank‟s function as the “lender of last resort” usually refers to
which of the following?
1. Lending to trade and industry bodies when they fail to borrow from other sources
2. Providing liquidity to the banks having a temporary crisis

https://t.me/prelimbits
375

3. Lending to governments to finance budgetary deficits


Select the correct answer using the code given below

A. 1 and 2
B. 2 only
C. 2 and 3
D. 3 only
Ans. B

LENDER OF LAST RESORT


 A lender of last resort (LoR) is an institution, usually a country's central bank, that offers
loans to banks or other eligible institutions that are experiencing financial
difficulty or are considered highly risky or near collapse.
 In India, RBI is the lender of last resort.
 Lender of last resort does not lend money to trade and industry bodies when they fail to
borrow from other sources. Hence, statement 1 is incorrect.
 The Reserve Bank extends this facility to protect the interest of the depositors of the
bank and to prevent possible failure of the bank, which in turn may also affect other
banks and institutions and can have an adverse impact on financial stability and
thus on the economy. Hence, statement 2 is correct.
 RBI does lend to governments to finance budget deficits but it does not come under Lender
of last resort. Hence, statement 3 is incorrect.
362. Consider the following statements:
1. The Governor of the Reserve Bank of India (RBI) is appointed by the Central Government.
2. Certain provisions in the Constitution of India give the Central Government the right to
issue directions to the RBI in the public interest.
3. The Governor of the RBI draws his power from the RBI Act.
Which of the above statements are correct?

A. 1 and 2 only
B. 2 and 3 only
C. 1 and 3 only
D. 1, 2 and 3
Ans. C

RBI
 The RBI Governors are appointed by the government of India for a fixed time period.
 The Reserve Bank of India was established on April 1, 1935, in accordance with the
provisions of the Reserve Bank of India Act, 1934.
 RBI is not a constitutional body.
 Hence, nothing is mentioned in the Constitution of India that gives the Central
Government the right to issue directions to the RBI in the public interest.

https://t.me/prelimbits
376

 The Governor of the Reserve Bank of India is the Chief Executive Officer of the Central Bank
of India and the Ex-officio Chairman of its Central Board of Directors.
 The Governor of the RBI draws his power from the RBI Act.
Power and Function of RBI Governor →

 The RBI governors are responsible for maintaining the monetary stability in an economy.
 He plays an important role in formulating the policies of the Reserve Bank of India.
 He has the responsibility for issuing licenses to open new foreign and private banks.
 The financial system of the nation is regulated and administered by the Governor and he
only sets the parameters within which the whole financial system functions.
 The Governor of RBI manages the external trade and payment also promotes orderly
development and maintenance of the foreign exchange market in India which comes under
the foreign exchange management act, 1999.
 RBI Governor also keeps a check on rules and regulations in order to make them more
customer-friendly.
 Through Urban Bank Departments RBI governor leads and supervise primary co-operative
banks.
 RBI Governor also has a part to play in facilitating and monitoring the flow of credit to small
scale industries, rural, and agricultural sectors.
 The responsibility for regulating state cooperative banks, regional rural banks, and various
local area banks.
363. With reference to the casual workers employed in India, consider the following
statements:
1. All casual workers are entitled to employees Provident Fund Coverage
2. All casual workers are entitled to regular working hours and overtime payment
3. The government can by notification specify that an establishment or industry shall pay
wages only through its bank account.
Which of the above statements are correct?

A. 1 and 2 only
B. 2 and 3 only
C. 1 and 3 only
D. 1, 2 and 3
Ans. B

According to UPSC 2021 Answer key statement, 1 is incorrect.

THE PROVISIONS OF THE PROVIDENT FUND ACT, 1952


 ―Employee‖ means any person who is employed for wages in any kind of work, manual or
otherwise, in or in connection with the work of (an establishment), and who gets his wages
directly or indirectly from the employer.
o Employe includes any person:
o Employed by or through a contractor in or in connection with the work of the

https://t.me/prelimbits
377

establishment.
 engaged as an apprentice, not being an apprentice engaged under the Apprentices Act,
1961 (52 of 1961), or under the standing orders of the establishment.
 The Act does not distinguish between a person employed on a permanent, temporary,
contractual, or casual basis under Section 2 (f) of the EPF Act. Hence statement 1 is not
correct.
 According to the Minimum Wages (Central) Rules, 1950, when a worker works in
employment for more than nine hours on any day or for more than forty-eight hours
in any week, he shall in respect of overtime work, be entitled to wages -
o In the case of employment in agriculture, at one and a half times the ordinary rate of
wages.
o In the case of any other scheduled employment, at double the ordinary rate of
wages. Hence statement 2 is correct.
 The Central government has notified the amendment under section 6 of The Payment of
Wages Act, 2017, with the provision that "the appropriate Government may, by notification
in the Official Gazette, specify the industrial or another establishment, the employer of
which shall pay to every person employed in such industrial or another establishment, the
wages only by cheque or by crediting the wages in his bank account". Hence
statement 3 is correct.
The counterargument for Statement 1

 The Act does not distinguish between a person employed on a permanent, temporary,
contractual, or casual basis under Section 2 (f) of the EPF Act.
 Holding that an employer cannot differentiate between contractual and permanent
employees, the Supreme Court in Jan 2020 has ruled that casual workers are also entitled to
social security benefits under the Employees' Provident Funds and Miscellaneous Provisions
Act.
 West Bengal also covers All casual/temporary workmen under EPF Act.
364. Which among the following steps is most likely to be taken at the time of an
economic recession?
A. Cut in tax rates accompanied by increase in interest rate
B. Increase in expenditure on public projects
C. Increase in tax rates accompanied by reduction of interest rate
D. Reduction of expenditure on public projects
Ans. B

 An economic recession is typically defined as a decline in the gross domestic product


(GDP) for two or more consecutive quarters.
 High-interest rates are a cause of recession because they limit liquidity, or the amount of
money available to invest. Hence statements 1 and 3 are not correct
 Inflation refers to a general rise in the prices of goods and services over a period of time. As
inflation increases, the percentage of goods and services that can be purchased with the

https://t.me/prelimbits
378

same amount of money decreases.


 An increase in public expenditure rises GDP by the same amount, other things equal.
 Moreover, since income is an important determinant of consumption, that increase in
income will be followed by a rise in consumption.
 Public expenditure plays four main roles →
o It contributes to current effective demand;
o It expresses a coordinated impulse on the economy, which can be used for
stabilization, business cycle inversion, and growth purposes;
o It increases the public endowment of goods for everybody;
o It gives rise to positive externalities to the economy and society as a whole (or in
specific sectors and geographical areas), the more so through its capital
component. Hence statement 2 is correct.
365. Consider the following statements
Other things remaining unchanged, market demand for a good might increase if

1. Price of its substitute increases


2. Price of its complement increases
3. The good is an inferior good and income of the consumers increases
4. Its price falls
Which of the above statements are correct?

A. 1 and 4 only
B. 2, 3 and 4
C. 1, 3 and 4
D. 1, 2 and 3
Ans. A

 Demand for a given commodity varies directly with the price of a substitute good.
o For example, if the price of a substitute good (say, coke) increases, then demand for
a given commodity (say, cane juice) will rise as cane juice will become relatively
cheaper in comparison to coke. Hence 1 is correct.
 A complementary good or service is an item used in conjunction with another good or
service.
o When the price of a good that complements good decreases, then the quantity
demanded of one increases and the demand for the other increases.
o For example, an increase in demand for cars will lead to an increase in demand for
fuel. If the price of the complement falls, the quantity demanded of the other goods
will increase. Hence 2 is incorrect.
 Inferior Good refers to an item that becomes less desirable as the incomes of its
consumer's increases.
o inferior goods are those whose price elasticity is negative.
o As consumers‘ incomes increase, they tend to decrease their purchases of inferior
goods, opting for normal goods or luxury goods instead. Hence 3 is incorrect.
https://t.me/prelimbits
379

 The quantity demanded of a good increase when its price falls is that the: lower price
increases the real incomes of buyers, enabling them to buy more. Hence 4 is correct.
366. With reference to Urban Cooperative Banks in India, consider the following
statements:
1. They are supervised and regulated by local boards set up by the State Governments.
2. They can issue equity shares and preference shares.
3. They were brought under the purview of the Banking Regulation Act, 1949 through an
Amendment in 1996
Which of the statements given above is/are correct?

A. 1 only
B. 2 and 3 only
C. 1 and 3 only
D. 1, 2 and 3
Ans. B

CO-OPERATIVE BANKS
 Co-operative banks are registered under the Cooperative Societies Act, 1912.
 Co-operative banks are regulated by the Reserve Bank of India. Hence statement 1 is
incorrect.
 Co-operative banks are regulated by the Reserve Bank of India under the Banking
Regulation Act, 1949 and Banking Laws (Application to Cooperative Societies) Act,
1965. Hence, statement 3 is correct.
 Urban Co-operative Banks:-
o Primary (urban) co-operative banks are registered and governed by state
governments under the respective co-operative societies acts of the concerned
states.
o Primary (urban) credit societies have to meet certain criteria in order to get a license
from RBI.
o The area of operation of primary (urban) co-operative banks is confined to
metropolitan, urban or semi-urban centres and caters to the needs of small
borrowers including retail traders, small entrepreneurs, professionals and the salaried
class
 The changes to The Banking Regulation Act approved by Parliament in September 2020,
brought cooperative banks under the direct supervision of the RBI.
 Urban cooperative banks will now be treated on a par with commercial banks and a
cooperative bank can, with prior approval of the RBI, issue equity shares, preference shares,
or special shares to its members or to any other person residing within its area of operation,
by way of public issue or private placements. Hence, statement 2 is correct.
 It can also issue unsecured debentures or bonds with a maturity of not less than 10 years.
 This essentially means non-members can become shareholders of the bank, and this will
allow the RBI to merge failing banks quickly.
https://t.me/prelimbits
380

367. Indian Government Bond yields are influenced by which of the following?
1. Actions of the United States Federal Reserve
2. Actions of the Reserve Bank of India
3. Inflation and short-term interest rates.
Select the correct answer using the code given below

A. 1 and 2 only
B. 2 only
C. 3 only
D. 1, 2 and 3
Ans. D

BOND YIELD
 A government bond is a debt instrument issued by the Central and State Governments of
India.
 A government bond in India is essentially a contract between the issuer and the investor,
wherein the issuer guarantees interest earnings on the face value of bonds held by investors
along with repayment of the principal value on a stipulated date.
 Government Bonds India, fall under the broad category of government securities (G-Sec)
and are primarily long-term investment tools issued for periods ranging from 5 to 40 years.
It can be issued by both the Central and State governments of India.
 Bond yield is the return on the bond an investor will get by investing in a bond.
 Bond yield is inversely proportional to Bond price. As bond prices increase,
bond yields fall.
 Indian Government Bond Yields are influenced by →
o The monetary policy of the Reserve Bank of India.
o The course of interest rates.
o The fiscal position of the government and its borrowing program.
o Global markets.
o Economy.
o Inflation.
368. Consider the following:
1. Foreign currency convertible bonds
2. Foreign institutional investment with certain conditions
3. Global depository receipts
4. Non-resident external deposits
Which of the above can be included in Foreign Direct Investments?

A. 1, 2 and 3
B. 3 only
C. 2 and 4
D. 1 and 4
https://t.me/prelimbits
381

Ans. A

FOREIGN DIRECT INVESTMENTS


 Foreign Currency Convertible Bonds - It is a bond issued by an Indian
company in foreign currency and subscribed by a non-resident in foreign currency and
convertible into ordinary shares of the issuing company, either in whole or in part.
o FCCBs represent a debt obligation of the corporate. Investors have the option to
redeem or convert them into underlying local shares or global depository receipts. If
investors prefer to hold the FCCBs until the redemption date, the corporate has to
redeem the FCCBs on the redemption date.
o Dilution would take place as and when debt is converted into equity. Since these
bonds are convertible into equity shares over a period of time as provided in the
instrument, therefore they are covered under FDI policy & counted towards FDI.
o If they are redeemed they count as ECB & a debt obligation, only on converting
into equity it is counted towards FDI. Hence, statement 1 is correct.
 Foreign institutional investment with certain conditions -
o In India, as per SEBI (FPI regulations), 2019, a particular FII is allowed to invest
upto 10% of the paid-up capital of a company, which implies that any
investment above 10% will be construed as FDI. Hence, statement 2 is correct.
o According to IMF and OECD definitions, the acquisition of at least 10% of the
ordinary shares or voting power in a public or private enterprise by non-
resident investors makes it eligible to be categorized as foreign direct investment.
o Global Depository Receipt - Any instrument issued in the form of a depository
receipt or certificate created by the oversees depository bank outside India and
issued to non-resident investors against underlying shares or foreign currency
convertible bonds of issuing company.
o GDRs are equity representing share-holders funds, foreign investment in the form of
equity shares issued outside India by a Depository Bank, on behalf of an Indian
company which is covered under the FDI policy. GDR proceeds are reckoned as
Foreign Direct Investment. Hence, statement 3 is correct.
 Non-resident external deposits - NRI investments that are repatriable are considered FDI
while non-repatriable investments are considered domestic investments. Hence, statement
4 is incorrect.
369. Consider the following statements:
The effect of the devaluation of a currency is that it necessarily

1. Improves the competitiveness of the domestic exports in the foreign markets


2. Increase the foreign value of the domestic currency
3. Improves the trade balance
Which of the above statements is/are

A. 1 only

https://t.me/prelimbits
382

B. 1 and 2
C. 3 only
D. 2 and 3
Ans. A

DEVALUATION OF A CURRENCY
 Devaluation of a currency means a reduction in the value of a currency vis-a-vis major
internationally traded currencies.
 Devaluation occurs when a country intentionally reduces the value of its
currency relative to one or more foreign countries. Hence, statement 2 is incorrect.
 When the country follows a fixed exchange rate regime the government constantly has to
revalue and devalue the currency to maintain the pegged exchange rate.
 When there is upwards market pressure on the currency to appreciate, the central bank will
artificially devalue the currency by buying up foreign reserves.
 Devaluation occurs when a government wishes to increase its balance of
trade by decreasing the relative value of its currency.
 The government does this by adjusting the fixed or semi-fixed exchange rate of its currency
versus that of another country.
 Exports become cheaper and more competitive to foreign buyers. Higher exports
relative to imports can increase aggregate demand as increased consumer spending on
domestic goods and services. Hence, it improves the competitiveness of the domestic
exports in the foreign markets. Hence, statement 1 is correct.
 With exports more competitive and imports more expensive, we may see higher
exports and lower imports, which will reduce the current account deficit.
 Devaluation of currency increases the volume of exports and reduces the volume of
imports, both of which have a favourable effect on the balance of trade, that is, they will
lower the trade deficit or increase the trade surplus. Hence, statement 3 is incorrect.
370. Which one of the following effects of creation of black money in India has been the
main cause of worry to the Government of India?
A. Diversion of resources to the purchase of real estate and investment in luxury housing
B. Investment in unproductive activities and purchase of precious stones, jewellery, gold etc.
C. Large donations to political parties and growth of regionalism
D. Loss of revenue to the State Exchequer due to tax evasion
Ans. D

BLACK MONEY
 Black Money in India, is a term used to describe money that is illegitimately owned.
 Black money includes all funds earned through illegal activity and otherwise legal income
that is not recorded for tax purposes.
 The generation of black income has been resulting in a huge loss of revenue to the

https://t.me/prelimbits
383

state exchequer due to continuous evasion of taxes both from direct and indirect
taxes.
 Black money and tax evasion which go hand in hand, have also the effect of seriously
undermining the equity concept of taxation and warping its progressiveness.
 Together, they throw a greater burden on the honest taxpayer and lead to economic
inequality and concentration of wealth in the hands of the unscrupulous few in the country.
371. Which one of the following is likely to be one of the most inflationary in its effects?
A. Repayment of public debt
B. Borrowing from the public to finance a budget deficit
C. Borrowing from the banks to finance a budget deficit
D. Creation of new money to finance a budget deficit
Ans. D

 When the government finances its budget deficit through the creation of new high
powered money and in the process causes inflation, the purchasing power of old
money balances held by the public falls.
 The monetary base (or M0) is the total amount of a currency that is either in general
circulation in the hands of the public or in the form of commercial bank deposits held in the
central bank's reserves.
 The creation of new money will increase the monetary base which in turn increase the
money supply that ultimately increases inflation.

https://t.me/prelimbits
384

Prelims 2022
372. Rapid Financing Instrument” and “Rapid Credit Facility” are related to the provisions
of lending by which of the following:
A. Asian Development Bank
B. International Monetary Fund
C. United Nations Environment Programme Finance Initiative
D. World Bank
Ans. B

RAPID FINANCING INSTRUMENT (RFI)


For higher income countries

 The Rapid Financing Instrument (RFI) provides rapid financial assistance, which is
available to all member countries facing an urgent balance of payments need.

 The RFI was created as part of a broader reform to make the IMF‘s financial support
more flexible to address the diverse needs of member countries.

 The RFI replaced the IMF‘s previous emergency assistance policy and can be used in a wide
range of circumstances.

https://www.imf.org/en/About/Factsheets/Sheets/2016/08/02/19/55/Rapid-Financing-Instrument

RAPID CREDIT FACILITY (RCF)


For low-income countries (LICs)

 The Rapid Credit Facility (RCF) provides rapid concessional financial assistance to low-
income countries (LICs) facing an urgent balance of payments (BoP) need with no ex post
conditionality where a full-fledged economic program is neither necessary nor
feasible.

 The RCF was created under the Poverty Reduction and Growth Trust (PRGT) as part of a
broader reform to make the Fund‘s financial support more flexible and better tailored to the
diverse needs of LICs, including in times of crisis.

https://www.imf.org/en/About/Factsheets/Sheets/2016/08/02/21/08/Rapid-Credit-Facility

373. With reference to the Indian economy, consider the following statements:
1. An increase in Nominal Effective Exchange Rate (NEER) indicates the appreciation of rupee.
2. An increase in the Real Effective Exchange Rate (REER) indicates an improvement in trade
competitiveness.
3. An increasing trend in domestic inflation relative to inflation in other countries is likely to
cause an increasing divergence between NEER and REER.
Which of the above statements are correct?

A. 1 and 2 only

https://t.me/prelimbits
385

B. 2 and 3 only
C. 1 and 3 only
D. 1, 2 and 3
Ans. C

THE NOMINAL EFFECTIVE EXCHANGE RATE (NEER)


 It is an unadjusted weighted average rate at which one country's currency exchanges
for a basket of multiple foreign currencies.
 The nominal exchange rate is the amount of domestic currency needed to purchase foreign
currency.
 It is a measure of the value of a currency against a weighted average of several foreign
currencies. An increase in NEER indicates an appreciation of the rupee. Hence, Statement
1 is correct.
THE REAL EFFECTIVE EXCHANGE RATE (REER)
 It is the weighted average of a country's currency in relation to an index or basket of
other major currencies.
 The weights are determined by comparing the relative trade balance of a country's
currency against that of each country in the index.
 An increase in REER implies that exports become more expensive and imports
become cheaper; therefore, an increase indicates a loss in trade competitiveness. Hence,
Statement 2 is not correct.
 The NEER is the weighted geometric average of the bilateral nominal exchange rates of
the home currency in terms of foreign currencies.
 The REER is the weighted average of NEER adjusted by the ratio of domestic prices to
foreign prices.
 An increasing trend in domestic inflation relative to inflation in other countries creates a
divergence in NEER and REER. Hence, Statement 3 is correct.
374. With reference to the Indian economy, consider the following statements:
1. If the inflation is too high, Reserve Bank of India (RBI) is likely to buy government securities.
2. If the rupee is rapidly depreciating, RBI is likely to sell dollars in the market.
3. If interest rates in the USA or European Union were to fall, that is likely to induce RBI to buy
dollars.
Which of the statements given above are correct?

A. 1 and 2 only
B. 2 and 3 only
C. 1 and 3 only
D. 1, 2 and 3
Ans. B

 Government security (G-Sec) →


o It is a tradeable instrument issued by the central government or state
https://t.me/prelimbits
386

government.
o It acknowledges the government‘s debt obligations.
o The G-Secs issuances are managed by the RBI, who on behalf of the Centre,
regularly conducts G-Sec auctions every Friday.
o State Government transactions are carried out by RBI in terms of the agreement
entered into with the State Governments.
o If the inflation is high RBI tries to reduce the liquidity of the market, by selling
Government securities to the public via open market operation. Hence,
Statement 1 is not correct.
 Rupee depreciation →
o It means to fall in the value of the rupee with respect to the dollar.
o In a free-floating exchange rate regime, depreciation takes place when the
demand for the dollar is more than the supply. Hence, RBI is likely to sell
dollars in the economy to increase the supply of the dollar. Hence, Statement
2 is correct.
 If the interest rate in US and EU falls, there will be an inflow of dollars in the Indian
market, leading to an appreciation of the rupee.
 To reduce the supply of dollars in the economy, RBI will like to buy the dollars from
the market. Hence, Statement 3 is correct.
375. With reference to the “G20 Common Framework”, consider the following
statements:
1. It is an initiative endorsed by the G20 together with the Paris Club.
2. It is an initiative to support Low Income Countries with unsustainable debt.
Which of the statements given above is/are correct?

A. 1 only
B. 2 only
C. Both 1 and 2
D. Neither 1 nor 2
Ans. C

G20 COMMON FRAMEWORK


 Chad has become the first country to officially request a debt restructuring under a new
common framework ―G20 Common Framework‖ introduced by China and other groups
of 20 countries last year with the help of the Paris Club.
 It is the Common Framework for Debt Treatments beyond the Debt Service Suspension
Initiative (DSSI).
 Initiative endorsed by G20 together with Paris Club. Hence, Statement 1 is correct.
 The Common Framework is intended to deal with insolvency and protracted
liquidity problems, along with the implementation of an IMF-supported
reform program.

https://t.me/prelimbits
387

 It was announced in November 2020 to deal with the issue of unsustainable debts
faced by various countries as an impact of COVID-19. Hence, Statement 2 is correct.
 Paris Club is a club or group of officials from major creditor countries.
o It was established in the year 1956.
o It aims to find sustainable solutions to the difficulties faced by debtor countries in
payments.
G20
 It is an international forum for global economic cooperation.
 It has 20 members, and these countries are as follows- Argentina, Australia, Brazil, Canada,
China, France, Germany, India, Indonesia, Italy, Japan, Republic of Korea, Mexico, Russia,
Saudi Arabia, South Africa, Turkey, the United Kingdom, the United States, and the
European Union.
 G20 members account for 85 percent of the world economy, 75 percent of global trade,
and two-thirds of the world's population.
 G20 was formed in 1999 and its present chairman is Joko Widodo, President of Indonesia
 Canadian finance minister Paul Martin was chosen as the first chairman and German finance
minister Hans Eichel hosted the inaugural meeting.

376. With reference to the India economy, what are the advantages of “Inflation-Indexed
Bonds (IIBs)”?
1. Government can reduce the coupon rates on its borrowing by way of IIBs.
2. IIGs provide protection to the investors from uncertainty regarding inflation.
3. The interest received as well as capital gains on IIBs are not taxable.
Which of the statements given above are correct?

https://t.me/prelimbits
388

A. 1 and 2 only
B. 2 and 3 only
C. 1 and 3 only
D. 1, 2 and 3
Ans. A

INFLATION-INDEXED BONDS
 It provides a continuous return to investors regardless of the amount of inflation in the
economy.
 The real coupon interest rate on IIBs is fixed, but the nominal principal value is
adjusted for inflation. Therefore, We can say Government can reduce the coupon rates on
its borrowing by way of IIBs. Hence, Statement 1 is correct.
 On the modified principal value, periodic coupon payments will be issued. In this way, both
the principal and the coupon payment will be protected from inflation. Hence, Statement
2 is correct.
 When the bond matures, the adjusted principal or face value, whichever is greater, will
be paid.
 IIBs are classified as government securities (G-Sec) and hence qualify for repo
transactions, as well as SLR status (i.e., they are eligible to be kept as part of Statutory
Liquidity Ratio requirements of banks).
 IIBs assist the small investor in protecting even the principal amount against inflation,
in addition to obtaining the investment's yield, which is dependent on the current inflation
rate.
 It's also predicted to enhance domestic savings and reverse the savings-to-GDP ratio's
downward trend.
Salient Features of Inflation index bonds

 Inflation-indexed bonds were announced in the 2013 budget.


 Anyone can invest in these bonds, however, they are distributed in such a way that
institutional investors (such as LICs and mutual funds) receive 80% of the bonds and retail
investors receive 20%.
 These bonds are auctioned directly by the RBI. The government receives this money.
 Minimum and maximum investment amounts are 10,000 and 25 lakhs, respectively.
 These bonds can only be redeemed after ten years, or else a penalty will be imposed.
 These bonds can be traded in the secondary market (through the BSE, NSE, and other stock
exchanges), however, if they are sold in the secondary market and profit is made, capital
gains tax is to be paid. Hence, Statement 3 is not correct.
377. With reference to foreign-owned e-commerce firms operating in India, which of the
following statements is/are correct?
1. They can sell their own goods in addition to offering their platforms as market-places.
2. The degree to which they can own big sellers on their platforms is limited.
Select the correct answer using the code given below:
https://t.me/prelimbits
389

A. 1 only
B. 2 only
C. Both 1 and 2
D. Neither 1 nor 2
Ans. B

E-COMMERCE RULES -2021


 The Consumer Protection (E-Commerce) Rules, 2020, notified on July 23, regulate all
commercial transactions involving goods or services, sold over a digital or electronic
network by retailers in India or overseas to consumers in India.
 The e-com rules currently recognise two e-commerce business models, namely, the
marketplace model and the inventory-based model. While regulating e-commerce entities,
the rules have separate specified provisions for the marketplace- and inventory-based
entities, and for sellers who sell on the platform operated by a marketplace e-commerce
entity.
 The rules barred online marketplaces from holding inventory of their own or
influencing the price of goods on their platforms.
 They also prohibited group companies or entities in which marketplaces have control of
inventory to sell on their platforms, among other things.
 E-commerce companies must ensure none of their "related parties and associated
enterprises" are listed as sellers on their shopping websites, and no related entity
should sell goods to an online seller operating on the same platform. Hence,
Statement 1 is not correct.
 Big sellers have a limit of 20% for sale on the e-commerce platform. Hence, Statement
2 is correct.
378. Which of the following activities constitute real sector in the economy?
1. Farmers harvesting their crops
2. Textile mills converting raw cotton into fabrics
3. A commercial bank lending money to a trading company
4. A corporate body issuing Rupee Denominated Bonds overseas
Select the correct answer using the code given below:

A. 1 and 2 only
B. 2, 3 and 4 only
C. 1, 3 and 4 only
D. 1, 2, 3 and 4
Ans. A

THE REAL SECTOR OF AN ECONOMY


 It is the key section as activities of this sector persuade economic output and is
represented by those economic segments that are essential for the progress of the
GDP of the economy.

https://t.me/prelimbits
390

 The sector generates better outcomes if accompanied by a healthier financial system; thus,
the advancement of the financial sector is a means for the growth of the real sector.
 Its statistics cover data and indicators of production in both agricultural and industrial
sectors, gross domestic products (GDP), private investment and consumption, wholesale
and consumer price indices/inflation, and employment.
 The real sector of the economy deals with the production side, while the nominal
economy deals with the financial side. Therefore, Farmers harvesting their crops and Textile
mills converting raw cotton into fabrics are the parts of the Real Sector of the
Economy. Hence, statements 1 and 2 are correct.
 Financial activities majorly support real(production) activity but do not contribute itself too
much except the factor incomer it generates. Therefore, A commercial bank lending money
to a trading company and A corporate body issuing Rupee Denominated Bonds overseas is
part of the financial sector. Hence, statements 3 and 4 are not correct.
379. Which one of the following situations best reflects “Indirect Transfers” often talked
about in media recently with reference to India?
A. An Indian company investing in a foreign enterprise and paying taxes to the foreign country
on the profits arising out of its investment
B. A foreign company investing in India and paying taxes to the country of its base on the
profits arising out of its investment
C. An Indian company purchases tangible assets in a foreign country and sells such assets after
their value increases and transfers the proceeds to India
D. A foreign company transfers shares and such shares derive their substantial value from
assets located in India
Ans. D

INDIRECT TRANSFER
 Its provisions deal with the taxation of transactions wherein even though the transfer of
shares took place overseas, the underlying assets were in India.
 It refers to situations where when foreign entities own shares or assets in India, the
shares of such foreign entities are transferred instead of a direct transfer of the underlying
assets in India. Hence, Option 4 is correct.
 Indirect transfer provisions were introduced in the I-T act in 2012 with retrospective effect,
as the government sought to bring Vodafone Group Plc.‘s $11 billion acquisition
of Hutchison Essar Ltd in 2007 (by acquiring a Cayman subsidiary owned by Hutchison
International) and the other such transactions under the tax net in India.
 Only those indirect transfer transactions wherein more than 50% of the underlying
assets are in India will be subject to a levy of capital gains tax in India. But the
clarifications also extended the tax to funds, including those outside India.

https://t.me/prelimbits
391

380. With reference to the expenditure made by an organisation or a company, which of


the following statements is/are correct?
1. Acquiring new technology is capital expenditure.
2. Debt financing is considered capital expenditure, while equity financing is considered
revenue expenditure.
Select the correct answer using the code given below:

A. 1 only
B. 2 only
C. Both 1 and 2
D. Neither 1 nor 2
Ans. A

CAPITAL EXPENDITURE
 It is the expenditure by the government for the development of fixed assets.
 If an item has a useful life of more than one year, it is capitalized (i.e., can be
considered CapEx). Capital expenditure is a payment for goods or services recorded - or
capitalized - on the balance sheet.
 Capital expenditure is used to create assets or to reduce liabilities.
 It consists of: Long-term investments by the government in creating assets such as roads
and hospitals, and
 The money was given by the government in the form of loans to states or repayment of its
borrowings.
 Therefore, Acquiring new technology is considered a capital expenditure as it will generate
profit in the future time and helps in the creation of new assets. Hence, Statement 1 is
correct.
 When a company borrows money to be paid back at a future date with interest it is known
as debt financing. Repayment of loan is an example of capital expenditure. Equity
financing is the process of raising capital through the sale of shares. It is an example of
non-debt capital receipts. Capital receipts are receipts that create liabilities or reduce
financial assets. They also refer to incoming cash flows.
o Examples of non-debt capital receipts: Recovery of loans and
advances, disinvestment, issue of bonus shares, etc.
REVENUE EXPENDITURE
 It is the expenditure by the government which does not impact its assets or liabilities. For
example, this includes salaries, interest payments, pensions, and administrative expenses.
381. With reference to the Indian economy, consider the following statements:
1. A share of the household financial savings goes towards government borrowings.
2. Dated securities issued at market-related rates in auctions form a large component of
internal debt.
Which of the above statements is/are correct?

https://t.me/prelimbits
392

A. 1 only
B. 2 only
C. Both 1 and 2
D. Neither 1 nor 2
Ans. C

HOUSEHOLD FINANCIAL SAVINGS


 It refers to currency, bank deposits, debt securities, mutual funds, pension funds,
insurance, and investments in small savings schemes.
 The total of these savings is referred to as gross household financial savings.
 Once financial liabilities, including loans from banks, non-banking financial companies
(NBFCs), and housing finance companies, are subtracted from gross savings, what remains is
referred to as net household financial savings.
 A share of household financial savings goes to government borrowings, as part of
public accounts of India. It mainly consists of provident fund. Hence, Statement 1 is
correct.
DATED GOVERNMENT SECURITIES
 It is government securities or bonds which are long-term, the tenor ranging from 5
years to 30 years.
 These have a fixed or floating coupon (or interest rate) associated with them which are paid
on the face value at fixed intervals.
 The securities can be issued by both the centre and the state governments to mobilise
funds.
 The government issues these funds to finance a fiscal deficit.
 Dated securities mean regular government bonds, whereas T-bills are considered
separately.
 Dated securities issued at market-related rates comprise a large share of internal
debt. Hence, Statement 2 is correct.
382. With reference to Ayushman Bharat Digital Mission, consider the following
statements:
1. Private and public hospitals must adopt it.
2. As it aims to achieve universal health coverage, every citizen of India should be part of it
ultimately.
3. It has seamless portability across the country.
Which of the statements given above is/are correct?

A. 1 and 2 only
B. 3 only
C. 1 and 3 only
D. 1, 2 and 3
Ans. B

https://t.me/prelimbits
393

AYUSHMAN BHARAT DIGITAL MISSION


 It was launched in September 2021 by the Prime Minister through a video conference.
 It aims to provide digital health IDs for all Indian citizens to help hospitals, insurance
firms, and citizens access health records electronically when required.
 The pilot project of the Mission had been announced by the Prime Minister from the
ramparts of the Red Fort on 15th August 2020.
 The project is being implemented in the pilot phase in six States & Union Territories.
 The National Health Authority (NHA) under the Ministry of Health and Family Welfare will
be the implementing Agency.
 Participation in Ayushman Bharat Digital Mission is voluntary including for citizens.
 Participation in a healthcare facility or an institution is also voluntary and shall be taken
by the respective management (government or private management). Hence statement 1
and 2 is not correct.
 However, once the management decides to register the respective healthcare
facility/institution in Ayushman Bharat Digital Mission, it is essential for all the healthcare
professionals serving the said facility/institution to register in Healthcare Professionals
Registry so that the institution can become fully integrated with the National Digital Health
Ecosystem (NDHE).
 It provides for seamless portability across the country. Hence statement 3 is correct.
383. Consider the following statements:
1. Tight monetary policy of US Federal Reserve could lead to capital flight.
2. Capital flight may increase the interest cost of firms with existing External Commercial
Borrowings (ECBs).
3. Devaluation of domestic currency decreases the currency risk associated with ECBS.
Which of the statements given above are correct?

A. 1 and 2 only
B. 2 and 3 only
C. 1 and 3 only
D. 1, 2 and 3
Ans. A

TIGHT MONETARY POLICY


 Tight monetary policy refers to the actions that a central bank takes to limit inflation and
an overheating economy. Tight monetary policy is commonly called contractionary
monetary policy.
 Tight monetary policy, or contractionary monetary policy, typically occurs when a central
bank wants to keep inflation under control.
 If there has been too much spending and borrowing by consumers and businesses, the
economy can become overheated and that could considerably raise the price level of goods
and services.

https://t.me/prelimbits
394

o Inflation is the rise in the price level of items, such as groceries or clothes, over time.
 To minimize or slow down inflation, a central bank could make it more expensive for
consumers to spend money and businesses to borrow money by raising interest rates.
This is a form of contractionary monetary policy—it restricts, or contracts,
spending.
Capital Flight

 In economics, capital flight is a phenomenon characterized by large outflows of assets


and/or capital from a country due to some events, resulting in negative economic
consequences to that country.
 In this context, Capital Flight will be induced due to the tight monetary policy of the US
federal reserve.
Tight Monetary policy and Its Fallout

 The US Federal Reserve is facing a tough job of walking the tightrope between controlling
the red-hot inflation and supporting growth.
 The situation has become more challenging in the backdrop of volatile crude and
geopolitical tensions casting a dome of uncertainty.
 The Fed policymakers aim to make borrowing more expensive so that consumers and
businesses hold making investments, thereby cooling off demand and hopefully the prices.
 Higher interest rates in the US usually lead to foreign investors pulling their money
from emerging markets like India back to the US for safer, and more secure
returns leading to capital flight. Hence statement 1 is correct.
 Foreign institutional investors have already sold over Rs 2 lakh crore worth of Indian
equities since October 2021.
 Further capital flight will put pressure on the RBI to hike interest rates or lead to rupee
depreciation against the dollar, which again would lead to imported inflation for India.
 Capital flight may increase the interest cost of firms with existing External Commercial
borrowing (ECBs) as the capital flight would lead to depreciation in the value of the currency
and create supply-side restraints for borrowers. Hence statement 2 is correct.
 Devaluation of domestic currency will inadvertently increase the currency risk associated
with ECBs and will result in higher interest costs for borrowers, Hence statement 3 is
incorrect. Hence statement 3 is incorrect.
384. Consider the following statements:
1. In India, credit rating agencies are regulated by Reserve Bank of India.
2. The rating agency popularly known as ICRA is a public limited company.
3. Brickwork Ratings is an Indian credit rating agency.
Which of the statements given above are correct?

A. 1 and 2 only
B. 2 and 3 only
C. 1 and 3 only

https://t.me/prelimbits
395

D. 1, 2 and 3
Ans. B

CREDIT RATING AGENCIES (CRA)


 It analyzes a debtor‘s ability to repay the debt and also rates its credit risk.
 All the credit rating agencies in India are regulated by SEBI (Credit Rating Agencies)
Regulations, 1999 of the Securities and Exchange Board of India Act, 1992. Hence,
Statement 1 is not correct.
 There are a total of six credit agencies in India viz, CRISIL, CARE, ICRA, SMREA, Brickwork
Rating, and India Rating and Research Pvt. Ltd.
 ICRA Limited It is a public limited company that was set up in 1991 in Gurugram.
Hence, Statement 2 is correct.
 The company was formerly known as Investment Information and Credit Rating Agency of
India Limited.
BRICKWORK RATINGS
 It is recognized as an external credit assessment agency (ECAI) by the Reserve Bank of
India (RBI) to carry out credit ratings in India. Hence, Statement 3 is correct.
 Brickwork Rating was established in 2007 and is promoted by Canara Bank. It offers ratings
for bank loans, SMEs, corporate governance ratings, municipal corporations, capital market
instruments, and financial institutions.
385. With reference to the „Banks Board Bureau (BBB)‟, which of the following
statements are correct?
1. The Governor of RBI is the Chairman of BBB.
2. BBB recommends for the selection of heads for Public Sector Banks.
3. BBB helps the Public Sector Banks in developing strategies and capital raising plans.
Select the correct answer using the code given below:

A. 1 and 2 only
B. 2 and 3 only
C. 1 and 3 only
D. 1, 2 and 3
Ans. B

Now being abolished (new body Financial Services Institutions)

Refer Q. 308

386. Convertible Bonds, consider the following statements:


1. As there is an option to exchange the bond for equity, Convertible Bonds pay a lower rate
of interest.
2. The option to convert to equity affords the bondholder a degree of indexation to rising
consumer prices.
Which of the statements given above is/are correct?

https://t.me/prelimbits
396

A. 1 only
B. 2 only
C. Both 1 and 2
D. Neither 1 nor 2
Ans. C

CONVERTIBLE BONDS
 A convertible bond pays fixed-income interest payments but can be converted into a
predetermined number of common stock shares.
 It is a hybrid security that offers investors the best of both stocks and bonds.
 It typically carries lower interest rate payments than straight corporate bonds the savings
in interest expense can be significant.
 Investors accept the lower interest payments because the conversion option offers the
opportunity to benefit from increases in the stock price. Hence, Statement 1 is correct.
 Governments might use indexation as a way to potentially alleviate the negative effects
inflation can have on the recipients of transfer payments and entitlements. Hence,
Statement 2 is correct.
 Social Security payments, for example, are indexed to the annual increase in the Consumer
Price Index.
 Issuing convertible bonds can help companies minimize the negative investor
sentiment that would surround equity issuance.
 Each time a company issues additional shares or equity, it adds to the number of shares
outstanding and dilutes existing investor ownership.
 The company might issue convertible bonds to avoid negative sentiment.
 Bondholders can, then, convert into equity shares should the company perform well.
387. In India, which one of the following is responsible for maintaining price stability by
controlling inflation?
A. Department of Consumer Affairs
B. Expenditure Management Commission
C. Financial Stability and Development Council
D. Reserve Bank of India
Ans. D

THE RESERVE BANK OF INDIA


 It was established on April 1, 1935, by the provisions of the Reserve Bank of India Act,
1934.
 The Central Office of the Reserve Bank was initially established in Calcutta but
was permanently moved to Mumbai in 1937. The Central Office is where the Governor
sits and where policies are formulated.
 Though originally privately owned, since its nationalization in 1949, the Reserve Bank is
fully owned by the Government of India.

https://t.me/prelimbits
397

Major Functions

 To regulate the issue of Bank notes and keeping of reserves to secure monetary
stability in India and generally to operate the currency and credit system of the country
to its advantage.
 To have a modern monetary policy framework to meet the challenge of an increasingly
complex economy.
 To maintain price stability while keeping in mind the objective of growth. Hence,
Option 4 is correct.
388. With reference to Non-Fungible Tokens (NFTs), consider the following statements:
1. They enable the digital representation of physical assets.
2. They are unique cryptographic tokens that exist on a blockchain.
3. They can be traded or exchanged at equivalency and therefore can be used as a medium
transactions. of commercial
Which of the statements given above are correct?

A. 1 and 2 only
B. 2 and 3 only
C. 1 and 3 only
D. 1, 2 and 3
Ans. A

NON FUNGIBLE TOKENS (NFTS)

 Anything that can be converted into a digital form can be an NFT. Everything from
your drawings, photos, videos, GIF, music, in-game items, selfies, and even a tweet
can be turned into an NFT, which can then be traded online using cryptocurrency.

 But what makes NFTs unique from other digital forms is that it is backed by Blockchain
technology. For the uninitiated, Blockchain is a distributed ledger where all transactions
are recorded. It is like your bank passbook, except all your transactions are transparent and
can be seen by anyone and cannot be changed or modified once recorded.

 NFTs are gaining massive popularity now because they are becoming an increasingly
popular way to showcase and sell your digital artwork. Billions of dollars have been spent
on NFTs since its inception—which date backs to 2015, and Terra Nulius was the first
NFT on Ethereum Blockchain, although this project was merely an idea which only allowed
to customise a short message which was then recorded on blockchain. Then came Curio
Cards, CryptoPunks and CryptoCats in 2017, before NFTS slowly moved into public
awareness, then expanding into mainstream adoption in early 2021.

https://t.me/prelimbits
398

How do NFTs work

 NFT works on blockchain as it gives users complete ownership of a digital asset. For
instance, if you‘re a sketch artist, and if you convert your digital asset to an NFT, what
you get is proof of ownership, powered by Blockchain.

So why are people willing to spend millions on something they could easily screenshot or
download?

 In simple words, when you list your NFT on a marketplace, you pay something called a Gas
Fee (Transaction Fee) for using the Blockchain, following which your digital art is then
recorded on Blockchain, mentioning that you (your address) own the particular NFT.
This gives you full ownership—which cannot be edited or modified by anyone, including
the marketplace owner.

How is an NFT different from cryptocurrency?

 NFTs and cryptocurrencies are very different from each other. While both are built on
Blockchain, that is where the similarity ends.

 Cryptocurrency is a currency and is fungible, meaning that it is interchangeable.

o For instance, if you hold one crypto token, say one Ethereum, the next Ethereum that
you hold will also be of the same value.

 But NFTs are non-fungible, that means the value of one NFT is not equal to another.
Every art is different from other, making it non fungible, and unique.

389. In India, which one of the following compiles information on industrial disputes,
closures, retrenchments and lay-offs in factories employing workers?
A. Central Statistics Office
B. Department for Promotion of Industry and Internal Trade
C. Labour Bureau
D. National Technical Manpower Information System
Ans. C

LABOUR BUREAU
 It is an attached office under the Ministry of Labour and Employment, which was set
up on 1st October 1946.
 It is entrusted with the work of compilation, collection, analysis and dissemination of
statistics on different aspects of labour.
 Labour Bureau has two main wings stationed in Shimla and Chandigarh.
 The functions/activities of the Labour Bureau can be classified under three major heads:
o Labour Intelligence
o Labour Research
o Monitoring and evaluation studies under the Minimum Wages Act 1948.

https://t.me/prelimbits
399

 Compilation and maintenance of Consumer Price Index Numbers for Industrial


Workers, Agricultural/Rural Labourers, Retail Price Index of Selected Essential
Commodities in Urban Areas etc.
 Quick Employment Survey and Employment-Unemployment survey are also being
conducted by Labour Bureau.
 The Departments of Labour in the States and Regional Labour Commissioners (Central)
collect the basic information from the affected Primary Units in respect of the work
stoppages in the State and Central Spheres respectively on account of strikes and lockouts.
 It compiles information on industrial disputes, closures, retrenchments and lay-offs in
factories employing workers. Hence, Option 3 is correct.

https://t.me/prelimbits
400

Prelims 2023
390. Consider the following statements with reference to India:
1. According to the ‗Micro, Small and Medium Enterprises Development (MSMED) Act, 2006‘,
the ‗medium enterprises‘ are those with investments in plant and machinery between Rs. 15
crore and Rs. 25 crore.

2. All bank loans to the Micro, Small and Medium Enterprises qualify under the priority sector.

Which of the statements given above is/are correct?

A. 1 only

B. 2 only

C. Both 1 and 2

D. Neither 1 nor 2

Ans. D

DEFINITIONS OF MICRO, SMALL & MEDIUM ENTERPRISES (2006)


In accordance with the provision of Micro, Small & Medium Enterprises Development (MSMED)
Act, 2006 the Micro, Small and Medium Enterprises (MSME) are classified in two Classes →

 Manufacturing Enterprises

o The enterprises engaged in the manufacture or production of goods pertaining to


any industry specified in the first schedule to the industries (Development and
regulation) Act, 1951) or employing plant and machinery in the process of value
addition to the final product having a distinct name or character or use. The
Manufacturing Enterprise are defined in terms of investment in Plant &
Machinery.

 Service Enterprises

o The enterprises engaged in providing or rendering of services and are defined in


terms of investment in equipment..

Manufacturing Sector

Enterprises Investment in Plant & Machinery

Micro Enterprises Does not exceed twenty five lakh rupees

Small Enterprises More than twenty five lakh rupees but does not exceed five crore rupees

Medium Enterprises More than five crore rupees but does not exceed ten crore rupees

Service Sector

https://t.me/prelimbits
401

Enterprises Investment in Equipment

Micro Enterprises Does not exceed ten lakh rupees:

Small Enterprises More than ten lakh rupees but does not exceed two crore rupees

Medium Enterprises More than two crore rupees but does not exceed five core rupees

REVISED MSME CLASSIFICATION

PRIORITY SECTOR LENDING (PSL) TO MSME


 All bank loans to MSMEs that meet the criteria outlined in the Master Direction on
―Priority Sector Lending (PSL) – Targets and Classification‖ dated September 4, 2020 are
eligible to be classified as priority sector lending.

391. With reference to Central Bank digital currencies, consider the following statements
1. It is possible to make payments in a digital currency without using US dollar or SWIFT
system.

2. A digital currency can be distributed with a condition programmed into it such as a time-
frame for spending it.

With of the statements given above is/are correct?

A. 1 only

B. 2 only

C. Both 1 and 2

D. Neither 1 nor 2

Ans. C

CENTRAL BANK DIGITAL CURRENCY (CBDC)


 CBDCs are a digital form of a paper currency and unlike cryptocurrencies that operate in
a regulatory vacuum, these are legal tenders issued and backed by a central bank.

 It is the same as a fiat currency and is exchangeable one-to-one with the fiat currency.

 A fiat currency is a national currency that is not pegged to the price of a commodity such as
gold or silver.

https://t.me/prelimbits
402

 The digital fiat currency or CBDC can be transacted using wallets backed by blockchain.

 Though the concept of CBDCs was directly inspired by Bitcoin, it is different from
decentralised virtual currencies and crypto assets, which are not issued by the state and lack
the ‗legal tender‘ status.

 A Central Bank Digital Currency (CBDC) has the potential to eliminate intermediaries
like SWIFT and the US dollar system, allowing for more direct and efficient
transactions

 Digital currencies can be programmed with time-based conditions using smart contracts
or time-locking mechanisms, allowing for automated enforcement of rules and
restrictions.

o These conditions can control spending, activate/deactivate features, offer time-


limited incentives, or even result in the expiration or destruction of the currency if not
used within a specified timeframe.

 For Example, Digital yuan is programmable to the extent that the currency can be designed
to expire, forcing users to spend it all by a specific date.

392. In the context of finance, the term ‘beta’ refers to


A. the process of simultaneous buying and selling of an asset from different platforms

B. an investment strategy of a portfolio manager to balance risk versus reward

C. a type of systemic risk that arises where perfect hedging is not possible

D. a numeric value that measures the fluctuations of a stock to changes in the overall stock
market

Ans. D

BETA
 Beta is a metric used to assess the volatility or price fluctuations of a particular stock
relative to the overall market. It quantifies the stock's sensitivity to market movements.

 A beta of 1 indicates that the stock's price tends to align with the market. A beta greater
than 1 suggests higher volatility, meaning the stock is more responsive to market changes.
A beta less than 1 indicates lower volatility, suggesting the stock is less affected by market
movements.

 Beta is used to evaluate the systematic risk associated with an individual stock. It helps
investors understand how the stock's price may be influenced by broader market trends and
assess the potential risks and rewards associated with owning the stock.

OTHER RELATED TERMS:


 Systematic Risk

https://t.me/prelimbits
403

o Systematic risk refers to the risk inherent in the overall market or a particular market
segment. It is non-diversifiable and affects a broad range of stocks or investments.
Beta captures this systematic risk, which cannot be eliminated through diversification.

 Market Index

o A market index, such as the S&P 500 or Dow Jones Industrial Average, represents a
basket of stocks selected to reflect the performance of a specific market or sector. It
serves as a benchmark against which the performance of individual stocks or
investments can be compared.

 Alpha

o Alpha measures the excess return of a stock or investment above the return
predicted by its beta. It reflects the stock's performance relative to the market,
considering its level of risk as measured by beta. Positive alpha suggests
outperformance, while negative alpha indicates underperformance.

 Risk-free Rate

o The risk-free rate represents the hypothetical return on an investment with no risk.
It is typically based on government bonds or similar instruments considered to have
minimal risk. In finance, the risk-free rate is used as a benchmark for comparing the
returns of other investments.

 Capital Asset Pricing Model (CAPM)

o CAPM is a widely used model in finance that relates a stock's expected return to its
beta and the risk-free rate. It provides a framework for estimating the appropriate
return on an investment based on its risk profile.

 Diversification

o Diversification is the strategy of spreading investments across various asset classes,


sectors, or regions to reduce risk.

o By holding a diversified portfolio, investors aim to minimize the impact of any


individual stock's performance on their overall portfolio.

 Unsystematic Risk

o Unsystematic risk, also known as specific risk or idiosyncratic risk, refers to the risk
associated with a specific company or investment.

o It can be reduced through diversification as it is specific to individual stocks and not


related to the overall market.

393. Consider the following statements:


1. The Self-Help Group (SHG) programme was originally initiated by the State Bank of India by
providing microcredit to the financially deprived.

https://t.me/prelimbits
404

2. In an SHG, all members of a group take responsibility for a loan that an individual member
takes.

3. The Regional Rural Banks and Scheduled Commercial Banks support SHGs.

How many of the above statements are correct

A. Only one

B. Only two

C. All three

D. None

Ans. B

SHGS
 NABARD began promoting self-help groups in 1991–1992. And it served as the actual
launch pad for the ―SHG movement.‖ The Reserve Bank of India additionally permitted SHGs
to open savings banks accounts in 1993. The ability to use banking services gave the
movement a significant boost. [Statement 1 is incorrect]

 The Banks provide the loan to the Self Help Group as a whole and it is the collective
responsibility of the SHG to repay that loan. Hence all members of a group take
responsibility for a loan that an individual member takes. Further, the SHG may decide to
not provide any share in further loans to the defaulter member. [Statement 2 is correct]

 . The Financial Inclusion Fund scheme to support Scheduled Commercial Banks (SCBs) and
Regional Rural Banks (RRBs) for enabling Dual Authentication option in the microATMs for
SHG transactions is being launched to provide an enabling eco-system for the SHGs to
seamlessly operate at Business Correspondent points with the proposed ―Dual
Authentication‖ feature. Thus, it can be said that SCBs and RRBs support the SHGs in
availing the credits as per their needs. [Statement 3 is correct]

394. Consider the following statements:


Statement-I: India‘s public sector health care system largely focuses on curative care with limited
preventive, promotive and rehabilitative care.

Statement-II: Under India‘s decentralized approach to health care delivery, the States are primarily
responsible for organizing health services.

Which one of the following is correct in respect of the above statements?

A. Both Statement-I and Statement-II are correct and Statement-II is the correct explanation
for Statement-I

B. Both Statement-I and Statement-II are correct and Statement-II is not the correct
explanation for Statement-I

C. Statement-I is correct but Statement-II is incorrect

https://t.me/prelimbits
405

D. Statement-I is incorrect but Statement-II is correct

Ans. B

PUBLIC SECTOR HEALTHCARE SYSTEM


 India's public sector healthcare system comprises a network of government-run healthcare
facilities at various levels. This includes primary health centers (PHCs), community health
centers (CHCs), district hospitals, and specialized research and education centers funded by
the central and state governments.

 These facilities provide a range of healthcare services from primary care to specialized
treatments.

 The Ministry of Health and Family Welfare at the central government level has regulatory
power and oversees national health programs.

 The state governments are responsible for organizing and delivering healthcare services,
managing the healthcare workforce, implementing national health programs, collecting
health information, and supervising local healthcare entities.

 This division of responsibilities leads to variations in service delivery models, insurance


coverage, and availability of healthcare services across different states.

 The central government establishes policies and regulations, while the state governments
are responsible for organizing and delivering healthcare services to their populations.
[Statement 2 is correct]

 This decentralized approach allows for flexibility and adaptation to local needs and
contexts.

 India's public sector healthcare system primarily focuses on curative care, with limited
emphasis on preventive, promotive, and rehabilitative care.

 The healthcare infrastructure in India has developed around the provision of curative
services, with a significant concentration of hospitals and specialized medical facilities. This
structural bias towards curative care is reflected in the allocation of resources and the
training of healthcare professionals who are primarily oriented towards diagnosis and
treatment. [Statement 1 is correct]

 Hence, Both Statements are correct but Statement-II is not the correct explanation for
Statement-I.

 Ayushman Bharat (PMJAY) was launched as a step towards Universal Health Coverage
(UHC). UHC entails ensuring all people have access to quality health services – including
prevention, promotion, treatment, rehabilitation, and palliation – without incurring
financial hardship.

https://t.me/prelimbits
406

395. Consider the following statements:


Statement-I: Recently, the United States of America (USA) and the European Union (EU) have
launched the Trade and technology council.

Statement-II: The USA and the EU claim that through this they are trying to bring technological
progress and physical productivity under their control.

Which one of the following is correct in respect of the above statements?

A. Both statement-I and Statement-II are correct and Statement-II is the correct explanation
for Statement-I

B. Both Statement-I and Statement-II are correct and Statement-II is not the correct
explanation for Statement-I

C. Statement-I is correct but Statement-II is incorrect.

D. Statement-I- is incorrect but Statement-II is correct.

Ans. C

USA-EU TRADE AND TECHNOLOGY COUNCIL


 Trade and Technology Council was launched by the the United States of America and
the European Union during the EU-US Summit on 15 June 2021 in Brussels.

 Trade and Technology council, the USA and EU does not aim to bring technological
progress and physical productivity under their control. Rather the initiative intends
to promote pooling of digital resources such as AI models and computing power, and
make them available to partner countries to address challenges in areas such as climate
change and extreme weather, healthcare or smart agriculture.

396. Consider the following statement


Statement – I: India accounts for 3.2% of global export of goods.

Statement-II: Many local companies and some foreign companies operating in India have taken
advantage of India‘s ‗Production-linked Incentive‘ scheme.

Which one of the following is correct in respect of the above statements?

A. Both statement-I and Statement-II are correct and Statement-II is the correct explanation
for Statement-I

B. Both Statement-I and Statement-II are correct and Statement-II is not the correct
explanation for Statement-I

C. Statement-I is correct but Statement-II is incorrect.

D. Statement-I- is incorrect but Statement-II is correct.

Ans. D

https://t.me/prelimbits
407

 As per the latest data available (Economic Survey 2022-23) India‘s accounts for 1.8% of
Global export of goods and 4% of global export of services. Thus, India accounts for
3.2% if global exports of goods are not correct. [Statement 1 is incorrect]

 Production Linked Incentive (PLI) scheme is a form of Performance-linked incentive given


to companies based on their incremental sales from products manufactured in domestic
units. It is aimed at boosting the manufacturing sector and to reduce imports of India.

 PLI schemes can be availed by both domestic as well as foreign companies operating in
India, thus it is true that many domestic and foreign companies operating in India have
taken advantage of PLI schemes [Statement 2 is correct]

397. Consider the following statements:


The ‗Stability and Growth Pact‘ of the European Union is a treaty that

1. Limits the levels of budgetary deficit of the countries of the European Union.

2. Makes the countries of the European Union to share their infrastructure facilities.

3. Enables the countries of the European Union to share their technologies.

How many of the above statements are correct?

A. Only one

B. Only two

C. All three

D. None

Ans. A

STABILITY AND GROWTH PACT (SGP)


 The Stability and Growth Pact (SGP) is an agreement, among all of the 27 member
states of the European Union, to facilitate and maintain the economic stability of the
EU countries. The European Commission and the Council of the European Union, monitors
the fiscal condition of EU member countries from time to time to ensure their fiscal stability.

 It is true that SGP aims to level the budget deficits of European countries. The
corrective arm of the Stability and Growth Pact ensures that Member States
adopt appropriate policy responses to correct excessive deficits (and/or debts) by
implementing the Excessive Deficit Procedure (EDP). [Statement 1 is correct]

 Also the SGP requires the EU Member States to lay out their fiscal plans for the next three
years to limit their budget deficits.

 The SGP treaty does not require its members to share their infrastructure facilities nor their
technologies with other countries. The purpose of the SGP was to ensure that fiscal
discipline would be maintained and enforced in the European Union. [Statement 2
and 3 are incorrect]

https://t.me/prelimbits
408

398. Consider the following statements:


1. Recently, all the countries of the United Nations have adopted the first-ever compact for
international migration, the ‗Global Compact for Safe, Orderly and Regular Migration
(GCM)‘.

2. The objective and commitments stated in the GCM are binding on the UN member
countries.

3. The GCM address internal migration or internally displaced people also in its objectives and
commitments.

How many of the above statements are correct?

A. Only one

B. Only two

C. All three

D. None

Ans. A

GLOBAL COMPACT FOR MIGRATION


 The Global Compact for Safe, Orderly and Regular Migration (GCM) is an
intergovernmentally negotiated agreement, prepared under the auspices of the United
Nations, that describes itself as covering "all dimensions of international migration in a
holistic and comprehensive manner".

 The compact was formally endorsed by the United Nations General Assembly on 19
December 2018.

 Importantly, the Compact is a "non-binding cooperative framework", meaning it has


limited weight under International Law.

https://t.me/prelimbits
409

399. Consider the following statements in relation to Janani Suraksha Yojana


1. It is a safe motherhood intervention of the State Health Departments.

2. Its objective is reduce maternal and neonatal mortality among poor pregnant women.

3. It aims to promote institutional delivery among poor pregnant women.

4. Its objective includes providing public health facilities to sick infants up one year of age.

How many of the statements given above are correct?

A. Only one

B. Only two

C. All three

D. None

Ans. B

 Janani Suraksha Yojana (JSY) is a safe motherhood intervention under the National
Health Mission (NHM). The Yojana was launched on 12th April 2005. It is being
implemented in all states and UTs with special focus on low performing states.

o JSY is a 100 % centrally sponsored scheme and it integrates cash assistance with
delivery and post-delivery care.

 Objectives

o It was Launched for reducing maternal and neonatal mortality, to promote


institutional delivery among pregnant women especially with weak socio-economic
status i.e. women from Scheduled Castes, Scheduled Tribes and BPL households.

 Cash assistance

https://t.me/prelimbits
410

o Under the JSY, eligible pregnant women are entitled to cash assistance irrespective of
the age of mother and number of children for giving birth in a government or
accredited private health facility.

 Area of Focus

o The scheme focuses on poor pregnant woman with special dispensation for states
that have low institutional delivery rates, namely, the states of Uttar Pradesh,
Uttarakhand, Bihar, Jharkhand, Madhya Pradesh, Chhattisgarh, Assam, Rajasthan,
Odisha, and Jammu and Kashmir.

 While these States have been named Low Performing States (LPS) under the
scheme, the remaining States/UTs have been named High Performing States
(HPS).

 The scheme also provides performance-based incentives to women health volunteers


known as ASHA (Accredited Social Health Activist) for promoting institutional delivery
among pregnant women.

JANANI SHISHU SURAKSHA KARYAKARAM (JSSK


https://nhm.gov.in/index1.php?lang=1&level=3&sublinkid=842&lid=308

 GoI launched Janani Shishu Suraksha Karyakaram (JSSK) in June 2011 to eliminate out-
of-pocket expenses for pregnant women delivering in public health institutions and
sick infants accessing public health institutions for treatment.

 The entitlements include →

o Free and Zero Expense delivery including C-section, free care in case of ante-natal &
post-natal complications

o Free drugs, diagnostics, blood and consumables

o Free diet during stay in facilities

o Free transport home to health institution, between health institutions in case of


referral and drop back home

o Exemption from all kinds of user charges

o Similar entitlements for sick Infants (upto 1 year of age)

o Extended to all antenatal & post-natal complications of pregnancy

o The number of beneficiaries of Supplementary Nutrition (Pregnant Women and


Lactating Mothers) under Anganwadi Services during the last three years and the
current year are tabulated below

https://t.me/prelimbits
411

400. Consider the following statements in the context of intervention being undertaken
under Anaemia Mukt Bharat Strategy
1. It provides prophylactic calcium supplementation for pre-school children, adolescents and
pregnant women.

2. It runs a campaign for delayed cord clamping at the time of child birth.

3. It provides for periodic deworming to children and adolescents.

4. It addresses non-nutritional causes of anemia in endemic pockets with special focus on


malaria, hemoglobinopathies and fluorosis.

How many of the statements given above are correct?

A. Only one

B. Only two

C. Only three

D. All Four

Ans. C

ANAEMIA MUKT BHARAT


https://pib.gov.in/Pressreleaseshare.aspx?PRID=1596308

 It is an initiative by the Government of India to combat anaemia.

 It operates under the umbrella of the National Nutrition Strategy. The AMB strategy
is comprehensive and involves multiple interventions.

 The AMB Strategy does provide for prophylactic iron and folic acid supplementation,
which is essential for haemoglobin production, but it doesn't focus on calcium
supplementation. [statement 1 is incorrect]

 It runs a campaign for delayed cord clamping at the time of childbirth.

 This is a component of the AMB Strategy. Delayed cord clamping is promoted because it
is known to reduce the risk of anaemia in the newborn.

 It provides for periodic deworming to children and adolescents.

 Deworming is a significant part of the AMB Strategy because intestinal worms can
interfere with the absorption of nutrients, contributing to anaemia.

 The AMB Strategy also targets non-nutritional causes of anaemia in areas where they
are prevalent. This includes diseases like malaria and conditions like hemoglobinopathies
and fluorosis, which can cause or exacerbate anaemia.

https://t.me/prelimbits
412

401. Consider the following statements:


Statement-I: Interest income from the deposits in Infrastructure Investment Trusts (InvITs)
distributed to their investors is exempted from tax, but the dividend is taxable.

Statement-II: InvITs are recognized as borrowers under the ―Secuntization and Reconstruction of
Financial Assets and Enforcement of Security Interest-Act, 2002:

Which one of the following is correct in respect of the above statements?

A. Both Statement-I and Statement-II are correct and Statement-II is the correct explanation
for Statement-I

B. Both Statement-I and Statement-II are correct and Statement-II is not the correct
explanation for Statement-I

C. Statement-I is correct but Statement-II is incorrect

D. Statement-I is incorrect but Statement-II is correct

Ans. D

INFRASTRUCTURE INVESTMENT TRUSTS (INVITS)


 InvITs are investment vehicles that pool funds from investors to invest in infrastructure
projects. InvITs are pass-through entities, which means that the income and expenses of the
InvIT are passed through to the unitholders. Under the latest budget 2023-24, all income
distributed by InvITs, including interest income, dividend income, and rental income, will be
taxable in the hands of unitholders. This is a change from the previous tax regime, under
which only dividend income from InvITs was taxable. The reason for this change is to widen
the tax base and to ensure that all income from InvITs is taxed, regardless of its source. This
will help to ensure that all investors in InvITs pay their fair share of taxes. It is important to
note that this change will only affect income distributed by InvITs. Income distributed by
other types of trusts, such as real estate investment trusts (REITs), will not be affected.
[Statement 1 is incorrect]

 Here are some additional details about the tax treatment of InvITs under the latest
budget:

o Interest income from deposits in InvITs will be taxed at the investor‘s marginal
income tax rate.

o Dividend income from InvITs will be taxed at the dividend distribution tax (DDT) rate
of 15%.

o Rental income from InvITs will be taxed at the investor‘s marginal income tax rate.

 InvITs are classified as borrowers under the Securitization and Reconstruction of Financial
Assets and Enforcement of Security Interest Act, 2002 (SARFAESI Act). The SARFAESI Act
provides a legal framework for the securitization of financial assets and enables the
enforcement of security interests in case of default. [Statement 2 is correct]

https://t.me/prelimbits
413

402. Consider the following statements


Statement-I: In the post-pandemic recent past, many Central Banks worldwide had carried out
interest rate hikes.

Statement-II: Central Banks generally assume that they have the ability to counteract the rising
consumer prices via monetary policy means.

Which one of the following is correct in respect of the above statements?

A. Both Statement-I and Statement-II are correct and Statement-II is the correct explanation
for Statement-I

B. Both Statement-I and Statement-II are correct and Statement-II is not the correct
explanation for Statement-I

C. Statement-I is correct but Statement II is incorrect

D. Statement-I is incorrect but Statement –II ic correct

Ans. A

 In the post-pandemic recent past, many central banks worldwide have indeed carried out
interest rate hikes. Central banks use interest rate adjustments as one of their monetary
policy tools to manage the economy. When the economy is recovering and inflationary
pressures increase, central banks may choose to increase interest rates to control inflation
and ensure price stability. By raising interest rates, central banks aim to reduce
borrowing and spending, which can help cool down an overheating economy and
prevent excessive inflation. [Statement 1 is correct]

 Monetary policy refers to the actions taken by central banks to influence the supply of
money and credit in the economy. Central banks believe that by adjusting interest rates,
they can influence borrowing costs, spending patterns, and overall economic activity. When
inflation rises, central banks may raise interest rates to make borrowing more expensive,
thereby discouraging excessive spending and slowing down economic growth. By doing so,
they aim to dampen inflationary pressures and maintain price stability. [Statement 2 is
correct]

 Therefore, statement 2 provides the correct explanation for statement 1. Central


banks believe that by using monetary policy tools, such as interest rate hikes, they can
effectively counteract rising consumer prices and control inflationary pressures in the
economy.

403. Which one of the following activities of the Reserve Bank of India is considered to be
part of 'sterilization'?
A. Conducting 'Open Market Operations'

B. Oversight of settlement and payment systems

C. Debt and cash management for the Central and State Governments
https://t.me/prelimbits
414

D. Regulating the functions of Non-banking Financial Institutions

Ans. A

STERILIZATION
 Sterilization refers to the actions taken by a central bank to offset the impact of its
foreign exchange operations on the domestic money supply.

 When a central bank intervenes in the foreign exchange market by buying or selling
foreign currencies, it affects the domestic money supply.

 Open Market Operations (OMO) is one of the primary tools used by central banks,
including the Reserve Bank of India (RBI), to conduct monetary policy.

 In OMO, the central bank buys or sells government securities (bonds) in the open
market to influence the liquidity in the economy.

 When the RBI conducts OMO, it impacts the money supply in the economy. If the RBI
buys government securities, it injects money into the system, increasing the money supply.
To prevent this injection of money from creating inflationary pressures, the RBI engages in
sterilization.

 Sterilization involves the simultaneous sale or purchase of other securities, typically


treasury bills, to offset the impact of the initial open market operation.

404. Consider the following markets


1. Government Bond Market

2. Call Money Market

3. Treasury Bill Market

4. Stock Market

How many of the above are included in Capital Markets?

A. Only one

B. Only two

C. Only three

D. All four

Ans. B

CAPITAL MARKETS & MONEY MARKET


 Capital markets are financial markets where long-term securities, such as stocks and
bonds, are traded. They provide a platform for raising capital for businesses and
governments.

https://t.me/prelimbits
415

 On the other hand, Money market are financial markets where short term securities such
as T-Bill, C-Paper, Cash Management Bills, Ways and Means advances, etc are trade.

 Government bonds are long-term debt securities issued by governments to finance their
activities. The government bond market is a part of the capital market as it involves the
trading of long-term debt securities. [Statement 1 is correct]

 The CALL MONEY market is a short-term market where funds are borrowed and lent for
very short durations, usually overnight. It deals with short-term funds, and its transactions
are not classified as part of the capital market. [Statement 2 is incorrect]

 TREASURY BILLS are short-term debt instruments issued by governments to finance their
short-term cash flow requirements. The treasury bill market, similar to the call money
market, deals with short-term instruments and is not considered part of the capital market.
[Statement 3 is incorrect]

 The Stock Market, also known as the equity market or share market, is where shares or
stocks of publicly listed companies are bought and sold. The stock market is a part of the
capital market as it involves the trading of ownership interests (equity securities) in
companies. [Statement 4 is correct]

405. Which one of the following best describes the concept of 'Small Farmer Large Field'?
A. Resettlement of a large number of people, uprooted from their countries due to war, by
giving them a large cultivable land which they cultivate collectively and share the produce

B. Many marginal farmers in an area organize themselves into groups and synchronize and
harmonize selected agricultural operations

C. Many marginal farmers in_ an area together make a contract with a corporate body are
surrender their land to the corporate body of a fixed term for which the corporate body
makes a payment of agreed amount to the farmers

D. A company extends loans, technical knowledge and material inputs to a number of small
farmers in an area so that they produce the agricultural commodity required by the
company for its manufacturing process and commercial production

Ans. B

SMALL FARMERS LARGE FIELD (SFLF)


 Small Farmers Large Field (SFLF) is a collective action model that aims to improve the
livelihood of small and marginal farmers by overcoming the disadvantages they face
due to diseconomies of scale and lack of bargaining power in the supply chain.

 The model is participatory and flexible, and allows small farmers to benefit from achieving
economies of scale by organizing themselves into groups and synchronizing and
harmonizing selected operations.

https://t.me/prelimbits
416

 The SFLF model was piloted in two villages of Odisha, an eastern Indian state, with 112
farmers (35 females and 77 males).

o The results of the pilot project showed that the model was effective in improving
the livelihoods of small farmers.

o The farmers who participated in the project were able to reduce their costs of
production, increase their yields, and get a better price for their produce.

o They also reported an increase in their income and a decrease in their indebtedness.

406. Consider the following statements:


1. The Government of India provides Minimum Support Price for Niger (Guizotia abyssinica)
seeds.

2. Niger is cultivated as a Kharif crop.

3. Some tribal people in India use Niger seed oil for cooking.

How many of the above statements are correct?

A. Only one

B. Only two

C. All three

D. None

Ans. C

NIGER SEED
https://www.downtoearth.org.in/news/agriculture/illusive-oilseed-india-s-niger-seed-cultivation-
is-declining-here-is-why-84380

https://vikaspedia.in/agriculture/crop-production/package-of-
practices/oilseeds/niger#:~:text=Niger%20is%20a%20minor%20oilseed,taste%20and%20a%20plea
sant%20odour.

 The Government of India provides Minimum Support Price (MSP) of Rs. 6930 (absolute
increase of Rs. 235 per quintal) for niger seeds for MSP 2021-22.

 Niger, a minor oilseed crop, is primarily cultivated in rainfed areas. The seeds of Niger are
utilized as a food source for humans. With an oil content ranging from 37% to 47%, the oil
extracted from the seeds has a pale yellow color, a nutty flavor, and a pleasant aroma.

 This oil finds application in various domains such as culinary purposes, anointing the
body, manufacturing paints and soft soaps, as well as for lighting and lubrication.
Additionally, the fragrance-absorbing properties of Niger oil make it valuable to the
perfume industry as a base oil. Moreover, it is believed that Niger oil can be used for
contraception and as a treatment for syphilis. [Statement 1 is correct]

https://t.me/prelimbits
417

 Niger, also known as Guizotia abyssinica, is primarily cultivated as a Kharif crop in India.
Kharif season refers to the monsoon season, typically from June to October, during which
crops are sown and harvested. [Statement 2 is correct]

 The tribal communities incorporate niger seed oil into their cooking practices, utilize
the press cake remaining after oil extraction as livestock feed, and even consume the seeds
themselves as a condiment. Niger seed oil possesses medicinal properties, leading to its
high demand in commercial sectors such as cosmetics, perfumeries, and related industries.
[Statement 3 is correct]

407. Consider the investments in the following assets:


1. Brand recognition

2. Inventory

3. Intellectual property

4. Mailing list of clients

How many of the above intangible investments?

A. Only one

B. Only two

C. Only three

D. All four

Ans. C

 A brand serves as a distinct symbol, logo, or name that companies utilize to differentiate
their product from others in the market. Brand equity is classified as an intangible asset
since its worth does not derive from a physical entity and is primarily influenced by how
consumers perceive the brand. [Statement 1 is correct]

 Inventory refers to the goods or products that a company holds for sale. It represents
tangible assets, as inventory consists of physical items that can be seen, touched, and
measured. Therefore, it is not considered an intangible investment. [Statement 2 is
incorrect]

 Intellectual property (IP) includes intangible assets such as patents, trademarks,


copyrights, and trade secrets. These are legal rights that protect creations of the mind, such
as inventions, artistic works, and business names. Intellectual property is considered an
intangible investment as it involves the acquisition and protection of intangible assets.
[Statement 3 is correct]

 The mailing list of client is among the intangible assets of a company, encompassing a
compilation of past customers. Building a customer list is a time-consuming process that
holds substantial value for a business in the long run. It serves as an invaluable asset for
targeted marketing efforts, enabling the company to promote new or existing products and
https://t.me/prelimbits
418

services to specific customer segments. Additionally, the customer list plays a crucial role in
attracting new business opportunities. [Statement 4 is correct]

408. Consider the following:


1. Demographic performance

2. Forest and ecology

3. Governance reforms

4. Stable government

5. Tax and fiscal efforts

For the horizontal tax devolution, the Fifteenth Finance Commission used how many of the above
as criteria other than population area and income distance?

A. Only two

B. Only three

C. Only four

D. All 'five

Ans. B

HORIZONTAL TAX DEVOLUTION- COMPARISON BETWEEN 14TH AND 15TH FINANCE COMMISSIO
Parameters 14th Finance 15th Finance 15th Finance
Commission (2015- Commission (2020- Commission (2021-26)
20) 21)

Population (1971) 17.5% 0% 0%

Population (2011) 10% 15% 15%

Area 15% 15% 15%

Forest Cover 7.5% 0% 0%

Forest and Ecology 0% 10% 10%

Income Distance 50% 45% 45%

Tax and Fiscal Efforts 0% 2.5% 2.5%

Demographic 0% 12.5% 12.5%


Performance

409. Consider the following infrastructure sectors:


1. Affordable housing

https://t.me/prelimbits
419

2. Mass rapid transport

3. Health rare

4. Renewable energy

On how man of the above does (UNOPS) Sustainable Investments in Infrastructure and Innovation
(S3i) ipitiative focus for its investments?

A. Only one

B. Only two

C. Only three

D. All four

Ans. C

SUSTAINABLE INVESTMENTS IN INFRASTRUCTURE AND INNOVATION (S3I)


 Established in 1973 as a component of the United Nations Development Programme
(UNDP), UNOPS gained autonomy and financial independence in 1995.

 UNOPS‘ overarching vision is to promote sustainable implementation practices in the


realms of development, humanitarian aid, and peacebuilding.

 The development of climate-resilient and sustainable infrastructure is crucial across various


sectors that play a significant role in sustainable development.

 Statement 1 is correct

o UNOPS S3i initiative prioritize affordable Housing. The objective in the field of
affordable housing is to provide significant support for affordable housing initiatives
targeting the middle-income and lower-middle-income groups of the population.
The focus is on implementing these projects on a large scale.

 Statement 2 is incorrect-

o Mass Rapid Transport is not included in the UNOPS S3i initiative.

 Statement 3 is correct

o UNOPS S3i initiative prioritize Healthcare. Regarding healthcare, the objective is to


enhance the quality of healthcare in the specified countries by providing support for
healthcare infrastructure projects. These projects aim to address the needs of both
private and public healthcare systems with a particular emphasis on primary care
clinics, hospitals, diagnostic centers, laboratories, and the potential implementation
of established healthcare technologies. The goal is to bring about effective and
sustainable solutions to meet healthcare demands.

 Statement 4 is correct

https://t.me/prelimbits
420

o UNOPS S3i initiative prioritize Renewable Energy. Within the realm of renewable
energy, the primary emphasis lies in investing in infrastructure projects related to
renewable energy sources, including but not limited to solar, hydro, wind, biomass,
and hydrogen, particularly in eligible emerging markets and developing nations. The
main focus is on the generation of renewable energy and the accompanying
distribution and storage infrastructure. However, investments in innovative
technologies and solutions throughout the renewable energy value chain may also
be taken into consideration.

https://t.me/prelimbits

You might also like